You are on page 1of 418

FUNDA

1. Using the principles of standard precautions, the nurse would wear gloves in what nursing interventions?
A. Providing a back massage
B. Feeding a client
C. Providing hair care
D. Providing oral hygiene
2. The nurse is preparing to take vital sign in an alert client admitted to the hospital with dehydration
secondary to vomiting and diarrhea. What is the best method used to assess the clients temperature?
A. Oral
B. Axillary
C. Radial
D. Heat sensitive tape
3. A nurse obtained a clients pulse and found the rate to be above normal. The nurse document this
findings as:
A. Tachypnea
B. Hyperpyrexia
C. Arrhythmia
D. Tachycardia
4. Which of the following actions should the nurse take to use a wide base support when assisting a client
to get up in a chair?
A. Bend at the waist and place arms under the clients arms and lift
B. Face the client, bend knees and place hands on clients forearm and lift
C. Spread his or her feet apart
D. Tighten his or her pelvic muscles
5. A client had oral surgery following a motor vehicle accident. The nurse assessing the client finds the skin
flushed and warm. Which of the following would be the best method to take the clients body temperature?
A. Oral
B. Axillary
C. Arterial line
D. Rectal

6. A client who is unconscious needs frequent mouth care. When performing a mouth care, the best
position of a client is:
A. Fowlers position
B. Side lying
C. Supine
D. Trendelenburg
7. A client is hospitalized for the first time, which of the following actions ensure the safety of the client?
A. Keep unnecessary furniture out of the way
B. Keep the lights on at all time
C. Keep side rails up at all time
D. Keep all equipment out of view
8. A walk-in client enters into the clinic with a chief complaint of abdominal pain and diarrhea. The nurse
takes the clients vital sign hereafter. What phrase of nursing process is being implemented here by the
nurse?
A. Assessment
B. Diagnosis
C. Planning
D. Implementation
9. It is best describe as a systematic, rational method of planning and providing nursing care for individual,
families, group and community
A. Assessment
B. Nursing Process
C. Diagnosis
D. Implementation
10. Exchange of gases takes place in which of the following organ?
A. Kidney
B. Lungs
C. Liver
D. Heart
11. The chamber of the heart that receives oxygenated blood from the lungs is the:

A. Left atrium
B. Right atrium
C. Left ventricle
D. Right ventricle
12. A muscular enlarge pouch or sac that lies slightly to the left which is used for temporary storage of
food
A. Gallbladder
B. Urinary bladder
C. Stomach
D. Lungs
13. The ability of the body to defend itself against scientific invading agent such as baceria, toxin, viruses
and foreign body
A. Hormones
B. Secretion
C. Immunity
D. Glands
14. Hormones secreted by Islets of Langerhans
A. Progesterone
B. Testosterone
C. Insulin
D. Hemoglobin
15. It is a transparent membrane that focuses the light that enters the eyes to the retina.
A. Lens
B. Sclera
C. Cornea
D. Pupils
16. Which of the following is included in Orems theory?
A. Maintenance of a sufficient intake of air
B. Self perception

C. Love and belongingness


D. Physiologic needs
17. Which of the following cluster of data belong to Maslows hierarchy of needs
A. Love and belonging
B. Physiologic needs
C. Self actualization
D. All of the above
18. This is characterized by severe symptoms relatively of short duration.
A. Chronic Illness
B. Acute Illness
C. Pain
D. Syndrome
19. Which of the following is the nurses role in the health promotion
A. Health risk appraisal
B. Teach client to be effective health consumer
C. Worksite wellness
D. None of the above
20. It is describe as a collection of people who share some attributes of their lives.
A. Family
B. Illness
C. Community
D. Nursing
21. Five teaspoon is equivalent to how many milliliters (ml)?
A. 30 ml
B. 25 ml
C. 12 ml
D. 22 ml
22. 1800 ml is equal to how many liters?

A. 1.8
B. 18000
C. 180
D. 2800
23. Which of the following is the abbreviation of drops?
A. Gtt.
B. Gtts.
C. Dp.
D. Dr.
24. The abbreviation for micro drop is
A. gtt
B. gtt
C. mdr
D. mgts
25. Which of the following is the meaning of PRN?
A. When advice
B. Immediately
C. When necessary
D. Now
26. Which of the following is the appropriate meaning of CBR?
A. Cardiac Board Room
B. Complete Bathroom
C. Complete Bed Rest
D. Complete Board Room
27. One (1) tsp is equals to how many drops?
A. 15
B. 60
C. 10
D. 30

28. 20 cc is equal to how many ml?


A. 2
B. 20
C. 2000
D. 20000
29. 1 cup is equals to how many ounces?
A. 8
B. 80
C. 800
D. 8000
30. The nurse must verify the clients identity before administration of medication. Which of the following is
the safest way to identify the client?
A. Ask the client his name
B. Check the clients identification band
C. State the clients name aloud and have the client repeat it
D. Check the room number
31. The nurse prepares to administer buccal medication. The medicine should be placed
A. On the clients skin
B. Between the clients cheeks and gums
C. Under the clients tongue
D. On the clients conjunctiva
32. The nurse administers cleansing enema. The common position for this procedure is
A. Sims left lateral
B. Dorsal Recumbent
C. Supine
D. Prone
33. A client complains of difficulty of swallowing, when the nurse try to administer capsule medication.
Which of the following measures the nurse should do?

A. Dissolve the capsule in a glass of water


B. Break the capsule and give the content with an applesauce
C. Check the availability of a liquid preparation
D. Crash the capsule and place it under the tongue
34. Which of the following is the appropriate route of administration for insulin?
A. Intramuscular
B. Intradermal
C. Subcutaneous
D. Intravenous
35. The nurse is ordered to administer ampicillin capsule TID p.o. The nurse should give the medication
A. Three times a day orally
B. Three times a day after meals
C. Two time a day by mouth
D. Two times a day before meals
36. Back Care is best describe as:
A. Caring for the back by means of massage
B. Washing of the back
C. Application of cold compress at the back
D. Application of hot compress at the back
37. It refers to the preparation of the bed with a new set of linens
A. Bed bath
B. Bed making
C. Bed shampoo
D. Bed lining
38. Which of the following is the most important purpose of handwashing
A. To promote hand circulation
B. To prevent the transfer of microorganism
C. To avoid touching the client with a dirty hand
D. To provide comfort

39. What should be done in order to prevent contaminating of the environment in bed making?
A. Avoid fanning soiled linens
B. Strip all linens at the same time
C. Finished both sides at the time
D. Embrace soiled linen
40. The most important purpose of cleansing bed bath is:
A. To cleanse, refresh and give comfort to the client who must remain in bed
B. To expose the necessary parts of the body
C. To develop skills in bed bath
D. To check the body temperature of the client in bed
41. Which of the following technique involves the sense of sight?
A. Inspection
B. Palpation
C. Percussion
D. Auscultation
42. The first techniques used examining the abdomen of a client is:
A. Palpation
B. Auscultation
C. Percussion
D. Inspection
43. A technique in physical examination that is use to assess the movement of air through the
tracheobronchial tree:
A. Palpation
B. Auscultation
C. Inspection
D. Percussion
44. An instrument used for auscultation is:
A. Percussion-hammer
B. Audiometer

C. Stethoscope
D. Sphygmomanometer
45. Resonance is best describe as:
A. Sounds created by air filled lungs
B. Short, high pitch and thudding
C. Moderately loud with musical quality
D. Drum-like
46. The best position for examining the rectum is:
A. Prone
B. Sims
C. Knee-chest
D. Lithotomy
47. It refers to the manner of walking
A. Gait
B. Range of motion
C. Flexion and extension
D. Hopping
48. The nurse asked the client to read the Snellen chart. Which of the following is tested:
A. Optic
B. Olfactory
C. Oculomotor
D. Trochlear
49. Another name for knee-chest position is:
A. Genu-dorsal
B. Genu-pectoral
C. Lithotomy
D. Sims
50. The nurse prepare IM injection that is irritating to the subcutaneous tissue. Which of the following is the
best action in order to prevent tracking of the medication

A. Use a small gauge needle


B. Apply ice on the injection site
C. Administer at a 45 angle
D. Use the Z-track technique

Answers and Rationale


1. Answer: D. Providing oral hygiene
Doing oral care requires the nurse to wear gloves.
2. Answer: B. Axillary
Axilla is the most accessible body part in this situation.
3. Answer: D. Tachycardia
Tachycardia means rapid heart rate. Tachypnea (Option A) refers to rapid respiratory rate. Hyperpyrexia (Option B)
means increase in temperature. Arrhythmia (Option C) means irregular heart rate.
4. Answer: B. Face the client, bend knees and place hands on clients forearm and lift
This is the proper way on supporting the client to get up in a chair that conforms to safety and proper body
mechanics.
5. Answer: B. Axillary
Taking the temperature via the oral route is incorrect since the client had oral surgery. Choice C and D are
unnecessary. Taking the temperature via the axilla is the most appropriate route.
6. Answer: B. Side lying
An unconscious client is best placed on his side when doing oral care to prevent aspiration.
7. Answer: C. Keep side rails up at all time
Although the other choices seem correct, they are not the best answer.
8. Answer: A. Assessment

Assessment is the first phase of the nursing process where a nurse collects information about the client. Diagnosis
is the formulation of the nursing diagnosis from the information collected during the assessment. In Planning, the
nurse sets achievable and measurable short and long term goals. Implementation is where nursing care is given.
9. Answer: B. Nursing Process
The statement describes the Nursing Process. The Nursing Process is the essential core of practice for the
registered nurse to deliver holistic, patient-focused care.
10. Answer: B. Lungs
11. Answer: A. Left atrium
The left atrium receives oxygenated blood from the lungs and pumps it to the left ventricle. The right atrium
receives blood from the veins and pumps it to the right ventricle. The right ventricle receives blood from the right
atrium and pumps it to the lungs, where it is loaded with oxygen. The left ventricle (the strongest chamber) pumps
oxygen-rich blood to the rest of the body, its vigorous contractions create the blood pressure.
12. Answer: C. Stomach
13. Answer: C. Immunity
14. Answer: C. Insulin
The Islets of Langerhans are the regions of the pancreas that contain its endocrine cells. Progesterone (Choice A) is
produced by the ovaries. Testosterone (Choice B) is secreted by the testicles of males and ovaries of females.
Hemoglobin (Choice D) is a protein molecule in the red blood cells that carries oxygen from the lungs to the bodys
tissues and returns carbon dioxide.
15. Answer: C. Cornea
The cornea is the transparent front part of the eye that covers the iris, pupil, and anterior chamber. The cornea is like
the crystal of a watch.
16. Answer: A. Maintenance of a sufficient intake of air
Dorothea Orems Self-Care Theory defined Nursing as The act of assisting others in the provision and management
of self-care to maintain or improve human functioning at home level of effectiveness. Choices B, C, and D are from
Abraham Maslows Hierarchy of Needs.

17. Answer: D. All of the above


All of the choices are part of Maslows Hierarchy of Needs.
18. Answer: B. Acute Illness
Chronic Illness (Choice A) are illnesses that are persistent or long-term.
19. Answer: B. Teach client to be effective health consumer
20. Answer: C. Community
Family is defined as a group consisting typically of parents and children living together in a household.
21. Answer: B. 25 ml
One teaspoon is equal to 5ml.
22. Answer: A. 1.8
23. Answer: B. Gtts.
Gtt (Choice A) is an abbreviation for drop. Dp and Dr are not recognized abbreviation for measurement.
24. Answer: A. gtt
25. Answer: C. When necessary
PRN comes from the Latin pro re nata meaning, for an occasion that has arisen or as circumstances require.
26. Answer: C. Complete Bed Rest
CBR means complete bed rest. For more abbreviations, please see this post.
27. Answer: B. 60
One teaspoon (tsp) is equal to 60 drops (gtts).
28. Answer: B. 20

One cubic centimeter is equal to one milliliter.


29. Answer: A. 8
One cup is equal to 8 ounces.
30. Answer: B. Check the clients identification band
The identification band is the safest way to know the identity of a patient whether he is conscious or unconscious.
Ask the client his name only after you have checked his ID band.
31. Answer: B. Between the clients cheeks and gums
32. Answer: A. Sims left lateral
This position provides comfort to the patient and an easy access to the natural curvature of the rectum.
33. Answer: C. Check the availability of a liquid preparation
The nurse should check first if the medication is available in liquid form before doing Choice A. Placing it under the
tongue is not the intended way of administering an oral medication.
34. Answer: C. Subcutaneous
The subcutaneous tissue of the abdomen is preferred because absorption of the insulin is more consistent from this
location than subcutaneous tissues in other locations.
35. Answer: A. Three times a day orally
TID is the Latin for ter in die which means three times a day. P.O. means per orem or through mouth.
36. Answer: A. Caring for the back by means of massage
37. Answer: B. Bed making
38. Answer: B. To prevent the transfer of microorganism
Hand washing is the single most effective infection control measure.
38. Answer: A. Avoid fanning soiled linens

Fanning soiled linens would scatter the lodged microorganisms and dead skin cells on the linens.
40. Answer: A. To cleanse, refresh and give comfort to the client who must remain in bed
41. Answer: A. Inspection
Palpation is a method of feeling with the fingers or hands during a physical examination. Percussion is a method of
tapping on a surface to determine the underlying structure, and is used in clinical examinations to assess the
condition of the thorax or abdomen. Auscultation (based on the Latin verb auscultare to listen) is listening to the
internal sounds of the body, usually using a stethoscope.
42. Answer: D. Inspection
For abdominal exam, auscultation is performed before palpation because the act of palpation could change what
was auscultated. Remember the mnemonic I-A-Per-Pal.
43. Answer: B. Auscultation
44. Answer: C. Stethoscope
45. Answer: A. Sounds created by air filled lungs
46. Answer: C. Knee-chest
To assume the genupectoral position the person kneels so that the weight of the body is supported by the knees and
chest, with the buttocks raised. The head is turned to one side and the arms are flexed so that the upper part of the
body can be supported in part by the elbows.
47. Answer: A. Gait
48. Answer: A. Optic
Cranial Nerve II or the optic nerve is tested through the use of the Snellen chart.
49. Answer: B. Genu-pectoral
50. Answer: D. Use the Z-track technique
During the procedure, skin and tissue are pulled and held firmly while a long needle is inserted into the muscle. After
the medication is injected, the skin and tissue are released. The needle track that forms during this procedure takes

the shape of the letter Z, which gives the procedure its name. This zigzag track line is what prevents medication
from leaking from the muscle into surrounding tissue.
1. The most appropriate nursing order for a patient who develops dyspnea and shortness of breath would
be
A. Maintain the patient on strict bed rest at all times
B. Maintain the patient in an orthopneic position as needed
C. Administer oxygen by Venturi mask at 24%, as needed
D. Allow a 1 hour rest period between activities
2. The nurse observes that Mr. Adams begins to have increased difficulty breathing. She elevates the head
of the bed to the high Fowler position, which decreases his respiratory distress. The nurse documents this
breathing as:
A. Tachypnea
B. Eupnea
C. Orthopnea
D. Hyperventilation
3. The physician orders a platelet count to be performed on Mrs. Smith after breakfast. The nurse is
responsible for:
A. Instructing the patient about this diagnostic test
B. Writing the order for this test
C. Giving the patient breakfast
D. All of the above
4. Mrs. Mitchell has been given a copy of her diet. The nurse discusses the foods allowed on a 500-mg low
sodium diet. These include:
A. A ham and Swiss cheese sandwich on whole wheat bread
B. Mashed potatoes and broiled chicken
C. A tossed salad with oil and vinegar and olives
D. Chicken bouillon
5. The physician orders a maintenance dose of 5,000 units of subcutaneous heparin (an anticoagulant)
daily. Nursing responsibilities for Mrs. Mitchell now include:
A. Reviewing daily activated partial thromboplastin time (APTT) and prothrombin time.
B. Reporting an APTT above 45 seconds to the physician

C. Assessing the patient for signs and symptoms of frank and occult bleeding
D. All of the above
6. The four main concepts common to nursing that appear in each of the current conceptual models are:
A. Person, nursing, environment, medicine
B. Person, health, nursing, support systems
C. Person, health, psychology, nursing
D. Person, environment, health, nursing
7. In Maslows hierarchy of physiologic needs, the human need of greatest priority is:
A. Love
B. Elimination
C. Nutrition
D. Oxygen
8. The family of an accident victim who has been declared brain-dead seems amenable to organ donation.
What should the nurse do?
A. Discourage them from making a decision until their grief has eased
B. Listen to their concerns and answer their questions honestly
C. Encourage them to sign the consent form right away
D. Tell them the body will not be available for a wake or funeral
9. A new head nurse on a unit is distressed about the poor staffing on the 11 p.m. to 7 a.m. shift. What
should she do?
A. Complain to her fellow nurses
B. Wait until she knows more about the unit
C. Discuss the problem with her supervisor
D. Inform the staff that they must volunteer to rotate
10. Which of the following principles of primary nursing has proven the most satisfying to the patient and
nurse?
A. Continuity of patient care promotes efficient, cost-effective nursing care
B. Autonomy and authority for planning are best delegated to a nurse who knows the patient well
C. Accountability is clearest when one nurse is responsible for the overall plan and its implementation.
D. The holistic approach provides for a therapeutic relationship, continuity, and efficient nursing care.
11. If nurse administers an injection to a patient who refuses that injection, she has committed:

A. Assault and battery


B. Negligence
C. Malpractice
D. None of the above
12. If patient asks the nurse her opinion about a particular physicians and the nurse replies that the
physician is incompetent, the nurse could be held liable for:
A. Slander
B. Libel
C. Assault
D. Respondent superior
13. A registered nurse reaches to answer the telephone on a busy pediatric unit, momentarily turning away
from a 3 month-old infant she has been weighing. The infant falls off the scale, suffering a skull fracture.
The nurse could be charged with:
A. Defamation
B. Assault
C. Battery
D. Malpractice
14. Which of the following is an example of nursing malpractice?
A. The nurse administers penicillin to a patient with a documented history of allergy to the drug. The patient
experiences an allergic reaction and has cerebral damage resulting from anoxia.
B. The nurse applies a hot water bottle or a heating pad to the abdomen of a patient with abdominal cramping.
C. The nurse assists a patient out of bed with the bed locked in position; the patient slips and fractures his right
humerus.
D. The nurse administers the wrong medication to a patient and the patient vomits. This information is documented
and reported to the physician and the nursing supervisor.
15. Which of the following signs and symptoms would the nurse expect to find when assessing an Asian
patient for postoperative pain following abdominal surgery?
A. Decreased blood pressure and heart rate and shallow respirations
B. Quiet crying
C. Immobility, diaphoresis, and avoidance of deep breathing or coughing
D. Changing position every 2 hours

16. A patient is admitted to the hospital with complaints of nausea, vomiting, diarrhea, and severe
abdominal pain. Which of the following would immediately alert the nurse that the patient has bleeding from
the GI tract?
A. Complete blood count
B. Guaiac test
C. Vital signs
C. Abdominal girth
17. The correct sequence for assessing the abdomen is:
A. Tympanic percussion, measurement of abdominal girth, and inspection
B. Assessment for distention, tenderness, and discoloration around the umbilicus.
C. Percussions, palpation, and auscultation
D. Auscultation, percussion, and palpation
18. High-pitched gurgles head over the right lower quadrant are:
A. A sign of increased bowel motility
B. A sign of decreased bowel motility
C. Normal bowel sounds
D. A sign of abdominal cramping
19. A patient about to undergo abdominal inspection is best placed in which of the following positions?
A. Prone
B. Trendelenburg
C. Supine
D. Side-lying
20. For a rectal examination, the patient can be directed to assume which of the following positions?
A. Genupectoral
B. Sims
C. Horizontal recumbent
D. All of the above
21. During a Romberg test, the nurse asks the patient to assume which position?
A. Sitting
B. Standing

C. Genupectoral
D. Trendelenburg
22. If a patients blood pressure is 150/96, his pulse pressure is:
A. 54
B. 96
C. 150
D. 246
23. A patient is kept off food and fluids for 10 hours before surgery. His oral temperature at 8 a.m. is 99.8 F
(37.7 C) This temperature reading probably indicates:
A. Infection
B. Hypothermia
C. Anxiety
D. Dehydration
24. Which of the following parameters should be checked when assessing respirations?
A. Rate
B. Rhythm
C. Symmetry
D. All of the above
25. A 38-year old patients vital signs at 8 a.m. are axillary temperature 99.6 F (37.6 C); pulse rate, 88;
respiratory rate, 30. Which findings should be reported?
A. Respiratory rate only
B. Temperature only
C. Pulse rate and temperature
D. Temperature and respiratory rate
26. All of the following can cause tachycardia except:
A. Fever
B. Exercise
C. Sympathetic nervous system stimulation
D. Parasympathetic nervous system stimulation
27. Palpating the midclavicular line is the correct technique for assessing

A. Baseline vital signs


B. Systolic blood pressure
C. Respiratory rate
D. Apical pulse
28. The absence of which pulse may not be a significant finding when a patient is admitted to the hospital?
A. Apical
B. Radial
C. Pedal
D. Femoral
29. Which of the following patients is at greatest risk for developing pressure ulcers?
A. An alert, chronic arthritic patient treated with steroids and aspirin
B. An 88-year old incontinent patient with gastric cancer who is confined to his bed at home
C. An apathetic 63-year old COPD patient receiving nasal oxygen via cannula
D. A confused 78-year old patient with congestive heart failure (CHF) who requires assistance to get out of bed.
30. The physician orders the administration of high-humidity oxygen by face mask and placement of the
patient in a high Fowlers position. After assessing Mrs. Paul, the nurse writes the following nursing
diagnosis: Impaired gas exchange related to increased secretions. Which of the following nursing
interventions has the greatest potential for improving this situation?
A. Encourage the patient to increase her fluid intake to 200 ml every 2 hours
B. Place a humidifier in the patients room.
C. Continue administering oxygen by high humidity face mask
D. Perform chest physiotherapy on a regular schedule
31. The most common deficiency seen in alcoholics is:
A. Thiamine
B. Riboflavin
C. Pyridoxine
D. Pantothenic acid
32. Which of the following statement is incorrect about a patient with dysphagia?
A. The patient will find pureed or soft foods, such as custards, easier to swallow than water
B. Fowlers or semi Fowlers position reduces the risk of aspiration during swallowing
C. The patient should always feed himself
D. The nurse should perform oral hygiene before assisting with feeding.

33. To assess the kidney function of a patient with an indwelling urinary (Foley) catheter, the nurse
measures his hourly urine output. She should notify the physician if the urine output is:
A. Less than 30 ml/hour
B. 64 ml in 2 hours
C. 90 ml in 3 hours
D. 125 ml in 4 hours
34. Certain substances increase the amount of urine produced. These include:
A. Caffeine-containing drinks, such as coffee and cola.
B. Beets
C. Urinary analgesics
D. Kaolin with pectin (Kaopectate)
35. A male patient who had surgery 2 days ago for head and neck cancer is about to make his first attempt
to ambulate outside his room. The nurse notes that he is steady on his feet and that his vision was
unaffected by the surgery. Which of the following nursing interventions would be appropriate?
A. Encourage the patient to walk in the hall alone
B. Discourage the patient from walking in the hall for a few more days
C. Accompany the patient for his walk.
D. Consult a physical therapist before allowing the patient to ambulate
36. A patient has exacerbation of chronic obstructive pulmonary disease (COPD) manifested by shortness
of breath; orthopnea: thick, tenacious secretions; and a dry hacking cough. An appropriate nursing
diagnosis would be:
A. Ineffective airway clearance related to thick, tenacious secretions.
B. Ineffective airway clearance related to dry, hacking cough.
C. Ineffective individual coping to COPD.
D. Pain related to immobilization of affected leg.
37. Mrs. Lim begins to cry as the nurse discusses hair loss. The best response would be:
A. Dont worry. Its only temporary
B. Why are you crying? I didnt get to the bad news yet
C. Your hair is really pretty
D. I know this will be difficult for you, but your hair will grow back after the completion of chemotheraphy
38. An additional Vitamin C is required during all of the following periods except:

A. Infancy
B. Young adulthood
C. Childhood
D. Pregnancy
39. A prescribed amount of oxygen s needed for a patient with COPD to prevent:
A. Cardiac arrest related to increased partial pressure of carbon dioxide in arterial blood (PaCO2)
B. Circulatory overload due to hypervolemia
C. Respiratory excitement
D. Inhibition of the respiratory hypoxic stimulus
40. After 1 week of hospitalization, Mr. Gray develops hypokalemia. Which of the following is the most
significant symptom of his disorder?
A. Lethargy
B. Increased pulse rate and blood pressure
C. Muscle weakness
D. Muscle irritability
41. Which of the following nursing interventions promotes patient safety?
A. Asses the patients ability to ambulate and transfer from a bed to a chair
B. Demonstrate the signal system to the patient
C. Check to see that the patient is wearing his identification band
D. All of the above
42. Studies have shown that about 40% of patients fall out of bed despite the use of side rails; this has led
to which of the following conclusions?
A. Side rails are ineffective
B. Side rails should not be used
C. Side rails are a deterrent that prevent a patient from falling out of bed.
D. Side rails are a reminder to a patient not to get out of bed
43. Examples of patients suffering from impaired awareness include all of the following except:
A. A semiconscious or over fatigued patient
B. A disoriented or confused patient
C. A patient who cannot care for himself at home
D. A patient demonstrating symptoms of drugs or alcohol withdrawal

44. The most common injury among elderly persons is:


A. Atheroscleotic changes in the blood vessels
B. Increased incidence of gallbladder disease
C. Urinary Tract Infection
D. Hip fracture
45. The most common psychogenic disorder among elderly person is:
A. Depression
B. Sleep disturbances (such as bizarre dreams)
C. Inability to concentrate
D. Decreased appetite
46. Which of the following vascular system changes results from aging?
A. Increased peripheral resistance of the blood vessels
B. Decreased blood flow
C. Increased workload of the left ventricle
D. All of the above
47. Which of the following is the most common cause of dementia among elderly persons?
A. Parkinsons disease
B. Multiple sclerosis
C. Amyotrophic lateral sclerosis (Lou Gehrigs disease)
D. Alzheimers disease
48. The nurses most important legal responsibility after a patients death in a hospital is:
A. Obtaining a consent of an autopsy
B. Notifying the coroner or medical examiner
C. Labeling the corpse appropriately
D. Ensuring that the attending physician issues the death certification
49. Before rigor mortis occurs, the nurse is responsible for:
A. Providing a complete bath and dressing change
B. Placing one pillow under the bodys head and shoulders
C. Removing the bodys clothing and wrapping the body in a shroud
D. Allowing the body to relax normally

50. When a patient in the terminal stages of lung cancer begins to exhibit loss of consciousness, a major
nursing priority is to:
A. Protect the patient from injury
B. Insert an airway
C. Elevate the head of the bed
D. Withdraw all pain medications

Answers and Rationale


The answers and rationale below will give you a better understanding of the exam. Counter-check your answers to
those below. If you have any disputes or objects, please direct them to the comments section.
1. Answer: B. Maintain the patient in an orthopneic position as needed
When a patient develops dyspnea and shortness of breath, the orthopneic position encourages maximum chest
expansion and keeps the abdominal organs from pressing against the diaphragm, thus improving ventilation. Bed
rest and oxygen by Venturi mask at 24% would improve oxygenation of the tissues and cells but must be ordered by
a physician. Allowing for rest periods decreases the possibility of hypoxia.
2. Answer: C. Orthopnea
Orthopnea is difficulty of breathing except in the upright position. Tachypnea is rapid respiration characterized by
quick, shallow breaths. Eupnea is normal respiration quiet, rhythmic, and without effort.
3. Answer: C. Giving the patient breakfast
A platelet count evaluates the number of platelets in the circulating blood volume. The nurse is responsible for giving
the patient breakfast at the scheduled time. The physician is responsible for instructing the patient about the test
and for writing the order for the test.
4. Answer: B. Mashed potatoes and broiled chicken
Mashed potatoes and broiled chicken are low in natural sodium chloride. Ham, olives, and chicken bouillon contain
large amounts of sodium and are contraindicated on a low sodium diet.
5. Answer: D. All of the above
All of the identified nursing responsibilities are pertinent when a patient is receiving heparin. The normal activated
partial thromboplastin time is 16 to 25 seconds and the normal prothrombin time is 12 to 15 seconds; these levels
must remain within two to two and one half the normal levels. All patients receiving anticoagulant therapy must be
observed for signs and symptoms of frank and occult bleeding (including hemorrhage, hypotension, tachycardia,

tachypnea, restlessness, pallor, cold and clammy skin, thirst and confusion); blood pressure should be measured
every 4 hours and the patient should be instructed to report promptly any bleeding that occurs with tooth brushing,
bowel movements, urination or heavy prolonged menstruation.
6. Answer: D. Person, environment, health, nursing
The focus concepts that have been accepted by all theorists as the focus of nursing practice from the time of
Florence Nightingale include the person receiving nursing care, his environment, his health on the health illness
continuum, and the nursing actions necessary to meet his needs.
7. Answer: D. Oxygen
Maslow, who defined a need as a satisfaction whose absence causes illness, considered oxygen to be the most
important physiologic need; without it, human life could not exist. According to this theory, other physiologic needs
(including food, water, elimination, shelter, rest and sleep, activity and temperature regulation) must be met before
proceeding to the next hierarchical levels on psychosocial needs.
8. Answer: B. Listen to their concerns and answer their questions honestly
The brain-dead patients family needs support and reassurance in making a decision about organ donation.
Because transplants are done within hours of death, decisions about organ donation must be made as soon as
possible. However, the familys concerns must be addressed before members are asked to sign a consent form. The
body of an organ donor is available for burial.
9. Answer: C. Discuss the problem with her supervisor
Although a new head nurse should initially spend time observing the unit for its strengths and weakness, she should
take action if a problem threatens patient safety. In this case, the supervisor is the resource person to approach.
10. Answer: D. The holistic approach provides for a therapeutic relationship, continuity, and efficient
nursing care.
Studies have shown that patients and nurses both respond well to primary nursing care units. Patients feel less
anxious and isolated and more secure because they are allowed to participate in planning their own care. Nurses
feel personal satisfaction, much of it related to positive feedback from the patients. They also seem to gain a greater
sense of achievement and esprit de corps.
11. Answer: A. Assault and battery
Assault is the unjustifiable attempt or threat to touch or injure another person. Battery is the unlawful touching of
another person or the carrying out of threatened physical harm. Thus, any act that a nurse performs on the patient
against his will is considered assault and battery.

12. Answer: A. Slander


Oral communication that injures an individuals reputation is considered slander. Written communication that does
the same is considered libel.
13. Answer: D. Malpractice
Malpractice is defined as injurious or unprofessional actions that harm another. It involves professional misconduct,
such as omission or commission of an act that a reasonable and prudent nurse would or would not do. In this
example, the standard of care was breached; a 3-month-old infant should never be left unattended on a scale.
14. Answer: A. The nurse administers penicillin to a patient with a documented history of allergy to the
drug. The patient experiences an allergic reaction and has cerebral damage resulting from anoxia.
The three elements necessary to establish a nursing malpractice are nursing error (administering penicillin to a
patient with a documented allergy to the drug), injury (cerebral damage), and proximal cause (administering the
penicillin caused the cerebral damage). Applying a hot water bottle or heating pad to a patient without a physicians
order does not include the three required components. Assisting a patient out of bed with the bed locked in position
is the correct nursing practice; therefore, the fracture was not the result of malpractice. Administering an incorrect
medication is a nursing error; however, if such action resulted in a serious illness or chronic problem, the nurse could
be sued for malpractice.
15. Answer: C. Immobility, diaphoresis, and avoidance of deep breathing or coughing
An Asian patient is likely to hide his pain. Consequently, the nurse must observe for objective signs. In an abdominal
surgery patient, these might include immobility, diaphoresis, and avoidance of deep breathing or coughing, as well
as increased heart rate, shallow respirations (stemming from pain upon moving the diaphragm and respiratory
muscles), and guarding or rigidity of the abdominal wall. Such a patient is unlikely to display emotion, such as
crying.
16. Answer: B. Guaiac test
To assess for GI tract bleeding when frank blood is absent, the nurse has two options: She can test for occult blood
in vomitus, if present, or in stool through guaiac (Hemoccult) test. A complete blood count does not provide
immediate results and does not always immediately reflect blood loss. Changes in vital signs may be cause by
factors other than blood loss. Abdominal girth is unrelated to blood loss.
17. Answer: D. Auscultation, percussion, and palpation
Because percussion and palpation can affect bowel motility and thus bowel sounds, they should follow auscultation
in abdominal assessment. Tympanic percussion, measurement of abdominal girth, and inspection are methods of

assessing the abdomen. Assessing for distention, tenderness and discoloration around the umbilicus can indicate
various bowel-related conditions, such as cholecystitis, appendicitis and peritonitis.
18. Answer: C. Normal bowel sounds
Hyperactive sounds indicate increased bowel motility; two or three sounds per minute indicate decreased bowel
motility. Abdominal cramping with hyperactive, high pitched tinkling bowel sounds can indicate a bowel obstruction.
19. Answer: C. Supine
The supine position (also called the dorsal position), in which the patient lies on his back with his face upward,
allows for easy access to the abdomen. In the prone position, the patient lies on his abdomen with his face turned to
the side. In the Trendelenburg position, the head of the bed is tilted downward to 30 to 40 degrees so that the upper
body is lower than the legs. In the lateral position, the patient lies on his side.
20. Answer: D. All of the above
All of these positions are appropriate for a rectal examination. In the genupectoral (knee-chest) position, the patient
kneels and rests his chest on the table, forming a 90 degree angle between the torso and upper legs. In Sims
position, the patient lies on his left side with the left arm behind the body and his right leg flexed. In the horizontal
recumbent position, the patient lies on his back with legs extended and hips rotated outward.
21. Answer: B. Standing
During a Romberg test, which evaluates for sensory or cerebellar ataxia, the patient must stand with feet together
and arms resting at the sidesfirst with eyes open, then with eyes closed. The need to move the feet apart to
maintain this stance is an abnormal finding.
22. Answer: A. 54
The pulse pressure is the difference between the systolic and diastolic blood pressure readings in this case, 54.
23. Answer: D. Dehydration
A slightly elevated temperature in the immediate preoperative or post operative period may result from the lack of
fluids before surgery rather than from infection. Anxiety will not cause an elevated temperature. Hypothermia is an
abnormally low body temperature.
24 Answer D. All of the above
The quality and efficiency of the respiratory process can be determined by appraising the rate, rhythm, depth, ease,
sound, and symmetry of respirations.

25. Answer: D. Temperature and respiratory rate


Under normal conditions, a healthy adult breathes in a smooth uninterrupted pattern 12 to 20 times a minute. Thus,
a respiratory rate of 30 would be abnormal. A normal adult body temperature, as measured on an oral thermometer,
ranges between 97 and 100F (36.1 and 37.8C); an axillary temperature is approximately one degree lower and a
rectal temperature, one degree higher. Thus, an axillary temperature of 99.6F (37.6C) would be considered
abnormal. The resting pulse rate in an adult ranges from 60 to 100 beats/minute, so a rate of 88 is normal.
26. Answer: D. Parasympathetic nervous system stimulation
Parasympathetic nervous system stimulation of the heart decreases the heart rate as well as the force of
contraction, rate of impulse conduction and blood flow through the coronary vessels. Fever, exercise, and
sympathetic stimulation all increase the heart rate.
27. Answer: D. Apical pulse
The apical pulse (the pulse at the apex of the heart) is located on the midclavicular line at the fourth, fifth, or sixth
intercostal space. Base line vital signs include pulse rate, temperature, respiratory rate, and blood pressure. Blood
pressure is typically assessed at the antecubital fossa, and respiratory rate is assessed best by observing chest
movement with each inspiration and expiration.
28. Answer: C. Pedal
Because the pedal pulse cannot be detected in 10% to 20% of the population, its absence is not necessarily a
significant finding. However, the presence or absence of the pedal pulse should be documented upon admission so
that changes can be identified during the hospital stay. Absence of the apical, radial, or femoral pulse is abnormal
and should be investigated.
29. Answer: B. An 88-year old incontinent patient with gastric cancer who is confined to his bed at home
Pressure ulcers are most likely to develop in patients with impaired mental status, mobility, activity level, nutrition,
circulation and bladder or bowel control. Age is also a factor. Thus, the 88-year old incontinent patient who has
impaired nutrition (from gastric cancer) and is confined to bed is at greater risk.
30. Answer: A. Encourage the patient to increase her fluid intake to 200 ml every 2 hours
Adequate hydration thins and loosens pulmonary secretions and also helps to replace fluids lost from elevated
temperature, diaphoresis, dehydration and dyspnea. High- humidity air and chest physiotherapy help liquefy and
mobilize secretions.
31. Answer: A. Thiamine

Chronic alcoholism commonly results in thiamine deficiency and other symptoms of malnutrition.
32. Answer: C. The patient should always feed himself
A patient with dysphagia (difficulty swallowing) requires assistance with feeding. Feeding himself is a long-range
expected outcome. Soft foods, Fowlers or semi-Fowlers position, and oral hygiene before eating should be part of
the feeding regimen.
33. Answer: A. Less than 30 ml/hour
A urine output of less than 30ml/hour indicates hypovolemia or oliguria, which is related to kidney function and
inadequate fluid intake.
34. Answer: A. Caffeine-containing drinks, such as coffee and cola.
Fluids containing caffeine have a diuretic effect. Beets and urinary analgesics, such as Pyridium (Phenazopyridine),
can color urine red. Kaopectate is an antidiarrheal medication.
35. Answer: C. Accompany the patient for his walk.
A hospitalized surgical patient leaving his room for the first time fears rejection and others staring at him, so he
should not walk alone. Accompanying him will offer moral support, enabling him to face the rest of the world.
Patients should begin ambulation as soon as possible after surgery to decrease complications and to regain
strength and confidence. Waiting to consult a physical therapist is unnecessary.
36. Answer: A. Ineffective airway clearance related to thick, tenacious secretions.
Thick, tenacious secretions, a dry, hacking cough, orthopnea, and shortness of breath are signs of ineffective airway
clearance. Ineffective airway clearance related to dry, hacking cough is incorrect because the cough is not the
reason for the ineffective airway clearance. Ineffective individual coping related to COPD is wrong because the
etiology for a nursing diagnosis should not be a medical diagnosis (COPD) and because no data indicate that the
patient is coping ineffectively. Pain related to immobilization of affected leg would be an appropriate nursing
diagnosis for a patient with a leg fracture.
37. Answer: D. I know this will be difficult for you, but your hair will grow back after the completion
of chemotherapy
I know this will be difficult acknowledges the problem and suggests a resolution to it. Dont worry.. offers some
relief but doesnt recognize the patients feelings. ..I didnt get to the bad news yet would be inappropriate at any
time. Your hair is really pretty offers no consolation or alternatives to the patient.
38. Answer: B. Young adulthood

Additional Vitamin C is needed in growth periods, such as infancy and childhood, and during pregnancy to supply
demands for fetal growth and maternal tissues. Other conditions requiring extra vitamin C include wound healing,
fever, infection and stress.
39. Answer: D. Inhibition of the respiratory hypoxic stimulus
Delivery of more than 2 liters of oxygen per minute to a patient with chronic obstructive pulmonary disease (COPD),
who is usually in a state of compensated respiratory acidosis (retaining carbon dioxide (CO2)), can inhibit the
hypoxic stimulus for respiration. An increased partial pressure of carbon dioxide in arterial blood (PACO2) would not
initially result in cardiac arrest. Circulatory overload and respiratory excitement have no relevance to the question.
40. Answer: C. Muscle weakness
Presenting symptoms of hypokalemia ( a serum potassium level below 3.5 mEq/liter) include muscle weakness,
chronic fatigue, and cardiac dysrhythmias. The combined effects of inadequate food intake and prolonged diarrhea
can deplete the potassium stores of a patient with GI problems.
41. Answer: D. All of the above
Assisting a patient with ambulation and transfer from a bed to a chair allows the nurse to evaluate the patients
ability to carry out these functions safely. Demonstrating the signal system and providing an opportunity for a return
demonstration ensures that the patient knows how to operate the equipment and encourages him to call for
assistance when needed. Checking the patients identification band verifies the patients identity and prevents
identification mistakes in drug administration.
42. Answer: D. Side rails are a reminder to a patient not to get out of bed
Since about 40% of patients fall out of bed despite the use of side rails, side rails cannot be said to prevent falls;
however, they do serve as a reminder that the patient should not get out of bed. The other answers are incorrect
interpretations of the statistical data.
43. Answer: C. A patient who cannot care for himself at home
A patient who cannot care for himself at home does not necessarily have impaired awareness; he may simply have
some degree of immobility.
44. Answer: D. Hip fracture
Hip fracture, the most common injury among elderly persons, usually results from osteoporosis. The other answers
are diseases that can occur in the elderly from physiologic changes.
45. Answer: A. Depression

Sleep disturbances, inability to concentrate and decreased appetite are symptoms of depression, the most common
psychogenic disorder among elderly persons. Other symptoms include diminished memory, apathy, disinterest in
appearance, withdrawal, and irritability. Depression typically begins before the onset of old age and usually is
caused by psychosocial, genetic, or biochemical factors
46. Answer: D. All of the above
Aging decreases elasticity of the blood vessels, which leads to increased peripheral resistance and decreased blood
flow. These changes, in turn, increase the workload of the left ventricle.
47. Answer: D. Alzheimers disease
Alzheimer;s disease, sometimes known as senile dementia of the Alzheimers type or primary degenerative
dementia, is an insidious; progressive, irreversible, and degenerative disease of the brain whose etiology is still
unknown. Parkinsons disease is a neurologic disorder caused by lesions in the extrapyramidal system and
manifested by tremors, muscle rigidity, hypokinesia, dysphagia, and dysphonia. Multiple sclerosis, a progressive,
degenerative disease involving demyelination of the nerve fibers, usually begins in young adulthood and is marked
by periods of remission and exacerbation. Amyotrophic lateral sclerosis, a disease marked by progressive
degeneration of the neurons, eventually results in atrophy of all the muscles; including those necessary for
respiration.
48. ANswer: C. Labeling the corpse appropriately
The nurse is legally responsible for labeling the corpse when death occurs in the hospital. She may be involved in
obtaining consent for an autopsy or notifying the coroner or medical examiner of a patients death; however, she is
not legally responsible for performing these functions. The attending physician may need information from the nurse
to complete the death certificate, but he is responsible for issuing it.
49. Answer: B. Placing one pillow under the bodys head and shoulders
The nurse must place a pillow under the decreased persons head and shoulders to prevent blood from settling in
the face and discoloring it. She is required to bathe only soiled areas of the body since the mortician will wash the
entire body. Before wrapping the body in a shroud, the nurse places a clean gown on the body and closes the eyes
and mouth.
50. Answer: A. Protect the patient from injury
Ensuring the patients safety is the most essential action at this time. The other nursing actions may be necessary
but are not a major priority.
1. Which element in the circular chain of infection can be eliminated by preserving skin integrity?

A. Host
B. Reservoir
C. Mode of transmission
D. Portal of entry
2. Which of the following will probably result in a break in sterile technique for respiratory isolation?
A. Opening the patients window to the outside environment
B. Turning on the patients room ventilator
C. Opening the door of the patients room leading into the hospital corridor
D. Failing to wear gloves when administering a bed bath
3. Which of the following patients is at greater risk for contracting an infection?
A. A patient with leukopenia
B. A patient receiving broad-spectrum antibiotics
C. A postoperative patient who has undergone orthopedic surgery
D. A newly diagnosed diabetic patient
4. Effective hand washing requires the use of:
A. Soap or detergent to promote emulsification
B. Hot water to destroy bacteria
C. A disinfectant to increase surface tension
D. All of the above
5. After routine patient contact, hand washing should last at least:
A. 30 seconds
B. 1 minute
C. 2 minute
D. 3 minutes
6. Which of the following procedures always requires surgical asepsis?
A. Vaginal instillation of conjugated estrogen
B. Urinary catheterization
C. Nasogastric tube insertion
D. Colostomy irrigation
7. Sterile technique is used whenever:

A. Strict isolation is required


B. Terminal disinfection is performed
C. Invasive procedures are performed
D. Protective isolation is necessary
8. Which of the following constitutes a break in sterile technique while preparing a sterile field for a
dressing change?
A. Using sterile forceps, rather than sterile gloves, to handle a sterile item
B. Touching the outside wrapper of sterilized material without sterile gloves
C. Placing a sterile object on the edge of the sterile field
D. Pouring out a small amount of solution (15 to 30 ml) before pouring the solution into a sterile container
9. A natural body defense that plays an active role in preventing infection is:
A. Yawning
B. Body hair
C. Hiccupping
D. Rapid eye movements
10. All of the following statement are true about donning sterile gloves except:
A. The first glove should be picked up by grasping the inside of the cuff.
B. The second glove should be picked up by inserting the gloved fingers under the cuff outside the glove.
C. The gloves should be adjusted by sliding the gloved fingers under the sterile cuff and pulling the glove over the
wrist
D. The inside of the glove is considered sterile
11. When removing a contaminated gown, the nurse should be careful that the first thing she touches is the:
A. Waist tie and neck tie at the back of the gown
B. Waist tie in front of the gown
C. Cuffs of the gown
D. Inside of the gown
12. Which of the following nursing interventions is considered the most effective form or universal
precautions?
A. Cap all used needles before removing them from their syringes
B. Discard all used uncapped needles and syringes in an impenetrable protective container
C. Wear gloves when administering IM injections
D. Follow enteric precautions

13. All of the following measures are recommended to prevent pressure ulcers except:
A. Massaging the reddened area with lotion
B. Using a water or air mattress
C. Adhering to a schedule for positioning and turning
D. Providing meticulous skin care
14. Which of the following blood tests should be performed before a blood transfusion?
A. Prothrombin and coagulation time
B. Blood typing and cross-matching
C. Bleeding and clotting time
D. Complete blood count (CBC) and electrolyte levels.
15. The primary purpose of a platelet count is to evaluate the:
A. Potential for clot formation
B. Potential for bleeding
C. Presence of an antigen-antibody response
D. Presence of cardiac enzymes
16. Which of the following white blood cell (WBC) counts clearly indicates leukocytosis?
A. 4,500/mm
B. 7,000/mm
C. 10,000/mm
D. 25,000/mm
17. After 5 days of diuretic therapy with 20mg of furosemide (Lasix) daily, a patient begins to exhibit fatigue,
muscle cramping and muscle weakness. These symptoms probably indicate that the patient is
experiencing:
A. Hypokalemia
B. Hyperkalemia
C. Anorexia
D. Dysphagia
18. Which of the following statements about chest X-ray is false?
A. No contradictions exist for this test
B. Before the procedure, the patient should remove all jewelry, metallic objects, and buttons above the waist

C. A signed consent is not required


D. Eating, drinking, and medications are allowed before this test
19. The most appropriate time for the nurse to obtain a sputum specimen for culture is:
A. Early in the morning
B. After the patient eats a light breakfast
C. After aerosol therapy
D. After chest physiotherapy
20. A patient with no known allergies is to receive penicillin every 6 hours. When administering the
medication, the nurse observes a fine rash on the patients skin. The most appropriate nursing action would
be to:
A. Withhold the moderation and notify the physician
B. Administer the medication and notify the physician
C. Administer the medication with an antihistamine
D. Apply corn starch soaks to the rash
21. All of the following nursing interventions are correct when using the Z-track method of drug injection
except:
A. Prepare the injection site with alcohol
B. Use a needle thats a least 1 long
C. Aspirate for blood before injection
D. Rub the site vigorously after the injection to promote absorption
22. The correct method for determining the vastus lateralis site for I.M. injection is to:
A. Locate the upper aspect of the upper outer quadrant of the buttock about 5 to 8 cm below the iliac crest
B. Palpate the lower edge of the acromion process and the midpoint lateral aspect of the arm
C. Palpate a 1 circular area anterior to the umbilicus
D. Divide the area between the greater femoral trochanter and the lateral femoral condyle into thirds, and select the
middle third on the anterior of the thigh
23. The mid-deltoid injection site is seldom used for I.M. injections because it:
A. Can accommodate only 1 ml or less of medication
B. Bruises too easily
C. Can be used only when the patient is lying down
D. Does not readily parenteral medication

24. The appropriate needle size for insulin injection is:


A. 18G, 1 long
B. 22G, 1 long
C. 22G, 1 long
D. 25G, 5/8 long
25. The appropriate needle gauge for intradermal injection is:
A. 20G
B. 22G
C. 25G
D. 26G
26. Parenteral penicillin can be administered as an:
A. IM injection or an IV solution
B. IV or an intradermal injection
C. Intradermal or subcutaneous injection
D. IM or a subcutaneous injection
27. The physician orders gr 10 of aspirin for a patient. The equivalent dose in milligrams is:
A. 0.6 mg
B. 10 mg
C. 60 mg
D. 600 mg
28. The physician orders an IV solution of dextrose 5% in water at 100ml/hour. What would the flow rate be if
the drop factor is 15 gtt = 1 ml?
A. 5 gtt/minute
B. 13 gtt/minute
C. 25 gtt/minute
D. 50 gtt/minute
29. Which of the following is a sign or symptom of a hemolytic reaction to blood transfusion?
A. Hemoglobinuria
B. Chest pain
C. Urticaria
D. Distended neck veins

30. Which of the following conditions may require fluid restriction?


A. Fever
B. Chronic Obstructive Pulmonary Disease
C. Renal Failure
D. Dehydration
31. All of the following are common signs and symptoms of phlebitis except:
A. Pain or discomfort at the IV insertion site
B. Edema and warmth at the IV insertion site
C. A red streak exiting the IV insertion site
D. Frank bleeding at the insertion site
32. The best way of determining whether a patient has learned to instill ear medication properly is for the
nurse to:
A. Ask the patient if he/she has used ear drops before
B. Have the patient repeat the nurses instructions using her own words
C. Demonstrate the procedure to the patient and encourage to ask questions
D. Ask the patient to demonstrate the procedure
33. Which of the following types of medications can be administered via gastrostomy tube?
A. Any oral medications
B. Capsules whole contents are dissolve in water
C. Enteric-coated tablets that are thoroughly dissolved in water
D. Most tablets designed for oral use, except for extended-duration compounds
34. A patient who develops hives after receiving an antibiotic is exhibiting drug:
A. Tolerance
B. Idiosyncrasy
C. Synergism
D. Allergy
35. A patient has returned to his room after femoral arteriography. All of the following are appropriate
nursing interventions except:
A. Assess femoral, popliteal, and pedal pulses every 15 minutes for 2 hours
B. Check the pressure dressing for sanguineous drainage

C. Assess a vital signs every 15 minutes for 2 hours


D. Order a hemoglobin and hematocrit count 1 hour after the arteriography
36. The nurse explains to a patient that a cough:
A. Is a protective response to clear the respiratory tract of irritants
B. Is primarily a voluntary action
C. Is induced by the administration of an antitussive drug
D. Can be inhibited by splinting the abdomen
37. An infected patient has chills and begins shivering. The best nursing intervention is to:
A. Apply iced alcohol sponges
B. Provide increased cool liquids
C. Provide additional bedclothes
D. Provide increased ventilation
38. A clinical nurse specialist is a nurse who has:
A. Been certified by the National League for Nursing
B. Received credentials from the Philippine Nurses Association
C. Graduated from an associate degree program and is a registered professional nurse
D. Completed a masters degree in the prescribed clinical area and is a registered professional nurse.
39. The purpose of increasing urine acidity through dietary means is to:
A. Decrease burning sensations
B. Change the urines color
C. Change the urines concentration
D. Inhibit the growth of microorganisms
40. Clay colored stools indicate:
A. Upper GI bleeding
B. Impending constipation
C. An effect of medication
D. Bile obstruction
41. In which step of the nursing process would the nurse ask a patient if the medication she administered
relieved his pain?

A. Assessment
B. Analysis
C. Planning
D. Evaluation
42. All of the following are good sources of vitamin A except:
A. White potatoes
B. Carrots
C. Apricots
D. Egg yolks
43. Which of the following is a primary nursing intervention necessary for all patients with a Foley Catheter
in place?
A. Maintain the drainage tubing and collection bag level with the patients bladder
B. Irrigate the patient with 1% Neosporin solution three times a daily
C. Clamp the catheter for 1 hour every 4 hours to maintain the bladders elasticity
D. Maintain the drainage tubing and collection bag below bladder level to facilitate drainage by gravity
44. The ELISA test is used to:
A. Screen blood donors for antibodies to human immunodeficiency virus (HIV)
B. Test blood to be used for transfusion for HIV antibodies
C. Aid in diagnosing a patient with AIDS
D. All of the above
45. The two blood vessels most commonly used for TPN infusion are the:
A. Subclavian and jugular veins
B. Brachial and subclavian veins
C. Femoral and subclavian veins
D. Brachial and femoral veins
46. Effective skin disinfection before a surgical procedure includes which of the following methods?
A. Shaving the site on the day before surgery
B. Applying a topical antiseptic to the skin on the evening before surgery
C. Having the patient take a tub bath on the morning of surgery
D. Having the patient shower with an antiseptic soap on the evening before and the morning of surgery

47. When transferring a patient from a bed to a chair, the nurse should use which muscles to avoid back
injury?
A. Abdominal muscles
B. Back muscles
C. Leg muscles
D. Upper arm muscles
48. Thrombophlebitis typically develops in patients with which of the following conditions?
A. Increases partial thromboplastin time
B. Acute pulsus paradoxus
C. An impaired or traumatized blood vessel wall
D. Chronic Obstructive Pulmonary Disease (COPD)
49. In a recumbent, immobilized patient, lung ventilation can become altered, leading to such respiratory
complications as:
A. Respiratory acidosis, atelectasis, and hypostatic pneumonia
B. Apneustic breathing, atypical pneumonia and respiratory alkalosis
C. Cheyne-Stokes respirations and spontaneous pneumothorax
D. Kussmauls respirations and hypoventilation
50. Immobility impairs bladder elimination, resulting in such disorders as
A. Increased urine acidity and relaxation of the perineal muscles, causing incontinence
B. Urine retention, bladder distention, and infection
C. Diuresis, natriuresis, and decreased urine specific gravity
D. Decreased calcium and phosphate levels in the urine

Answers and Rationale


Gauge your performance by counter checking your answers to the answers below. Learn more about the question
by reading the rationale. If you have any disputes or questions, please direct them to the comments section.
1. Answer: D. Portal of entry
In the circular chain of infection, pathogens must be able to leave their reservoir and be transmitted to a susceptible
host through a portal of entry, such as broken skin.
2. Answer: C. Opening the door of the patients room leading into the hospital corridor

Respiratory isolation, like strict isolation, requires that the door to the door patients room remain closed. However,
the patients room should be well ventilated, so opening the window or turning on the ventricular is desirable. The
nurse does not need to wear gloves for respiratory isolation, but good hand washing is important for all types of
isolation.
3. Answer: A. A patient with leukopenia
Leukopenia is a decreased number of leukocytes (white blood cells), which are important in resisting infection. None
of the other situations would put the patient at risk for contracting an infection; taking broad-spectrum antibiotics
might actually reduce the infection risk.
4. Answer: A. Soap or detergent to promote emulsification
Soaps and detergents are used to help remove bacteria because of their ability to lower the surface tension of water
and act as emulsifying agents. Hot water may lead to skin irritation or burns.
5. Answer: A. 30 seconds
Depending on the degree of exposure to pathogens, hand washing may last from 10 seconds to 4 minutes. After
routine patient contact, hand washing for 30 seconds effectively minimizes the risk of pathogen transmission.
6. Answer: B. Urinary catheterization
The urinary system is normally free of microorganisms except at the urinary meatus. Any procedure that involves
entering this system must use surgically aseptic measures to maintain a bacteria-free state.
7. Answer: C. Invasive procedures are performed
All invasive procedures, including surgery, catheter insertion, and administration of parenteral therapy, require sterile
technique to maintain a sterile environment. All equipment must be sterile, and the nurse and the physician must
wear sterile gloves and maintain surgical asepsis. In the operating room, the nurse and physician are required to
wear sterile gowns, gloves, masks, hair covers, and shoe covers for all invasive procedures. Strict isolation requires
the use of clean gloves, masks, gowns and equipment to prevent the transmission of highly communicable diseases
by contact or by airborne routes. Terminal disinfection is the disinfection of all contaminated supplies and equipment
after a patient has been discharged to prepare them for reuse by another patient. The purpose of protective
(reverse)isolation is to prevent a person with seriously impaired resistance from coming into contact who potentially
pathogenic organisms.
8. Answer: C. Placing a sterile object on the edge of the sterile field
The edges of a sterile field are considered contaminated. When sterile items are allowed to come in contact with the
edges of the field, the sterile items also become contaminated.

9. Answer: B. Body hair


Hair on or within body areas, such as the nose, traps and holds particles that contain microorganisms. Yawning and
hiccupping do not prevent microorganisms from entering or leaving the body. Rapid eye movement marks the stage
of sleep during which dreaming occurs.
10. Answer: D. The inside of the glove is considered sterile
The inside of the glove is always considered to be clean, but not sterile.
11. Answer: A. Waist tie and neck tie at the back of the gown
The back of the gown is considered clean, the front is contaminated. So, after removing gloves and washing hands,
the nurse should untie the back of the gown; slowly move backward away from the gown, holding the inside of the
gown and keeping the edges off the floor; turn and fold the gown inside out; discard it in a contaminated linen
container; then wash her hands again.
12. Answer: B. Discard all used uncapped needles and syringes in an impenetrable protective container
According to the Centers for Disease Control (CDC), blood-to-blood contact occurs most commonly when a health
care worker attempts to cap a used needle. Therefore, used needles should never be recapped; instead they should
be inserted in a specially designed puncture resistant, labeled container. Wearing gloves is not always necessary
when administering an I.M. injection. Enteric precautions prevent the transfer of pathogens via feces.
13. Answer: A. Massaging the reddened area with lotion
Nurses and other health care professionals previously believed that massaging a reddened area with lotion would
promote venous return and reduce edema to the area. However, research has shown that massage only increases
the likelihood of cellular ischemia and necrosis to the area.
14. Answer: B. Blood typing and cross-matching
Before a blood transfusion is performed, the blood of the donor and recipient must be checked for compatibility. This
is done by blood typing (a test that determines a persons blood type) and cross-matching (a procedure that
determines the compatibility of the donors and recipients blood after the blood types has been matched). If the
blood specimens are incompatible, hemolysis and antigen-antibody reactions will occur.
15. Answer: A. Potential for clot formation
Platelets are disk-shaped cells that are essential for blood coagulation. A platelet count determines the number of
thrombocytes in blood available for promoting hemostasis and assisting with blood coagulation after injury. It also is
used to evaluate the patients potential for bleeding; however, this is not its primary purpose. The normal count

ranges from 150,000 to 350,000/mm3. A count of 100,000/mm3 or less indicates a potential for bleeding; count of
less than 20,000/mm3 is associated with spontaneous bleeding.
16. Answer: D. 25,000/mm
Leukocytosis is any transient increase in the number of white blood cells (leukocytes) in the blood. Normal WBC
counts range from 5,000 to 100,000/mm3. Thus, a count of 25,000/mm3 indicates leukocytosis.
17. Answer: A. Hypokalemia
Fatigue, muscle cramping, and muscle weaknesses are symptoms of hypokalemia (an inadequate potassium level),
which is a potential side effect of diuretic therapy. The physician usually orders supplemental potassium to prevent
hypokalemia in patients receiving diuretics. Anorexia is another symptom of hypokalemia. Dysphagia means
difficulty swallowing.
18. Answer: A. No contradictions exist for this test
Pregnancy or suspected pregnancy is the only contraindication for a chest X-ray. However, if a chest X-ray is
necessary, the patient can wear a lead apron to protect the pelvic region from radiation. Jewelry, metallic objects,
and buttons would interfere with the X-ray and thus should not be worn above the waist. A signed consent is not
required because a chest X-ray is not an invasive examination. Eating, drinking and medications are allowed
because the X-ray is of the chest, not the abdominal region.
19. Answer: A. Early in the morning
Obtaining a sputum specimen early in this morning ensures an adequate supply of bacteria for culturing and
decreases the risk of contamination from food or medication.
20. Answer: A. Withhold the moderation and notify the physician
Initial sensitivity to penicillin is commonly manifested by a skin rash, even in individuals who have not been allergic
to it previously. Because of the danger of anaphylactic shock, he nurse should withhold the drug and notify the
physician, who may choose to substitute another drug. Administering an antihistamine is a dependent nursing
intervention that requires a written physicians order. Although applying corn starch to the rash may relieve
discomfort, it is not the nurses top priority in such a potentially life-threatening situation.
21. Answer: D. Rub the site vigorously after the injection to promote absorption
The Z-track method is an I.M. injection technique in which the patients skin is pulled in such a way that the needle
track is sealed off after the injection. This procedure seals medication deep into the muscle, thereby minimizing skin
staining and irritation. Rubbing the injection site is contraindicated because it may cause the medication to
extravasate into the skin.

22. Answer: D. Divide the area between the greater femoral trochanter and the lateral femoral condyle into
thirds, and select the middle third on the anterior of the thigh
The vastus lateralis, a long, thick muscle that extends the full length of the thigh, is viewed by many clinicians as the
site of choice for I.M. injections because it has relatively few major nerves and blood vessels. The middle third of the
muscle is recommended as the injection site. The patient can be in a supine or sitting position for an injection into
this site.
23. Answer: A. Can accommodate only 1 ml or less of medication
The mid-deltoid injection site can accommodate only 1 ml or less of medication because of its size and location (on
the deltoid muscle of the arm, close to the brachial artery and radial nerve).
24. Answer: D. 25G, 5/8 long
A 25G, 5/8 needle is the recommended size for insulin injection because insulin is administered by the
subcutaneous route. An 18G, 1 needle is usually used for I.M. injections in children, typically in the vastus
lateralis. A 22G, 1 needle is usually used for adult I.M. injections, which are typically administered in the vastus
lateralis or ventrogluteal site.
25. Answer: C. 25G
Because an intradermal injection does not penetrate deeply into the skin, a small-bore 25G needle is recommended.
This type of injection is used primarily to administer antigens to evaluate reactions for allergy or sensitivity studies. A
20G needle is usually used for I.M. injections of oil-based medications; a 22G needle for I.M. injections; and a 25G
needle, for I.M. injections; and a 25G needle, for subcutaneous insulin injections.
26. Answer: A. IM injection or an IV solution
Parenteral penicillin can be administered I.M. or added to a solution and given I.V. It cannot be administered
subcutaneously or intradermally.
27. Answer: D. 600 mg
gr 10 x 60 mg/gr 1 = 600 mg
28. Answer: C. 25 gtt/minute
100ml/60 min X 15 gtt/ 1 ml = 25 gtt/minute
29. Answer: A. Hemoglobinuria

Hemoglobinuria, the abnormal presence of hemoglobin in the urine, indicates a hemolytic reaction (incompatibility of
the donors and recipients blood). In this reaction, antibodies in the recipients plasma combine rapidly with donor
RBCs; the cells are hemolyzed in either circulatory or reticuloendothelial system. Hemolysis occurs more rapidly in
ABO incompatibilities than in Rh incompatibilities. Chest pain and urticaria may be symptoms of impending
anaphylaxis. Distended neck veins are an indication of hypervolemia.
30. Answer: C. Renal Failure
In real failure, the kidney loses their ability to effectively eliminate wastes and fluids. Because of this, limiting the
patients intake of oral and I.V. fluids may be necessary. Fever, chronic obstructive pulmonary disease, and
dehydration are conditions for which fluids should be encouraged.
31. Answer: D. Frank bleeding at the insertion site
Phlebitis, the inflammation of a vein, can be caused by chemical irritants (I.V. solutions or medications), mechanical
irritants (the needle or catheter used during venipuncture or cannulation), or a localized allergic reaction to the
needle or catheter. Signs and symptoms of phlebitis include pain or discomfort, edema and heat at the I.V. insertion
site, and a red streak going up the arm or leg from the I.V. insertion site.
32. Answer: D. Ask the patient to demonstrate the procedure
Return demonstration provides the most certain evidence for evaluating the effectiveness of patient teaching.
33. Answer: D. Most tablets designed for oral use, except for extended-duration compounds
Capsules, enteric-coated tablets, and most extended duration or sustained release products should not be dissolved
for use in a gastrostomy tube. They are pharmaceutically manufactured in these forms for valid reasons, and altering
them destroys their purpose. The nurse should seek an alternate physicians order when an ordered medication is
inappropriate for delivery by tube.
34. Answer: D. Allergy
A drug-allergy is an adverse reaction resulting from an immunologic response following a previous sensitizing
exposure to the drug. The reaction can range from a rash or hives to anaphylactic shock. Tolerance to a drug means
that the patient experiences a decreasing physiologic response to repeated administration of the drug in the same
dosage. Idiosyncrasy is an individuals unique hypersensitivity to a drug, food, or other substance; it appears to be
genetically determined. Synergism, is a drug interaction in which the sum of the drugs combined effects is greater
than that of their separate effects.
35. Answer: D. Order a hemoglobin and hematocrit count 1 hour after the arteriography

A hemoglobin and hematocrit count would be ordered by the physician if bleeding were suspected. The other
answers are appropriate nursing interventions for a patient who has undergone femoral arteriography.
36. Answer: A. Is a protective response to clear the respiratory tract of irritants
Coughing, a protective response that clears the respiratory tract of irritants, usually is involuntary; however it can be
voluntary, as when a patient is taught to perform coughing exercises. An antitussive drug inhibits coughing. Splinting
the abdomen supports the abdominal muscles when a patient coughs.
37. Answer: C. Provide additional bedclothes
In an infected patient, shivering results from the bodys attempt to increase heat production and the production of
neutrophils and phagocytic action through increased skeletal muscle tension and contractions. Initial
vasoconstriction may cause skin to feel cold to the touch. Applying additional bed clothes helps to equalize the body
temperature and stop the chills. Attempts to cool the body result in further shivering, increased metabolism, and thus
increased heat production.
38. Answer: D. Completed a masters degree in the prescribed clinical area and is a registered professional
nurse.
A clinical nurse specialist must have completed a masters degree in a clinical specialty and be a registered
professional nurse. The National League of Nursing accredits educational programs in nursing and provides a
testing service to evaluate student nursing competence but it does not certify nurses. The American Nurses
Association identifies requirements for certification and offers examinations for certification in many areas of
nursing., such as medical surgical nursing. These certification (credentialing) demonstrates that the nurse has the
knowledge and the ability to provide high quality nursing care in the area of her certification. A graduate of an
associate degree program is not a clinical nurse specialist: however, she is prepared to provide bed side nursing
with a high degree of knowledge and skill. She must successfully complete the licensing examination to become a
registered professional nurse.
39. Answer: D. Inhibit the growth of microorganisms
Microorganisms usually do not grow in an acidic environment.
40. Answer: D. Bile obstruction
Bile colors the stool brown. Any inflammation or obstruction that impairs bile flow will affect the stool pigment,
yielding light, clay-colored stool. Upper GI bleeding results in black or tarry stool. Constipation is characterized by
small, hard masses. Many medications and foods will discolor stool for example, drugs containing iron turn stool
black.; beets turn stool red.
41. Answer: D. Evaluation

In the evaluation step of the nursing process, the nurse must decide whether the patient has achieved the expected
outcome that was identified in the planning phase.
42. Answer: A. White potatoes
The main sources of vitamin A are yellow and green vegetables (such as carrots, sweet potatoes, squash, spinach,
collard greens, broccoli, and cabbage) and yellow fruits (such as apricots, and cantaloupe). Animal sources include
liver, kidneys, cream, butter, and egg yolks.
43. Answer: D. Maintain the drainage tubing and collection bag below bladder level to facilitate drainage by
gravity
Maintaining the drainage tubing and collection bag level with the patients bladder could result in reflux of urine into
the kidney. Irrigating the bladder with Neosporin and clamping the catheter for 1 hour every 4 hours must be
prescribed by a physician.
44. Answer: D. All of the above
The ELISA test of venous blood is used to assess blood and potential blood donors to human immunodeficiency
virus (HIV). A positive ELISA test combined with various signs and symptoms helps to diagnose acquired
immunodeficiency syndrome (AIDS)
45. Answer: A. Subclavian and jugular veins
Total Parenteral Nutrition (TPN) requires the use of a large vessel, such as the subclavian or jugular vein, to ensure
rapid dilution of the solution and thereby prevent complications, such as hyperglycemia. The brachial and femoral
veins usually are contraindicated because they pose an increased risk of thrombophlebitis.
46. Answer: D. Having the patient shower with an antiseptic soap on the evening before and the morning of
surgery
Studies have shown that showering with an antiseptic soap before surgery is the most effective method of removing
microorganisms from the skin. Shaving the site of the intended surgery might cause breaks in the skin, thereby
increasing the risk of infection; however, if indicated, shaving, should be done immediately before surgery, not the
day before. A topical antiseptic would not remove microorganisms and would be beneficial only after proper cleaning
and rinsing. Tub bathing might transfer organisms to another body site rather than rinse them away.
47. Answer: C. Leg muscles
The leg muscles are the strongest muscles in the body and should bear the greatest stress when lifting. Muscles of
the abdomen, back, and upper arms may be easily injured.

48. Answer: C. An impaired or traumatized blood vessel wall


The factors, known as Virchows triad, collectively predispose a patient to thrombophlebitis; impaired venous return
to the heart, blood hypercoagulability, and injury to a blood vessel wall. Increased partial thromboplastin time
indicates a prolonged bleeding time during fibrin clot formation, commonly the result of anticoagulant (heparin)
therapy. Arterial blood disorders (such as pulsus paradoxus) and lung diseases (such as COPD) do not necessarily
impede venous return of injure vessel walls.
49. Answer: A. Respiratory acidosis, atelectasis, and hypostatic pneumonia
Because of restricted respiratory movement, a recumbent, immobilize patient is at particular risk for respiratory
acidosis from poor gas exchange; atelectasis from reduced surfactant and accumulated mucus in the bronchioles,
and hypostatic pneumonia from bacterial growth caused by stasis of mucus secretions.
50. Answer: B. Urine retention, bladder distention, and infection
The immobilized patient commonly suffers from urine retention caused by decreased muscle tone in the perineum.
This leads to bladder distention and urine stagnation, which provide an excellent medium for bacterial growth
leading to infection. Immobility also results in more alkaline urine with excessive amounts of calcium, sodium and
phosphate, a gradual decrease in urine production, and an increased specific gravity.
1. The four major concepts in nursing theory are the
A. Person, Environment, Nurse, Health
B. Nurse, Person, Environment, Cure
C. Promotive, Preventive, Curative, Rehabilitative
D. Person, Environment, Nursing, Health
2. The act of utilizing the environment of the patient to assist him in his recovery is theorized by
A. Nightingale
B. Benner
C. Swanson
D. King
3. For her, Nursing is a theoretical system of knowledge that prescribes a process of analysis and action
related to care of the ill person
A. King
B. Henderson
C. Roy
D. Leininger

4. According to her, Nursing is a helping or assisting profession to persons who are wholly or partly
dependent or when those who are supposedly caring for them are no longer able to give care.
A. Henderson
B. Orem
C. Swanson
D. Neuman
5. Nursing is a unique profession, Concerned with all the variables affecting an individuals response to
stressors, which are intra, inter and extra personal in nature.
A. Neuman
B. Johnson
C. Watson
D. Parse
6. The unique function of the nurse is to assist the individual, sick or well, in the performance of those
activities contributing to health that he would perform unaided if he has the necessary strength, will and
knowledge, and do this in such a way as to help him gain independence as rapidly as possible.
A. Henderson
B. Abdellah
C. Levin
D. Peplau
7. Caring is the essence and central unifying, a dominant domain that distinguishes nursing from other
health disciplines. Care is an essential human need.
A. Benner
B. Watson
C. Leininger
D. Swanson
8. Caring involves 5 processes: KNOWING, BEING WITH, DOING FOR, ENABLING and MAINTAINING
BELIEF.
A. Benner
B. Watson
C. Leininger
D. Swanson

9. Caring is healing, it is communicated through the consciousness of the nurse to the individual being
cared for. It allows access to higher human spirit.
A. Benner
B. Watson
C. Leininger
D. Swanson
10. Caring means that person, events, projects and things matter to people. It reveals stress and coping
options. Caring creates responsibility. It is an inherent feature of nursing practice. It helps the nurse assist
clients to recover in the face of the illness.
A. Benner
B. Watson
C. Leininger
D. Swanson
11. Which of the following is NOT TRUE about profession according to Marie Jahoda?
A. A profession is an organization of an occupational group based on the application of special knowledge
B. It serves specific interest of a group
C. It is altruistic
D. Quality of work is of greater importance than the rewards
12. Which of the following is NOT an attribute of a professional?
A. Concerned with quantity
B. Self directed
C. Committed to spirit of inquiry
D. Independent
13. The most unique characteristic of nursing as a profession is
A. Education
B. Theory
C. Caring
D. Autonomy
14. This is the distinctive individual qualities that differentiate a person to another
A. Philosophy
B. Personality

C. Charm
D. Character
15. Refers to the moral values and beliefs that are used as guides to personal behavior and actions
A. Philosophy
B. Personality
C. Charm
D. Character
16. As a nurse manager, which of the following best describes this function?
A. Initiate modification on clients lifestyle
B. Protect clients right
C. Coordinates the activities of other members of the health team in managing patient care
D. Provide in service education programs, Use accurate nursing audit, formulate philosophy and vision of the
institution
17. What best describes nurses as a care provider?
A. Determine clients need
B. Provide direct nursing care
C. Help client recognize and cope with stressful psychological situation
D. Works in combined effort with all those involved in patients care
18. The nurse questions a doctors order of Morphine sulfate 50 mg, IM for a client with pancreatitis. Which
role best fit that statement?
A. Change agent
B. Client advocate
C. Case manager
D. Collaborator
19. These are nursing intervention that requires knowledge, skills and expertise of multiple health
professionals.
A. Dependent
B. Independent
C. Interdependent
D. Intradependent
20. What type of patient care model is the most common for student nurses and private duty nurses?

A. Total patient care


B. Team nursing
C. Primary Nursing
D. Case management
21. This is the best patient care model when there are many nurses but few patients.
A. Functional nursing
B. Team nursing
C. Primary nursing
D. Total patient care
22. This patient care model works best when there are plenty of patient but few nurses
A. Functional nursing
B. Team nursing
C. Primary nursing
D. Total patient care
23. RN assumes 24 hour responsibility for the client to maintain continuity of care across shifts, days or
visits.
A. Functional nursing
B. Team nursing
C. Primary nursing
D. Total patient care
24. Who developed the first theory of nursing?
A. Hammurabi
B. Alexander
C. Fabiola
D. Nightingale
25. She introduces the NATURE OF NURSING MODEL.
A. Henderson
B. Nightingale
C. Parse
D. Orlando
26. She described the four conservation principle.

A. Levin
B. Leininger
C. Orlando
D. Parse
27. Proposed the HEALTH CARE SYSTEM MODEL.
A. Henderson
B. Orem
C. Parse
D. Neuman
28. Conceptualized the BEHAVIORAL SYSTEM MODEL
A. Orem
B. Johnson
C. Henderson
D. Parse
29. Developed the CLINICAL NURSING A HELPING ART MODEL
A. Swanson
B. Hall
C. Weidenbach
D. Zderad
30. Developed the ROLE MODELING and MODELING theory
A. Erickson,Tomlin,Swain
B. Neuman
C. Newman
D. Benner and Wrubel
31. Proposed the GRAND THEORY OF NURSING AS CARING
A. Erickson, Tomlin, Swain
B. Peterson,Zderad
C. Bnner,Wrubel
D. Boykin,Schoenhofer
32. Postulated the INTERPERSONAL ASPECT OF NURSING

A. Travelbee
B. Swanson
C. Zderad
D. Peplau
33. He proposed the theory of morality that is based on MUTUAL TRUST
A. Freud
B. Erikson
C. Kohlberg
D. Peters
34. He proposed the theory of morality based on PRINCIPLES
A. Freud
B. Erikson
C. Kohlberg
D. Peters
35. Freud postulated that child adopts parental standards and traits through
A. Imitation
B. Introjection
C. Identification
D. Regression
36. According to them, Morality is measured of how people treat human being and that a moral child strives
to be kind and just
A. Zderad and Peterson
B. Benner and Wrubel
C. Fowler and Westerhoff
D. Schulman and Mekler
37. Postulated that FAITH is the way of behaving. He developed four theories of faith and development
based on his experience.
A. Giligan
B. Westerhoff
C. Fowler
D. Freud

38. He described the development of faith. He suggested that faith is a spiritual dimension that gives
meaning to a persons life. Faith according to him, is a relational phenomenon.
A. Giligan
B. Westerhoff
C. Fowler
D. Freud
39. Established in 1906 by the Baptist foreign mission society of America. Miss rose nicolet, was its first
superintendent.
A. St. Paul Hospital School of nursing
B. Iloilo Mission Hospital School of nursing
C. Philippine General Hospital School of nursing
D. St. Lukes Hospital School of nursing
40. Anastacia Giron-Tupas was the first Filipino nurse to occupy the position of chief nurse in this hospital.
A. St. Paul Hospital
B. Iloilo Mission Hospital
C. Philippine General Hospital
D. St. Lukes Hospital
41. She was the daughter of Hungarian kings, who feed 300-900 people everyday in their gate, builds
hospitals, and care of the poor and sick herself.
A. Elizabeth
B. Catherine
C. Nightingale
D. Sairey Gamp
42. She dies of yellow fever in her search for truth to prove that yellow fever is carried by a mosquitoes.
A. Clara louise Maas
B. Pearl Tucker
C. Isabel Hampton Robb
D. Caroline Hampton Robb
43. He was called the father of sanitation.
A. Abraham
B. Hippocrates

C. Moses
D. Willam Halstead
44. The country where SHUSHURUTU originated
A. China
B. Egypt
C. India
D. Babylonia
45. They put girls clothes on male infants to drive evil forces away
A. Chinese
B. Egyptian
C. Indian
D. Babylonian
46. In what period of nursing does people believe in TREPHINING to drive evil forces away?
A. Dark period
B. Intuitive period
C. Contemporary period
D. Educative period
47. This period ended when Pastor Fliedner, build Kaiserwerth institute for the training of Deaconesses
A. Apprentice period
B. Dark period
C. Contemporary period
D. Educative period
48. Period of nursing where religious Christian orders emerged to take care of the sick
A. Apprentice period
B. Dark period
C. Contemporary period
D. Educative period
49. Founded the second order of St. Francis of Assisi
A. St. Catherine
B. St. Anne

C. St. Clare
D. St. Elizabeth
50. This period marked the religious upheaval of Luther, Who questions the Christian faith.
A. Apprentice period
B. Dark period
C. Contemporary period
D. Educative period
51. According to the Biopsychosocial and spiritual theory of Sister Callista Roy, Man, As a SOCIAL being is
A. Like all other men
B. Like some other men
C. Like no other men
D. Like men
52. She conceptualized that man, as an Open system is in constant interaction and transaction with a
changing environment.
A. Roy
B. Levin
C. Neuman
D. Newman
53. In a CLOSED system, which of the following is true?
A. Affected by matter
B. A sole island in vast ocean
C. Allows input
D. Constantly affected by matter, energy, information
54. Who postulated the WHOLISTIC concept that the totality is greater than sum of its parts?
A. Roy
B. Rogers
C. Henderson
D. Johnson
55. She theorized that man is composed of sub and supra systems. Subsystems are cells, tissues, organs
and systems while the suprasystems are family, society and community.

A. Roy
B. Rogers
C. Henderson
D. Johnson
56. Which of the following is not true about the human needs?
A. Certain needs are common to all people
B. Needs should be followed exactly in accordance with their hierarchy
C. Needs are stimulated by internal factors
D. Needs are stimulated by external factors
57. Which of the following is TRUE about the human needs?
A. May not be deferred
B. Are not interrelated
C. Met in exact and rigid way
D. Priorities are alterable
58. According to Maslow, which of the following is NOT TRUE about a self actualized person?
A. Understands poetry, music, philosophy, science etc.
B. Desires privacy, autonomous
C. Follows the decision of the majority, uphold justice and truth
D. Problem centered
59. According to Maslow, which of the following is TRUE about a self actualized person?
A. Makes decision contrary to public opinion
B. Do not predict events
C. Self centered
D. Maximum degree of self conflict
60. This is the essence of mental health
A. Self awareness
B. Self actualization
C. Self esteem
D. Self worth
61. Florence Nightingale was born in

A. Germany
B. Britain
C. France
D. Italy
62. Which is unlikely of Florence Nightingale?
A. Born May 12, 1840
B. Built St. Thomas school of nursing when she was 40 years old
C. Notes in nursing
D. Notes in hospital
63. What country did Florence Nightingale train in nursing?
A. Belgium
B. US
C. Germany
D. England
64. Which of the following is recognized for developing the concept of HIGH LEVEL WELLNESS?
A. Erikson
B. Madaw
C. Peplau
D. Dunn
65. One of the expectations is for nurses to join professional association primarily because of
A. Promotes advancement and professional growth among its members
B. Works for raising funds for nurses benefit
C. Facilitate and establishes acquaintances
D. Assist them and securing jobs abroad
66. Founder of the PNA
A. Julita Sotejo
B. Anastacia Giron Tupas
C. Eufemia Octaviano
D. Anesia Dionisio
67. Which of the following provides that nurses must be a member of a national nurse organization?

A. R.A 877
B. 1981 Code of ethics approved by the house of delegates and the PNA
C. Board resolution No. 1955 Promulgated by the BON
D. RA 7164
68. Which of the following best describes the action of a nurse who documents her nursing diagnosis?
A. She documents it and charts it whenever necessary
B. She can be accused of malpractice
C. She does it regularly as an important responsibility
D. She charts it only when the patient is acutely ill
69. Which of the following does not govern nursing practice?
A. RA 7164
B. RA 9173
C. BON Res. Code Of Ethics
D. BON Res. Scope of Nursing Practice
70. A nurse who is maintaining a private clinic in the community renders service on maternal and child
health among the neighborhood for a fee is:
A. Primary care nurse
B. Independent nurse practitioner
C. Nurse-Midwife
D. Nurse specialist
71. When was the PNA founded?
A. September 22, 1922
B. September 02, 1920
C. October 21, 1922
D. September 02, 1922
72. Who was the first president of the PNA ?
A. Anastacia Giron-Tupas
B. Loreto Tupas
C. Rosario Montenegro
D. Ricarda Mendoza

73. Defines health as the ability to maintain internal milieu. Illness according to him/her/them is the failure to
maintain internal environment.
A. Cannon
B. Bernard
C. Leddy and Pepper
D. Roy
74. Postulated that health is a state and process of being and becoming an integrated and whole person.
A. Cannon
B. Bernard
C. Dunn
D. Roy
75. What regulates HOMEOSTASIS according to the theory of Walter Cannon?
A. Positive feedback
B. Negative feedback
C. Buffer system
D. Various mechanisms
76. Stated that health is WELLNESS. A termed define by the culture or an individual.
A. Roy
B. Henderson
C. Rogers
D. King
77. Defined health as a dynamic state in the life cycle, and Illness as interference in the life cycle.
A. Roy
B. Henderson
C. Rogers
D. King
78. She defined health as the soundness and wholness of developed human structure and bodily mental
functioning.
A. Orem
B. Henderson

C. Neuman
D. Clark
79. According to her, Wellness is a condition in which all parts and subparts of an individual are in harmony
with the whole system.
A. Orem
B. Henderson
C. Neuman
D. Johnson
80. Postulated that health is reflected by the organization, interaction, interdependence and integration of
the subsystem of the behavioral system.
A. Orem
B. Henderson
C. Neuman
D. Johnson
81. According to them, Well being is a subjective perception of BALANCE, HARMONY and VITALITY
A. Leavell and Clark
B. Peterson and Zderad
C. Benner and Wruber
D. Leddy and Pepper
82. He describes the WELLNESS-ILLNESS Continuum as interaction of the environment with well being and
illness.
A. Cannon
B. Bernard
C. Dunn
D. Clark
83. An integrated method of functioning that is oriented towards maximizing ones potential within the
limitation of the environment.
A. Well being
B. Health
C. Low level Wellness
D. High level Wellness

84. What kind of illness precursor, according to DUNN is cigarette smoking?


A. Heredity
B. Social
C. Behavioral
D. Environmental
85. According to DUNN, Overcrowding is what type of illness precursor?
A. Heredity
B. Social
C. Behavioral
D. Environmental
86. Health belief model was formulated in 1975 by who?
A. Becker
B. Smith
C. Dunn
D. Leavell and Clark
87. In health belief model, Individual perception matters. Which of the following is highly UNLIKELY to
influence preventive behavior?
A. Perceived susceptibility to an illness
B. Perceived seriousness of an illness
C. Perceived threat of an illness
D. Perceived curability of an illness
88. Which of the following is not a PERCEIVED BARRIER in preventive action?
A. Difficulty adhering to the lifestyle
B. Economic factors
C. Accessibility of health care facilities
D. Increase adherence to medical therapies
89. Conceptualizes that health is a condition of actualization or realization of persons potential. Avers that
the highest aspiration of people is fulfillment and complete development actualization.
A. Clinical Model
B. Role performance Model

C. Adaptive Model
D. Eudaemonistic Model
90. Views people as physiologic system and Absence of sign and symptoms equates health.
A. Clinical Model
B. Role performance Model
C. Adaptive Model
D. Eudaemonistic Model
91. Knowledge about the disease and prior contact with it is what type of VARIABLE according to the health
belief model?
A. Demographic
B. Sociopsychologic
C. Structural
D. Cues to action
92. It includes internal and external factors that leads the individual to seek help
A. Demographic
B. Sociopsychological
C. Structural
D. Cues to action
93. Influence from peers and social pressure is included in what variable of HBM?
A. Demographic
B. Sociopsychological
C. Structural
D. Cues to action
94. Age, Sex, Race etc. is included in what variable of HBM?
A. Demographic
B. Sociopsychological
C. Structural
D. Cues to action
95. According to Leavell and Clarks ecologic model, All of this are factors that affects health and illness
except

A. Reservoir
B. Agent
C. Environment
D. Host
96. Is a multi dimensional model developed by PENDER that describes the nature of persons as they
interact within the environment to pursue health
A. Ecologic Model
B. Health Belief Model
C. Health Promotion Model
D. Health Prevention Model
97. Defined by Pender as all activities directed toward increasing the level of well being and self
actualization.
A. Health prevention
B. Health promotion
C. Health teaching
D. Self actualization
98. Defined as an alteration in normal function resulting in reduction of capacities and shortening of life
span.
A. Illness
B. Disease
C. Health
D. Wellness
99. Personal state in which a person feels unhealthy
A. Illness
B. Disease
C. Health
D. Wellness
100. According to her, Caring is defined as a nurturant way of responding to a valued client towards whom
the nurse feels a sense of commitment and responsibility.
A. Benner
B. Watson

C. Leininger
D. Swanson

Answers & Rationale


Gauge your performance by counter-checking your answers to those below. If you have disputes or further
questions, please direct them to the comments section.
1. Answer: D. Person, Environment, Nursing, Health
Theorist always describes The nursing profession by first defining what is NURSING, followed by the PERSON,
ENVIRONMENT and HEALTH CONCEPT. The most popular theory was perhaps Nightingales. She defined nursing
as the utilization of the persons environment to assist him towards recovery. She defined the person as somebody
who has a reparative capabilities mediated and enhanced by factors in his environment. She describes the
environment as something that would facilitate the persons reparative process and identified different factors like
sanitation, noise, etc. that affects a persons reparative state.
2. Answer: A. Nightingale
Florence Nightingale do not believe in the Germ Theory, and perhaps this was her biggest mistake. Yet, her theory
was the first in nursing. She believed that manipulation of environment that includes appropriate noise, nutrition,
hygiene, light, comfort, sanitation etc. could provide the clients body the nurturance it needs for repair and recovery.
3. Answer: C. Roy
Remember the word theoROYtical for Callista Roy. Nursing is a theoretical body of knowledge that prescribes
analysis and action to care for an ill person. Roy introduced the Adaptation Model and viewed a person as a
biopsychosocial being. She believed that by adaptation, a person can maintain homeostasis.
4. Answer: B. Orem
In self care deficit theory, Nursing is defined as a helping or assistive profession to person who are wholly or partly
dependent or when people who are to give care to them are no longer available. Self care are the activities that a
person do for himself to maintain health, life and well being.
5. Answer: A. Neuman
Neuman divided stressors as either intra, inter and extra personal in nature. She said that NURSING is concerned
with eliminating these stressors to obtain a maximum level of wellness. The nurse helps the client through
PRIMARY, SECONDARY AND TERTIARY prevention modes.
6. Answer: A. Henderson

Remember this definition and associate it with Virginia Henderson. Henderson also describes the NATURE OF
NURSING theory. She identified 14 basic needs of the client. She describes nursing roles as:

Substitutive: Doing everything for the client;

Supplementary: Helping the client; and

Complementary: Working with the client.

Breathing normally, eliminating waste, eating and drinking adequately, worship and play are some of the basic needs
according to her.
7. Answer: C. Leininger
There are many theorists that describes nursing as CARE. The most popular was JEAN WATSONS Human Caring
Model. But this question pertains to Leiningers definition of caring. CUD I LIE IN GER? [ Could I Lie In There ] Is the
Mnemonics I am using not to get confused. C stands for CENTRAL , U stands for UNIFYING, D stands for
DOMINANT DOMAIN.
8. Answer: D. Swanson
Caring according to Swanson involves 5 processes. Knowing means understanding the client. Being with
emphasizes the Physical presence of the nurse for the patient. Doing for means doing things for the patient when he
is incapable of doing it for himself. Enabling means helping client transcend maturational and developmental
stressors in life while Maintaining belief is the ability of the Nurse to inculcate meaning to these events.
9. Answer: B. Watson
The deepest and spiritual definition of caring came from Jean Watson. For her, caring expands the limits of
openness and allows access to higher human spirit.
10. Answer: A. Benner
11. Answer: B. It serves specific interest of a group
A profession should serve the whole community and not just a specific interest of a group. All other choices are
correct.
12. Answer: A. Concerned with quantity
A professional is concerned with quality and not quantity. In nursing, We have methods of quality assurance and
control to evaluate the effectiveness of nursing care. Nurses, are never concerned with quantity of care provided.
13. Answer: C. Caring

Caring and caring alone, is the most unique quality of the Nursing Profession. It is the one the delineate nursing
from other professions.
14. Answer: B. Personality
Personality are qualities that make us different from each other. These are impressions that we made, or the
footprints that we leave behind. This is the result of the integration of ones talents, behavior, appearance, mood,
character, morals and impulses into one harmonious whole. Philosophy is the basic truth that fuel our soul and give
our life a purpose, it shapes the facets of a persons character. Charm is to attract other people to be a change
agent. Character is our moral values and belief that guides our actions in life.
15. Answer: D. Character
Character is our moral values and belief that guides our actions in life.
16. Answer: D. Provide in service education programs. Use accurate nursing audit, formulate philosophy
and vision of the institution
A refers to being a change agent. B is a role of a patient advocate. C is a case manager while D basically
summarized functions of a nurse manager. If you havent read Lydia Venzons Book : NURSING MANAGEMENT
TOWARDS QUALITY CARE, I suggest reading it in advance for your management subjects in the graduate school.
Formulating philosophy and vision is in PLANNING. Nursing Audit is in CONTROLLING, In service education
programs are included in DIRECTING. These are the processes of Nursing Management, I just forgot to add
ORGANIZING which includes formulating an organizational structure and plans, Staffing and developing
qualifications and job descriptions.
17. Answer: A. Determine clients need
You can never provide nursing care if you dont know what are the needs of the client. How can you provide an
effective postural drainage if you do not know where is the bulk of the clients secretion. Therefore, the best
description of a care provider is the accurate and prompt determination of the clients need to be able to render an
appropriate nursing care.
18. Answer: B. Client advocate
As a clients advocate, nurses are to protect the clients rights and promotes what is best for the them. Knowing
that morphine causes spasm of the Sphincter of Oddi and will lead to further increase in the clients pain, the nurse
should know that the the best treatment option for the client was not provided and intervene to provide the best
possible care.
19. Answer: C. Interdependent

Interdependent functions are those that needs expertise and skills of multiple health professionals.
20. Answer: A. Total patient care
This is also known as case nursing. It is a method of nursing care wherein, one nurse is assigned to one patient for
the delivery of total care. These are the method use by nursing students. Private duty nurses and those in critical or
isolation units.
21. Answer: D. Total patient care
Total patient care works best if there are many nurses but few patients.
22. Answer: A. Functional nursing
Functional nursing is task oriented. One nurse is assigned on a particular task leading to task expertise and
efficiency. The nurse will work fast because the procedures are repetitive leading to task mastery. This care is not
recommended as this leads fragmented nursing care.
23. Answer: C. Primary nursing
Your keyword in Primary Nursing is the 24 hours. This does not necessarily mean the nurse is awake for 24 hours.
The nurse can have secondary nurses that can take care of the the patient during shifts where the primary nurse is
not around.
24. Answer: D. Nightingale
Hammurabi is a king of Babylon that introduced the Lex Taliones law or better be described as an eye for an eye
and a tooth for a tooth. Alexander the Great was the son of King Philip II and is from Macedonia but he ruled and
conquered Greece, Persia and Egypt. He is known to use a hammer to crush a dying soldiers medulla giving a
speedy death. Fabiola was a beautiful Roman matron who converted her house into a hospital.
25. Answer: A. Henderson
Refer to question # 6.
26. Answer: A. Levin
Myra Levin described the 4 Conservation principles which are concerned with the Unity and Integrity of an individual.

Energy: Output to facilitate meeting our needs.

Structural Integrity: Maintaining the integrity of our organs, tissues and systems to prevent harmful agents
from entering ones body.

Personal Integrity: These refers to ones self-esteem, self worth, self concept, identity and personality.

Social Integrity: Reflects ones societal roles to ones society, community, family, friends and fellow
individuals.

27. Answer: D. Neuman


Betty Neuman asserted that nursing is a unique profession and is concerned with all the variables affecting the
individuals response to stressors. These are INTRA or within ourselves, EXTRA or outside the individual, INTER
means between two or more people.
She proposed the Health Care System Model which states that by the three levels of prevention primary,
secondary, tertiary the nurse can help the client maintain stability against these stressors.
28. Answer: B. Johnson
According to Dorothy Johnson, each person is a behavioral system that is composed of seven (7) subsystems. Man
adjusts or adapts to stressors by a using a learned pattern of response. Man uses his behavior to meet the demands
of the environment, and is able to modify his behavior to support these demands.
29. Answer: C. Weidenbach
The Helping Art of Clinical Nursing was developed by Ernestine Wiedenbach. It defines nursing as the practice of
identifying a patients need for help through the observation of presenting behavior and symptoms, exploration of the
meaning of those symptoms, determination of the cause of discomfort, the determination of the patients ability to
resolve the patients discomfort, or determining if the patient has a need of help from the nurse or another health
care professional.
30. Answer: A. Erickson, Tomlin, Swain
Modeling and Role Modeling theory was developed by Helen C. Erickson, Evelyn M. Tomlin, and Mary Ann P.
Swain. It enables nurses to care for and nurture each client with an awareness of and respect for the individuals
uniqueness which exemplifies theory-based clinical practice that focuses on the clients needs.
31. Answer: D. Boykin,Schoenhofer
This theory was called GRAND THEORY because Boykin and Schoenhofer thinks that all men are caring and that
nursing is a response to this unique call. According to them, caring is a moral imperative meaning all people will
tend to help a man even if he is not trained to do so.
32. Answer: A. Travelbee

Travelbees theory was referred to as Interpersonal Theory because she postulated that nursing is to assist the
individual and all people that affects this individual to cope with illness, recover and find meaning to this experience.
For her, nursing is a human to human relationship that is formed during illness.
33. Answer: C. Kohlberg
Kohlber states that relationships are based on mutual trust. He postulated the levels of morality development. At the
first stage called the premoral or preconventional. A child do things and label them as bad or good depending on the
punishment or reward they get.
34. Answer: D. Peters
Peters believes that morality has 3 components:

Emotions or how one feels;

Judgement or how one reasons;

Behavior or how one actuates his emotions and judgement.

35. Answer: C. Identification


A child, according to Freud adopts parental standards, traits, habits and norms through identication. A good example
is the corned beef commercial WALK LIKE A MAN, TALK LIKE A MAN Where the child identifies with his father
by wearing the same clothes and doing the same thing.
36. Answer: D. Schulman and Mekler
According to Schulman and Mekler, there are 2 components that makes an action MORAL : The intention should be
good and the Act must be just. A good example is ROBIN HOOD, His intention is GOOD but the act is UNJUST,
which makes his action IMMORAL.
37. Answer: B. Westerhoff
There are only 2 theorist of FAITH that might be asked in the board examinations. Fowler and Westerhoff. What
differs them is that, FAITH of fowler is defined abstractly, Fowler defines faith as a FORCE that gives a meaning to a
persons life while Westerhoff defines faith as a behavior that continuously develops through time.
38. Answer: C. Fowler
Refer to # 37
39. Answer: B. Iloilo Mission Hospital School of Nursing

This is the first School of Nursing in the Philippines which started in 1906 and produced the three graduate nurses in
1909. Now, the School of Nursing was transferred at Central Philippine University.
40. Answer: C. Philippine General Hospital
In 1917, Tupas was named the first Filipino chief nurse and superintendent of the Philippine General Hospital School
of Nursing. She also headed the committee that prepared the bill systematizing Philippine nursing education passed
in 1919. In 1959, through the initiative of the Civic Assembly of Women of the Philippines, she received the
Presidential medal of merit.
41. Answer: A. Elizabeth
Saint Elizabeth of Hungary was a daughter of a King and is the patron saint of nurses. She build hospitals and feed
hungry people everyday using the kingdoms money. She is a princess, but devoted her life in feeding the hungry
and serving the sick.
42. Answer: A. Clara Louise Maass
Clara Louise Maass sacrificed her life in research of YELLOW FEVER. People during her time do not believe that
yellow fever was brought by mosquitoes. To prove that they are wrong, She allowed herself to be bitten by the vector
and after days, She died.
43. Answer: C. Moses
More than 1000 years before Christ, Moses was recognized as the Father of Sanitation. He wrote rules for
sanitation. He stated that all people preparing and serving public food must be neat and clean. Moses also required
that serving dishes and cooking utensils be washed between customers and public restaurants.
44. Answer: C. India
45. Answer: A. Chinese
Chinese believes that male newborns are demon magnets. To fool those demons, they put female clothes to their
male newborn.
46. Answer: B. Intuitive period
Egyptians believe that a sick person is someone with an evil force or demon that is inside their heads. To release
these evil spirits, They would tend to drill holes on the patients skull and it is called TREPHINING.
47. Answer: A. Apprentice period

What delineates apprentice period among others is that, it ENDED when formal schools were established. During
the apprentice period, There is no formal educational institution for nurses. Most of them receive training inside the
convent or church. Some of them are trained just for the purpose of nursing the wounded soldiers. But almost all of
them are influenced by the christian faith to serve and nurse the sick. When Fliedner build the first formal school for
nurses, It marked the end of the APPRENTICESHIP period.
48. Answer: A. Apprentice period
Apprentice period is marked by the emergence of religious orders the are devoted to religious life and the practice of
nursing.
49. Answer: C. St. Clare
The poor St. Clare is the second order of St. Francis of Assisi. The first order was founded by St. Francis himself. St.
Catherine of Sienna was the first lady with the lamp. St. Anne is the mother of Mary. St. Elizabeth is the patron saint
of Nursing.
50. Answer: B. Dark period
Protestantism emerged with Martin Luther questions the Pope and Christianity. This started the Dark period of
nursing when the christian faith was smeared by controversies. These leads to closure of some hospital and schools
run by the church. Nursing became the work of prostitutes, slaves, mother and least desirable of women.
51. Answer: B. Like some other men
According to ROY, Man as a social being is like some other man. As a spiritual being and Biologic being, Man are all
alike. As a psychologic being, No man thinks alike. This basically summarized her BIOPSYHOSOCIAL theory which
is included in our licensure exam coverage.
52. Answer: A. Roy
OPEN system theory is ROY. As an open system, man continuously allows input from the environment. Example is
when you tell me Im good looking, I will be happy the entire day, Because I am an open system and continuously
interact and transact with my environment. A close system is best exemplified by a CANDLE. When you cover the
candle with a glass, it will die because it will eventually use all the oxygen it needs inside the glass for combustion. A
closed system do not allow inputs and output in its environment.
53. Answer: B. A sole island in vast ocean
54. Answer: B. Rogers

The holistic theory by Martha Rogers states that MAN is greater than the sum of all its parts and that his dignity and
worth will not be lessen even if one of this part is missing. A good example is ANNE BOLEYN, The mother of Queen
Elizabeth and the wife of King Henry VIII. She was beheaded because Henry wants to marry another wife and that
his divorce was not approved by the pope. Outraged, He insisted on the separation of the Church and State and
divorce Anne himself by making everyone believe that Anne is having an affair to another man. Anne was beheaded
while her lips is still saying a prayer. Even without her head, People still gave respect to her diseased body and a
separate head. She was still remembered as Anne Boleyn, Mother of Elizabeth who lead England to their GOLDEN
AGE.
55. Answer: B. Rogers
According to Martha Rogers, Man is composed of 2 systems : SUB which includes cells, tissues, organs and system
and SUPRA which includes our family, community and society. She stated that when any of these systems are
affected, it will affect the entire individual.
56. Answer: B. Needs should be followed exactly in accordance with their hierarchy
Needs can be deferred. I can urinate later as not to miss the part of the movies climax. I can save my money that
are supposedly for my lunch to watch my idols in concert. The physiologic needs can be meet later for some other
needs and need not be strictly followed according to their hierarchy.
57. Answer: D. Priorities are alterable
58. Answer: C. Follows the decision of the majority, uphold justice and truth
A,B and D are all qualities of a self actualized person. A self actualized person do not follow the decision of majority
but is self directed and can make decisions contrary to a popular opinion.
59. Answer: A. Makes decision contrary to public opinion
60. Answer: B. Self actualization
The peak of Maslows hierarchy is the essence of mental health.
61. Answer: D. Italy
Florence Nightingale was born in Florence, Italy, May 12, 1820. Studied in Germany and Practiced in England.
62. Answer: A. Born May 12, 1840
63. Answer: C. Germany
64. Answer: D. Dunn

According to Dunn, High level wellness is the ability of an individual to maximize his full potential with the limitations
imposed by his environment. According to him, An individual can be healthy or ill in both favorable and unfavorable
environment.
65. Answer: A. Promotes advancement and professional growth among its members
66. Answer: B. Anastacia Giron Tupas
Anastasia Giron Tupas is the founder of PNA (formerly Filipino Nurses Association) and the Dean of Philippine
Nursing. Founded on September 2, 1922 as Filipino Nurses Association (FNA) in a meeting of 150 nurses, the FNA
was incorporated in 1924.
67. Answer: C. Board resolution No. 1955 Promulgated by the BON
This is an old board resolution. The new Board resolution is No. 220 series of 2004 also known as the Nursing Code
Of Ethics which states that A nurse should be a member of an accredited professional organization which is the
PNA.
68. Answer: C. She does it regularly as an important responsibility
69. Answer: A. RA 7164
7164 is an old law. This is the 1991 Nursing Law which was repealed by the newer RA 9173.
70. Answer: B. Independent nurse practitioner
71. Answer: D. September 02, 1922
According to the official PNA website, they are founded September 02, 1922.
72. Answer: C. Rosario Montenegro
Anastacia Giron Tupas founded the FNA, the former name of the PNA but the first President was Rosario
Montenegro.
73. Answer: B. Bernard
According to Bernard, Health is the ability to maintain and Internal Milieu and Illness is the failure to maintain the
internal environment.
74. Answer: D. Roy
According to ROY, Health is a state and process of becoming a WHOLE AND INTEGRATED Person.

75. Answer: B. Negative feedback


The theory of Health as the ability to maintain homeostasis was postulated by Walter Cannon. According to him,
There are certain FEEDBACK Mechanism that regulates our Homeostasis. A good example is that when we overuse
our arm, it will produce pain. PAIN is a negative feedback that signals us that our arm needs a rest.
76. Answer: C. Rogers
Martha Rogers states that HEALTH is synonymous with WELLNESS and that HEALTH and WELLNESS is
subjective depending on the definition of ones culture.
77. Answer: D. King
Emogene King states that health is a state in the life cycle and Illness is any interference on this cycle. I enjoyed the
Movie LION KING and like what Mufasa said that they are all part of the CIRCLE OF LIFE, or the Life cycle.
78. Answer: A. Orem
Orem defined health as the SOUNDNESS and WHOLENESS of developed human structure and of bodily and
mental functioning.
79. Answer: C. Neuman
Neuman believe that man is composed of sub parts and when this sub parts are in harmony with the whole system,
Wellness results. Please do not confuse this with the SUB and SUPRA systems of Martha rogers.
80. Answer: D. Johnson
Once you see the phrase BEHAVIORAL SYSTEM, answer Dorothy Johnson.
81. Answer: D. Leddy and Pepper
According to Leedy and Pepper, Wellness is subjective and depends on an individuals perception of balance,
harmony and vitality. Leavell and Clark postulated the ecologic model of health and illness or the AGENT-HOSTENVIRONMENT model. Paterson and Zderad developed the HUMANISTIC NURSING PRACTICE theory while
Benner and Wrubel postulate the PRIMACY OF CARING MODEL.
82. Answer: C. Dunn
83. Answer: D. High level Wellness
84. Answer: C. Behavioral

Behavioral precursors includes smoking, alcoholism, high fat intake and other lifestyle choices. Environmental
factors involved poor sanitation and over crowding. Heredity includes congenital and diseases acquired through the
genes. There are no social precursors according to DUNN.
85. Answer: D. Environmental
86. Answer: A. Becker
According to Becker, The belief of an individual greatly affects his behavior. If a man believes that he is susceptible
to an illness, He will alter his behavior in order to prevent its occurrence. For example, If a man thinks that diabetes
is acquired through high intake of sugar and simple carbohydrates, then he will limit the intake of foods rich in these
components.
87. Answer: D. Perceived curability of an illness
If a man think he is susceptible to a certain disease, thinks that the disease is serious and it is a threat to his life and
functions, he will use preventive behaviors to avoid the occurrence of this threat.
88. Answer: A. Difficulty adhering to the lifestyle or B. Economic factors
Perceived barriers are those factors that affects the individuals health preventive actions. Both A and B can affect
the individuals ability to prevent the occurrence of diseases. C and D are called Preventive Health Behaviors which
enhances the individuals preventive capabilities.
89. Answer: D. Eudaemonistic Model
Smith formulated 5 models of health. Clinical model simply states that when people experience sign and symptoms,
they would think that they are unhealthy therefore, Health is the absence of clinical sign and symptoms of a disease.
Role performance model states that when a person does his role and activities without deficits, he is healthy and the
inability to perform usual roles means that the person is ill. Adaptive Model states that if a person adapts well with
his environment, he is healthy and maladaptation equates illness. Eudaemonistic Model of health according to smith
is the actualization of a persons fullest potential. If a person functions optimally and develop self actualization, then,
no doubt that person is healthy.
90. Answer: A. Clinical Model
Refer to question # 89.
91. Answer: C. Structural
Modifying variables in Beckers health belief model includes DEMOGRAPHIC : Age, sex, race etc.
SOCIOPSYCHOLOGIC : Social and Peer influence. STRUCTURAL : Knowledge about the disease and prior

contact with it and CUES TO ACTION : Which are the sign and symptoms of the disease or advice from friends,
mass media and others that forces or makes the individual seek help.
92. Answer: D. Cues to action
93. Answer: B. Sociopsychologic
94. Answer: A. Demographic
95. Answer: A. Reservoir
According to L&Cs Ecologic model, there are 3 factors that affect health and illness. These are the AGENT or the
factor the leads to illness, either a bacteria or an event in life. HOST are persons that may or may not be affected by
these agents. ENVIRONMENT are factors external to the host that may or may not predispose him to the AGENT.
96. Answer: C. Health Promotion Model
Pender developed the concept of HEALTH PROMOTION MODEL which postulated that an individual engages in
health promotion activities to increase well being and attain self actualization. These includes exercise,
immunization, healthy lifestyle, good food, self responsibility and all other factors that minimize if not totally eradicate
risks and threats of health.
97. Answer: B. Health promotion
98. Answer: B. Disease
Diseases are alteration in body functions resulting in reduction of capabilities or shortening of lifespan.
99. Answer: A. Illness
Illness is something personal in perspective. Unlike disease, illnesses are in which a person feels a state of being
unhealthy. An old person may think he is ill but in fact, he is not, due to diminishing functions and capabilities of his
body.
100. Answer: B. Watson
This is Jean Watsons definition of Nursing as caring.
1. When the General adaptation syndrome is activated, FLIGHT OR FIGHT response sets in. Sympathetic
nervous system releases norepinephrine while the adrenal medulla secretes epinephrine. Which of the
following is true with regards to that statement?

A. Pupils will constrict


B. Client will be lethargic
C. Lungs will bronchodilate
D. Gastric motility will increase
2. Which of the following response is not expected to a person whose GAS is activated and the FIGHT OR
FLIGHT response sets in?
A. The client will not urinate due to relaxation of the detrusor muscle
B. The client will be restless and alert
C. Clients BP will increase, there will be vasodilation
D. There will be increase glycogenolysis, Pancrease will decrease insulin secretion
3. State in which a persons physical, emotional, intellectual and social development or spiritual functioning
is diminished or impaired compared with a previous experience.
A. Illness
B. Disease
C. Health
D. Wellness
4. This is the first stage of illness wherein, the person starts to believe that something is wrong. Also known
as the transition phase from wellness to illness.
A. Symptom Experience
B. Assumption of sick role
C. Medical care contact
D. Dependent patient role
5. In this stage of illness, the person accepts or rejects a professionals suggestion. The person also
becomes passive and may regress to an earlier stage.
A. Symptom Experience
B. Assumption of sick role
C. Medical care contact
D. Dependent patient role
6. In this stage of illness, the person learns to accept the illness.
A. Symptom Experience
B. Assumption of sick role

C. Medical care contact


D. Dependent patient role
7. In this stage, the person tries to find answers for his illness. He wants his illness to be validated, his
symptoms explained and the outcome reassured or predicted
A. Symptom Experience
B. Assumption of sick role
C. Medical care contact
D. Dependent patient role
8. The following are true with regards to aspect of the sick role except
A. One should be held responsible for his condition
B. One is excused from his societal role
C. One is obliged to get well as soon as possible
D. One is obliged to seek competent help
9. Refers to conditions that increases vulnerability of individual or group to illness or accident
A. Predisposing factor
B. Etiology
C. Risk factor
D. Modifiable Risks
10. Refers to the degree of resistance the potential host has against a certain pathogen
A. Susceptibility
B. Immunity
C. Virulence
D. Etiology
11. A group of symptoms that sums up or constitute a disease
A. Syndrome
B. Symptoms
C. Signs
D. Etiology
12. A woman undergoing radiation therapy developed redness and burning of the skin around the best. This
is best classified as what type of disease?

A. Neoplastic
B. Traumatic
C. Nosocomial
D. Iatrogenic
13. The classification of CANCER according to its etiology Is best described as:
1. Nosocomial
2. Idiopathic
3. Neoplastic
4. Traumatic
5. Congenital
6. Degenrative
A. 5 and 2
B. 2 and 3
C. 3 and 4
D. 3 and 5
14. Term to describe the reactiviation and recurrence of pronounced symptoms of a disease
A. Remission
B. Emission
C. Exacerbation
D. Sub acute
15. A type of illness characterized by periods of remission and exacerbation
A. Chronic
B. Acute
C. Sub acute
D. Sub chronic
16. Diseases that results from changes in the normal structure, from recognizable anatomical changes in an
organ or body tissue is termed as
A. Functional
B. Occupational
C. Inorganic
D. Organic

17. It is the science of organism as affected by factors in their environment. It deals with the relationship
between disease and geographical environment.
A. Epidemiology
B. Ecology
C. Statistics
D. Geography
18. This is the study of the patterns of health and disease. Its occurrence and distribution in man, for the
purpose of control and prevention of disease.
A. Epidemiology
B. Ecology
C. Statistics
D. Geography
19. Refers to diseases that produced no anatomic changes but as a result from abnormal response to a
stimuli.
A. Functional
B. Occupational
C. Inorganic
D. Organic
20. In what level of prevention according to Leavell and Clark does the nurse support the client in obtaining
OPTIMAL HEALTH STATUS after a disease or injury?
A. Primary
B. Secondary
C. Tertiary
D. None of the above
21. In what level of prevention does the nurse encourage optimal health and increases persons
susceptibility to illness?
A. Primary
B. Secondary
C. Tertiary
D. None of the above
22. Also known as HEALTH MAINTENANCE prevention.

A. Primary
B. Secondary
C. Tertiary
D. None of the above
23. PPD In occupational health nursing is what type of prevention?
A. Primary
B. Secondary
C. Tertiary
D. None of the above
24. BCG in community health nursing is what type of prevention?
A. Primary
B. Secondary
C. Tertiary
D. None of the above
25. A regular pap smear for woman every 3 years after establishing normal pap smear for 3 consecutive
years Is advocated. What level of prevention does this belongs?
A. Primary
B. Secondary
C. Tertiary
D. None of the above
26. Self monitoring of blood glucose for diabetic clients is on what level of prevention?
A. Primary
B. Secondary
C. Tertiary
D. None of the above
27. Which is the best way to disseminate information to the public?
A. Newspaper
B. School bulletins
C. Community bill boards
D. Radio and Television
28. Who conceptualized health as integration of parts and subparts of an individual?

A. Newman
B. Neuman
C. Watson
D. Rogers
29. The following are concept of health:
1. Health is a state of complete physical, mental and social wellbeing and not merely an absence of disease or
infirmity.
2. Health is the ability to maintain balance
3. Health is the ability to maintain internal milieu
4. Health is integration of all parts and subparts of an individual
A. 1,2,3
B. 1,3,4
C. 2,3,4
D. 1,2,3,4
30. The theorist the advocated that health is the ability to maintain dynamic equilibrium is
A. Bernard
B. Selye
C. Cannon
D. Rogers
31. Excessive alcohol intake is what type of risk factor?
A. Genetics
B. Age
C. Environment
D. Lifestyle
32. Osteoporosis and degenerative diseases like Osteoarthritis belongs to what type of risk factor?
A. Genetics
B. Age
C. Environment
D. Lifestyle
33. Also known as STERILE TECHNIQUE

A. Surgical Asepsis
B. Medical Asepsis
C. Sepsis
D. Asepsis
34. This is a person or animal, who is without signs of illness but harbors pathogen within his body and can
be transferred to another
A. Host
B. Agent
C. Environment
D. Carrier
35. Refers to a person or animal, known or believed to have been exposed to a disease.
A. Carrier
B. Contact
C. Agent
D. Host
36. A substance usually intended for use on inanimate objects, that destroys pathogens but not the spores.
A. Sterilization
B. Disinfectant
C. Antiseptic
D. Autoclave
37. This is a process of removing pathogens but not their spores
A. Sterilization
B. Auto claving
C. Disinfection
D. Medical asepsis
38. The third period of infectious processes characterized by development of specific signs and symptoms
A. Incubation period
B. Prodromal period
C. Illness period
D. Convalescent period

39. A child with measles developed fever and general weakness after being exposed to another child with
rubella. In what stage of infectious process does this child belongs?
A. Incubation period
B. Prodromal period
C. Illness period
D. Convalescent period
40. A 50 year old mailman carried a mail with anthrax powder in it. A minute after exposure, he still hasnt
developed any signs and symptoms of anthrax. In what stage of infectious process does this man belongs?
A. Incubation period
B. Prodromal period
C. Illness period
D. Convalescent period
41. Considered as the WEAKEST LINK in the chain of infection that nurses can manipulate to prevent
spread of infection and diseases
A. Etiologic/Infectious agent
B. Portal of Entry
C. Susceptible host
D. Mode of transmission
42. Which of the following is the exact order of the infection chain?
1. Susceptible host
2. Portal of entry
3. Portal of exit
4. Etiologic agent
5. Reservoir
6. Mode of transmission
A. 1, 2, 3, 4, 5, 6
B. 5, 4, 2, 3, 6, 1
C. 4, 5, 3, 6, 2, 1
D. 6, 5, 4, 3, 2, 1
43. Markee, A 15 year old high school student asked you. What is the mode of transmission of Lyme
disease. You correctly answered him that Lyme disease is transmitted via

A. Direct contact transmission


B. Vehicle borne transmission
C. Air borne transmission
D. Vector borne transmission
44. The ability of the infectious agent to cause a disease primarily depends on all of the following except
A. Pathogenicity
B. Virulence
C. Invasiveness
D. Non Specificity
45. Contact transmission of infectious organism in the hospital is usually cause by
A. Urinary catheterization
B. Spread from patient to patient
C. Spread by cross contamination via hands of caregiver
D. Cause by unclean instruments used by doctors and nurses
46. Transmission occurs when an infected person sneezes, coughs or laugh that is usually projected at a
distance of 3 feet.
A. Droplet transmission
B. Airborne transmission
C. Vehicle transmission
D. Vector borne transmission
47. Considered as the first line of defense of the body against infection
A. Skin
B. WBC
C. Leukocytes
D. Immunization
48. All of the following contributes to host susceptibility except
A. Creed
B. Immunization
C. Current medication being taken
D. Color of the skin

49. Graciel has been injected TT5, her last dosed for tetanus toxoid immunization. Graciel asked you, what
type of immunity is TT Injections? You correctly answer her by saying Tetanus toxoid immunization is a/an
A. Natural active immunity
B. Natural passive immunity
C. Artificial active immunity
D. Artificial passive immunity
50. Agatha, was hacked and slashed by a psychotic man while she was crossing the railway. She suffered
multiple injuries and was injected Tetanus toxoid Immunoglobulin. Agatha asked you, What immunity does
TTIg provides? You best answered her by saying TTIg provides
A. Natural active immunity
B. Natural passive immunity
C. Artificial active immunity
D. Artificial passive immunity
51. This is the single most important procedure that prevents cross contamination and infection
A. Cleaning
B. Disinfecting
C. Sterilizing
D. Handwashing
52. This is considered as the most important aspect of handwashing
A. Time
B. Friction
C. Water
D. Soap
53. In handwashing by medical asepsis, hands are held
A. Above the elbow, The hands must always be above the waist
B. Above the elbow, The hands are cleaner than the elbow
C. Below the elbow, Medical asepsis do not require hands to be above the waist
D. Below the elbow, Hands are dirtier than the lower arms
54. The suggested time per hand on handwashing using the time method is
A. 5 to 10 seconds each hand
B. 10 to 15 seconds each hand

C. 15 to 30 seconds each hand


D. 30 to 60 seconds each hand
55. The minimum time in washing each hand should never be below
A. 5 seconds
B. 10 seconds
C. 15 seconds
D. 30 seconds
56. How many ml of liquid soap is recommended for handwashing procedure?
A. 1-2 ml
B. 2-3 ml
C. 2-4 ml
D. 5-10 ml
57. Which of the following is not true about sterilization, cleaning and disinfection?
A. Equipment with small lumen are easier to clean
B. Sterilization is the complete destruction of all viable microorganism including spores
C. Some organism are easily destroyed, while other, with coagulated protein requires longer time
D. The number of organism is directly proportional to the length of time required for sterilization
58. Karlita asked you, How long should she boil her glass baby bottle in water? You correctly answered her
by saying
A. The minimum time for boiling articles is 5 minutes
B. Boil the glass baby bottler and other articles for atleast 10 minutes
C. For boiling to be effective, a minimum of 15 minutes is required
D. It doesnt matter how long you boil the articles, as long as the water reached 100 degree Celsius
59. This type of disinfection is best done in sterilizing drugs, foods and other things that are required to be
sterilized before taken in by the human body
A. Boiling Water
B. Gas sterilization
C. Steam under pressure
D. Radiation
60. A TB patient was discharged in the hospital. A UV Lamp was placed in the room where he stayed for a
week. What type of disinfection is this?

A. Concurrent disinfection
B. Terminal disinfection
C. Regular disinfection
D. Routine disinfection
61. Which of the following is not true in implementing medical asepsis
A. Wash hand before and after patient contact
B. Keep soiled linens from touching the clothings
C. Shake the linens to remove dust
D. Practice good hygiene
62. Which of the following is true about autoclaving or steam under pressure?
A. All kinds of microorganism and their spores are destroyed by autoclave machine
B. The autoclaved instruments can be used for 1 month considering the bags are still intact
C. The instruments are put into unlocked position, on their hinge, during the autoclave
D. Autoclaving different kinds of metals at one time is advisable
63. Which of the following is true about masks?
A. Mask should only cover the nose
B. Mask functions better if they are wet with alcohol
C. Masks can provide durable protection even when worn for a long time and after each and every patient care
D. N95 Mask or particulate masks can filter organism as mall as 1 micromillimeter
64. Where should you put a wet adult diaper?
A. Green trashcan
B. Black trashcan
C. Orange trashcan
D. Yellow trashcan
65. Needles, scalpels, broken glass and lancets are considered as injurious wastes. As a nurse, it is correct
to put them at disposal via a/an
A. Puncture proof container
B. Reused PET Bottles
C. Black trashcan
D. Yellow trashcan with a tag INJURIOUS WASTES

66. Miranda Priestly, An executive of RAMP magazine, was diagnosed with cancer of the cervix. You noticed
that the radioactive internal implant protrudes to her vagina where supposedly, it should be in her cervix.
What should be your initial action?
A. Using a long forceps, Push it back towards the cervix then call the physician
B. Wear gloves, remove it gently and place it on a lead container
C. Using a long forceps, Remove it and place it on a lead container
D. Call the physician, You are not allowed to touch, re insert or remove it
67. After leech therapy, Where should you put the leeches?
A. In specially marked BIO HAZARD Containers
B. Yellow trashcan
C. Black trashcan
D. Leeches are brought back to the culture room, they are not thrown away for they are reusable
68. Which of the following should the nurse AVOID doing in preventing spread of infection?
A. Recapping the needle before disposal to prevent injuries
B. Never pointing a needle towards a body part
C. Using only Standard precaution to AIDS Patients
D. Do not give fresh and uncooked fruits and vegetables to Mr. Gatchie, with Neutropenia
69. Where should you put Mr. Alejar, with Category II TB?
A. In a room with positive air pressure and atleast 3 air exchanges an hour
B. In a room with positive air pressure and atleast 6 air exchanges an hour
C. In a room with negative air pressure and atleast 3 air exchanges an hour
D. In a room with negative air pressure and atleast 6 air exchanges an hour
70. A client has been diagnosed with RUBELLA. What precaution is used for this patient?
A. Standard precaution
B. Airborne precaution
C. Droplet precaution
D. Contact precaution
71. A client has been diagnosed with MEASLES. What precaution is used for this patient?
A. Standard precaution
B. Airborne precaution

C. Droplet precaution
D. Contact precaution
72. A client has been diagnosed with IMPETIGO. What precaution is used for this patient?
A. Standard precaution
B. Airborne precaution
C. Droplet precaution
D. Contact precaution
73. The nurse is to insert an NG Tube when suddenly, she accidentally dip the end of the tube in the clients
glass containing distilled drinking water which is definitely not sterile. As a nurse, what should you do?
A. Dont mind the incident, continue to insert the NG Tube
B. Obtain a new NG Tube for the client
C. Disinfect the NG Tube before reinserting it again
D. Ask your senior nurse what to do
74. All of the following are principle of SURGICAL ASEPSIS except
A. Microorganism travels to moist surfaces faster than with dry surfaces
B. When in doubt about the sterility of an object, consider it not sterile
C. Once the skin has been sterilized, considered it sterile
D. If you can reach the object by overreaching, just move around the sterile field to pick it rather than reaching for it
75. Which of the following is true in SURGICAL ASEPSIS?
A. Autoclaved linens and gowns are considered sterile for about 4 months as long as the bagging is intact
B. Surgical technique is a sole effort of each nurse
C. Sterile conscience, is the best method to enhance sterile technique
D. If a scrubbed person leaves the area of the sterile field, He/she must do handwashing and gloving again, but the
gown need not be changed.
76. In putting sterile gloves, Which should be gloved first?
A. The dominant hand
B. The non dominant hand
C. The left hand
D. No specific order, Its up to the nurse for her own convenience
77. As the scrubbed nurse, when should you apply the goggles, shoe cap and mask prior to the operation?

A. Immediately after entering the sterile field


B. After surgical hand scrub
C. Before surgical hand scrub
D. Before entering the sterile field
78. Which of the following should the nurse do when applying gloves prior to a surgical procedure?
A. Slipping gloved hand with all fingers when picking up the second glove
B. Grasping the first glove by inserting four fingers, with thumbs up underneath the cuff
C. Putting the gloves into the dominant hand first
D. Adjust only the fitting of the gloves after both gloves are on
79. Which gloves should you remove first?
A. The glove of the non dominant hand
B. The glove of the dominant hand
C. The glove of the left hand
D. Order in removing the gloves Is unnecessary
80. Before a surgical procedure, Give the sequence on applying the protective items listed below
1. Eye wear or goggles
2. Cap
3. Mask
4. Gloves
5. Gown
A. 3, 2, 1, 5, 4
B. 3, 2, 1, 4, 5
C. 2, 3, 1, 5, 4
D. 2, 3, 1, 4, 5
81. In removing protective devices, which should be the exact sequence?
1. Eye wear or goggles
2. Cap
3. Mask
4. Gloves
5. Gown
A. 4, 3, 5, 1, 2
B. 2, 3, 1, 5, 4

C. 5, 4, 3, 2, 1
D. 1, 2, 3, 4, 5
82. In pouring a plain NSS into a receptacle located in a sterile field, how high should the nurse hold the
bottle above the receptacle?
A. 1 inch
B. 3 inches
C. 6 inches
D. 10 inches
83. The tip of the sterile forceps is considered sterile. It is used to manipulate the objects in the sterile field
using the non sterile hands. How should the nurse hold a sterile forceps?
A. The tip should always be lower than the handle
B. The tip should always be above the handle
C. The handle and the tip should be at the same level
D. The handle should point downward and the tip, always upward
84. The nurse enters the room of the client on airborne precaution due to tuberculosis. Which of the
following are appropriate actions by the nurse?
1. She wears mask, covering the nose and mouth
2. She washes her hands before and after removing gloves, after suctioning the clients secretion
3. She removes gloves and hands before leaving the clients room
4. She discards contaminated suction catheter tip in trashcan found in the clients room
A. 1, 2
B. 1, 2, 3
C. 1, 2, 3, 4
D. 1, 3
85. When performing surgical hand scrub, which of the following nursing action is required to prevent
contamination?
1. Keep fingernail short, clean and with nail polish
2. Open faucet with knee or foot control
3. Keep hands above the elbow when washing and rinsing
4. Wear cap, mask, shoe cover after you scrubbed
A. 1, 2
B. 2, 3

C. 1, 2, 3
D. 2, 3, 4
86. When removing gloves, which of the following is an inappropriate nursing action?
A. Wash gloved hand first
B. Peel off gloves inside out
C. Use glove to glove skin to skin technique
D. Remove mask and gown before removing gloves
87. Which of the following is TRUE in the concept of stress?
A. Stress is not always present in diseases and illnesses
B. Stress are only psychological and manifests psychological symptoms
C. All stressors evoke common adaptive response
D. Hemostasis refers to the dynamic state of equilibrium
88. According to this theorist, in his modern stress theory, Stress is the non specific response of the body
to any demand made upon it.
A. Hans Selye
B. Walter Cannon
C. Claude Bernard
D. Martha Rogers
89. Which of the following is NOT TRUE with regards to the concept of Modern Stress Theory?
A. Stress is not a nervous energy
B. Man, whenever he encounters stresses, always adapts to it
C. Stress is not always something to be avoided
D. Stress does not always lead to distress
90. Which of the following is TRUE with regards to the concept of Modern Stress Theory?
A. Stress is essential
B. Man does not encounter stress if he is asleep
C. A single stress can cause a disease
D. Stress always leads to distress
91. Which of the following is TRUE in the stage of alarm of general adaptation syndrome?

A. Results from the prolonged exposure to stress


B. Levels or resistance is increased
C. Characterized by adaptation
D. Death can ensue
92. The stage of GAS where the adaptation mechanism begins
A. Stage of Alarm
B. Stage of Resistance
C. Stage of Homeostasis
D. Stage of Exhaustion
93. Stage of GAS Characterized by adaptation
A. Stage of Alarm
B. Stage of Resistance
C. Stage of Homeostasis
D. Stage of Exhaustion
94. Stage of GAS wherein, the Level of resistance are decreased
A. Stage of Alarm
B. Stage of Resistance
C. Stage of Homeostasis
D. Stage of Exhaustion
95. Where in stages of GAS does a person moves back into HOMEOSTASIS?
A. Stage of Alarm
B. Stage of Resistance
C. Stage of Homeostasis
D. Stage of Exhaustion
96. Stage of GAS that results from prolonged exposure to stress. Here, death will ensue unless extra
adaptive mechanisms are utilized
A. Stage of Alarm
B. Stage of Resistance
C. Stage of Homeostasis
D. Stage of Exhaustion
97. All but one is a characteristic of adaptive response

A. This is an attempt to maintain homeostasis


B. There is a totality of response
C. Adaptive response is immediately mobilized, doesnt require time
D. Response varies from person to person
98. Andy, a newly hired nurse, starts to learn the new technology and electronic devices at the hospital.
Which of the following mode of adaptation is Andy experiencing?
A. Biologic/Physiologic adaptive mode
B. Psychologic adaptive mode
C. Sociocultural adaptive mode
D. Technological adaptive mode
99. Andy is not yet fluent in French, but he works in Quebec where majority speaks French. He is starting to
learn the language of the people. What type of adaptation is Andy experiencing?
A. Biologic/Physiologic adaptive mode
B. Psychologic adaptive mode
C. Sociocultural adaptive mode
D. Technological adaptive mode
100. Andy made an error and his senior nurse issued a written warning. Andy arrived in his house mad and
kicked the door hard to shut it off. What adaptation mode is this?
A. Biologic/Physiologic adaptive mode
B. Psychologic adaptive mode
C. Sociocultural adaptive mode
D. Technological adaptive mode

Answers and Rationale


Here are the answers and rationale for this 100-item Fundamentals of Nursing exam. If you have any disputes or
questions, direct them to the comments section below! Tell us also your scores!
1. Answer: C. Lungs will bronchodilate
To better understand the concept, the autonomic nervous system is composed of
the sympathetic and parasympathetic nervous system. It is called autonomic because it is involuntary and stimulibased. You cannot dictate your heart to beat 60 per minute, nor tell your blood vessels when to constrict and dilate.
Sympathetic nervous system is the fight or flight mechanism.
2. Answer: C. Clients BP will increase, there will be vasodilation

If vasodilation will occur, the BP will not increase but decrease.


3. Answer: A. Illness
Disease is a proven fact based on a medical theory, standards, diagnosis and clinical features. Illness, on the other
hand, is a subjective state of not feeling well based on subjective appraisal, previous experience, peer advice and
etc.
4. Answer: A. Symptom Experience
Assumption of the sick role is when a client accepts he is ill. Medical care contact is where the client asks someone
to confirm what he is experiencing. During this stage, the client seeks professional advice for validation, reassurance
clarification and explanation of the symptoms he is experiencing. Last stage of illness is the recovery stage where
the patient gives up the sick role and assumes the previous normal functions.
5. Answer: D. Dependent patient role
In the dependent patient role stage, patients needs professionals for help. They have a choice either to accept or
reject the professionals decisions but patients are usually passive and accepting. Regression tends to occur more in
this period.
6. Answer: B. Assumption of sick role
Acceptance of illness occurs in the assumption of sick role phase of illness.
7. Answer: C. Medical care contact
At this stage, The patient seeks for validation of his symptom experience. He wants to find out if what he feels are
normal or not normal. He wants someone to explain why is he feeling these signs and symptoms and wants to know
the probable outcome of this experience.
8. Answer: A. One should be held responsible for his condition
The nurse should not judge the patient and not view the patient as the cause or someone responsible for his illness.
A sick client is excused from his societal roles, oblige to get well as soon as possible and obliged to seek competent
help.
9. Answer: C. Risk factor
10. Answer: A. Susceptibility
Immunity is the absolute resistance to a pathogen considering that person has an intact immunity. Susceptibility is
the degree of resistance or how well would the individual combat the pathogens and repel infection or invasion of

these disease causing organisms. A susceptible person is someone who has a very low degree of resistance to
combat pathogens. An immune person is someone that can easily repel specific pathogens.
11. Answer: A. Syndrome
Symptoms are individual manifestation of a certain disease (i.e., tics in Tourette syndrome) but a symptom alone is
not enough to diagnose the patient as other diseases may have the symptom manifestation. Syndrome, on the other
hand, means collection of these symptoms that occurs together and has a pattern that characterizes a certain
disease.
12. Answer: D. Iatrogenic
Iatrogenic diseases refers to those that resulted from treatment of a certain disease. For example, a child develops
redness and partial thickness burns over his chest area due to frequent exposure to X-ray. Neoplastic diseases
are malignant diseases caused by proliferation of abnormally growing cells. Traumatic are brought about by injuries
like vehicular accidents. Nosocomial are infections that are acquired inside the hospital.
13. Answer: B. 2 and 3 (Idiopathic and Neoplastic)
14. Answer: C. Exacerbation
15. Answer: A. Chronic
Chronic diseases are characterized by periods of remissions and exacerbations and persists longer than six (6)
months that is why remissions and exacerbations are observable. The duration of acute and sub acute diseases are
too short to manifest remissions.
16. Answer: D. Organic
Organic diseases are caused by a change in structure of a certain organ and organ systems. Inorganic diseases, on
the other hand, are synonymous with functional diseases where there is no evident structural, anatomical or physical
change in the structure of the organ or organ system but its function is altered due to other causes.
17. Answer: B. Ecology
Ecology is the science that deals with the ecosystem and its effects on living things in the biosphere. It deals with
diseases in relationship with the environment. Epidemiology is simply the study of diseases and its occurrence and
distribution in man for the purpose of controlling and preventing diseases.
18. Answer: A. Epidemiology
19. Answer: C. Inorganic

20. Answer: C. Tertiary


Primary refers to the measures that aim in preventing the diseases (i.e., healthy lifestyle, good nutrition, knowledge
seeking behaviors, etc). Secondary prevention are those measures that deal with early diagnostics, case finding
treatments (i.e, breast self exam, X-rays, antibiotic treatment to cure infection, iron therapy for anemia, etc). Tertiary
prevention aims on maintaining optimum level of functioning during or after the impact of disease that threatens to
alter the normal body functioning (i.e., prosthesis fitting for an amputated leg, glucose monitoring among diabetics,
and TPA therapy after stroke).
21. Answer: D. None of the above.
The nurse never increases the persons susceptibility to illness but rather, LESSEN the persons susceptibility to
illness.
22. Answer: B. Secondary
Secondary prevention is also known as health maintenance prevention.
23. Answer: A. Primary
Personal Protective Devices or PPD are worn by workers in a hazardous work environment to protect them from
injuries. This is considered as primary prevention because the nurse prevents the occurrence of injuries.
24. Answer: A. Primary
25. Answer: B. Secondary
Pap-smear is a diagnostic procedure thus falls under the secondary level of prevention.
26. Answer: C. Tertiary
27. Answer: D. Radio and Television
The best way to disseminate information to the public is via television followed by radio. The two media outlets have
the widest reach and this is how the Department of Health (DOH) establishes its IEC programs.
28. Answer: B. Neuman
The supra and subsystems are theories of Martha Rogers but the parts and subparts are Betty Neumans. She
stated that health is a state where in all parts and subparts of an individual are in harmony with the whole system.
Margarex Newman defined health as an expanding consciousness.
29. Answer: D. 1,2,3,4

Rationale:All of the following are correct statement about health. The first one is the definition by World Health
Organization (WHO). The second is taken from Walter Cannons Homeostasis Theory. The third is from Claude
Bernards concept of Health as Internal Milieu. The last one is from Neumans theory.
30. Answer: C. Cannon
Walter Cannon advocated health as homeostasis or the ability to maintain dynamic equilibrium. Hans Selye
postulated concepts about stress and adaptation. Bernard defined health as the ability to maintain internal milieu
and Rogers defined Health as Wellness that is influenced by individuals culture.
31. Answer: D. Lifestyle
32. Answer: B. Age
33. Answer: A. Surgical Asepsis
Surgical Asepsis is otherwise known as the sterile technique, while Medical Asepsis is synonymous with the clean
technique.
34. Answer: D. Carrier
35. Answer: B. Contact
36. Answer: B. Disinfectant
Disinfectants are used on inanimate objects. Antiseptics are intended on persons and other living things. Both can
kill and inhibit growth of microorganisms but cannot kill spores. Autoclaving or steam under pressure kills almost all
types of microorganisms including their spores.
37. Answer: C. Disinfection
38. Answer: C. Illness period
In incubation period, the disease has been introduced to the body but no sign and symptom appear because the
pathogen is not yet strong enough to cause it and may still need to multiply. The second period is called prodromal
period where appearance of non-specific signs and symptoms sets in. Illness period is characterized by appearance
of specific signs and symptoms. Acme is the peak of an illness while convalescent period is characterized by the
abatement of the disease or its gradual dissipation.
39. Answer: B. Prodromal period
40. Answer: A. Incubation period

Anthrax can have an incubation period of hours to seven (7) days with an average of 48 (hours). Since the question
stated exposure, we can assume that the mailman is in the incubation phase.
41. Answer: D. Mode of transmission
Mode of transmission is the weakest link in the chain of infection. It is easily manipulated by the nurses using the
tiers of prevention, either by instituting transmission based precautions, universal precaution or isolation techniques.
42. Answer: C. 4,5,3,6,2,1
Source is the etiological agent itself, this is where the chain of infection starts. The source will first proliferate on a
reservoir and will need a portal of exit to be able to transmit itself using a portal of entry to a susceptible host.
43. Answer: D. Vector borne transmission
Lymes disease is caused by borrelia burgdorferi and is transmitted via a tick bite.
44. Answer: D. Non Specificity
To be able to cause a disease, a pathogen should have a target organ and should be specific to these organs to
cause an infection.
45. Answer: C. Spread by cross contamination via hands of caregiver
A caregivers hands like any other healthcare workers is the main cause of cross-contamination in a hospital setting.
Handwashing is the single most important procedure to prevent the occurrence of cross-contamination and
nosocomial infections.
46. Answer: A. Droplet transmission
47. Answer: A. Skin
Remember that intact skin and mucous membrane is our first line of defense against infection.
48. Answer: A. Creed
Creed, Faith or religious belief do not affect persons susceptibility to illness. Medication like corticosteroids could
supress a persons immune system that will lead to increase susceptibility. Color of the skin could affect persons
susceptibility to certain skin diseases. A dark skinned person has lower risk of skin cancer than a fair skinned
person. Fair skinned person also has a higher risk for cholecystitis and cholelithiasis.
49. Answer: C. Artificial active immunity

TT1 and TT2 are considered the primary doses while TT3, TT4 and TT5 are booster doses. A woman with a
complete immunization of DPT need not to receive the primary doses TT1 and TT2. Tetanus toxoid is an actual but
weakened and inactivated toxin produced by clostridium tetani. It is artificial because it did not occur in the course of
an actual illness or infection.
50. Answer: D. Artificial passive immunity
In this scenario, Agatha was already wounded and has injuries. Giving the Tetanus Toxoid vaccine itself would not
be beneficial to her as it will take time before the body can produce antitoxins. What Agatha needs now is a ready
made antitoxin in the form of ATS or TTIg which is considered artificial because her body did not produce it
and passive because her immune system was not stimulated but rather, given a ready-made immunoglobulin to
immediately suppress the infection.
51. Answer: D. Handwashing
TIP: Most of the time, when you see the word handwashing as one of the options, there is a big chance that it is the
correct answer.
52. Answer: B. Friction
The most important aspect of handwashing is friction. The rest of the components will just enhance friction. Soap
lowers the surface tension thereby increasing the effectiveness of friction. Water helps remove transient bacteria by
working with soap to create lather that reduces surface tension. Time, on the other hand, is of the essence but
friction is the most essential aspect of handwashing. Its friction whether you like it or not.
53. Answer: D. Below the elbow, Hands are dirtier than the lower arms
Hands are put below the elbow in medical asepsis in contrast with surgical asepsis where hands are required to be
kept above the waist. In medical asepsis, hands are considered dirtier than the elbow and therefore, to limit
contamination of the lower arm, the hands should always be below the elbow.
54. Answer: C. 15 to 30 seconds each hand
Each hand requires 15 to 30 seconds of hand washing as a minimum to effectively remove transient germs.
55. Answer: B. 10 seconds
According to Kozier, the minimum time required for watching each hands is 10 seconds and should not be lower
than that. The recommended time, again, is 15 to 30 seconds.
56. Answer: C. 2-4 ml
If a liquid soap is to be used, 1 tsp (5 ml) of liquid soap is recommended for handwashing procedure.

57. Answer: A. Equipment with small lumen are easier to clean


Equipment with large lumen are easier to clean compared to those with small lumen. Other choices are correct.
58. Answer: C. For boiling to be effective, a minimum of 15 minutes is required
Boiling is the most common and least expensive method of sterilization used in home. For it to be effective, you
should boil articles for at least 15 minutes.
59. Answer: D. Radiation
If food and drugs are to be sterilized by boiling water, ethylene oxide gas and autoclave, most probably theyll be
ineffective after the process. Ethylene oxide gas is toxic to humans, boiling food will alter its consistency and lower it
nutritional value. Autoclaving food may sound fun but it is the dumbest thing to do. Radiation using a microwave
oven or an ionization penetrate foods and drugs thus, sterilizing them.
60. Answer: B. Terminal disinfection
Terminal disinfection refers to practices to remove pathogens that stayed in the belongings or immediate
environment of an infected client who has been discharged. Concurrent disinfection refers to ongoing efforts
implemented during the clients stay to remove or limit pathogens in his supplies, belongings, immediate
environment in order to control the spread of the disease.
61. Answer: C. Shake the linens to remove dust
Never shake soiled linens. Once soiled fold it inwards with the clean surface facing out. Shaking the linen can
dislodge and further spread pathogens harbored in its fabric.
62. Answer: C. The instruments are put into unlocked position, on their hinge, during the autoclave
Metals with locks, like clamps and scissors should be unlocked in order to minimize stiffening caused by autoclaving
the hinges. Not all microorganisms are destroyed by autoclaving, there are still microorganisms that are invulnerable
to extreme heat. Autoclaved instruments are to be used within two (2) weeks. Only the same type of metals should
be autoclaved as this will alteration in plating of these metals.
63. Answer: D. N95 Mask or particulate masks can filter organism as mall as 1 micromillimeter
Mask should cover both nose and mouth and will not function optimally when wet. They should never be worn
greater than four (4) hours and will gradually lose its effectiveness after four (4) horus. N95 masks or particulate
masks can filter organisms as small as one (1) micromillimeter.
64. Answer: D. Yellow trashcan

Infectious waste like blood and blood products, wet diapers and dressings are thrown in yellow-colored trashbins.
65. Answer: A. Puncture proof container
Needles, scalpels and other sharps are to be disposed in a puncture proof container.
66. Answer: C. Using a long forceps, Remove it and place it on a lead container
A dislodged radioactive cervical implant in brachytherapy are to be picked by a long forceps and to be stored in a
lead container in order to prevent damage on the clients normal tissue. Calling the physician is the most appropriate
action among the choices, a nurse should never attempt to put it back nor touch it with her bare or even gloved
hands.
67. Answer: A. In specially marked BIO HAZARD Containers
Leeches, in leech therapy or leech phlebotomy are to be disposed on a BIOHAZARD container. They are never re
used as this could cause transfer of infection. These leeches are hospital grown and not the usual leeches found in
swamps because that would just be disgusting.
68. Answer: A. Recapping the needle before disposal to prevent injuries
Never recap needles. After using, they are directly disposed in a puncture proof container after used. Recapping
could cause injury to the nurse and spread infection. Choices B, C and D are all appropriate. Standard precaution is
sufficient for a patient with HIV. A client with neutropenia are not given fresh and uncooked fruits and vegetables as
it may cause severe infection due to immunosuppression.
69. Answer: D. In a room with negative air pressure and atleast 6 air exchanges an hour
Patients with tuberculosis should have a private room with a negative air pressure and at least 6 to 12 air exchanges
per hour. Negative pressure rooms will prevent air inside from escaping. Air exchanges are necessary since the
clients room do not allow air to get out of the room.
70. Answer: C. Droplet precaution
Droplet precaution is sufficient on clients with RUBELLA or german measles.
71. Answer: B. Airborne precaution
Measles is highly communicable and more contagious than Rubella. It requires airborne precaution as it is spread
by small particle droplets that remains suspended in air and dispersed by air movements.
72. Answer: D. Contact precaution

Impetigo causes blisters or sores in the skin. It is generally caused by GABS or Staph Aureaus. It is spread by skin
to skin contact or by scratching the lesions and touching another persons skin.
73. Answer: A. Dont mind the incident, continue to insert the NG Tube
The digestive tract is not sterile, and therefore, simple errors like this would not cause harm to the patient. NGT tube
need not be sterile, and so is colostomy and rectal tubes. Clean technique is sufficient during NGT and colostomy
care.
74. Answer: C. Once the skin has been sterilized, considered it sterile
Human skin is impossible to be sterilized. It contains normal flora of microorganism. A, B and D are all correct.
75. Answer: C. Sterile conscience, is the best method to enhance sterile technique
Sterile conscience, or the moral imperative of a nurse to be honest in practicing sterile technique, is the best method
to enhance sterile technique. Autoclaved linens are considered sterile only within two (2) weeks even if the bagging
is intact. Surgical technique is a team effort of each nurse. If a scrubbed person leave the sterile field and area, he
must do the process all over again.
76. Answer: B. The non dominant hand
Gloves are put on the non dominant hands first and then, the dominant hand. The rationale is simply because
humans tend to use the dominant hand first before the non dominant hand. Out of 10 humans that will put on their
sterile gloves, 8 of them will put the gloves on their non dominant hands first.
77. Answer: C. Before surgical hand scrub
The nurse should put his goggles, cap and mask prior to washing the hands. If he wash his hands prior to putting all
these equipments, he must wash his hands again as these equipments are said to be unsterile.
78. Answer: D. Adjust only the fitting of the gloves after both gloves are on
The nurse should only adjust fitting of the gloves when they are both on the hands. Not doing so will break the sterile
technique. Only 4 fingers are slipped when picking up the second gloves. You cannot slip all of your fingers as the
cuff is limited and the thumb would not be able to enter the cuff. The first glove is grasp by simply picking it up with
the first 2 fingers and a thumb in a pinching motion. Gloves are put on the non dominant hands first.
79. Answer: A. The glove of the non dominant hand
Gloves are worn in the non dominant hand first, and is removed also from the non dominant hand first. Rationale is
simply because in 10 people removing gloves, 8 of them will use the dominant hand first and remove the gloves of
the non dominant hand.

80. Answer: D. 2,3,1,4,5


81. Answer: A. 4,3,5,1,2
When removing protective devices, one must remove the gloves first, followed by the mask and gown, then other
devices like cap, shoe cover, and etc. Doing it in this manner will prevent contamination of hair, neck and face
area. Go ahead, try removing your mask, hair cap and others before removing your soiled and bloodied gloves.
82. Answer: C. 6 inches
Even if you do not know the answer to this question, you can answer it correctly by imagining. If you pour the NSS
into a receptacle 1 to 3 inch above it, chances are, the mouth of the NSS bottle would dip into the receptacle as you
fill it, making it contaminated. If you pour the NSS bottle into a receptacle 10 inches above it, that is too high,
chances are, as you pour the NSS, most will spill out because the force will be too much for the buoyant force to
handle. It will also be difficult to pour something precisely into a receptacle as the height increases between the
receptacle and the bottle. 6 inches is the correct answer. It is not too low nor too high.
83. Answer: A. The tip should always be lower than the handle
Sterile forceps are usually dipped into a disinfectant or germicidal solution. If the tip is higher than the handle, the
solution will flow down into the handle and into your hands, and as you use the forceps, youll eventually lower its tip
making the solution in your hand flow to the tip thus contaminating the sterile area of the forceps. To prevent this, the
tip should always be lower than the handle.
84. Answer: C. 1,2,3,4
All soiled equipment used in an infectious patient are disposed inside the clients room to prevent contamination
outside the room. Using the mask to cover both nose and mouth is correct. Hands are washed before removing the
gloves and before and after your enter the clients room. Gloves and contaminated suction tip are thrown in the trash
found in the clients room.
85. Answer: C. 1,2,3
Caps, masks and shoe covers are worn before scrubbing in.
86. Answer: D. Remove mask and gown before removing gloves
Gloves are the dirtiest personal protective device used and therefore, should be the first to be removed to prevent
spread of microorganisms as you remove your masks and gown.
87. Answer: C. All stressors evoke common adaptive response

All stressors evoke common adaptive response. A psychologic fear like nightmare and a real fear or real perceive
threat evokes common manifestation like tachycardia, tachypnea, sweating, increase muscle tension etc. ALL
diseases and illness causes stress. Stress can be both real or imaginary. Hemostasis refers to the arrest of blood
flowing abnormally through a damage vessel. Homeostasis is the one that refers to dynamic state of equilibrium
according to Walter Cannon.
88. Answer: A. Hans Selye
Hans Selye is the only theorist who proposed an intriguing theory about stress that has been widely used and
accepted by professionals today. He conceptualized two types of human response to stress, The GAS or general
adaptation syndrome which is characterized by stages of ALARM, RESISTANCE and EXHAUSTION. The local
adaptation syndrome controls stress through a particular body part.
89. Answer: B. Man, whenever he encounters stresses, always adapts to it
Man do not always adapt to stress. Sometimes, stress can lead to exhaustion and eventually, death. Choices A, C
and D are all correct.
90. Answer: A. Stress is essential
Stress is ESSENTIAL. No man can live normally without stress. It is essential because it is evoked by the bodys
normal pattern of response and leads to a favorable adaptive mechanism that are utilized in the future when more
stressors are encountered by the body. Man can encounter stress even while asleep (i.e., nightmares).
Disease are multifactorial and are not caused by a single stressor. Stress is sometimes favorable and are not always
a cause for distress.
91. Answer: D. Death can ensue
Death can ensue as early as the stage of alarm. Exhaustion results to a prolonged exposure to stress. Resistance is
when the levels of resistance increases and characterized by being able to adapt.
92. Answer: A. Stage of Alarm
Adaptation mechanisms begin in the stage of alarm. This is when the adaptive mechanism are mobilized.
93. Answer: B. Stage of Resistance
94. Answer: A. Stage of Alarm
Resistance are decreased in the stage of alarm. Resistance is absent in the stage of exhaustion. Resistance is
increased in the stage of resistance.

95. Answer: B. Stage of Resistance


96. Answer: D. Stage of Exhaustion
97. Answer: C. Adaptive response is immediately mobilized, doesnt require time
Aside from having limits that leads to exhaustion, adaptive response requires time for it to act. It requires energy,
physical and psychological taxes that needs time for our body to mobilize and utilize.
98. Answer: D. Technological adaptive mode
99. Answer: C. Sociocultural adaptive mode
Sociocultural adaptive modes include language, communication, dressing, acting and socializing in line with the
social and cultural standard of the people around the adapting individual.
100. Answer: B. Psychologic adaptive mode
1. The coronary vessels, unlike any other blood vessels in the body, respond to sympathetic stimulation by
A. Vasoconstriction
B. Vasodilatation
C. Decreases force of contractility
D. Decreases cardiac output
2. What stress response can you expect from a patient with blood sugar of 50 mg / dl?
A. Body will try to decrease the glucose level
B. There will be a halt in release of sex hormones
C. Client will appear restless
D. Blood pressure will increase
3. All of the following are purpose of inflammation except
A. Increase heat, thereby produce abatement of phagocytosis
B. Localized tissue injury by increasing capillary permeability
C. Protect the issue from injury by producing pain
D. Prepare for tissue repair
4. The initial response of tissue after injury is
A. Immediate Vasodilation
B. Transient Vasoconstriction

C. Immediate Vasoconstriction
D. Transient Vasodilation
5. The last expected process in the stages of inflammation is characterized by
A. There will be sudden redness of the affected part
B. Heat will increase on the affected part
C. The affected part will lose its normal function
D. Exudates will flow from the injured site
6. What kind of exudates is expected when there is an antibody-antigen reaction as a result of
microorganism infection?
A. Serous
B. Serosanguinous
C. Purulent
D. Sanguinous
7. The first manifestation of inflammation is
A. Redness on the affected area
B. Swelling of the affected area
C. Pain, which causes guarding of the area
D. Increase heat due to transient vasodilation
8. The client has a chronic tissue injury. Upon examining the clients antibody for a particular cellular
response, Which of the following WBC component is responsible for phagocytosis in chronic tissue injury?
A. Neutrophils
B. Basophils
C. Eosinophils
D. Monocytes
9. Which of the following WBC component proliferates in cases of Anaphylaxis?
A. Neutrophils
B. Basophils
C. Eosinophil
D. Monocytes

10. Icheanne, ask you, her Nurse, about WBC Components. She got an injury yesterday after she twisted
her ankle accidentally at her gymnastic class. She asked you, which WBC Component is responsible for
proliferation at the injured site immediately following an injury. You answer:
A. Neutrophils
B. Basophils
C. Eosinophils
D. Monocytes
11. Icheanne then asked you, what is the first process that occurs in the inflammatory response after injury,
You tell her:
A. Phagocytosis
B. Emigration
C. Pavementation
D. Chemotaxis
12. Icheanne asked you again, What is that term that describes the magnetic attraction of injured tissue to
bring phagocytes to the site of injury?
A. Icheanne, you better sleep now, you asked a lot of questions
B. It is Diapedesis
C. We call that Emigration
D. I dont know the answer, perhaps I can tell you after I find it out later
13. This type of healing occurs when there is a delayed surgical closure of infected wound
A. First intention
B. Second intention
C. Third intention
D. Fourth intention
14. Type of healing when scars are minimal due to careful surgical incision and good healing
A. First intention
B. Second intention
C. Third intention
D. Fourth intention
15. Imelda, was slashed and hacked by an unknown suspects. She suffered massive tissue loss and
laceration on her arms and elbow in an attempt to evade the criminal. As a nurse, you know that the type of
healing that will most likely occur to Miss Imelda is

A. First intention
B. Second intention
C. Third intention
D. Fourth intention
16. Imelda is in the recovery stage after the incident. As a nurse, you know that the diet that will be
prescribed to Miss Imelda is
A. Low calorie, High protein with Vitamin A and C rich foods
B. High protein, High calorie with Vitamin A and C rich foods
C. High calorie, Low protein with Vitamin A and C rich foods
D. Low calorie, Low protein with Vitamin A and C rich foods
17. Miss Imelda asked you, What is WET TO DRY Dressing method? Your best response is
A. It is a type of mechanical debridement using Wet dressing that is applied and left to dry to remove dead tissues
B. It is a type of surgical debridement with the use of Wet dressing to remove the necrotic tissues
C. It is a type of dressing where in, The wound is covered with Wet or Dry dressing to prevent contamination
D. It is a type of dressing where in, A cellophane or plastic is placed on the wound over a wet dressing to stimulate
healing of the wound in a wet medium
18. The primary cause of pain in inflammation is
A. Release of pain mediators
B. Injury to the nerve endings
C. Compression of the local nerve endings by the edema fluids
D. Circulation is lessen, Supply of oxygen is insufficient
19. The client is in stress because he was told by the physician he needs to undergo surgery for removal of
tumor in his bladder. Which of the following are effects of sympatho-adreno-medullary response by the
client?
1. Constipation
2. Urinary frequency
3. Hyperglycemia
4. Increased blood pressure
A. 3,4
B. 1,3,4
C.1,2,4
D.1,4

20. The client is on NPO post midnight. Which of the following, if done by the client, is sufficient to cancel
the operation in the morning?
A. Eat a full meal at 10:00 P.M
B. Drink fluids at 11:50 P.M
C. Brush his teeth the morning before operation
D. Smoke cigarette around 3:00 A.M
21. The client place on NPO for preparation of the blood test. Adreno-cortical response is activated and
which of the following below is an expected response?
A. Low BP
B. Decrease Urine output
C. Warm, flushed, dry skin
D. Low serum sodium levels
22. Which of the following is true about therapeutic relationship?
A. Directed towards helping an individual both physically and emotionally
B. Bases on friendship and mutual trust
C. Goals are set by the solely nurse
D. Maintained even after the client doesnt need anymore of the Nurses help
23. According to her, A nurse patient relationship is composed of 4 stages : Orientation, Identification,
Exploitation and Resolution
A. Roy
B. Peplau
C. Rogers
D. Travelbee
24. In what phase of Nurse patient relationship does a nurse review the clients medical records thereby
learning as much as possible about the client?
A. Pre Orientation
B. Orientation
C. Working
D. Termination
25. Nurse Aida has seen her patient, Roger for the first time. She establish a contract about the frequency of
meeting and introduce to Roger the expected termination. She started taking baseline assessment and set
interventions and outcomes. On what phase of NPR Does Nurse Aida and Roger belong?

A. Pre Orientation
B. Orientation
C. Working
D. Termination
26. Roger has been seen agitated, shouting and running. As Nurse Aida approaches, he shouts and swear,
calling Aida names. Nurse Aida told Roger That is an unacceptable behavior Roger, Stop and go to your
room now. The situation is most likely in what phase of NPR?
A. Pre Orientation
B. Orientation
C. Working
D. Termination
27. Nurse Aida, in spite of the incident, still consider Roger as worthwhile simply because he is a human
being. What major ingredient of a therapeutic communication is Nurse Aida using?
A. Empathy
B. Positive regard
C. Comfortable sense of self
D. Self awareness
28. Nurse Irma saw Roger and told Nurse Aida Oh look at that psychotic patient Nurse Aida should
intervene and correct Nurse Irma because her statement shows that she is lacking?
A. Empathy
B. Positive regard
C. Comfortable sense of self
D. Self awareness
29. Which of the following statement is not true about stress?
A. It is a nervous energy
B. It is an essential aspect of existence
C. It has been always a part of human experience
D. It is something each person has to cope
30. Martina, a tennis champ was devastated after many new competitors outpaced her in the Wimbledon
event.
She became depressed and always seen crying. Martina is clearly on what kind of situation?

A. Martina is just stressed out


B. Martina is Anxious
C. Martina is in the exhaustion stage of GAS
D. Martina is in Crisis
31. Which of the following statement is not true with regards to anxiety?
A. It has physiologic component
B. It has psychologic component
C. The source of dread or uneasiness is from an unrecognized entity
D. The source of dread or uneasiness is from a recognized entity
32. Lorraine, a 27 year old executive was brought to the ER for an unknown reason. She is starting to speak
but her speech is disorganized and cannot be understood. On what level of anxiety does this features
belongs?
A. Mild
B. Moderate
C. Severe
D. Panic
33. Elton, 21 year old nursing student is taking the board examination. She is sweating profusely, has
decreased awareness of his environment and is purely focused on the exam questions characterized by his
selective attentiveness. What anxiety level is Elton exemplifying?
A. Mild
B. Moderate
C. Severe
D. Panic
34. You noticed the patient chart : ANXIETY +3 What will you expect to see in this client?
A. An optimal time for learning, hearing and perception is greatly increased
B. Dilated pupils
C. Unable to communicate
D. Palliative Coping Mechanism
35. When should the nurse starts giving XANAX?
A. When anxiety is +1
B. When the client starts to have a narrow perceptual field and selective inattentiveness

C. When problem solving is not possible


D. When the client is immobile and disorganized
36. Which of the following behavior is not a sign or a symptom of Anxiety?
A. Frequent hand movement
B. Somatization
C. The client asks a question
D. The client is acting out
37. Which of the following intervention is inappropriate for clients with anxiety?
A. Offer choices
B. Provide a quiet and calm environment
C. Provide detailed explanation on each and every procedures and equipments
D. Bring anxiety down to a controllable level
38. Which of the following statement, if made by the nurse, is considered not therapeutic?
A. How did you deal with your anxiety before?
B. It must be awful to feel anxious.
C. How does it feel to be anxious?
D. What makes you feel anxious?
39. Marissa Salva, Uses Bensons relaxation. How is it done?
A. Systematically tensing muscle groups from top to bottom for 5 seconds, and then releasing them
B. Concentrating on breathing without tensing the muscle, Letting go and repeating a word or sound after each
exhalation
C. Using a strong positive, feeling-rich statement about a desired change
D. Exercise combined with meditation to foster relaxation and mental alacrity
40. What type of relaxation technique does Lyza uses if a machine is showing her pulse rate, temperature
and muscle tension which she can visualize and assess?
A. Biofeedback
B. Massage
C. Autogenic training
D. Visualization and Imagery
41. This is also known as Self-suggestion or Self-hypnosis

A. Biofeedback
B. Meditation
C. Autogenic training
D. Visualization and Imagery
42. Which among these drugs is NOT an anxiolytic?
A. Valium
B. Ativan
C. Milltown
D. Luvox
43. Kenneth, 25 year old diagnosed with HIV felt that he had not lived up with Gods expectation. He fears
that in the course of his illness, God will be punitive and not be supportive. What kind of spiritual crisis is
Kenneth experiencing?
1. Spiritual Pain
2. Spiritual Anxiety
3. Spiritual Guilt
4. Spiritual Despair
A. 1,2
B. 2,3
C. 3,4
D. 1,4
44. Grace, believes that her relationship with God is broken. She tried to go to church to ask forgiveness
everyday to remedy her feelings. What kind of spiritual distress is Grace experiencing?
A. Spiritual Pain
B. Spiritual Alienation
C. Spiritual Guilt
D. Spiritual Despair
45. Remedios felt EMPTY She felt that she has already lost Gods favor and love because of her sins. This
is a type of what spiritual crisis?
A. Spiritual Anger
B. Spiritual Loss
C. Spiritual Despair
D. Spiritual Anxiety

46. Blake is working with a schizophrenic patient. He noticed that the client is agitated, pacing back and
forth, restless and experiencing Anxiety +3. Blake said You appear restless What therapeutic technique
did Blake used?
A. Offering general leads
B. Seeking clarification
C. Making observation
D. Encouraging description of perception
47. Ronny told Blake I see dead people. Blake responded, You see dead people? This exchange is an
example of what therapeutic communication technique?
A. Reflecting
B. Restating
C. Exploring
D. Seeking clarification
48. Ronny told Blake, Do you think Im crazy? Blake responded, Do you think youre crazy? Blake uses
what example of therapeutic communication?
A. Reflecting
B. Restating
C. Exploring
D. Seeking clarification
49. Myra, 21 year old nursing student has difficulty sleeping. She told Nurse Blake I really think a lot about
my ex-boyfriend recently. Blake told Myra And that causes you difficulty sleeping? Which therapeutic
technique is used in this situation?
A. Reflecting
B. Restating
C. Exploring
D. Seeking clarification
50. Myra told Blake I cannot sleep, I stay away all night. Blake told her You have difficulty sleeping? This
is what type of therapeutic communication technique?
A. Reflecting
B. Restating
C. Exploring
D. Seeking clarification

51. Myra said I saw my dead grandmother here at my bedside a while ago. Blake responded Really? That
is hard to believe. How do you feel about it? What technique did Blake used?
A. Disproving
B. Disagreeing
C. Voicing Doubt
D. Presenting Reality
52. Which of the following is a therapeutic communication in response to I am a GOD, bow before me or ill
summon the dreaded thunder to burn you and purge you to pieces!
A. You are not a GOD, you are Professor Tadle and you are a PE Teacher, not a Nurse. I am Glen, your nurse.
B. Oh hail GOD Tadle, everyone bow or face his wrath!
C. Hello Mr. Tadle, You are here in the hospital, I am your nurse and you are a patient here
D. How can you be a GOD Mr. Tadle? Can you tell me more about it?
53. Erik John Senna, Told Nurse Blake I dont want to that, I dont want that thing.. thats too painful!
Which of the following response is NON THERAPEUTIC?
A. This must be difficult for you, but I need to inject you this for your own good
B. You sound afraid
C. Are you telling me you dont want this injection?
D. Why are you so anxious? Please tell me more about your feelings Erik
54. Mr. Poncho was caught by the police because of his illegal activities. When he got home after paying for
the bail, he shouted at his son. What defense mechanism did Mr. Poncho used?
A. Restitution
B. Projection
C. Displacement
D. Undoing
55. Later that day, he bought his son ice cream and food. What defense mechanism is Mr. Poncho
unconsciously doing?
A. Restitution
B. Conversion
C. Redoing
D. Reaction formation
56. Crisis is a sudden event in ones life that disturbs a persons homeostasis. Which of the following is NOT
TRUE in crisis?

A. The person experiences heightened feeling of stress


B. Inability to function in the usual organized manner
C. Lasts for 4 months
D. Indicates unpleasant emotional feelings
57. Which of the following is a characteristic of crisis?
A. Lasts for an unlimited period of time
B. There is a triggering event
C. Situation is not dangerous to the person
D. Person totality is not involved
58. Levito Devin, the Italian prime minister, is due to retire next week. He feels depressed due to the
enormous loss of influence, power, fame and fortune. What type of crisis is Devin experiencing?
A. Situational
B. Maturational
C. Social
D. Phenomenal
59. Estrada, the Philippine president, has been unexpectedly impeached and was out of office before the
end of his term. He is in what type of crisis?
A. Situational
B. Maturational
C. Social
D. Phenomenal
60. The tsunami in Thailand and Indonesia took thousands of people and change million lives. The people
affected by the tsunami are saddened and do not know how to start all over again. What type of crisis is
this?
A. Situational
B. Maturational
C. Social
D. Phenomenal
61. Which of the following is the BEST goal for crisis intervention?
A. Bring back the client in the pre crisis state
B. Make sure that the client becomes better

C. Achieve independence
D. Provide alternate coping mechanism
62. What is the best intervention when the client has just experienced the crisis and still at the first phase of
the crisis?
A. Behavior therapy
B. Gestalt therapy
C. Cognitive therapy
D. Milieu Therapy
63. Therapeutic nurse client relationship is describes as follows
1. Based on friendship and mutual interest
2. It is a professional relationship
3. It is focused on helping the patient solve problems and achieve health-related goals
4. Maintained only as long as the patient requires professional help
A. 1,2,3
B. 1,2,4
C. 2,3,4
D. 1,3,4
64. The client is scheduled to have surgical removal of the tumor on her left breast. Which of the following
manifestation indicates that she is experiencing Mild Anxiety?
A. She has increased awareness of her environmental details
B. She focused on selected aspect of her illness
C. She experiences incongruence of action, thoughts and feelings
D. She experiences random motor activities
65. Which of the following nursing intervention would least likely be effective when dealing with a client with
aggressive behavior?
A. Approach him in a calm manner
B. Provide opportunities to express feelings
C. Maintain eye contact with the client
D. Isolate the client from others
66. Whitney, a patient of nurse Blake, verbalizes I have nothing, nothing nothing! Dont make me close
one more door, I dont wanna hurt anymore! Which of the following is the most appropriate response by
Blake?

A. Why are you singing?


B. What makes you say that?
C. Ofcourse you are everything!
D. What is that you said?
67. Whitney verbalizes that she is anxious that the diagnostic test might reveal laryngeal cancer. Which of
the following is the most appropriate nursing intervention?
A. Tell the client not to worry until the results are in
B. Ask the client to express feelings and concern
C. Reassure the client everything will be alright
D. Advice the client to divert his attention by watching television and reading newspapers
68. Considered as the most accurate expression of persons thought and feelings
A. Verbal communication
B. Non verbal communication
C. Written communication
D. Oral communication
69. Represents inner feeling that a person do not like talking about.
A. Overt communication
B. Covert communication
C. Verbal communication
D. Non verbal communication
70. Which of the following is NOT a characteristic of an effective Nurse-Client relationship?
A. Focused on the patient
B. Based on mutual trust
C. Conveys acceptance
D. Discourages emotional bond
71. A type of record wherein each person or department makes notation in separate records. A nurse will
use the nursing notes, the doctor will use the Physicians order sheet etc. Data is arranged according to
information source.
A. POMR
B. POR
C. Traditional
D. Resource oriented

72. Type of recording that integrates all data about the problem, gathered by members of the health team.
A. POMR
B. Traditional
C. Resource oriented
D. Source oriented
73. These are data that are monitored by using graphic charts or graphs that indicated the progression or
fluctuation of clients Temperature and Blood pressure.
A. Progress notes
B. Kardex
C. Flow chart
D. Flow sheet
74. Provides a concise method of organizing and recording data about the client. It is a series of flip cards
kept in portable file used in change of shift reports.
A. Kardex
B. Progress Notes
C. SOAPIE
D. Change of shift report
75. You are about to write an information on the Kardex. There are four (4) available writing instruments to
use. Which of the following should you use?
A. Mongol #2
B. Permanent Ink
C. A felt or fountain pen
D. Pilot Pentel Pen marker
76. The client has an allergy to Iodine-based dye. Where should you put this vital information in the clients
chart?
A. In the first page of the clients chart
B. At the last page of the clients chart
C. At the front metal plate of the chart
D. In the Kardex
77. Which of the following is NOT TRUE about the Kardex

A. It provides readily available information


B. It is a tool of end of shift reports
C. The primary basis of endorsement
D. Where Allergies information are written
78. Which of the following, if seen on the Nurses notes, violates characteristic of good recording?
A. The client has a blood pressure of 120/80, Temperature of 36.6 C, Pulse rate of 120 and Respiratory rate of 22
B. Ate 50% of food served
C. Refused administration of betaxolol
D. Visited and seen by Dr. Santiago
79. The physician ordered: Mannerix a.c , what does a.c means?
A. As desired
B. Before meals
C. After meals
D. Before bed time
80. The physician ordered, Maalox, 2 hours p.c, what does p.c means?
A. As desired
B. Before meals
C. After meals
D. Before bedtime
81. The physician ordered, Maxitrol, Od. What does Od means?
A. Left eye
B. Right eye
C. Both eye
D. Once a day
82. The physician ordered, Magnesium Hydroxide cc Aluminum Hydroxide. What does cc means?
A. without
B. with
C. one half
D. With one half dose
83. Physician ordered, Paracetamol tablet ss. What does ss means?

A. without
B. with
C. one half
D. With one half dose
84. Which of the following indicates that learning has been achieved?
A. Martin starts exercising every morning and eating a balance diet after you taught her mag HL tayo program
B. Donya Delilah has been able to repeat the steps of insulin administration after you taught it to her
C. Marsha said I understand after you a health teaching about family planning
D. John rated 100% on your given quiz about smoking and alcoholism
85. In his theory of learning as a BEHAVIORISM, he stated that transfer of knowledge occurs if a new
situation closely resembles an old one.
A. Bloom
B. Lewin
C. Thorndike
D. Skinner
86. Which of the following is TRUE with regards to learning?
A. Start from complex to simple
B. Goals should be hard to achieve so patient can strive to attain unrealistic goals
C. Visual learning is the best for every individual
D. Do not teach a client when he is in pain
87. According to Bloom, there are 3 domains in learning. Which of these domains is responsible for the
ability of Donya Delilah to inject insulin?
A. Cognitive
B. Affective
C. Psychomotor
D. Motivative
88. Which domains of learning is responsible for making John and Marsha understand the different kinds of
family planning methods?
A. Cognitive
B. Affective
C. Psychomotor
D. Motivative

89. Which of the following statement clearly defines therapeutic communication?


A. Therapeutic communication is an interaction process which is primarily directed by the nurse
B. It conveys feeling of warmth, acceptance and empathy from the nurse to a patient in relaxed atmosphere
C. Therapeutic communication is a reciprocal interaction based on trust and aimed at identifying patient needs and
developing mutual goals
D. Therapeutic communication is an assessment component of the nursing process
90. Which of the following concept is most important in establishing a therapeutic nurse patient
relationship?
A. The nurse must fully understand the patients feelings, perception and reactions before goals can be established
B. The nurse must be a role model for health fostering behavior
C. The nurse must recognize that the patient may manifest maladaptive behavior after illness
D. The nurse should understand that patients might test her before trust is established
91. Which of the following communication skill is most effective in dealing with covert communication?
A. Validation
B. Listening
C. Evaluation
D. Clarification
92. Which of the following are qualities of a good recording?
1. Brevity
2. Completeness and chronology
3. Appropriateness
4. Accuracy
A. 1,2
B. 3,4
C. 1,2,3
D. 1,2,3,4
93. All of the following chart entries are correct except
A. V/S 36.8 C,80,16,120/80
B. Complained of chest pain
C. Seems agitated
D. Able to ambulate without assistance

94. Which of the following teaching method is effective in client who needs to be educated about self
injection of insulin?
A. Detailed explanation
B. Demonstration
C. Use of pamphlets
D. Film showing
95. What is the most important characteristic of a nurse patient relationship?
A. It is growth facilitating
B. Based on mutual understanding
C. Fosters hope and confidence
D. Involves primarily emotional bond
96. Which of the following nursing intervention is needed before teaching a client post spleenectomy deep
breathing and coughing exercises?
A. Tell the patient that deep breathing and coughing exercises is needed to promote good breathing, circulation and
prevent complication
B. Tell the client that deep breathing and coughing exercises is needed to prevent Thrombophlebitis, hydrostatic
pneumonia and atelectasis
C. Medicate client for pain
D. Tell client that cooperation is vital to improve recovery
97. The client has an allergy with penicillin. What is the best way to communicate this information?
A. Place an allergy alert in the Kardex
B. Notify the attending physician
C. Write it on the patients chart
D. Take note when giving medications
98. An adult client is on extreme pain. He is moaning and grimacing. What is the best way to assess the
clients pain?
A. Perform physical assessment
B. Have the client rate his pain on the smiley pain rating scale
C. Active listening on what the patient says
D. Observe the clients behavior
99. Therapeutic communication begins with?

A. Knowing your client


B. Knowing yourself
C. Showing empathy
D. Encoding
100. The PCS gave new guidelines including leaflets to educate cancer patients. As a nurse, When using
materials like this, what is your responsibility?
A. Read it for the patient
B. Give it for the patient to read himself
C. Let the family member read the material for the patient
D. Read it yourself then, Have the client read the material

Answers and Rationale


Countercheck your answers to those below. Rationales are not given on this exam. If you have any disputes or
clarifications, please direct them to the comments section.
1. Answer: B. Vasodilatation
Sympathetic activation to the heart results in coronary vasodilation and increased coronary flow due to increased
metabolic activity (increased heart rate, contractility) despite direct vasoconstrictor effects of sympathetic activation
on the coronaries. This is termed functional sympatholysis.
2. Answer: D. Blood pressure will increase
Blood pressure elevation secondary to hypoglycemia has been demonstrated in human experimentation through the
activation of the sympathoadrenal system.
3. Answer: A. Increase heat, thereby produce abatement of phagocytosis
The inflammatory response is a defense mechanism that evolved in higher organisms to protect them from infection
and injury. Its purpose is to localize and eliminate the injurious agent and to remove damaged tissue components so
that the body can begin to heal. The response consists of changes in blood flow, an increase in permeability of
blood vessels, and the migration of fluid, proteins, and white blood cells (leukocytes) from the circulation to the site
of tissue damage. An inflammatory response that lasts only a few days is called acute inflammation, while a
response of longer duration is referred to as chronic inflammation.
4. Answer: C. Immediate Vasoconstriction

Inflammation can be divided into several phases. The earliest, gross event of an inflammatory response is
temporary vasoconstriction, i.e. narrowing of blood vessels caused by contraction of smooth muscle in the vessel
walls, which can be seen as blanching (whitening) of the skin.
5. Answer: C. The affected part will lose its normal function
A fifth consequence of inflammation is the loss of function of the inflamed area, a feature noted by German
pathologist Rudolf Virchow in the 19th century. Loss of function may result from pain that inhibits mobility or from
severe swelling that prevents movement in the area.
6. Answer: C. Purulent
Purulent effluent refers to a thick yellow, gray or green drainage that comes out of a wound when infection invades
the area. The fluid contains pathogenic microorganisms along with white cells, inflammatory cells and dead or dying
bacteria. When the wound is infected, the volume of the purulent exudate increases.
7. Answer: A. Redness on the affected area
Vasodilation occurs first at the arteriole level, progressing to the capillary level, and brings about a net increase in
the amount of blood present, causing the redness and heat of inflammation.
8. Answer: D. Monocytes
Monocytes and their macrophage and dendritic-cell progeny serve three main functions in the immune system.
These are phagocytosis, antigen presentation, and cytokine production. Phagocytosis is the process of uptake of
microbes and particles followed by digestion and destruction of this material. Monocytes can perform phagocytosis
using intermediary (opsonising) proteins such as antibodies or complement that coat the pathogen, as well as by
binding to the microbe directly via pattern-recognition receptors that recognize pathogens.
9. Answer: C. Eosinophil
It is known that eosinophils appear as an aftermath of anaphylaxis in sensitized tissues that are reexposed to
specific antigens.
10. Answer: A. Neutrophils
Activated neutrophils are capable of presenting antigens via MHCII, thereby stimulating T-cell activation and
proliferation.
11. Answer: C. Pavementation
Pavementation or the sticking of white blood cells to the linings of the finest blood vessels (capillaries) when
inflammation occurs is the first process that occurs in the inflammatory response after injury.

12. Answer: D. I dont know the answer, perhaps I can tell you after I find it out later
13. Answer: C. Third intention
Healing by third intention is a method of closing a grossly contaminated wound in which the wound is left open until
contamination has been markedly reduced and inflammation has subsided and then is closed by first intention. Also
called delayed primary closure.
14. Answer: A. First intention
Healing by first intention aka. primary wound healing or primary closure describes a wound closed by approximation
of wound margins or by placement of a graft or flap, or wounds created and closed in the operating room.
15. Answer: B. Second intention
Healing by second intention aka. secondary wound healing or spontaneous healing Describes a wound left open
and allowed to close by epithelialization and contraction.
16. Answer: B. High protein, High calorie with Vitamin A and C rich foods
Even if you do not currently have any aches or pains, adding these foods into your diet can help prepare your body
for healing if you should happen to get hurt.
17. Answer: A. It is a type of mechanical debridement using Wet dressing that is applied and left to dry to
remove dead tissues
This type of mechanical debridement is done by placing a wet (or moist) gauze dressing on your wound and
allowing it to dry, wound drainage and dead tissue can be removed when you take off the old dressing.
18. Answer: C. Compression of the local nerve endings by the edema fluids
Chemicals that stimulate nerve endings are released, making the area much more sensitive and the inflamed area is
likely to be painful, especially when touched.
19. Answer: B. 1,3,4
20. Answer: D. Smoke cigarette around 3:00 A.M
The phrase NPO After Midnight is one of the most common in medicine. It is present not only in physicians preoperative orders, but repeated by nurses, ward secretaries and dietary workers. The goal of NPO after midnight
was to ensure an empty stomach for the morning procedure.
21. Answer: B. Decrease Urine output

22. Answer: A. Directed towards helping an individual both physically and emotionally
Therapeutic nurse-patient communication helps nurses to build positive relationships with patients by showing
warmth, respect, and empathy. It also increases confidence of the nurse by enabling nurses to ask for support, open
up for feedbacks, and overcome anxiety. Therefore allows nurses to provide the best possible care for patients.
Effective nurse-healthcare provider communications assure optimal patient-centered care.
23. Answer: B. Peplau
Peplau published her Theory of Interpersonal Relations in 1952, and in 1968, interpersonal techniques became the
crux of psychiatric nursing. The nursing model identifies four sequential phases in the interpersonal relationship:
orientation, identification, exploitation, and resolution.
24. Answer: A. Pre Orientation
Pre-orientation phase begins when the nurse is assigned to the patient. It includes all that the nurse thinks and does
before interacting with the patient such as when the nurse review the clients medical records.
25. Answer: B. Orientation
Orientation Phase begins when the nurse and the patient meet for the patient. In this phase, parameters of the
relationship is done. Explanation of roles is also done during this phase which includes the responsibilities and
expectation of the patient and nurse, with the expectations of both parties of what they can and cant do. It is during
the orientation phase when the nurse begins to know the patient.
26. Answer: C. Working
The working phase is highly individualized. it is more structured than the orientation phase- meaning most of the
therapeutic work is done during this phase.
27. Answer: B. Positive regard
Unconditional positive regard often described as acceptance is the third core condition and this involves taking a
non-judgemental attitude towards the client accepting and respecting them for who and what they are.
28. Answer: B. Positive regard
Refer to #27
29. Answer: A. It is a nervous energy
Stress is a normal physical response to events that make you feel threatened or upset your balance in some way. A
nervous energy is more part of anxiety.

30. Answer: D. Martina is in Crisis


Martina is in Crisis. She feels sad and down during this tough time. She can have intense negative feelings for
weeks, months or more.
31. Answer: D. The source of dread or uneasiness is from a recognized entity
Anxiety is a general term for several disorders that cause nervousness, fear, apprehension, and worrying. These
disorders affect how we feel and behave, and they can manifest real physical symptoms. Mild anxiety is vague and
unsettling, while severe anxiety can be extremely debilitating, having a serious impact on daily life.
32. Answer: D. Panic
Panic is the highest level of anxiety and is associated with dread, terror and a sense of impending doom. You may
not be able to communicate, function or concentrate because you are unable to think rationally. You may start
uncontrollably pacing and become increasingly active without absolute purpose.
33. Answer: B. Moderate
At this level, you lose the broader picture and focus only on the cause of the anxiety. You may not be able to pay
attention like usual, and even though your perceptual field is narrowed, you are still able to solve a problem. Mild and
moderate levels of anxiety are considered normal.
34. Answer: B. Dilated pupils
35. Answer: B. When the client starts to have a narrow perceptual field and selective inattentiveness
36. Answer: C. The client asks a question
37. Answer: A. Offer choices
38. Answer: D. What makes you feel anxious?
39. Answer: B. Concentrating on breathing without tensing the muscle, Letting go and repeating a word or
sound after each exhalation
The Relaxation Response is essentially the opposite reaction to the fight or flight response. According to Dr.
Benson, using the Relaxation Response is beneficial as it counteracts the physiological effects of stress and the
fight or flight response.
40. Answer: A. Biofeedback

Biofeedback is a technique you can use to learn to control your bodys functions, such as your heart rate. With
biofeedback, youre connected to electrical sensors that help you receive information (feedback) about your body
(bio). This feedback helps you focus on making subtle changes in your body, such as relaxing certain muscles, to
achieve the results you want, such as reducing pain.
In essence, biofeedback gives you the power to use your thoughts to control your body, often to help with a health
condition or physical performance. Biofeedback is often used as a relaxation technique.
41. Answer: C. Autogenic training
Autogenic Therapy (AT) is a powerful mind and body technique involving simple relaxation and body awareness
exercises. These reduce the intensity of the bodys stress response, and replace it with a calmer physiological state
in which self-healing naturally begins to occur.
42. Answer: D. Luvox
Luvox (fluvoxamine) is a selective serotonin reuptake inhibitor (SSRI) antidepressant. Fluvoxamine affects chemicals
in the brain that may become unbalanced and cause obsessive-compulsive symptoms. It is frequently used along
with benzodiazepine anxiolytics in clinics.
43: Answer: B. 2,3
Spiritual anxiety is the expression of fear of Gods wrath and punishment; fear that God might not take care of one,
either immediately or in the future; and/or worry that God is displeased with ones behavior. Spiritual guilt is an
expression suggesting that one has failed to do the things which he should have done in life and/or done things
which were not pleasing to God; articulation of concerns about the kind of life one has lived. Spiritual pain is the
expression of discomfort of suffering relative to ones relationship with God, verbalization of feelings of having a void
or lack of spiritual fulfillment, and/or a lack of peace in terms of ones relationship to ones creator. And spiritual
despair is the expression suggesting that there is no hope of ever having a relationship with God or of pleasing Him
and/or a feeling that God no longer can or does care for one.
44. Answer: B. Spiritual Alienation
Spiritual alienation is the expressions of loneliness or the feeling that God seems very far away and remote from
ones everyday life, verbalization that one has to depend upon ones self in times of trial or need, and/or a negative
attitude toward receiving any comfort or help from God.
45. Answer: B. Spiritual Loss
Spiritual loss is the expression of feelings of having temporarily lost or terminated the love of God, fear that ones
relationship with God has been threatened, and/or a feeling of emptiness with regard to spiritual things.

46. Answer: C. Making observation


Making observation is a therapeutic observation that is verbalizing what is observed or perceived.
47. Answer: B. Restating
Restating lets client know whether an expressed statement has or has not been understood.
48. Answer: A. Reflecting
Reflecting directs questions or feelings back to client so that they may be recognized and accepted.
49. Answer: A. Reflecting
Reflecting is directing back to the patient questions, feelings, and ideas.
50. Answer: B. Restating
Restating is repeating the main idea expressed.
51. Answer: C. Voicing Doubt
Voicing doubt is expressing uncertainty as to the reality of clients perception.
52. Answer: C. Hello Mr. Tadle, You are here in the hospital, I am your nurse and you are a patient here
This is an example of presenting reality. It is clarifying misconceptions that client may be expressing.
53. Answer: D. Why are you so anxious? Please tell me more about your feelings Erik
Requesting an explanation is an ineffective behavior and response.
54. Answer: C. Displacement
Displacement is the redirecting of thoughts feelings and impulses directed at one person or object, but taken out
upon another person or object. People often use displacement when they cannot express their feelings in a safe
manner to the person they are directed at.
55. Answer: A. Restitution
Restitution is the mechanism of relieving the mind of a load of guilt by making up or reparation (paying up with
interest).
56. Answer: C. Lasts for 4 months

57: Answer: B. There is a triggering event


Crisis is any event that is, or is expected to lead to, an unstable and dangerous situation affecting an individual,
group, community, or whole society.
58. Answer: B. Maturational
Maturational crisis a life crisis in which usual coping mechanisms are inadequate in dealing with a stress common to
a particular stage in the life cycle or with stress caused by a transition from one stage to another.
59. Answer: A. Situational
Situational crisis is an unexpected crisis that arises suddenly in response to an external event or a conflict
concerning a specific circumstance.
60. Answer: C. Social
Social Crisis is the crisis which hampers the social life of an individual. It can include recession, World War, terrorism
etc., these are the major factor which is responsible for social crisis.
61. Answer: D. Provide alternate coping mechanism
One of the goals for crisis intervention is that the victim and the counselor begin to collaboratively generate and
explore alternatives for coping. Although this situation will be unlike any other experience before, the counselor
should assist the individual in looking at what has worked in the past for other situations; this is typically the most
difficult to achieve in crisis counseling.
62. Answer D. Milieu Therapy
The goal of milieu therapy is to manipulate the environment so that all aspects of the clients hospital experience are
considered therapeutic.
63. Answer: C. 2,3,4
A therapeutic nurse-client relationship is established for the benefit of the client. It includes nurses working with the
client to create goals directed at improving their health status. Goals are centered on the clients values, beliefs and
needs. A partnership is formed between nurse and client. The nurse empowers patient and families to get involved
in their health. This relationship has three phases, a beginning (first time contact/introduction), a middle (develop a
relationship to deliver care) and an end (the patient is no longer dependant on the nurse).
64. Answer: A. She has increased awareness of her environmental details

Mild anxiety is anxiety that is manageable without any additional techniques. Mild anxiety tends to be when you have
irritating symptoms that dont seem to go away, but otherwise dont control you.
65. Answer: B. Provide opportunities to express feelings
Providing opportunities for the client to express his or her feelings will likely trigger his or her aggressiveness more.
66. Answer: D. What is that you said?
Clarification should be sought at each step of the way. The patient is usually quite aware when he is not being
understood. Eventually he may cease trying to communicate. Peplau states that it is always possible to ask: What is
that you said?
67. Answer: B. Ask the client to express feelings and concern
Asking the client to verbalize what she or he perceives may relieve the tension the client is feeling and he or she
might be less likely to take action on ideas that are harmful or frightening.
68. Answer: B. Non verbal communication
Non verbal communication is considered as the most accurate expression of persons thought and feelings.
Nonverbal communication represents two-thirds of all communication.
69. Answer: B. Covert communication
Overt means done or shown openly or plainly apparent in the Oxford English dictionary. This can refer to all sorts
of actions which are done in plain sight or with clear manifestations. Covert, on the other hand, means the exact
opposite of overt not openly acknowledged or displayed.
70. Answer: D. Discourages emotional bond
Emotional support is important in order for a nurse-client relationship to be effective. It is giving and receiving
reassurance and encouragement done through understanding.
71. Answer: D. Resource oriented
72. Answer: A. POMR
Problem-oriented medical record (POMR) is a method of recording data about the health status of a patient in a
problem-solving system. The POMR preserves the data in an easily accessible way that encourages ongoing
assessment and revision of the health care plan by all members of the health care team.
73. Answer: D. Flow sheet

Flowsheet (in a patient record) is a graphic summary of several changing factors, especially the patients vital signs
or weight and the treatments and medications given.
74. Answer: A. Kardex
The Kardex is a trademark for a card-filing system that allows quick reference to the particular needs of each patient
for certain aspects of nursing care.
75. Answer: A. Mongol #2
Kardexes are a huge risk factor for med errors that is why they are written in pencil. They are not legal documents
and are discarded when the patient is transferred out. And they are only as good as the nurse responsible for
checking the orders
76. Answer: C. At the front metal plate of the chart
77. Answer: C. The primary basis of endorsement
Refer to #75.
78. Answer: A. The client has a blood pressure of 120/80, Temperature of 36.6 C Pulse rate of 120 and
Respiratory rate of 22
Shortcuts would be safe to use on records when commenting on blood pressure, temperature, pulse rate, and
respiratory rate respectively. Do not use an abbreviation unless you are sure that it is commonly understood and in
general use.
79. Answer: B. Before meals
80. Answer: C. After meals
81. Answer: D. Once a day
82. Answer: B. with
83. Answer: C. one half
84. Answer: A. Martin starts exercising every morning and eating a balance diet after you taught her mag HL
tayo program
Learning has been achieved by the patient if he or she applies and acts upon the teachings of the health care
provider or the nurse. Verbalizing such learning does not guarantee an effective nurse teaching.

85. Answer: C. Thorndike


After several experiments on animals and learning development, Thorndike posited that learning was actually merely
a change in behavior as a result of a consequence. Furthermore, if an action brought a reward, it was stamped into
the mind and available for recall later. These two suppositions together came to be known as the Law of Effect, and
now inform much of what we know about operant conditioning and behaviorism.
86. Answer: D. Do not teach a client when he is in pain
Pain tells us something is wrong. Pain does not suggest that the patient is ready to listen and perceive properly the
teaching that the nurse may impart.
87. Answer: C. Psychomotor
Blooms Taxonomy model is in three parts, or overlapping domains. Again, Bloom used rather academic language,
but the meanings are simple to understand: The learner should benefit from development of knowledge and intellect
(Cognitive Domain); attitude and beliefs (Affective Domain); and the ability to put physical and bodily skills into effect
to act (Psychomotor Domain).
88. Answer: A. Cognitive
Refer to #87.
89. Answer: C. Therapeutic communication is a reciprocal interaction based on trust and aimed at
identifying patient needs and developing mutual goals
Therapeutic communication is a process in which the nurse consciously influences a client or helps the client to a
better understanding through verbal or nonverbal communication. Therapeutic communication involves the use of
specific strategies that encourage the patient to express feelings and ideas and that convey acceptance and respect.
The goal of therapeutic communication is to increase self-worth or decrease psychological distress by collecting
information to determine the illness, assessing and modifying the behavior, and providing health education.
90. Answer: D. The nurse should understand that patients might test her before trust is established
A therapeutic nurse-patient relationship is defined as a helping relationship thats based on mutual trust and respect,
the nurturing of faith and hope, being sensitive to self and others, and assisting with the gratification of your patients
physical, emotional, and spiritual needs through your knowledge and skill.
91. Answer: A. Validation
92. Answer: D. 1,2,3,4

Documentation in nursing is also an integral part of providing quality and safe care to patients. Qualities of a good
recording include Brevity, Completeness and chronology, Appropriateness and Accuracy.
93. Answer: C. Seems agitated
Being accurate is very important. For example, do not use vague terms such as good urine output. How many ccs
are good? Chart the specific amount and be accurate on things that you see and observe.
94. Answer: B. Demonstration
In teaching through demonstration, patients are set up to potentially conceptualize material more effectively.
95. Answer: A. It is growth facilitating
You can help your patient achieve harmony in mind, body, and spirit when engaging in a therapeutic relationship
based on effective communication that incorporates caring behaviors. Its a win-win situation in which you and your
patient can experience growth by sharing the moment with each other.
96. Answer: A. Tell the patient that deep breathing and coughing exercises is needed to promote good
breathing, circulation and prevent complication
Deep breathing and coughing will decrease your risk for a lung infection. Take a deep breath and hold it for as long
as you can. Let the air out and then cough strongly. Deep breaths help open your airway. You may be given an
incentive spirometer to help you take deep breaths. Put the plastic piece in your mouth and take a slow, deep breath.
Then let the air out and cough. Repeat these steps 10 times every hour.
97. Answer: B. Notify the attending physician
98. Answer: C. Active listening on what the patient says
Assessment of the patient experiencing pain is the cornerstone to optimal pain management. However, the quality
and utility of any assessment tool is only as good as the clinicians ability to thoroughly focus on the patient. This
means listening empathically, believing and legitimizing the patients pain, and understanding, to the best of his or
her capability, what the patient may be experiencing. A health care professionals empathic understanding of the
patients pain experience and accompanying symptoms confirms that there is genuine interest in the patient as a
person. This can influence a positive pain management outcome.
99. Answer: B. Knowing yourself
It is important for the nurse to know herself to identify kinds of behavior of ideas that make her anxious and to seed
help for her problems. Otherwise, she is likely to add new problems to those with which the patient is already
struggling.

100. Answer: D. Read it yourself then, Have the client read the material
1. She is the first one to coin the term NURSING PROCESS. She introduced three (3) steps of nursing
process which are: Observation, Ministration and Validation.
A. Nightingale
B. Johnson
C. Rogers
D. Hall
2. The American Nurses Association formulated an innovation of the Nursing process. Today, how many
distinct steps are there in the nursing process?
A. APIE 4
B. ADPIE 5
C. ADOPIE 6
D. ADOPIER 7
3. They are the first ones to suggest a four (4) step nursing process called APIE or assessment, planning,
implementation, and evaluation.
1. Yura
2. Walsh
3. Roy
4. Knowles
A. 1,2
B. 1,3
C. 3,4
D. 2,3
4. Which characteristic of nursing process is responsible for proper utilization of human resources, time
and cost resources?
A. Organized and Systematic
B. Humanistic
C. Efficient
D. Effective
5. Which characteristic of nursing process addresses the INDIVIDUALIZED care a client must receive?

A. Organized and Systematic


B. Humanistic
C. Efficient
D. Effective
6. A characteristic of the nursing process that is essential to promote client satisfaction and progress. The
care should also be relevant with the clients needs.
A. Organized and Systematic
B. Humanistic
C. Efficient
D. Effective
7. Rhina, who has Menieres disease, said that her environment is moving. Which of the following is a valid
assessment?
1. Rhina is giving an objective data
2. Rhina is giving a subjective data
3. The source of the data is primary
4. The source of the data is secondary
A. 1,3
B. 2,3
C. 2.4
D. 1,4
8. Nurse Angela, observe Joel who is very apprehensive over the impending operation. The client is
experiencing dyspnea, diaphoresis and asks lots of questions. Angela made a diagnosis of ANXIETY R/T
INTRUSIVE PROCEDURE. This is what type of Nursing Diagnosis?
A. Actual
B. Probable
C. Possible
D. Risk
9. Nurse Angela diagnosed Mrs. Delgado, who have undergone a BKA. Her diagnosis is SELF ESTEEM
DISTURBANCE R/T CHANGE IN BODY IMAGE. Although the client has not yet seen her lost leg, Angela
already anticipated the diagnosis. This is what type of Diagnosis?
A. Actual
B. Probable

C. Possible
D. Risk
10. Nurse Angela is about to make a diagnosis but very unsure because the S/S the client is experiencing is
not specific with her diagnosis of POWERLESSNESS R/T DIFFICULTY ACCEPTING LOSS OF LOVED ONE.
She then focus on gathering data to refute or prove her diagnosis but her plans and interventions are
already ongoing for the diagnosis. Which type of Diagnosis is this?
A. Actual
B. Probable
C. Possible
D. Risk
11. Nurse Angela knew that Stephen Lee Mu Chin, has just undergone an operation with an incision near
the diaphragm. She knew that this will contribute to some complications later on. She then should develop
what type of Nursing diagnosis?
A. Actual
B. Probable
C. Possible
D. Risk
12. Which of the following Nursing diagnosis is INCORRECT?
A. Fluid volume deficit R/T Diarrhea
B. High risk for injury R/T Absence of side rails
C. Possible ineffective coping R/T Loss of loved one
D. Self esteem disturbance R/T Effects of surgical removal of the leg
13. Among the following statements, which should be given the HIGHEST priority?
A. Client is in extreme pain
B. Clients blood pressure is 60/40
C. Clients temperature is 40 deg. Centigrade
D. Client is cyanotic
14. Which of the following need is given a higher priority among others?
A. The client has attempted suicide and safety precaution is needed
B. The client has disturbance in his body image because of the recent operation
C. The client is depressed because her boyfriend left her all alone
D. The client is thirsty and dehydrated

15. Which of the following is TRUE with regards to Client Goals?


A. They are specific, measurable, attainable and time bounded
B. They are general and broadly stated
C. They should answer for WHO, WHAT ACTIONS, WHAT CIRCUMSTANCES, HOW WELL and WHEN.
D. Example is : After discharge planning, Client demonstrated the proper psychomotor skills for insulin injection.
16. Which of the following is a NOT a correct statement of an Outcome criteria?
A. Ambulates 30 feet with a cane before discharge
B. Discusses fears and concerns regarding the surgical procedure
C. Demonstrates proper coughing and breathing technique after a teaching session
D. Reestablishes a normal pattern of elimination
17. Which of the following is a OBJECTIVE data?
A. Dizziness
B. Chest pain
C. Anxiety
D. Blue nails
18. A patients chart is what type of data source?
A. Primary
B. Secondary
C. Tertiary
D. Can be A and B
19. All of the following are characteristic of the Nursing process except
A. Dynamic
B. Cyclical
C. Universal
D. Intrapersonal
20. Which of the following is true about the NURSING CARE PLAN?
A. It is nursing centered
B. Rationales are supported by interventions
C. Verbal
D. At least 2 goals are needed for every nursing diagnosis

21. A framework for health assessment that evaluates the effects of stressors to the mind, body and
environment in relation with the ability of the client to perform ADL.
A. Functional health framework
B. Head to toe framework
C. Body system framework
D. Cephalocaudal framework
22. Client has undergone Upper GI and Lower GI series. Which type of health assessment framework is
used in this situation?
A. Functional health framework
B. Head to toe framework
C. Body system framework
D. Cephalocaudal framework
23. Which of the following statement is true regarding temperature?
A. Oral temperature is more accurate than rectal temperature
B. The bulb used in Rectal temperature reading is pear shaped or round
C. The older the person, the higher his BMR
D. When the client is swimming, BMR Decreases
24. A type of heat loss that occurs when the heat is dissipated by air current
A. Convection
B. Conduction
C. Radiation
D. Evaporation
25. Which of the following is TRUE about temperature?
A. The highest temperature usually occurs later in a day, around 8 P.M to 12 M.N
B. The lowest temperature is usually in the Afternoon, Around 12 P.M
C. Thyroxin decreases body temperature
D. Elderly people are risk for hyperthermia due to the absence of fats, Decreased thermoregulatory control and
sedentary lifestyle.
26. Hyperpyrexia is a condition in which the temperature is greater than
A. 40 degree Celsius
B. 39 degree Celsius

C. 100 degree Fahrenheit


D. 105.8 degree Fahrenheit
27. Tympanic temperature is taken from John, A client who was brought recently into the ER due to frequent
barking cough. The temperature reads 37.9 Degrees Celsius. As a nurse, you conclude that this temperature
is
A. High
B. Low
C. At the low end of the normal range
D. At the high end of the normal range
28. John has a fever of 38.5 Deg. Celsius. It surges at around 40 Degrees and go back to 38.5 degrees 6
times today in a typical pattern. What kind of fever is John having?
A. Relapsing
B. Intermittent
C. Remittent
D. Constant
29. John has a fever of 39.5 degrees 2 days ago, But yesterday, he has a normal temperature of 36.5
degrees. Today, his temperature surges to 40 degrees. What type of fever is John having?
A. Relapsing
B. Intermittent
C. Remittent
D. Constant
30. Johns temperature 10 hours ago is a normal 36.5 degrees. 4 hours ago, He has a fever with a
temperature of 38.9 Degrees. Right now, his temperature is back to normal. Which of the following best
describe the fever john is having?
A. Relapsing
B. Intermittent
C. Remittent
D. Constant
31. The characteristic fever in Dengue Virus is characterized as:
A. Tricyclic
B. Bicyclic

C. Biphasic
D. Triphasic
32. When John has been given paracetamol, his fever was brought down dramatically from 40 degrees
Celsius to 36.7 degrees in a matter of 10 minutes. The nurse would assess this event as:
A. The goal of reducing johns fever has been met with full satisfaction of the outcome criteria
B. The desired goal has been partially met
C. The goal is not completely met
D. The goal has been met but not with the desired outcome criteria
33. What can you expect from Marianne, who is currently at the ONSET stage of fever?
A. Hot, flushed skin
B. Increase thirst
C. Convulsion
D. Pale,cold skin
34. Marianne is now at the Defervescence stage of the fever, which of the following is expected?
A. Delirium
B. Goose flesh
C. Cyanotic nail beds
D. Sweating
35. Considered as the most accessible and convenient method for temperature taking
A. Oral
B. Rectal
C. Tympanic
D. Axillary
36. Considered as Safest and most non invasive method of temperature taking
A. Oral
B. Rectal
C. Tympanic
D. Axillary
37. Which of the following is NOT a contraindication in taking ORAL temperature?

A. Quadriplegic
B. Presence of NGT
C. Dyspnea
D. Nausea and Vomitting
38. Which of the following is a contraindication in taking RECTAL temperature?
A. Unconscious
B. Neutropenic
C. NPO
D. Very young children
39. How long should the Rectal Thermometer be inserted to the clients anus?
A. 1 to 2 inches
B. .5 to 1.5 inches
C. 3 to 5 inches
D. 2 to 3 inches
40. In cleaning the thermometer after use, The direction of the cleaning to follow Medical Asepsis is :
A. From bulb to stem
B. From stem to bulb
C. From stem to stem
D. From bulb to bulb
41. How long should the thermometer stay in the Clients Axilla?
A. 3 minutes
B. 4 minutes
C. 7 minutes
D. 10 minutes
42. Which of the following statement is TRUE about pulse?
A. Young person have higher pulse than older persons
B. Males have higher pulse rate than females after puberty
C. Digitalis has a positive chronotropic effect
D. In lying position, Pulse rate is higher
43. The following are correct actions when taking radial pulse except:

A. Put the palms downward


B. Use the thumb to palpate the artery
C. Use two or three fingers to palpate the pulse at the inner wrist
D. Assess the pulse rate, rhythm, volume and bilateral quality
44. The difference between the systolic and diastolic pressure is termed as
A. Apical rate
B. Cardiac rate
C. Pulse deficit
D. Pulse pressure
45. Which of the following completely describes PULSUS PARADOXICUS?
A. A greater-than-normal increase in systolic blood pressure with inspiration
B. A greater-than-normal decrease in systolic blood pressure with inspiration
C. Pulse is paradoxically low when client is in standing position and high when supine.
D. Pulse is paradoxically high when client is in standing position and low when supine.
46. Which of the following is TRUE about respiration?
A. I:E 2:1
B. I:E : 4:3
C I:E 1:1
D. I:E 1:2
47. Contains the pneumotaxic and the apneustic centers
A. Medulla oblongata
B. Pons
C. Carotid bodies
D. Aortic bodies
48. Which of the following is responsible for deep and prolonged inspiration
A. Medulla oblongata
B. Pons
C. Carotid bodies
D. Aortic bodies
49. Which of the following is responsible for the rhythm and quality of breathing?

A. Medulla oblongata
B. Pons
C. Carotid bodies
D. Aortic bodies
50. The primary respiratory center
A. Medulla oblongata
B. Pons
C. Carotid bodies
D. Aortic bodies
51. Which of the following is TRUE about the mechanism of action of the Aortic and Carotid bodies?
A. If the BP is elevated, the RR increases
B. If the BP is elevated, the RR decreases
C. Elevated BP leads to Metabolic alkalosis
D. Low BP leads to Metabolic acidosis
52. All of the following factors correctly influence respiration except one. Which of the following is
incorrect?
A. Hydrocodone decreases RR
B. Stress increases RR
C. Increase temperature of the environment, Increase RR
D. Increase altitude, Increase RR
53. When does the heart receives blood from the coronary artery?
A. Systole
B. Diastole
C. When the valves opens
D. When the valves closes
54. Which of the following is more life threatening?
A. BP = 180/100
B. BP = 160/120
C. BP = 90/60
D. BP = 80/50
55. Refers to the pressure when the ventricles are at rest

A. Diastole
B. Systole
C. Preload
D. Pulse pressure
56. Which of the following is TRUE about the blood pressure determinants?
A. Hypervolemia lowers BP
B. Hypervolemia increases GFR
C. HCT of 70% might decrease or increase BP
D. Epinephrine decreases BP
57. Which of the following do not correctly correlates the increase BP of Ms. Aida, a 70 year old diabetic?
A. Females, after the age 65 tends to have lower BP than males
B. Disease process like Diabetes increase BP
C. BP is highest in the morning, and lowest during the night
D. Africans, have a greater risk of hypertension than Caucasian and Asians.
58. How many minutes are allowed to pass if the client had engaged in strenuous activities, smoked or
ingested caffeine before taking his/her BP?
A. 5
B. 10
C. 15
D. 30
59. Too narrow cuff will cause what change in the Clients BP?
A. True high reading
B. True low reading
C. False high reading
D. False low reading
60. Which is a preferable arm for BP taking?
A. An arm with the most contraptions
B. The left arm of the client with a CVA affecting the right brain
C. The right arm
D. The left arm
61. Which of the following is INCORRECT in assessing clients BP?

A. Read the mercury at the upper meniscus, preferably at the eye level to prevent error of parallax
B. Inflate and deflate slowly, 2-3 mmHg at a time
C. The sound heard during taking BP is known as KOROTKOFF sound
D. If the BP is taken on the left leg using the popliteal artery pressure, a BP of 160/80 is normal.
62. Which of the following is the correct interpretation of the ERROR OF PARALLAX
A. If the eye level is higher than the level of the meniscus, it will cause a false high reading
B. If the eye level is higher than the level of the meniscus, it will cause a false low reading
C. If the eye level is lower than the level of the meniscus, it will cause a false low reading
D. If the eye level is equal to that of the level of the upper meniscus, the reading is accurate
63. How many minute/s is/are allowed to pass before making a re-reading after the first one?
A. 1
B. 5
C. 15
D. 30
64. Which of the following is TRUE about the auscultation of blood pressure?
A. Pulse + 4 is considered as FULL
B. The bell of the stethoscope is use in auscultating BP
C. Sound produced by BP is considered as HIGH frequency sound
D. Pulse +1 is considered as NORMAL
65. In assessing the abdomen, Which of the following is the correct sequence of the physical assessment?
A. Inspection, Auscultation, Percussion, Palpation
B. Palpation, Auscultation, Percussion, Inspection
C. Inspection, Palpation, Auscultation, Percussion
D. Inspection, Auscultation, Palpation, Percussion
66. The sequence in examining the quadrants of the abdomen is:
A. RUQ, RLQ, LUQ, LLQ
B. RLQ, RUQ, LLQ, LUQ
C. RUQ, RLQ, LLQ, LUQ
D. RLQ, RUQ, LUQ, LLQ
67. In inspecting the abdomen, which of the following is NOT DONE?

A. Ask the client to void first


B. Knees and legs are straighten to relax the abdomen
C. The best position in assessing the abdomen is Dorsal recumbent
D. The knees and legs are externally rotated
68. Dr. House is about to conduct an ophthalmoscope examination. Which of the following, if done by a
nurse, is a correct preparation before the procedure?
A. Provide the necessary draping to ensure privacy
B. Open the windows, curtains and light to allow better illumination
C. Pour warm water over the ophthalmoscope to ensure comfort
D. Darken the room to provide better illumination
69. If the client is female, and the doctor is a male and the patient is about to undergo a vaginal and cervical
examination, why is it necessary to have a female nurse in attendance?
A. To ensure that the doctor performs the procedure safely
B. To assist the doctor
C. To assess the clients response to examination
D. To ensure that the procedure is done in an ethical manner
70. In palpating the clients breast, which of the following position is necessary for the patient to assume
before the start of the procedure?
A. Supine
B. Dorsal recumbent
C. Sitting
D. Lithotomy
71. When is the best time to collect urine specimen for routine urinalysis and C/S?
A. Early morning
B. Later afternoon
C. Midnight
D. Before breakfast
72. Which of the following is among an ideal way of collecting a urine specimen for culture and sensitivity?
A. Use a clean container
B. Discard the first flow of urine to ensure that the urine is not contaminated
C. Collect around 30-50 ml of urine
D. Add preservatives, refrigerate the specimen or add ice according to the agencys protocol

73. In a 24 hour urine specimen started Friday, 9:00 A.M, which of the following if done by a Nurse indicate a
NEED for further procedural debriefing?
A. The nurse ask the client to urinate at 9:00 A.M, Friday and she included the urine in the 24 hour urine specimen
B. The nurse discards the Friday 9:00 A M urine of the client
C. The nurse included the Saturday 9:00 A.M urine of the client to the specimen collection
D. The nurse added preservatives as per protocol and refrigerates the specimen
74. This specimen is required to assess glucose levels and for the presence of albumin the the urine
A. Midstream clean catch urine
B. 24 hours urine collection
C. Postprandial urine collection
D. Second voided urine
75. When should the client test his blood sugar levels for greater accuracy?
A. During meals
B. In between meals
C. Before meals
D. 2 Hours after meals
76. In collecting a urine from a catheterized patient, Which of the following statement indicates an accurate
performance of the procedure?
A. Clamp above the port for 30 to 60 minutes before drawing the urine from the port
B. Clamp below the port for 30 to 60 minutes before drawing the urine from the port
C. Clamp above the port for 5 to 10 minutes before drawing the urine from the port
D. Clamp below the port for 5 to 10 minutes before drawing the urine from the port
77. A community health nurse should be resourceful and meet the needs of the client. A villager ask him,
Can you test my urine for glucose? Which of the following technique allows the nurse to test a clients
urine for glucose without the need for intricate instruments.
A. Acetic Acid test
B. Nitrazine paper test
C. Benedicts test
D. Litmus paper test
78. A community health nurse is assessing clients urine using the Acetic Acid solution. Which of the
following, if done by a nurse, indicates lack of correct knowledge with the procedure?

A. The nurse added the Urine as the 2/3 part of the solution
B. The nurse heats the test tube after adding 1/3 part acetic acid
C. The nurse heats the test tube after adding 2/3 part of Urine
D. The nurse determines abnormal result if she noticed that the test tube becomes cloudy
79. Which of the following is incorrect with regards to proper urine testing using Benedicts Solution?
A. Heat around 5ml of Benedicts solution together with the urine in a test tube
B. Add 8 to 10 drops of urine
C. Heat the Benedicts solution without the urine to check if the solution is contaminated
D. If the color remains BLUE, the result is POSITIVE
80. +++ Positive result after Benedicts test is depicted by what color?
A. Blue
B. Green
C. Yellow
D. Orange
81. Clinitest is used in testing the urine of a client for glucose. Which of the following, if committed by a
nurse, indicates error?
A. Specimen is collected after meals
B. The nurse puts 1 clinitest tablet into a test tube
C. She added 5 drops of urine and 10 drops of water
D. If the color becomes orange or red, It is considered postitive
82. Which of the following nursing intervention is important for a client scheduled to have a Guaiac Test?
A. Avoid turnips, radish and horseradish 3 days before procedure
B. Continue iron preparation to prevent further loss of Iron
C. Do not eat read meat 12 hours before procedure
D. Encourage caffeine and dark colored foods to produce accurate results
83. In collecting a routine specimen for fecalysis, which of the following, if done by a nurse, indicates
inadequate knowledge and skills about the procedure?
A. The nurse scoop the specimen specifically at the site with blood and mucus
B. She took around 1 inch of specimen or a teaspoonful
C. Ask the client to call her for the specimen after the client wiped off his anus with a tissue
D. Ask the client to defecate in a bedpan, Secure a sterile container

84. In a routine sputum analysis, which of the following indicates proper nursing action before sputum
collection?
A. Secure a clean container
B. Discard the container if the outside becomes contaminated with the sputum
C. Rinse the clients mouth with Listerine after collection
D. Tell the client that 4 tablespoon of sputum is needed for each specimen for a routine sputum analysis
85. Who collects blood specimen?
A. The nurse
B. Medical technologist
C. Physician
D. Physical therapist
86. David, 68 year old male client is scheduled for Serum Lipid analysis. Which of the following health
teaching is important to ensure accurate reading?
A. Tell the patient to eat fatty meals 3 days prior to the procedure
B. NPO for 12 hours pre procedure
C. Ask the client to drink 1 glass of water 1 hour prior to the procedure
D. Tell the client that the normal serum lipase level is 50 to 140 U/L
87. The primary factor responsible for body heat production is the
A. Metabolism
B. Release of thyroxin
C. Muscle activity
D. Stress
88. The heat regulating center is found in the
A. Medulla oblongata
B. Thalamus
C. Hypothalamus
D. Pons
89. A process of heat loss which involves the transfer of heat from one surface to another is
A. Radiation
B. Conduction

C. Convection
D. Evaporation
90. Which of the following is a primary factor that affects the BP?
A. Obesity
B. Age
C. Stress
D. Gender
91. The following are social data about the client except
A. Patients lifestyle
B. Religious practices
C. Family home situation
D. Usual health status
92. The best position for any procedure that involves vaginal and cervical examination is
A. Dorsal recumbent
B. Side lying
C. Supine
D. Lithotomy
93. Measure the leg circumference of a client with bipedal edema is best done in what position?
A. Dorsal recumbent
B. Sitting
C. Standing
D. Supine
94. In palpating the clients abdomen, Which of the following is the best position for the client to assume?
A. Dorsal recumbent
B. Side lying
C. Supine
D. Lithotomy
95. Rectal examination is done with a client in what position?
A. Dorsal recumbent
B. Sims position

C. Supine
D. Lithotomy
96. Which of the following is a correct nursing action when collecting urine specimen from a client with an
Indwelling catheter?
A. Collect urine specimen from the drainage bag
B. Detach catheter from the connecting tube and draw the specimen from the port
C. Use sterile syringe to aspirate urine specimen from the drainage port
D. Insert the syringe straight to the port to allow self sealing of the port
97. Which of the following is inappropriate in collecting mid stream clean catch urine specimen for urine
analysis?
A. Collect early in the morning, First voided specimen
B. Do perineal care before specimen collection
C. Collect 5 to 10 ml for urine
D. Discard the first flow of the urine
98. When palpating the clients neck for lymphadenopathy, where should the nurse position himself?
A. At the clients back
B. At the clients right side
C. At the clients left side
D. In front of a sitting client
99. Which of the following is the best position for the client to assume if the back is to be examined by the
nurse?
A. Standing
B. Sitting
C. Side lying
D. Prone
100. In assessing the clients chest, which position best show chest expansion as well as its movements?
A. Sitting
B. Prone
C. Sidelying
D. Supine

Answers

Here are the answers for this examination. Countercheck your answers to those below. If you have any disputes or
clarifications, please direct them to the comments section.
1. Answer: D. Hall
The term Nursing Process was first used by Lydia E. Hall where she introduced the 3 steps: observation,
administration of care and validation.Florence Nightingale introduced the Environmental Theory. Dorothy Johnson is
known for her Behavioral Systems Model while Martha Rogers introduced the Theory of Unitary Human Beings.
2. Answer: C. ADOPIE 6
ADOPIE stands for: Assessment, Diagnosis, Outcome Identification, Planning, Intervention and Evaluation.
3. Answer: A. 1,2
Helen Yura and Mary B. Walsh are the first ones to suggest a four step nursing process.
4. Answer: C. Efficient
Other than being efficient, a nursing process should also be effective in planning care, and utilization of resources.
5. Answer: B. Humanistic
In the humanistic approach, a plan to care is developed and implemented by taking into consideration the unique
needs of the individual client. It also states that no two person has the same health needs even if they have the
same health condition or illness.
6. Answer: D. Effective
See rationale for #4.
7. Answer: B. 2,3
The data is subjective and primary since it was said by the client herself. Objective data are observable by the
nurse.
8. Answer: A. Actual
An actual nursing diagnosis describes a clinical judgement that the nurse has validated because of the presence of
major defining characteristics.
9. Answer: D. Risk

A risk nursing diagnosis describes a clinical judgement that an individual or group is more vulnerable to develop the
problem than others in the same or a similar situation because of risk factors.
10. Answer: C. Possible
Possible nursing diagnosis is not a type of diagnosis as are actual, risk, and syndrome. These are the diagnostician
option to indicate that some data are present to confirm a diagnosis but are insufficient at this time.
11. Answer: D. Risk
See rationale for #9.
12. Answer: B. High risk for injury R/T Absence of side rails
13. Answer: D. Client is cyanotic
Cyanosis (cyanotic) is the appearance of a blue or purple coloration of the skin or mucous membranes due to the
tissues near the skin surface having low oxygen saturation. Following the principles of Airway, Breathing, and
Circulation, the first priority should be option D.
14. Answer: D. The client is thirsty and dehydrated
Based on Abraham Maslows Hierarchy of Needs, physiological needs should be prioritized first. The other options
are not physiological needs.
15. Answer: B. They are general and broadly stated
16. Answer: D. Reestablishes a normal pattern of elimination
17. Answer: D. Blue nails
18. Answer: B. Secondary
19. Answer: D. Intrapersonal
20. Answer: A. It is nursing centered
21. Answer: A. Functional health framework
22. Answer: C. Body system framework
23. Answer: B. The bulb used in Rectal temperature reading is pear shaped or round
24. Answer: A. Convection

25. Answer: A. The highest temperature usually occurs later in a day, around 8 P.M to 12 M.N
26. Answer: D. 105.8 degree Fahrenheit
27. Answer: D. At the high end of the normal range
28. Answer: C. Remittent
29. Answer: A. Relapsing
30. Answer: B. Intermittent
31. Answer: C. Biphasic
32. Answer: D. The goal has been met but not with the desired outcome criteria
33. Answer: D. Pale,cold skin
34. Answer: D. Sweating
35. Answer: A. Oral
36. Answer: D. Axillary
37. Answer: A. Quadriplegic
38. Answer: B. Neutropenic
39. Answer: B. .5 to 1.5 inches
40. Answer: B. From stem to bulb
41. Answer: C. 7 minutes
42. Answer: A. Young person have higher pulse than older persons
43. Answer: B. Use the thumb to palpate the artery
44. Answer: D. Pulse pressure
45. Answer: B. A greater-than-normal decrease in systolic blood pressure with inspiration
46. Answer: D. I:E 1:2

47. Answer: B. Pons


48. Answer: B. Pons
49. Answer: B. Pons
50. Answer: A. Medulla oblongata
51. Answer: B. If the BP is elevated, the RR decreases
52. Answer: C. Increase temperature of the environment, Increase RR
53. Answer: B. Diastole
54. Answer: B. BP = 160/120
55. Answer: A. Diastole
56. Answer: D. Epinephrine decreases BP
57. Answer: A. Females, after the age 65 tends to have lower BP than males
58. Answer: D. 30
59. Answer: C. False high reading
60. Answer: D. The left arm
61. Answer: A. Read the mercury at the upper meniscus, preferably at the eye level to prevent error of
parallax
62. Answer: B. If the eye level is higher than the level of the meniscus, it will cause a false low reading
63. Answer: A. 1
64. Answer: B. The bell of the stethoscope is use in auscultating BP
65. Answer: A. Inspection, Auscultation, Percussion, Palpation
66. Answer: D. RLQ,RUQ,LUQ,LLQ
67. Answer: B. Knees and legs are straighten to relax the abdomen
68. Answer: D. Darken the room to provide better illumination

69. Answer: D. To ensure that the procedure is done in an ethical manner


70. Answer: A. Supine
71. Answer: A. Early morning
72. Answer: B. Discard the first flow of urine to ensure that the urine is not contaminated
73. Answer: A. The nurse ask the client to urinate at 9:00 A.M, Friday and she included the urine in the 24
hour urine specimen
74. Answer: D. Second voided urine
75. Answer: C. Before meals
76. Answer: B. Clamp below the port for 30 to 60 minutes before drawing the urine from the port
77. Answer: C. Benedicts test
78. Answer: B. The nurse heats the test tube after adding 1/3 part acetic acid
79. Answer: D. If the color remains BLUE, the result is POSITIVE
80. Answer: D. Orange
81. Answer: Specimen is collected after meals
82. Answer: A. Avoid turnips, radish and horseradish 3 days before procedure
83. Answer: C. Ask the client to call her for the specimen after the client wiped off his anus with a tissue
84. Answer: C. Rinse the clients mouth with Listerine after collection
85. Answer: B. Medical technologist
86. Answer: B. NPO for 12 hours pre procedure
87. Answer: A. Metabolism
88. Answer: C. Hypothalamus
89. Answer: B. Conduction
90. Answer: C. Stress

91. Answer: A. Patients lifestyle


92. Answer: D. Lithotomy
93. Answer: A. Dorsal recumbent
94. Answer: A. Dorsal recumbent
95. Answer: B. Sims position
96. Answer: C. Use sterile syringe to aspirate urine specimen from the drainage port
97. Answer: C. Collect 5 to 10 ml for urine
98. Answer: A. At the clients back
99. Answer: A. Standing
100. Answer: A. Sitting
1. Which one of the following is NOT a function of the Upper airway?
A. For clearance mechanism such as coughing
B. Transport gases to the lower airways
C. Warming, Filtration and Humidification of inspired air
D. Protect the lower airway from foreign mater
2. It is the hair that lines the vestibule which function as a filtering mechanism for foreign objects
A. Cilia
B. Nares
C. Carina
D. Vibrissae
3. This is the paranasal sinus found between the eyes and the nose that extends backward into the skull
A. Ehtmoid
B. Sphenoid
C. Maxillary
D. Frontal
4. Which paranasal sinus is found over the eyebrow?

A. Ehtmoid
B. Sphenoid
C. Maxillary
D. Frontal
5. Gene De Vonne Katrouchuacheulujiki wants to change her surname to something shorter, The court
denied her request which depresses her and find herself binge eating. She accidentally aspirate a large
piece of nut and it passes the carina. Probabilty wise, Where will the nut go?
A. Right main stem bronchus
B. Left main stem bronchus
C. Be dislodged in between the carina
D. Be blocked by the closed epiglottis
6. Which cell secretes mucus that help protect the lungs by trapping debris in the respiratory tract?
A. Type I pneumocytes
B. Type II pneumocytes
C. Goblet cells
D. Adipose cells
7. How many lobes are there in the RIGHT LUNG?
A. One
B. Two
C. Three
D. Four
8. The presence of the liver causes which anatomical difference of the Kidneys and the Lungs?
A. Left kidney slightly lower, Left lung slightly shorter
B. Left kidney slightly higher, Left lung slightly shorter
C. Right kidney lower, Right lung shorter
D. Right kidney higher, Right lung shorter
9. Surfactant is produced by what cells in the alveoli?
A. Type I pneumocytes
B. Type II pneumocytes
C. Goblet cells
D. Adipose cells

10. The normal L:S Ratio to consider the newborn baby viable is
A. 1:2
B. 2:1
C. 3:1
D. 1:3
11. Refers to the extra air that can be inhaled beyond the normal tidal volume
A. Inspiratory reserve volume
B. Expiratory reserve volume
C. Functional residual capacity
D. Residual volume
12. This is the amount of air remained in the lungs after a forceful expiration
A. Inspiratory reserve volume
B. Expiratory reserve volume
C. Functional residual capacity
D. Residual volume
13. Casssandra, A 22 year old grade Agnostic, Asked you, how many spikes of bones are there in my ribs?
Your best response is which of the following?
A. We have 13 pairs of ribs Cassandra
B. We have 12 pairs of ribs Cassandra
C. Humans have 16 pairs of ribs, and that was noted by Vesalius in 1543
D. Humans have 8 pairs of ribs. 4 of which are floating
14. Which of the following is considered as the main muscle of respiration?
A. Lungs
B. Intercostal Muscles
C. Diaphragm
D. Pectoralis major
15. Cassandra asked you : How many air is there in the oxygen and how many does human requires? Which
of the following is the best response :
A. God is good, Man requires 21% of oxygen and we have 21% available in our air
B. Man requires 16% of oxygen and we have 35% available in our air

C. Man requires 10% of oxygen and we have 50% available in our air
D. Human requires 21% of oxygen and we have 21% available in our air
16. Which of the following is TRUE about Expiration?
A. A passive process
B. The length of which is half of the length of Inspiration
C. Stridor is commonly heard during expiration
D. Requires energy to be carried out
17. Which of the following is TRUE in postural drainage?
A. Patient assumes position for 10 to 15 minutes
B. Should last only for 60 minutes
C. Done best P.C
D. An independent nursing action
18. All but one of the following is a purpose of steam inhalation
A. Mucolytic
B. Warm and humidify air
C. Administer medications
D. Promote bronchoconstriction
19. Which of the following is NOT TRUE in steam inhalation?
A. It is a dependent nursing action
B. Spout is put 12-18 inches away from the nose
C. Render steam inhalation for atleast 60 minutes
D. Cover the clients eye with wash cloth to prevent irritation
20. When should a nurse suction a client?
A. As desired
B. As needed
C. Every 1 hour
D. Every 4 hours
21. Ernest Arnold Hamilton, a 60 year old American client was mobbed by teen gangsters near New york,
Cubao. He was rushed to John John Hopio Medical Center and was Unconscious. You are his nurse and
you are to suction his secretions. In which position should you place Mr. Hamilton?

A. High fowlers
B. Semi fowlers
C. Prone
D. Side lying
22. You are about to set the suction pressure to be used to Mr. Hamilton. You are using a Wall unit suction
machine. How much pressure should you set the valve before suctioning Mr. Hamilton?
A. 50-95 mmHg
B. 200-350 mmHg
C. 100-120 mmHg
D. 10-15 mmHg
23. The wall unit is not functioning; You then try to use the portable suction equipment available. How much
pressure of suction equipment is needed to prevent trauma to mucus membrane and air ways in case of
portable suction units?
A. 2-5 mmHg
B. 5-10 mmHg
C. 10-15 mmHg
D. 15-25 mmHg
24. There are four catheter sizes available for use, which one of these should you use for Mr. Hamilton?
A. Fr. 18
B. Fr. 12
C. Fr. 10
D. Fr. 5
25. Which of the following, if done by the nurse, indicates incompetence during suctioning an unconscious
client?
A. Measure the length of the suction catheter to be inserted by measuring from the tip of the nose, to the earlobe, to
the xiphoid process
B. Use KY Jelly if suctioning nasopharyngeal secretion
C. The maximum time of suctioning should not exceed 15 seconds
D. Allow 30 seconds interval between suctioning
26. Which of the following is the initial sign of hypoxemia in an adult client?
1. Tachypnea
2. Tachycardia

3. Cyanosis
4. Pallor
5. Irritability
6. Flaring of Nares
A. 1,2
B. 2,5
C. 2,6
D. 3,4
27. Which method of oxygenation least likely produces anxiety and apprehension?
A. Nasal Cannula
B. Simple Face mask
C. Non Rebreather mask
D. Partial Rebreather mask
28. Which of the following oxygen delivery method can deliver 100% Oxygen at 15 LPM?
A. Nasal Cannula
B. Simple Face mask
C. Non Rebreather mask
D. Partial Rebreather mask
29. Which of the following is not true about OXYGEN?
A. Oxygen is odorless, tasteless and colorless gas.
B. Oxygen can irritate mucus membrane
C. Oxygen supports combustion
D. Excessive oxygen administration results in respiratory acidosis
30. Roberto San Andres, A new nurse in the hospital is about to administer oxygen on patient with
Respiratory distress. As his senior nurse, you should intervene if Roberto will:
A. Uses venture mask in oxygen administration
B. Put a non rebreather mask in the patient before opening the oxygen source
C. Use a partial rebreather mask to deliver oxygen
D. Check for the doctors order for Oxygen administration
31. Which of the following will alert the nurse as an early sign of hypoxia?

A. Client is tired and dyspneic


B. The client is coughing out blood
C. The clients heart rate is 50 BPM
D. Client is frequently turning from side to side
32. Miguelito de balboa, An OFW presents at the admission with an A:P Diameter ratio of 2:1, Which of the
following associated finding should the nurse expect?
A. Pancytopenia
B. Anemia
C. Fingers are Club-like
D. Hematocrit of client is decreased
33. The best method of oxygen administration for client with COPD uses:
A. Cannula
B. Simple Face mask
C. Non rebreather mask
D. Venturi mask
34. Mang Dagul, a 50 year old chronic smoker was brought to the E.R because of difficulty in breathing.
Pleural effusion was the diagnosis and CTT was ordered. What does C.T.T Stands for?
A. Chest tube thoracotomy
B. Chest tube thoracostomy
C. Closed tube thoracotomy
D. Closed tube thoracostmy
35. Where will the CTT be inserted if we are to drain fluids accumulated in Mang daguls pleura?
A. 2nd ICS
B. 4th ICS
C. 5th ICS
D. 8th ICS
36. There is a continuous bubbling in the water sealed drainage system with suction. And oscillation is
observed. As a nurse, what should you do?
A. Consider this as normal findings
B. Notify the physician
C. Check for tube leak
D. Prepare a petrolatum gauze dressing

37. Which of the following is true about nutrition?


A. It is the process in which food are broken down, for the body to use in growth and development
B. It is a process in which digested proteins, fats, minerals, vitamins and carbohydrates are transported into the
circulation
C. It is a chemical process that occurs in the cell that allows for energy production, energy use, growth and tissue
repair
D. It is the study of nutrients and the process in which they are use by the body
38. The majority of the digestion processes take place in the
A. Mouth
B. Small intestine
C. Large intestine
D. Stomach
39. All of the following is true about digestion that occurs in the Mouth except
A. It is where the digestion process starts
B. Mechanical digestion is brought about by mastication
C. The action of ptyalin or the salivary trypsin breaks down starches into maltose
D. Deglutition occurs after food is broken down into small pieces and well mixed with saliva
40. Which of the following foods lowers the cardiac sphincter pressure?
A. Roast beef, Steamed cauliflower and Rice
B. Orange juice, Non fat milk, Dry crackers
C. Decaffeinated coffee, Sky flakes crackers, Suman
D. Coffee with coffee mate, Bacon and Egg
41. Where does the digestion of carbohydrates start?
A. Mouth
B. Esophagus
C. Small intestine
D. Stomach
42. Protein digestion begins where?
A. Mouth
B. Esophagus

C. Small intestine
D. Stomach
43. All but one is true about digestion that occurs in the Stomach
A. Carbohydrates are the fastest to be digested, in about an hour
B. Fat is the slowest to be digested, in about 5 hours
C. HCl inhibits absorption of Calcium in the gastric mucosa
D. HCl converts pepsinogen to pepsin, which starts the complex process of protein digestion
44. Which of the following is NOT an enzyme secreted by the small intestine?
A. Sucrase
B. Enterokinase
C. Amylase
D. Enterokinase
45. The hormone secreted by the Small intestine that stimulates the production of pancreatic juice which
primarily aids in buffering the acidic bolus passed by the Stomach
A. Enterogastrone
B. Ghrelin
C. Pancreozymin
D. Enterokinase
46. When the duodenal enzyme sucrase acts on SUCROSE, which 2 monosaccharides are formed?
A. Galactose + Galactose
B. Glucose + Fructose
C. Glucose + Galactose
D. Fructose + Fructose
47. This is the enzyme secreted by the pancrease that completes the protein digestion
A. Trypsin
B. Enterokinase
C. Enterogastrone
D. Amylase
48. The end product of protein digestion or the Building blocks of Protein is what we call

A. Nucleotides
B. Fatty acids
C. Glucose
D. Amino Acids
49. Enzyme secreted by the small intestine after it detects a bolus of fatty food. This will contract the
gallbladder to secrete bile and relax the sphincter of Oddi to aid in the emulsification of fats and its
digestion.
A. Lipase
B. Amylase
C. Cholecystokinin
D. Pancreozymin
50. Which of the following is not true about the Large Intestine?
A. It absorbs around 1 L of water making the feces around 75% water and 25% solid
B. The stool formed in the transverse colon is not yet well formed
C. It is a sterile body cavity
D. It is called large intestine because it is longer than the small intestine
51. This is the amount of heat required to raise the temperature of 1 kg water to 1 degree Celsius
A. Calorie
B. Joules
C. Metabolism
D. Basal metabolic rate
52. Assuming a cup of rice provides 50 grams of carbohydrates. How many calories are there in that cup of
rice?
A. 150 calories
B. 200 calories
C. 250 calories
D. 400 calories
53. An average adult Filipino male requires how many calories in a day?
A. 1,000 calories
B. 1,500 calories
C. 2,000 calories
D. 2,500 calories

54. Which of the following is true about an individuals caloric needs?


A. All individual have the same caloric needs
B. Females in general have higher BMR and therefore, require more calories
C. During cold weather, people need more calories due to increase BMR
D. Dinner should be the heaviest meal of the day
55. Among the following people, who requires the greatest caloric intake?
A. An individual in a long state of gluconeogenesis
B. An individual in a long state of glycogenolysis
C. A pregnant individual
D. An adolescent with a BMI of 25
56. Which nutrient deficiency is associated with the development of Pellagra, Dermatitis and Diarrhea?
A. Vitamin B1
B. Vitamin B2
C. Vitamin B3
D. Vitamin B6
57. Which Vitamin is not given in conjunction with the intake of LEVODOPA in cases of Parkinsons Disease
due to the fact that levodopa increases its level in the body?
A. Vitamin B1
B. Vitamin B2
C. Vitamin B3
D. Vitamin B6
58. A vitamin taken in conjunction with ISONIAZID to prevent peripheral neuritis
A. Vitamin B1
B. Vitamin B2
C. Vitamin B3
D. Vitamin B6
59. The inflammation of the Lips, Palate and Tongue is associated in the deficiency of this vitamin
A. Vitamin B1
B. Vitamin B2
C. Vitamin B3
D. Vitamin B6

60. Beri beri is caused by the deficiency of which Vitamin?


A. Vitamin B1
B. Vitamin B2
C. Vitamin B3
D. Vitamin C
61. Which of the following is the best source of Vitamin E?
A. Green leafy vegetables
B. Vegetable oil
C. Fortified Milk
D. Fish liver oil
62. Among the following foods, which food should you emphasize giving on an Alcoholic client?
A. Pork liver and organ meats, Pork
B. Red meat, Eggs and Dairy products
C. Green leafy vegetables, Yellow vegetables, Cantaloupe and Dairy products
D. Chicken, Peanuts, Bananas, Wheat germs and yeasts
63. Which food group should you emphasize giving on a pregnant mother in first trimester to prevent neural
tube defects?
A. Broccoli, Guava, Citrus fruits, Tomatoes
B. Butter, Sardines, Tuna, Salmon, Egg yolk
C. Wheat germ, Vegetable Oil, soybeans, corn, peanuts
D. Organ meats, Green leafy vegetables, Liver, Eggs
64. A client taking Coumadin is to be educated on his diet. As a nurse, which of the following food should
you instruct the client to avoid?
A. Spinach, Green leafy vegetables, Cabbage, Liver
B. Salmon, Sardines, Tuna
C. Butter, Egg yolk, breakfast cereals
D. Banana, Yeast, Wheat germ, Chicken
65. Vitamin E plus this mineral works as one of the best anti oxidant in the body according to the latest
research. They are combined with 5 Alpha reductase inhibitor to reduce the risk of acquiring prostate
cancer

A. Zinc
B. Iron
C. Selenium
D. Vanadium
66. Incident of prostate cancer is found to have been reduced on a population exposed in tolerable amount
of sunlight. Which vitamin is associated with this phenomenon?
A. Vitamin A
B. Vitamin B
C. Vitamin C
D. Vitamin D
67. Micronutrients are those nutrients needed by the body in a very minute amount. Which of the following
vitamin is considered as a MICRONUTRIENT
A. Phosphorous
B. Iron
C. Calcium
D. Sodium
68. Deficiency of this mineral results in tetany, osteomalacia, osteoporosis and rickets.
A. Vitamin D
B. Iron
C. Calcium
D. Sodium
69. Among the following foods, which has the highest amount of potassium per serving?
A. Cantaloupe
B. Avocado
C. Raisin
D. Banana
70. A client has HEMOSIDEROSIS. Which of the following drug would you expect to be given to the client?
A. Acetazolamide
B. Deferoxamine
C. Calcium EDTA
D. Activated charcoal

71. Which of the following provides the richest source of Iron per area of their meat?
A. Pork meat
B. Lean red meat
C. Pork liver
D. Green mongo
72. Which of the following is considered the best indicator of nutritional status of an individual?
A. Height
B. Weight
C. Arm muscle circumference
D. BMI
73. Jose Miguel, a 50 year old business man is 60 Tall and weights 179 lbs. As a nurse, you know that Jose
Miguel is :
A. Overweight
B. Underweight
C. Normal
D. Obese
74. Jose Miguel is a little bit nauseous. Among the following beverages, Which could help relieve JMs
nausea?
A. Coke
B. Sprite
C. Mirinda
D. Orange Juice or Lemon Juice
75. Which of the following is the first sign of dehydration?
A. Tachycardia
B. Restlessness
C. Thirst
D. Poor skin turgor
76. What Specific gravity lab result is compatible with a dehydrated client?
A. 1.007
B. 1.020

C. 1.039
D. 1.029
77. Which hematocrit value is expected in a dehydrated male client?
A. 67%
B. 50%
C. 36%
D. 45%
78. Which of the following statement by a client with prolonged vomiting indicates the initial onset of
hypokalemia?
A. My arm feels so weak
B. I felt my heart beat just right now
C. My face muscle is twitching
D. Nurse, help! My legs are cramping
79. Which of the following is not an anti-emetic?
A. Marinol
B. Dramamine
C. Benadryl
D. Alevaire
80. Which is not a clear liquid diet?
A. Hard candy
B. Gelatin
C. Coffee with Coffee mate
D. Bouillon
81. Which of the following is included in a full liquid diet?
A. Popsicles
B. Pureed vegetable meat
C. Pineapple juice with pulps
D. Mashed potato
82. Which food is included in a BLAND DIET?

A. Steamed broccoli
B. Creamed potato
C. Spinach in garlic
D. Sweet potato
83. Which of the following if done by the nurse, is correct during NGT Insertion?
A. Use an oil based lubricant
B. Measure the amount of the tube to be inserted from the Tip of the nose, to the earlobe, to the xiphoid process
C. Soak the NGT in a basin of ice water to facilitate easy insertion
D. Check the placement of the tube by introducing 10 cc of sterile water and auscultating for bubbling sound
84. Which of the following is the BEST method in assessing for the correct placement of the NGT?
A. X-Ray
B. Immerse tip of the tube in water to check for bubbles produced
C. Aspirating gastric content to check if the content is acidic
D. Instilling air in the NGT and listening for a gurgling sound at the epigastric area
85. A terminally ill cancer patient is scheduled for an NGT feeding today. How should you position the
patient?
A. Semi fowlers in bed
B. Bring the client into a chair
C. Slightly elevated right side lying position
D. Supine in bed
86. A client is scheduled for NGT Feeding. Checking the residual volume, you determined that he has 40 cc
residual from the last feeding. You reinstill the 40 cc of residual volume and added the 250 cc of feeding
ordered by the doctor. You then instill 60 cc of water to clear the lumen and the tube. How much will you put
in the clients chart as input?
A. 250 cc
B. 290 cc
C. 350 cc
D. 310 cc
87. Which of the following if done by a nurse indicates deviation from the standards of NGT feeding?
A. Do not give the feeding and notify the doctor of residual of the last feeding is greater than or equal to 50 ml
B. Height of the feeding should be 12 inches about the tube point of insertion to allow slow introduction of feeding

C. Ask the client to position in supine position immediately after feeding to prevent dumping syndrome
D. Clamp the NGT before all of the water is instilled to prevent air entry in the stomach
88. What is the most common problem in TUBE FEEDING?
A. Diarrhea
B. Infection
C. Hyperglycemia
D. Vomiting
89. Which of the following is TRUE in colostomy feeding?
A. Hold the syringe 18 inches above the stoma and administer the feeding slowly
B. Pour 30 ml of water before and after feeding administration
C. Insert the ostomy feeding tube 1 inch towards the stoma
D. A Pink stoma means that circulation towards the stoma is all well
90. A client with TPN suddenly develops tremors, dizziness, weakness and diaphoresis. The client said I
feel weak You saw that his TPN is already empty and another TPN is scheduled to replace the previous one
but its provision is already 3 hours late. Which of the following is the probable complication being
experienced by the client?
A. Hyperglycemia
B. Hypoglycemia
C. Infection
D. Fluid overload
91. To assess the adequacy of food intake, which of the following assessment parameters is best used?
A. Food likes and dislikes
B. Regularity of meal times
C. 3 day diet recall
D. Eating style and habits
92. The vomiting center is found in the
A. Medulla Oblongata
B. Pons
C. Hypothalamus
D. Cerebellum
93. The most threatening complication of vomiting in clients with stroke is

A. Aspiration
B. Dehydration
C. Fluid and electrolyte imbalance
D. Malnutrition
94. Which among this food is the richest source of Iron?
A. Ampalaya
B. Broccoli
C. Mongo
D. Malunggay leaves
95. Which of the following is a good source of Vitamin A?
A. Egg yolk
B. Liver
C. Fish
D. Peanuts
96. The most important nursing action before gastrostomy feeding is
A. Check V/S
B. Assess for patency of the tube
C. Measure residual feeding
D. Check the placement of the tube
97. The primary advantage of gastrostomy feeding is
A. Ensures adequate nutrition
B. It prevents aspiration
C. Maintains Gastro esophageal sphincter integrity
D. Minimizes fluid-electrolyte imbalance
98. What is the BMI Of Budek, weighing 120 lbs and has a height of 5 feet 7 inches.
A. 20
B. 19
C. 15
D. 25
99. Which finding is consistent with PERNICIOUS ANEMIA?

A. Strawberry tongue
B. Currant Jelly stool
C. Beefy red tongue
D. Pale [ HYPOCHROMIC ] RBC
100. The nurse is browsing the chart of the patient and notes a normal serum lipase level. Which of the
following is a normal serum lipase value?
A. 10 U/L
B. 100 U/L
C. 200 U/L
D. 350 U/L

Answers and Rationale


Here are the answers for this exam. Gauge your performance by counter checking your answers to those below. If
you have any disputes or clarifications, please direct them to the comments section.
1. Answer: A. For clearance mechanism such as coughing
The the function of the cough reflex is to dislodge foreign substances from the trachea. The upper respiratory tract
refers to the external nose, nasal cavity, pharynx and associated structures while the lower respiratory tract includes
the larynx, trachea, bronchi, and lungs. However, there are alternative definitions just like the larynx being placed on
the upper respiratory tract.
2. Answer: D. Vibrissae
Vibrissae are the thick hairs which grow inside the nostrils to help keep large particles from entering the nasal
passages. Cilia are hair-like projection lining the bronchus that move microbes and debris up and out of the airways.
3. Answer: A. Ehtmoid
The ethmoidal sinuses are formed from several discrete air cells within the ethmoid bone between the nose and the
eyes. The maxillary sinuses, also called the maxillary antrechea and the largest of the paranasal sinuses, are under
the eyes, in the maxillary bones. The frontal sinuses, superior to the eyes, are in the frontal bone, which forms the
hard part of the forehead. The sphenoidal sinuses are in the sphenoid bone.
4. Answer: D. Frontal
The frontal sinuses are found superior to the eyes and eyebrows in the frontal bone, which forms the hard part of the
forehead.

5. Answer: A. Right main stem bronchus


Foreign objects that enter the trachea usually lodge in the right main bronchus, because it is more vertical than the
left main bronchus and therefore for in direct line with the trachea.
6. Answer: C. Goblet cells
Goblet cells are found scattered among the epithelial lining of organs, such as the intestinal and respiratory tracts.
They are found inside the trachea, bronchus, and larger bronchioles in respiratory tract. The main role of goblet cells
is to secrete mucus in order to protect the mucosae where they are found.
7. Answer: C. Three
The right lung has three lobes called the superior, middle and inferior lobes. the left lung, on the other hand, has
only two lobes namely the superior and inferior lobes.
8. Answer: C. Right kidney lower, Right lung shorter
The kidneys are paired retroperitoneal structures that are normally located between the transverse processes of
T12-L3 vertebrae, with the left kidney typically somewhat more superior in position than the right. The right lung,
meanwhile, has a higher volume, total capacity and weight, than that of the left lung. Although it is 5 cm shorter due
to the diaphragm rising higher on the right side to accommodate the liver, it is broader than the left lung due to the
cardiac notch of the left lung.
9. Answer: B. Type II pneumocytes
Pulmonary surfactant is a surface-active lipoprotein complex (phospho lipoprotein) formed by type II alveolar cells.
10. Answer: B. 2:1
An LS ratio of 2 or more indicates fetal lung maturity and a relatively low risk of infant respiratory distress
syndrome, and an L/S ratio of less than 1.5 is associated with a high risk of infant respiratory distress syndrome.
11. Answer: A. Inspiratory reserve volume
Inspiratory reserve volume is the maximal amount of additional air that can be drawn into the lungs by determined
effort after normal inspiration. Average inspiratory reserve volumes in healthy adults are 3.0 L in men and 1.9 L in
women.
12. Answer: D. Residual volume
Residual volume is the volume of air remaining in the lungs after a maximal exhalation. Average residual volumes in
healthy adults are 1.2 L in men and 1.1 L in women.

13. Answer: B. We have 12 pairs of ribs Cassandra


Humans have 24 ribs (12 pairs). The first seven sets of ribs, known as true ribs (costae verae), are directly
attached to the sternum through the costal cartilage. The following five sets are known as false ribs (costae
spuriae), three of these sharing a common cartilaginous connection to the sternum, while the last two (eleventh and
twelfth ribs) are termed floating ribs (costae fluctuantes) or vertebral ribs.
14. Answer: C. Diaphragm
The main muscle of breathing is the diaphragm, therefore it gets its own page. The muscles of inspiration elevate
the ribs and sternum, and the muscles of expiration depress them. The primary inspiratory muscles are the external
intercostals and the diaphragm.
15. Answer: D. Human requires 21% of oxygen and we have 21% available in our air
Air is a mixture of 21% oxygen, 78% nitrogen, and approximately 1% other trace gases, primarily argon; to simplify
calculations this last 1% is usually treated as if it were nitrogen. The gas is essential for living, but only up to a point.
Humans can only breathe 21 percent oxygen.
16. Answer: A. A passive process
Expiration is a passive process. That means that we dont have to expend any energy to exhale. When our
diaphragm has stopped contracting for inspiration, it can now begin to relax.
17. Answer: A. Patient assumes position for 10 to 15 minutes
Postural drainage is the positioning techniques that drain secretions from specific segments of the lugs and bronchi
into the trachea. The client is instructed to remain in each position for 10 to 15 minutes. During this time, perform
percussion and vibration, as ordered.
18. Answer: D. Promote bronchoconstriction
Steam inhalation is a method of introducing warm, moist air into the lungs via the nose and throat for therapeutic
benefit. Essential oils are often added to provide additional relief. Inhaling steam is a great treatment for respiratory
complications and is recommended for dealing with common cold, flu, bronchitis, sinusitis, asthma, and allergies.
Dry air passages are moistened, and mucus is loosened/eliminated easier by coughing or blowing the nose. The
moist air also alleviates difficulty breathing, throat irritation and inflammation.
19. Answer: C. Render steam inhalation for atleast 60 minutes
A common method of inhaling steam is to boil a few cups of filtered water and then pour the steaming water into a
large bowl. Essential oils can be added at this point if desired. Next, a towel can be placed over the head, while

leaning over the bowl of water, breathing deeply through the nose for approximately 15 minutes. Humidifiers also
provide a gentle form of steam inhalation.
20. Answer: B. As needed
A nurse should suction a patient as needed and indicated to maintain patency and integrity of airway.
21. Answer: D. Side lying
Position the unconscious client in a side-lying position facing you. A side-lying position facilitates drainage of
secretions by gravity and prevents aspiration.
22. Answer: C. 100-120 mmHg
Turn on suction device and adjust pressure: infants and children, 50 to 75 mm Hg; adults, 100 to 120 mm Hg.
Excessive negative pressure traumatizes mucosa and can induce hypoxia.
23. Answer: C. 10-15 mmHg
The pressure for a portable suction equipment depends on the age of the patient. For adults (10-15 mmHg), for
children (5-10 mmHg) and for infants (2-5 mmHg).
24. Answer: A. Fr. 18
Appropriate-sized catheter: infants, 5 to 8 Fr; children, 8 to 10 Fr; adults, 12 to 18 Fr.
25. Answer: A. Measure the length of the suction catheter to be inserted by measuring from the tip of the
nose, to the earlobe, to the xiphoid process
Determine the depth the suction device will be inserted, by measuring the device against the distance from the
corner of the patients mouth to the tip of the earlobe on the same side.
26. Answer: B. 2,5
Restlessness, irritability and tachycardia are early signs of hypoxia. Chronic hypoxemia may result in cognitive
changes, such as memory changes.
27. Answer: A. Nasal Cannula
Among the methods of oxygenation, nasal cannulas least likely produce anxiety and apprehension. However,
patients with nasal cannulas sometimes complain of nasal dryness, particularly when receiving oxygen at high
levels. New devices can help with this by adding moisture and warmth to the delivery process.

28. Answer: C. Non Rebreather mask


A non rebreather mask has a reservoir bag that is inflated with pure oxygen. Between the mask and the bag is
another one-way valve that allows the patient to breathe in the oxygen supplied by the source as well as oxygen
from the reservoir. This provides the patient with an oxygen concentration of nearly 100%. A piece of tubing, usually
connected to extension tubing, connects the mask to the oxygen source.
29. Answer: D. Excessive oxygen administration results in respiratory acidosis
Oxygen, in its natural state, is a colorless, odorless, and tasteless gas. Oxygen is considered to be the most
important of all the elements to life. Oxygen does not burn, but it does support combustion. Oxygen may also dry out
the nasal mucosa and can irritate the nares.
30. Answer: B. Put a non rebreather mask in the patient before opening the oxygen source
In using a non rebreather mask, nurses should connect oxygen flow meter to an oxygen source first and preset the
oxygen flow to 15 liters per minute and check the system before placing the mask over patients face covering the
nose and mouth.
31. Answer: D. Client is frequently turning from side to side
Restlessness, irritability and tachycardia are early signs of hypoxia.
32. Answer: C. Fingers are Club-like
The normal AP to Lateral diameter in normal adult is 1:2. Signs of possible chronic pulmonary disease include
clubbing, barrel chest (the increased anterior-posterior diameter of the chest present in some patients with
emphysema), and pursed lip breathing. Clubbing is enlargement of the fingertips (or toes) due to proliferation of
connective tissue between the fingernail and the bone.
33. Answer: A. Cannula
The nasal cannula is used when a low-flow oxygen is indicated. The room air mixes with the oxygen from the tank. It
can deliver 24-40 percent of oxygen at 2-6 liters per minute. The cannula is used for patients with COPD, asthma,
emphysema, and uncomplicated heart attack.
34. Answer: B. Chest tube thoracostomy
CTT stands for Chest tube thoracostomy. It is done to drain fluid, blood, or air from the space around the lungs.
Some diseases, such as pneumonia and cancer, can cause an excess amount of fluid or blood to build up in the
space around the lungs (called a pleural effusion).
35. Answer: D. 8th ICS

Chest tubes are indicated when the normally airtight pleural space has been penetrated through surgery or trauma,
when a defect in the alveoli allows air to enter the intrapleural space, and when there is an accumulation of fluid, as
from pleural effusion. In some cases one tube is inserted higher in the thorax (usually in the 2nd intercostal space)
to remove air, and a second tube is placed lower (in the 8th or 9th intercostal space) to drain off fluids.
36. Answer: C. Check for tube leak
Intermittent bubbling in water seal chamber with forced expiration or cough is okay. Continuous bubbling in the water
seal is abnormal and indicates an air leak. IF the nurse notes that there is CONTINUOUS bubbling in the water seal
chamber, check for leaks in the system. With physicians order, RN places padded clamp closest to dressing. If leak
stops, air leak is at insertion site. If bubbling continues, leak is between clamp and drainage system.
37. Answer: D. It is the study of nutrients and the process in which they are use by the body
The science or study that deals with food and nourishment, especially in humans. It is the process of nourishing or
being nourished, especially the process by which a living organism assimilates food and uses it for growth and for
replacement of tissues.
38. Answer: B. Small intestine
Chemical processes that contribute to digestion begin in the mouth with action of saliva on food. However, most of
the chemical digestive processes occur in the stomach and small intestine where the partly-digested materials are
subjected to gastric juices, pancreatic juice, succus entericus and so on. To be slightly more detailed, most digestion
takes place in the duodenum section of the small intestine.
39. Answer: C. The action of ptyalin or the salivary trypsin breaks down starches into maltose
Salivary amylase (ptyalin) starts the breakdown of high-molecular-weight carbohydrates while trypsin breaks down
proteins.
40. Answer: D. Coffee with coffee mate, Bacon and Egg
Certain foods and drinks loosen the lower esophageal sphincter. These include chocolate, peppermint, caffeinecontaining beverages (such as coffee, tea, and soft drinks), fatty foods, and alcohol.
41. Answer: A. Mouth
Chemical processes that contribute to digestion begin in the mouth with action of saliva on food.
42. Answer: D. Stomach

Protein digestion occurs in the stomach and duodenum in which 3 main enzymes, pepsin secreted by the stomach
and trypsin and chymotrypsin secreted by the pancreas, break down food proteins into polypeptides that are then
broken down by various exopeptidases and dipeptidases into amino acids.
43. Answer: C. HCl inhibits absorption of Calcium in the gastric mucosa
Hydrochloric acid, or HCL, is secreted in the stomach during digestion to begin breakdown of dietary fats. HCL is
needed for absorption of calcium in the duodenum, which is the first part of small intestine. The duodenum is where
calcium is actively absorbed from food into the body through the intestinal wall into the bloodstream.
44. Answer: C. Amylase
Amylase is an enzyme that helps digest carbohydrates. It is produced in the pancreas and the glands that make
saliva.
45. Answer: C. Pancreozymin
Pancreozymin is a hormone of the duodenal mucosa that stimulates the external secretory activity of the pancreas,
especially its production of amylase; identical with cholecystokinin.
46. Answer: B. Glucose + Fructose
Sucrose is a disaccharide combination of the monosaccharides glucose and fructose with the formula C12H22O11.
47. Answer: A. Trypsin
In the duodenum, trypsin catalyzes the hydrolysis of peptide bonds, breaking down proteins into smaller peptides.
The peptide products are then further hydrolyzed into amino acids via other proteases, rendering them available for
absorption into the blood stream. Tryptic digestion is a necessary step in protein absorption as proteins are generally
too large to be absorbed through the lining of the small intestine.
48. Answer: D. Amino Acids
Twenty percent of the human body is made up of protein. Protein plays a crucial role in almost all biological
processes and amino acids are the building blocks of it.
49. Answer: C. Cholecystokinin
CCK mediates a number of physiological processes, including digestion and satiety. It is released by I cells located
in the mucosal epithelium of the small intestine (mostly in the duodenum and jejunum). CCK also causes the
increased production of hepatic bile, and stimulates the contraction of the gall bladder and the relaxation of the
Sphincter of Oddi (Glissons sphincter), resulting in the delivery of bile into the duodenal part of the small intestine.
Bile salts form amphipathic micelles that emulsify fats, aiding in their digestion and absorption.

50. Answer: C. It is a sterile body cavity


The large intestine is the end section of the intestine. It is about 5 ft (1.5 m) long, is wider than the small intestine,
and has a smooth inner wall. In the first half, enzymes from the small intestine complete digestion, and bacteria
produce many B vitamins and vitamin K. Over 2430 hours, churning movements break down tough cellulose fibres
and expose chyme to the colons walls, which absorb water and electrolytes; absorption is its main function, along
with storing fecal matter for expulsion.
51. Answer: A. Calorie
Calorie, a unit of energy or heat variously defined. The calorie was originally defined as the amount of heat required
at a pressure of 1 standard atmosphere to raise the temperature of 1 gram of water 1 Celsius.
52. Answer: B. 200 calories
Determine if your food is a carbohydrate, protein, or fat. The conversion multiple is different for each type of food. 1
gram of carbohydrate is equal to 4 calories. Therefore, a cup of rice having 50 grams of carbohydrates contains 200
calories.
53. Answer: C. 2,000 calories
The Department of Health consultant on non-communicable diseases, noted that the recommended calorie intake
for women is 1,500 and 2,000 for men daily. It is advisable that Filipinos should aim for 500 calories a meal only.
54. Answer: C. During cold weather, people need more calories due to increase BMR
Temperature affects how many calories we burn. According to the American Dietetic Association Complete Food
and Nutrition Guide, both the heat and cold raise the BMR. If we are too cold we shiver. Shivering burns up much
energy from the constant contraction and relaxation of muscle cells trying to produce heat to maintain body
temperature. When we are hot we also burn more energy through the process of sweating.
55. Answer: A. An individual in a long state of gluconeogenesis
The human body requires glucose for the brain and nervous system, and a diet that has very few or no dietary
carbohydrates forces it to generate this glucose from protein through gluconeogenesis, with an efficiency of
approximately 57% (protein and carbohydrate are approximately equal in calorific value; each has about four
kilocalories per gram, but gluconeogenesis can produce only 57g of glucose from 100g of protein). This could be a
significant contributor to metabolic advantage.
56. Answer: C. Vitamin B3

Also known as vitamin B3 or nicotinic acid, Niacin deficiency is a condition that occurs when a person doesnt get
enough or cant absorb niacin or tryptophan. Severe deficiency, called pellagra, can cause symptoms related to the
skin, digestive system, and nervous system.
57. Answer: D. Vitamin B6
Vitamin B6 reduces the effectiveness of levodopa, a medication used to treat Parkinsons disease. However, your
doctor may be able to determine a dose of B6 that can help reduce side effects of levodopa without interfering with
the drugs action. Taking vitamin B6 along with levodopa should be done only under the strict guidance of a
physician.
58. Answer: D. Vitamin B6
Vitamin B6 (pyridoxine) supplementation during isoniazid (INH) therapy is necessary in some patients to prevent the
development of peripheral neuropathy. In vivo pyridoxine is converted into coenzymes which play an essential role in
the metabolism of protein, carbohydrates, fatty acids, and several other substances, including brain amines, INH
apparently competitively inhibits the action of pyridoxine in these metabolic functions. The reported frequency of
INH-induced neuropathy in various studies is reviewed and population groups at relatively high risk of developing
this complication are identified. The routine use of pyridoxine supplementation to prevent peripheral neuropathy in
high risk populations is recommended.
59. Answer: B. Vitamin B2
Riboflavin deficiency is also called ariboflavinosis. In humans the classical syndrome affects the mouth (sore throat,
inflammation of the lining of mouth and tongue), angular cheilitis), the eyes (photophobia with bloodshot, itchy,
watery eyes), the skin (moist, scaly skin particularly affecting the scrotum or labia majora and the nasolabial folds)
and the blood (decreased red blood cell count with normal cell size and hemoglobin content i.e. normochromic
normocytic anemia). In children it also results in reduced growth.
60. Answer: A. Vitamin B1
Beriberi is a disease brought on by a Vitamin B-1 (thiamine) deficiency. There are two types of the disease: wet
beriberi and dry beriberi. Wet beriberi can effect heart function and, in the most extreme cases, heart failure. Dry
beriberi damages the nerves and can lead to a loss of muscle strength and, eventually, muscle paralysis. If left
unchecked and untreated, beriberi will cause death.
61. Answer: B. Vegetable oil
Because vitamin E is naturally present in plant-based diets and animal products and is often added by
manufacturers to vegetable oils and processed foods, intakes are probably adequate to avoid overt deficiency in
most situations.

62. Answer: A. Pork liver and organ meats, Pork


It is important for alcoholics to avoid refined sugars and caffeine, as they stress blood sugar control mechanisms
and may increase the craving for alcohol. In one study, excluding caffeine, junk food, dairy products, and peanut
butter was compared to a control diet for six months. Excellent food sources of thiamine include asparagus,
mushrooms, peanuts, pork, soybeans, sunflower seeds, and yeast.
63. Answer: D. Organ meats, Green leafy vegetables, Liver, Eggs
Foods rich in folic acid such as leafy green vegetable, yeast, wheat germ, nuts, eggs, bananas, oranges, and organ
meats taken during the first trimester of pregnancy can help prevent spina bifida.
64. Answer: A. Spinach, Green leafy vegetables, Cabbage, Liver
Certain foods and beverages can make it so warfarin doesnt effectively prevent blood clots. In order to maintain
stable PT/INR levels one should not eat more than 1 serving of a high vitamin K food, and no more than 3 servings
of a food with moderate amounts of vitamin K. What is important is that your intake of vitamin K stays consistent.
The nurse must instruct the client to avoid eating or drinking large amounts of Kale, Spinach, Brussels sprouts,
Parsley, Collard greens, Mustard greens, Chard, and Green tea.
65. Answer: C. Selenium
Lycopene (as beta-carotene) and selenium supplementation have been associated with a reduced risk of prostate
cancer in nested case-control studies, but only in subgroups of men with low baseline plasma lycopene (or betacarotene) and selenium levels respectively. The Prostate Cancer Prevention Trial prospectively evaluated finasteride,
a 5-alpha-reductase inhibitor, as chemoprevention.
66. Answer: D. Vitamin D
Vitamin D is made naturally by the body when exposed to sunlight. A study in patients with prostate cancer
suggested that medium or high levels of vitamin D in the blood may be linked with better outcomes than lower levels.
These findings indicate that vitamin D levels may play a role in whether or not the disease will worsen and may be a
factor in predicting outcome in prostate cancer patients.
67. Answer: B. Iron
Micronutrients are nutrients for humans required in small quantities throughout life. The microminerals or trace
elements include at least iron, cobalt, chromium, copper, iodine, manganese, selenium, zinc and molybdenum.
Micronutrients also include vitamins, which are organic compounds required as nutrients in tiny amounts by an
organism.
68. Answer: C. Calcium

Calcium and phosphorus are needed to keep bones healthy and strong. A mild lack it may not cause symptoms but
can cause tiredness and general aches and pains. A more severe lack can cause serious problems such as rickets
(in children) and osteomalacia (in adults). Vitamin D is also important because it increases the rate at which calcium
is absorbed into your blood.
69. Answer: B. Avocado
Cantaloupe: 267 mg Potassium, Avocado: 485 mg Potassium, Banana: 358 mg Potassium
70. Answer: B. Deferoxamine
Combined therapy with deferoxamine and hemofiltration offers promises as an effective means of iron mobilization in
dialysis patients with hemosiderosis.
71. Answer: C. Pork liver
Pork liver has 23 mg of iron in every 100 g while lean red meat only has 3.7 mg of iron in every 100 g.
72. Answer: D. BMI
Body Mass Index (BMI) is a number calculated from a persons weight and height. BMI provides a reliable indicator
of body fatness for most people and is used to screen for weight categories that may lead to health problems.
73. Answer: C. Normal

74. Answer: A. Coke


One of the ingredients in coke is sodium bicarbonate, or baking soda. This is an alkaline substance, and therefore
can help balance the pH level in the stomach. So, if the cause of the nausea has to do with too much acid in the
stomach, or acid rising from the stomach into the esophagus, coke can have a relieving effect on the condition.
Doctors recommend letting the soda go flat before drinking, or adding a pinch of salt over the top.
75. Answer: C. Thirst

Dehydration can be mild, moderate or severe, depending on how much of your body weight is lost through fluids.
Two early signs of dehydration are thirst and dark colored urine. This is the bodys way of trying to increase water
intake and decrease water loss.
76. Answer: C. 1.039
Adults generally have a specific gravity in the range of 1.000 to 1.030. Increases in specific gravity (hypersthenuria,
i.e. increased concentration of solutes in the urine) may be associated with dehydration, diarrhea, emesis, excessive
sweating, urinary tract/bladder infection, glucosuria, renal artery stenosis, hepatorenal syndrome, decreased blood
flow to the kidney (especially as a result of heart failure), and excess of anti-diuretic hormone caused by Syndrome
of inappropriate anti-diuretic hormone. A specific gravity greater than 1.035 is consistent with frank dehydration.
77. Answer: A. 67%
Higher than normal hematocrit levels represent abnormally elevated red blood cell counts. High hematocrits can be
seen in people living at high altitudes and in chronic smokers. Dehydration produces a falsely high hematocrit that
disappears when proper fluid balance is restored. Some other infrequent causes of an elevated hematocrit are lung
disease, certain tumors, a disorder of the bone marrow known as polycythemia rubra Vera, and abuse of the drug
erythropoietin (Epogen) by athletes for blood doping purposes.
78. Answer: D. Nurse, help! My legs are cramping
The effects of low potassium include may cause the following symptoms: weakness, tiredness, or cramping in arm
or leg muscles, sometimes severe enough to cause inability to move arms or legs due to weakness (much like a
paralysis).
79. Answer: D. Alevaire
Alevaire is a Mucolytic Agent.
80. Answer: C. Coffee with Coffee mate
A clear liquid diet consists of clear liquids such as water, broth and plain gelatin that are easily digested and
leave no undigested residue in your intestinal tract. A tea or coffee without milk or cream is considered a clear liquid
diet.
81. Answer: A. Popsicles
A clear liquid diet is made up of only clear fluids and foods that turn to clear fluids when they are at room
temperature. It includes things like clear broth, tea, cranberry juice, Jell-O, and Popsicles.
82. Answer: B. Creamed potato

The bland diet omits all foods that are bowel stimulants and are irritating to the gastrointestinal tract. Potatoes
allowed include potato, mashed, creamed, baked, or broiled without skins, sweet potato or yams. Rice. Spaghetti,
noodles or macaroni are also allowed. Avoid potato chips and potato skins.
83. Answer: B. Measure the amount of the tube to be inserted from the Tip of the nose, to the earlobe, to the
xiphoid process
To measure the length of the tube to be inserted, stand to the patients right, if you are right handed, and measure
from the tip of the nose to the earlobe and to the xiphoid process. Experience has shown that in tall people, it may
be necessary to add 2 inches to the length of the tube to ensure entrance into the stomach. If you are measure the
tube for an infant, extend it from the tip of the nose to the earlobe and then from the nose to a point half-way
between the xiphoid process and the umbilicus, because the body proportions are different in infants and adults.
Mark the tube with a piece of tape.
84. Answer: A. X-Ray
The gold standard for nasoenteric feeding tube placement is radiographic confirmation with chest and abdominal xrays.
85. Answer: B. Bring the client into a chair
This minimizes the possibility of aspiration (inhaling food into the lungs) and its inherent complication (pneumonia). If
choking or difficulty of breathing occurs during a feeding, stop the feeding and call the doctor immediately.
86. Answer: D. 310 cc
250 cc of feeding ordered by the doctor plus the 60 cc of water to clear the lumen and the tube is 310 cc.
87. Answer: C. Ask the client to position in supine position immediately after feeding to prevent dumping
syndrome
It is important to sit up or prop your patients head up while receiving feeding and remain in that position for 30-60
minutes. This minimizes the possibility of aspiration (inhaling food into the lungs) and its inherent complication
(pneumonia).
88. Answer: A. Diarrhea
The most common reported complication of tube feeding is diarrhea, defined as stool weight > 200 mL per 24 hours.
89. Answer: B. Pour 30 ml of water before and after feeding administration
The tube should be flushed with water before and after feeds.

90. Answer: B. Hypoglycemia


Hypoglycemia upon abrupt discontinuation of TPN is a complication that may result from endogenous insulin levels
not adjusting to the sudden reduction in dextrose.
91. Answer: C. 3 day diet recall
A 3 day diet recall provides more representative intake information.
92. Answer: A. Medulla Oblongata
The medulla oblongata is the lower half of the brainstem, which is continuous with the spinal cord; the upper half
being the pons. It is often referred to simply as the medulla. The medulla contains the cardiac, respiratory, vomiting
and vasomotor centers and therefore deals with the autonomic (involuntary) functions of breathing, heart rate and
blood pressure.
93. Answer: A. Aspiration
Impaired consciousness with drug or alcohol abuse, general anaesthesia, seizures, sedation, acute stroke, central
nervous system lesions or head injury are considered risk factors for aspiration. Others include swallowing disorders
such as esophageal stricture, dysphagia, stroke, bulbar palsy, pharyngeal disease (eg, malignancy), neuromuscular
disorders (eg, multiple sclerosis).
94. Answer: C. Mongo
Bean sprouts have the richest source of amino acids (for protein), vitamins and minerals, and also contain a good
amount of fiber. They contain all types of vitamins (A, B, C, D, E and K), folate and are an excellent source of iron,
potassium, calcium, phosphorous, magnesium and zinc.
95. Answer: B. Liver
Liver is a particularly rich source of vitamin A, although this means you may be at risk of having too much vitamin A
if you eat liver more than once a week.
96. Answer: B. Assess for patency of the tube
A gastrostomy tube allows the delivery of supplemental nutrition and medications directly into the stomach.
Maintaining its patency is the most important nursing action to be considered before gastrostomy feeding in order for
it to be successful.
97. Answer: C. Maintains Gastro esophageal sphincter integrity
One of the advantages of gastrostomy feeding is that it maintains the integrity of the gastroesophageal sphincter.

98. Answer: B. 19

99. Answer: C. Beefy red tongue


Pernicious anemia is a type of anemia caused by a lack of vitamin B12. Not having enough vitamin B12, or being
unable to absorb it, can lead to symptoms such as nerve damage, confusion, dementia, memory loss, depression,
nausea, heartburn, weight loss, and a smooth, beefy red tongue.
100. Answer: C. 200 U/L
1. A 2 year-old child is receiving temporary total parental nutrition (TPN) through a central venous line. This
is the first day of TPN therapy. Although all of the following nursing actions must be included in the plan of
care of this child, which one would be a priority at this time?
A. Use aseptic technique during dressing changes
B. Maintain central line catheter integrity
C. Monitor serum glucose levels
D. Check results of liver function tests
2. Nurse Jamie is administering the initial total parenteral nutrition solution to a client. Which of the
following assessments requires the nurses immediate attention?
A. Temperature of 37.5 degrees Celsius
B. Urine output of 300 cc in 4 hours
C. Poor skin turgor
D. Blood glucose of 350 mg/dl
3. Nurse Susan administered intravenous gamma globulin to an 18 month-old child with AIDS. The parent
asks why this medication is being given. What is the nurses best response?
A. It will slow down the replication of the virus.
B. This medication will improve your childs overall health status.
C. This medication is used to prevent bacterial infections.
D. It will increase the effectiveness of the other medications your child receives.

4. When caring for a client with total parenteral nutrition (TPN), what is the most important action on the
part of the nurse?
A. Record the number of stools per day
B. Maintain strict intake and output records
C. Sterile technique for dressing change at IV site
D. Monitor for cardiac arrhythmias
5. The nurse is administering an intravenous vesicant chemotherapeutic agent to a client. Which
assessment would require the nurses immediate action?
A. Stomatitis lesion in the mouth
B. Severe nausea and vomiting
C. Complaints of pain at site of infusion
D. A rash on the clients extremities
6. Nurse Celine is caring for a client with clinical depression who is receiving a MAO inhibitor. When
providing instructions about precautions with this medication, the nurse should instruct the client to:
A. Avoid chocolate and cheese
B. Take frequent naps
C. Take the medication with milk
D. Avoid walking without assistance
7. While providing home care to a client with congestive heart failure, the nurse is asked how long diuretics
must be taken. The BEST response to this client should be:
A. As you urinate more, you will need less medication to control fluid.
B. You will have to take this medication for about a year.
C. The medication must be continued so the fluid problem is controlled.
D. Please talk to your physician about medications and treatments.
8. George, age 8, is admitted with rheumatic fever. Which clinical finding indicates to the nurse that George
needs to continue taking the salicylates he had received at home?
A. Chorea.
B. Polyarthritis.
C. Subcutaneous nodules.
D. Erythema marginatum.
9. An order is written to start an IV on a 74-year-old client who is getting ready to go to the operating room
for a total hip replacement. What gauge of catheter would best meet the needs of this client?

A. 18
B. 20
C. 21 butterfly
D. 25
10. A client with an acute exacerbation of rheumatoid arthritis is admitted to the hospital for treatment.
Which drug, used to treat clients with rheumatoid arthritis, has both an anti-inflammatory and
immunosuppressive effect?
A. Gold sodium thiomalate (Myochrysine)
B. Azathioprine (Imuran)
C. Prednisone (Deltasone)
D. Naproxen (Naprosyn)
11. Which of the following is least likely to influence the potential for a client to comply with lithium therapy
after discharge?
A. The impact of lithium on the clients energy level and lifestyle.
B. The need for consistent blood level monitoring.
C. The potential side effects of lithium.
D. What the clients friends think of his need to take medication
12. Which of the following is least likely to influence the potential for a client to comply with lithium therapy
after discharge?
A. The impact of lithium on the clients energy level and lifestyle.
B. The need for consistent blood level monitoring.
C. The potential side effects of lithium.
D. What the clients friends think of his need to take medication.
13. The nurse is caring for an elderly client who has been diagnosed as having sundown syndrome. He is
alert and oriented during the day but becomes disoriented and disruptive around dinnertime. He is
hospitalized for evaluation. The nurse asks the client and his family to list all of the medications,
prescription and nonprescription, he is currently taking. What is the primary reason for this action?
A. Multiple medications can lead to dementia
B. The medications can provide clues regarding his medical background
C. Ability to recall medications is a good assessment of the clients level of orientation.
D. Medications taken by a client are part of every nursing assessment.
14. A 25-year-old woman is in her fifth month of pregnancy. She has been taking 20 units of NPH insulin for
diabetes mellitus daily for six years. Her diabetes has been well controlled with this dosage. She has been

coming for routine prenatal visits, during which diabetic teaching has been implementeD. Which of the
following statements indicates that the woman understands the teaching regarding her insulin needs during
her pregnancy?
A. Are you sure all this insulin wont hurt my baby?
B. Ill probably need my daily insulin dose raised.
C. I will continue to take my regular dose of insulin.
D. These finger sticks make my hand sore. Can I do them less frequently?
15. Mrs. Johansons physician has prescribed tetracycline 500 mg po q6h. While assessing Mrs. Johansons
nursing history for allergies, the nurse notes that Mrs. Johansons is also taking oral contraceptives. What
is the most appropriate initial nursing intervention?
A. Administer the dose of tetracycline.
B. Notify the physician that Mrs. Johanson is taking oral contraceptives.
C. Tell Mrs. Johanson, she should stop taking oral contraceptives since they are inactivated by tetracycline.
D. Tell Mrs. Johanson, to use another form of birth control for at least two months.
16. An adult clients insulin dosage is 10 units of regular insulin and 15 units of NPH insulin in the morning.
The client should be taught to expect the first insulin peak:
A. as soon as food is ingested.
B. in two to four hours.
C. in six hours.
D. in ten to twelve hours.
17. An adult is hospitalized for treatment of deep electrical burns. Burn wound sepsis develops and
mafenide acetate 10% (Sulfamylon) is ordered bid. While applying the Sulfamylon to the wound, it is
important for the nurse to prepare the client for expected responses to the topical application, which
include:
A. severe burning pain for a few minutes following application.
B. possible severe metabolic alkalosis with continued use.
C. black discoloration of everything that comes in contact with this drug.
D. chilling due to evaporation of solution from the moistened dressings.
18. Ms.Clark has hyperthyroidism and is scheduled for a thyroidectomy. The physician has ordered Lugols
solution for the client. The nurse understands that the primary reason for giving Lugols solution
preoperatively is to:
A. decrease the risk of agranulocytosis postoperatively.
B. prevent tetany while the client is under general anesthesia.

C. reduce the size and vascularity of the thyroid and prevent hemorrhage.
D. potentiate the effect of the other preoperative medication so less medicine can be given while the client is under
anesthesia.
19. A two-year-old child with congestive heart failure has been receiving digoxin for one week. The nurse
needs to recognize that an early sign of digitalis toxicity is:
A. bradypnea.
B. failure to thrive.
C. tachycardia.
D. vomiting.
20. Mr. Bates is admitted to the surgical ICU following a left adrenalectomy. He is sleepy but easily aroused.
An IV containing hydrocortisone is running. The nurse planning care for Mr. Bates knows it is essential to
include which of the following nursing interventions at this time?
A. Monitor blood glucose levels every shift to detect development of hypo- or hyperglycemia.
B. Keep flat on back with minimal movement to reduce risk of hemorrhage following surgery.
C. Administer hydrocortisone until vital signs stabilize, then discontinue the IV.
D. Teach Mr. Bates how to care for his wound since he is at high risk for developing postoperative infection.

Answers & Rationale


Here are the answers and rationale for this exam. Counter check your answers to those below and tell us your
scores. If you have any disputes or need more clarification to a certain question, please direct them to the comments
section.
1. Answer C. Monitor serum glucose levels
Monitor serum glucose levels. Hyperglycemia may occur during the first day or 2 as the child adapts to the highglucose load of the TPN solution. Thus, a chief nursing responsibility is blood glucose testing.
2. Answer D. Blood glucose of 350 mg/dl
Total parenteral nutrition formulas contain dextrose in concentrations of 10% or greater to supply 20% to 50% of the
total calories. Blood glucose levels should be checked every 4 to 6 hours. A sliding scale dose of insulin may be
ordered to maintain the blood glucose level below 200mg/dl.
3. Answer C. This medication is used to prevent bacterial infections.
Intravenous gamma globulin is given to help prevent as well as to fight bacterial infections in young children with
AIDS.

4. Answer C. Sterile technique for dressing change at IV site


Clients receiving TPN are very susceptible to infection. The concentrated glucose solutions are a good medium for
bacterial growth. Strict sterile technique is crucial in preventing infection at IV infusion site.
5. Answer C. Complaints of pain at site of infusion
A vesicant is a chemotherapeutic agent capable of causing blistering of tissues and possible tissue necrosis if there
is extravasation. These agents are irritants which cause pain along the vein wall, with or without inflammation.
6. Answer A. Avoid chocolate and cheese
Foods high in tryptophan, tyramine and caffeine, such as chocolate and cheese may precipitate hypertensive crisis.
7. Answer C. The medication must be continued so the fluid problem is controlled.
This is the most therapeutic response and gives the client accurate information.
8. Answer B. Polyarthritis.
Chorea is the restless and sudden aimless and irregular movements of the extremities suddenly seen in persons
with rheumatic fever, especially girls. Polyarthritis is characterized by swollen, painful, hot joints that respond to
salicylates. Subcutaneous nodules are nontender swellings over bony prominences sometimes seen in persons
with rheumatic fever.Erythema marginatum is a skin condition characterized by nonpruritic rash, affecting trunk and
proximal extremities, seen in persons with rheumatic fever.
9. Answer A. 18
Clients going to the operating room ideally should have an 18- gauge catheter. This is large enough to handle blood
products safely and to allow rapid administration of large amounts of fluid if indicated during the perioperative period.
An 18-gauge catheter is recommended. A 20-gauge catheter is a second choice. A 21-gauge needle is too small
and a butterfly too unstable for a client going to surgery. A 25-gauge needle is too small.
10. Answer C. Prednisone (Deltasone)
Gold sodium thiomalate is usually used in combination with aspirin and nonsteroidal anti-inflammatory drugs to
relieve pain. Gold has an immunosuppressive affect. Azathioprine is used for clients with life-threatening
rheumatoid arthritis for its immunosuppressive effects.Prednisone is used to treat persons with acute exacerbations
of rheumatoid arthritis. This medication is given for its anti-inflammatory and immunosuppressive
effects. Naproxen is a nonsteroidal anti-inflammatory drug. Immunosuppression does not occur.
11. Answer D. What the clients friends think of his need to take medication

The impact of lithium on the clients energy level and lifestyle are great determinants to compliance. The frequent
blood level monitoring required is difficult for clients to follow for a long period of time. Potential side effects such as
fine tremor, drowsiness, diarrhea, polyuria, thirst, weight gain, and fatigue can be disturbing to the client. While the
clients social network can influence the client in terms of compliance, the influence is typically secondary to that of
the other factors listed.
12. Answer D. What the clients friends think of his need to take medication.
The impact of lithium on the clients energy level and lifestyle are great determinants to compliance. The frequent
blood level monitoring required is difficult for clients to follow for a long period of time. Potential side effects such as
fine tremor, drowsiness, diarrhea, polyuria, thirst, weight gain, and fatigue can be disturbing to the client. While the
clients social network can influence the client in terms of compliance, the influence is typically secondary to that of
the other factors listed.
13. Answer A. Multiple medications can lead to dementia
Drugs commonly used by elderly people, especially in combination, can lead to dementia. Assessment of the
medication taken may or may not provide information on the clients medical background. However, this is not the
primary reason for assessing medications in a client who is exhibiting sundown syndrome. Ability to recall
medications may indicate short-term memory and recall. However, that is not the primary reason for assessing
medications in a client with sundown syndrome. Medication history should be a part of the nursing assessment. In
this client there is an even more important reason for evaluating the medications taken.
14. Answer B. Ill probably need my daily insulin dose raised.
The client starts to need increased insulin in the second trimester. This statement indicates a lack of understanding.
As a result of placental maturation and placental production of lactogen, insulin requirements begin increasing in the
second trimester and may double or quadruple by the end of pregnancy. The client starts to need increased insulin
in the second trimester. This statement indicates a lack of understanding. Insulin doses depend on blood glucose
levels. Finger sticks for glucose levels must be continued.
15. Answer B. Notify the physician that Mrs. Johanson is taking oral contraceptives.
The nurse should be aware that tetracyclines decrease the effectiveness of oral contraceptives. The physician
should be notified. The physician should be notified. Tetracycline decreases the effectiveness of oral contraceptives.
There may be an equally effective antibiotic available that can be prescribed. Note on the clients chart that the
physician was notified. The nurse should be aware that tetracyclines decrease the effectiveness of oral
contraceptives. The nurse should not tell the client to stop taking oral contraceptives unless the physician orders
this. The nurse should be aware that tetracyclines decrease the effectiveness of oral contraceptives. If the physician
chooses to keep the client on tetracycline, the client should be encouraged to use another form of birth control. The
first intervention is to notify the physician.

16. Answer B. in two to four hours.


The first insulin peak will occur two to four hours after administration of regular insulin. Regular insulin is classified
as rapid acting and will peak two to four hours after administration. The second peak will be eight to twelve hours
after the administration of NPH insulin. This is why a snack must be eaten mid-morning and also three to four hours
after the evening meal. The first insulin peak will occur two to four hours after administration of regular insulin. The
first insulin peak will occur two to four hours after administration of regular insulin. The second peak will occur eight
to twelve hours after the administration of NPH insulin.
17. Answer A. severe burning pain for a few minutes following application.
Mafenide acetate 10% (Sulfamylon) does cause burning on application. An analgesic may be required before the
ointment is applied. Mafenide acetate 10% (Sulfamylon) is a strong carbonic anhydrase inhibitor that affects the
renal tubular buffering system, resulting in metabolic acidosis. Mafenide acetate 10% (Sulfamylon) does not cause
discoloration. Silver nitrate solution, another topical antibiotic used to treat burn sepsis, has the disadvantage of
turning everything it touches black. Mafenide acetate 10% (Sulfamylon) is an ointment that is applied directly to the
wound. It has the ability to diffuse rapidly through the eschar. The wound may be left open or dry dressing may be
applied. Silver nitrate solution is applied by soaking the wound dressings and keeping them constantly wet, which
may cause chilling and hypotension.
18. Answer C. reduce the size and vascularity of the thyroid and prevent hemorrhage.
Doses of over 30 mg/day may increase the risk of agranulocytosis. Lugols solution does not act to prevent tetany.
Calcium is used to treat tetany. The client may receive iodine solution (Lugols solution) for 10 to 14 days before
surgery to decrease vascularity of the thyroid and thus prevent excess bleeding. Lugols solution does not potentiate
any other preoperative medication.
19. Answer D. vomiting.
Bradypnea (slow breathing) is not associated with digitalis toxicity. Bradycardia is associated with digitalis toxicity.
Although children with congestive heart failure often have a related condition of failure to thrive, it is not directly
related to digitalis administration. It is more related to chronic hypoxia. Tachycardia is not a sign of digitalis toxicity.
Bradycardia is a sign of digitalis toxicity. The earliest sign of digitalis toxicity is vomiting, although one episode does
not warrant discontinuing medication.
20. Answer A. Monitor blood glucose levels every shift to detect development of hypo- or hyperglycemia.
Hydrocortisone promotes gluconeogenesis and elevates blood glucose levels. Following adrenalectomy the normal
supply of hydrocortisone is interrupted and must be replaced to maintain the blood glucose at normal levels. Care for
the client following adrenalectomy is similar to that for any abdominal operation. The client is encouraged to change
position, cough, and deep breathe to prevent postoperative complications such as pneumonia or thrombophlebitis.
Maintenance doses of hydrocortisone will be administered IV until the client is able to take it by mouth and will be

necessary for six months to two years or until the remaining gland recovers. The client undergoing an
adrenalectomy is at increased risk for infection and delayed wound healing and will need to learn about wound care,
but not at this time while he is in the ICU.
1. The nursery nurse is putting erythromycin ointment in the newborns eyes to prevent infection. She
places it in the following area of the eye:
A. under the eyelid
B. on the cornea.
C. in the lower conjunctival sac
D. by the optic disc.
2. The physician orders penicillin for a patient with streptococcal pharyngitis. The nurse administers the
drug as ordered, and the patient has an allergic reaction. The nurse checks the medication order sheet and
finds that the patient is allergic to penicillin. Legal responsibility for the error is:
A. only the nursesshe should have checked the allergies before administering the medication.
B. only the physiciansshe gave the order, the nurse is obligated to follow it.
C. only the pharmacistshe should alert the floor to possible allergic reactions.
D. the pharmacist, physician, and nurse are all liable for the mistake
3. James Perez, a nurse on a geriatric floor, is administering a dose of digoxin to one of his patients. The
woman asks why she takes a different pill than her niece, who also has heart trouble. James replies that as
people get older, liver and kidney function decline, and if the dose is as high as her nieces, the drug will
tend to:
A. have a shorter half-life.
B. accumulate.
C. have decreased distribution.
D. have increased absorption.
4. The nurse is administering augmentin to her patient with a sinus infection. Which is the best way for her
to insure that she is giving it to the right patient?
A. Call the patient by name
B. Read the name of the patient on the patients door
C. Check the patients wristband
D. Check the patients room number on the unit census list
5. The most important instructions a nurse can give a patient regarding the use of the
antibiotic ampicillin prescribed for her are to

A. call the physician if she has any breathing difficulties.


B. take it with meals so it doesnt cause an upset stomach.
C. take all of the medication prescribed even if the symptoms stop sooner.
D. not share the pills with anyone else.
6. Mr. Jessie Ray, a newly admitted patient, has a seizure disorder which is being treated with medication.
Which of the following drugs would the nurse question if ordered for him?
A. Phenobarbitol, 150 mg hs
B. Amitriptylene (Elavil), 10 mg QID.
C. Valproic acid (Depakote), 150 mg BID
D. Phenytoin (Dilantin), 100 mg TID
7. Mrs. Jane Gately has been dealing with uterine cancer for several months. Pain management is the
primary focus of her current admission to your oncology unit. Her vital signs on admission are BP 110/64,
pulse 78, respirations 18, and temperature 99.2 F. Morphine sulfate 6mg IV, q 4 hours, prn has been ordered.
During your assessment after lunch, your findings are: BP 92/60, pulse 66, respirations 10, and temperature
98.8. Mrs. Gately is crying and tells you she is still experiencing severe pain. Your action should be to
A. give her the next ordered dose of MS.
B. give her a back rub, put on some light music, and dim the lights in the room.
C. report your findings to the MD, requesting an alternate medication order
D. be obtained from the physician.
E. call her daughter to come and sit with her.
8. When counseling a patient who is starting to take MAO (monoamine oxidase) inhibitors such as Nardil for
depression, it is essential that they be warned not to eat foods containing tyramine, such as:
A. Roquefort, cheddar, or Camembert cheese.
B. grape juice, orange juice, or raisins.
C. onions, garlic, or scallions.
D. ground beef, turkey, or pork.
9. The physician orders an intramuscular injection of Demerol for the postoperativepatients pain. When
preparing to draw up the medication, the nurse is careful to remove the correct vial from the narcotics
cabinet. It is labeled
A. simethicone.
B. albuterol.
C. meperidine.
D. ibuprofen.

10. The nurse is administering an antibiotic to her pediatric patient. She checks the patients armband and
verifies the correct medication by checking the physicians order, medication kardex, and vial. Which of the
following is not considered one of the five rights of drug administration?
A. Right dose
B. Right route
C. Right frequency
D. Right time
11. A nurse is preparing the clients morning NPH insulin dose and notices a clumpy precipitate inside the
insulin vial. The nurse should:
A. draw up and administer the dose
B. shake the vial in an attempt to disperse the clumps
C. draw the dose from a new vial
D. warm the bottle under running water to dissolve the clump
12. A client with histoplasmosis has an order for ketoconazole (Nizoral). The nurse teaches the client to do
which of the following while taking this medication?
A. take the medication on an empty stomach
B. take the medication with an antacid
C. avoid exposure to sunlight
D. limit alcohol to 2 ounces per day
13. A nurse has taught a client taking a xanthine bronchodilator about beverages to avoid. The nurse
determines that the client understands the information if the client chooses which of the following
beverages from the dietary menu?
A. chocolate milk
B. cranberry juice
C. coffee
D. cola
14. A client is taking famotidine (Pepcid) asks the home care nurse what would be the best medication to
take for a headache. The nurse tells the client that it would be best to take:
A. aspirin (acetylsalicylic acid, ASA)
B. ibuprofen (Motrin)
C. acetaminophen (Tylenol)
D. naproxen (Naprosyn)

15. A nurse is planning dietary counseling for the client taking triamterene (Dyrenium). The nurse plans to
include which of the following in a list of foods that are acceptable?
A. baked potato
B. bananas
C. oranges
D. pears canned in water
16. A client with advanced cirrhosis of the liver is not tolerating protein well, as eveidenced by abnormal
laboratory values. The nurse anticipates that which of the following medications will be prescribed for the
client?
A. lactulose (Chronulac)
B. ethacrynic acid (Edecrin)
C. folic acid (Folvite)
D. thiamine (Vitamin B1)
17. A female client tells the clinic nurse that her skin is very dry and irritated. Which product would the
nurse suggest that the client apply to the dry skin?
A. glycerin emollient
B. aspercreame
C. myoflex
D. acetic acid solution
18. A nurse is providing instructions to a client regarding quinapril hydrochloride (Accupril). The nurse tells
the client:
A. to take the medication with food only
B. to rise slowly from a lying to a sitting position
C. to discontinue the medication if nausea occurs
D. that a therapeutic effect will be noted immediately
19. Auranofin (Ridaura) is prescribed for a client with rheumatoid arthritis, and the nurse monitors the client
for signs of an adverse effect related to the medication. Which of the following indicates an adverse effect?
A. nausea
B. diarrhea
C. anorexia
D. proteinuria

20. A client has been taking benzonatate (Tessalon) as ordered. The nurse tells the client that this
medication should do which of the following?
A. take away nausea and vomiting
B. calm the persistent cough
C. decrease anxiety level
D. increase comfort level

Answers & Rationale


Here are the answers and rationale for this exam. Counter check your answers to those below and tell us your
scores. If you have any disputes or need more clarification to a certain question, please direct them to the comments
section.
1. Answer: C. in the lower conjunctival sac
The ointment is placed in the lower conjunctival sac so it will not scratch the eye itself and will get well distributed.
2. Answer: D. the pharmacist, physician, and nurse are all liable for the mistake
The physician, nurse, and pharmacist all are licensed professionals and share responsibility for errors.
3. Answer: B. accumulate.
The decreased circulation to the kidney and reduced liver function tend to allow drugs to accumulate and have toxic
effects.
4. Answer: C. Check the patients wristband
The correct way to identify a patient before giving a medication is to check the name on the medication
administration record with the patients identification band. The nurse should also ask the patient to state their name.
The name on the door or the census list are not sufficient proof of identification. Calling the patient by name is not as
effective as having the patient state their name; patients may not hear well or understand what the nurse is saying,
and may respond to a name which is not their own.
5. Answer: C. take all of the medication prescribed even if the symptoms stop sooner.
Frequently patients do not complete an entire course of antibiotic therapy, and the bacteria are not destroyed.
6. Answer: B. Amitriptyline (Elavil), 10 mg QI
Elavil is an antidepressant that lowers the seizure threshold, so would not be appropriate for this patient. The other
medications are anti-seizure drugs.

7. Answer: C. report your findings to the MD, requesting an alternate medication order
Morphine sulfate depresses the respiratory center. When the rate is less than 10, the MD should be notified.
8. Answer: A. Roquefort, cheddar, or Camembert cheese.
Monoamine oxidase inhibitors react with foods high in the amino acid tyramine to cause dangerously high blood
pressure. Aged cheeses are all high in this amino acid; the other foods are not.
9. Answer: C. meperidine.
The generic name for Demerol is meperidine.
10. Answer: C. Right frequency
The five rights of medication administration are right drug, right dose, right route, right time, right patient. Frequency
is not included.
11. Answer: C. draw the dose from a new vial
The nurse should always inspect the vial of insulin before use for solution changes that may signify loss of potency.
NPH insulin is normally uniformly cloudy. Clumping, frosting, and precipitates are signs of insulin damage. In this
situation, because potency is questionable, it is safer to discard the vial and draw up the dose from a new vial.
12. Answer: C. avoid exposure to sunlight
The client should be taught that ketoconazole is an antifungal medication. It should be taken with food or milk.
Antacids should be avoided for 2 hours after it is taken because gastric acid is needed to activate the medication.
The client should avoid concurrent use of alcohol, because the medication is hepatotoxic. The client should also
avoid exposure to sunlight, because the medication increases photosensitivity.
13. Answer: B. cranberry juice
Cola, coffee, and chocolate contain xanthine and should be avoided by the client taking a xanthine bronchodilator.
This could lead to an increased incidence of cardiovascular and central nervous system side effects that can occur
with the use of these types of bronchodilators.
14. Answer: C. acetaminophen (Tylenol)
The client is taking famotidine, a histamine receptor antagonist. This implies that the client has a disorder
characterized by gastrointestinal (GI) irritation. The only medication of the ones listed in the options that is not
irritating to the GI tract is acetaminophen. The other medications could aggravate an already existing GI problem.

15. Answer: D. pears canned in water


Triamterene is a potassium-sparing diuretic, and clients taking this medication should be cautioned against eating
foods that are high in potassium, including many vegetables, fruits, and fresh meats. Because potassium is very
water-soluble, foods that are prepared in water are often lower in potassium.
16. Answer: A. lactulose (Chronulac)
The client with cirrhosis has impaired ability to metabolize protein because of liver dysfunction. Administration of
lactulose aids in the clearance of ammonia via the gastrointestinal (GI) tract. Ethacrynic acid is a diuretic. Folic acid
and thiamine are vitamins, which may be used in clients with liver disease as supplemental therapy.
17. Answer: A. glycerin emollient
Glycerin is an emollient that is used for dry, cracked, and irritated skin. Aspercreme and Myoflex are used to treat
muscular aches. Acetic acid solution is used for irrigating, cleansing, and packing wounds infected by Pseudomonas
aeruginosa.
18. Answer: B. to rise slowly from a lying to a sitting position
Accupril is an angiotensin-converting enzyme (ACE) inhibitor. It is used in the treatment of hypertension. The client
should be instructed to rise slowly from a lying to sitting position and to permit the legs to dangle from the bed
momentarily before standing to reduce the hypotensive effect. The medication does not need to be taken with meals.
It may be given without regard to food. If nausea occurs, the client should be instructed to take a non cola
carbonated beverage and salted crackers or dry toast. A full therapeutic effect may be noted in 1 to 2 weeks.
19. Answer: D. proteinuria
Auranofin (Ridaura) is a gold preparation that is used as an antirheumatic. Gold toxicity is an adverse effect and is
evidenced by decreased hemoglobin, leukopenia, reduced granulocyte counts, proteinuria, hematuria, stomatitis,
glomerulonephritis, nephrotic syndrome, or cholestatic jaundice. Anorexia, nausea, and diarrhea are frequent side
effects of the medication.
20. Answer: B. calm the persistent cough
Benzonatate is a locally acting antitussive. Its effectiveness is measured by the degree to which it decreases the
intensity and frequency of cough, without eliminating the cough reflex.
1. An infection in a central venous access device is not eliminated by giving antibiotics through the
catheter. How would bacterial glycocalyx contribute to this?

A. It protects the bacteria from antibiotic and immunologic destruction.


B. Glycocalyx neutralizes the antibiotic rendering it ineffective.
C. It competes with the antibiotic for binding sites on the microbe.
D. Glycocalyx provides nutrients for microbial growth.
2. Central venous access devices are beneficial in pediatric therapy because:
A. They dont frighten children.
B. Use of the arms is not restricted.
C. They cannot be dislodged.
D. They are difficult to see.
3. How can central venous access devices (CVADs) be of value in a patient receiving chemotherapy who
has stomatitis and severe diarrhea?
A. The chemotherapy can be rapidly completed allowing the stomatitis and diarrhea to resolve.
B. Crystalloid can be administered to prevent dehydration.
C. Concentrated hyperalimentation fluid can be administered through the CVAD.
D. The chemotherapy dose can be reduced.
4. Some central venous access devices (CVAD) have more than one lumen. These multi lumen catheters:
A. Have an increased risk of infiltration.
B. Only work a short while because the small bore clots off.
C. Are beneficial to patient care but are prohibitively expensive.
D. Allow different medications or solutions to be administered simultaneously.
5. Some institutions will not infuse a fat emulsion, such as Intralipid, into central venous access devices
(CVAD) because:
A. Lipid residue may accumulate in the CVAD and occlude the catheter.
B. If the catheter clogs, there is no treatment other than removal and replacement.
C. Lipids are necessary only in the most extreme cases to prevent essential fatty acid (EFA) deficiency.
D. Fat emulsions are very caustic.
6. A male patient needs a percutaneously inserted central catheter (PICC) for prolonged IV therapy. He
knows it can be inserted without going to the operating room. He mentions that, at least the doctor wont
be wearing surgical garb, will he? How will the nurse answer the patient?
A. You are correct. It is a minor procedure performed on the unit and does not necessitate surgical attire.
B. To decrease the risk of infection, the doctor inserting the PICC will wear a cap, mask, and sterile gown and
gloves.

C. It depends on the doctors preference.


D. Most doctors only wear sterile gloves, not a cap, mask, or sterile gown.
7. A male patient is to receive a percutaneously inserted central catheter (PICC). He asks the nurse whether
the insertion will hurt. How will the nurse reply?
A. You will have general anesthesia so you wont feel anything.
B. It will be inserted rapidly, and any discomfort is fleeting.
C. The insertion site will be anesthetized. Threading the catheter through the vein is not painful.
D. You will receive sedation prior to the procedure.
8. What volume of air can safely be infused into a patient with a central venous access device (CVAD)?
A. It is dependent on the patients weight and height.
B. Air entering the patient through a CVAD will follow circulation to the lungs where it will be absorbed and cause no
problems.
C. It is dependent on comorbidities such as asthma or chronic obstructive lung disease.
D. None.
9. Kent a new staff nurse asks her preceptor nurse how to obtain a blood sample from a patient with a
portacath device. The preceptor nurse teaches the new staff nurse:
A. The sample will be withdrawn into a syringe attached to the portacath needle and then placed into a vacutainer.
B. Portacath devices are not used to obtain blood samples because of the risk of clot formation.
C. The vacutainer will be attached to the portacath needle to obtain a direct sample.
D. Any needle and syringe may be utilized to obtain the sample.
10. What is the purpose of tunneling (inserting the catheter 2-4 inches under the skin) when the surgeon
inserts a Hickman central catheter device? Tunneling:
A. Increases the patients comfort level.
B. Decreases the risk of infection.
C. Prevents the patients clothes from having contact with the catheter
D. Makes the catheter less visible to other people.
11. The primary complication of a central venous access device (CVAD) is:
A. Thrombus formation in the vein.
B. Pain and discomfort.
C. Infection.
D. Occlusion of the catheter as the result of an intra-lumen clot.

12. Nurse Blessy is doing some patient education related to a patients central venous access device.
Which of the following statements will the nurse make to the patient?
A. These type of devices are essentially risk free.
B. These devices seldom work for more than a week or two necessitating replacement.
C. The dressing should only the changed by your doctor.
D. Heparin in instilled into the lumen of the catheter to decrease the risk of clotting.
13. The chemotherapeutic DNA alkylating agents such as nitrogen mustards are effective because they:
A. Cross-link DNA strands with covalent bonds between alkyl groups on the drug and guanine bases on DNA.
B. Have few, if any, side effects.
C. Are used to treat multiple types of cancer.
D. Are cell cycle-specific agents.
14. Hormonal agents are used to treat some cancers. An example would be:
A. Thyroxine to treat thyroid cancer.
B. ACTH to treat adrenal carcinoma.
C. Estrogen antagonists to treat breast cancer.
D. Glucagon to treat pancreatic carcinoma.
15. Chemotherapeutic agents often produce a certain degree of myelosuppression including leukopeniA.
Leukopenia does not present immediately but is delayed several days to weeks because:
A. The patients hemoglobin and hematocrit are normal.
B. Red blood cells are affected first.
C. Folic acid levels are normal.
D. The current white cell count is not affected by chemotherapy.
16. Currently, there is no way to prevent myelosuppression. However, there are medications available to
elicit a more rapid bone marrow recovery. An example is:
A. Epoetin alfa (Epogen, Procrit).
B. Glucagon.
C. Fenofibrate (Tricor).
D. Lamotrigine (Lamictal).
17. Estrogen antagonists are used to treat estrogen hormone-dependent cancer, such as breast carcinomA.
Androgen antagonists block testosterone stimulation of androgen-dependent cancers. An example of an
androgen-dependent cancer would be:

A. Prostate cancer.
B. Thyroid cancer.
C. Renal carcinoma.
D. neuroblastoma.
18. Serotonin release stimulates vomiting following chemotherapy. Therefore, serotonin antagonists are
effective in preventing and treating nausea and vomiting related to chemotherapy. An example of an
effective serotonin antagonist antiemetic is:
A. ondansetron (Zofran).
B. fluoxetine (Prozac).
C. paroxetine (Paxil).
D. sertraline (Zoloft).
19. Methotrexate, the most widely used antimetabolite in cancer chemotherapy does not penetrate the
central nervous system (CNS). To treat CNS disease this drug must be administered:
A. Intravenously.
B. Subcutaneously.
C. Intrathecally.
D. By inhalation.
20. Methotrexate is a folate antagonist. It inhibits enzymes required for DNA base synthesis. To prevent
harm to normal cells, a fully activated form of folic acid known as leucovorin (folinic acid; citrovorum factor)
can be administered. Administration of leucovorin is known as:
A. Induction therapy.
B. Consolidation therapy.
C. Pulse therapy.
D. Rescue therapy.
21. A male Patient is undergoing chemotherapy may also be given the drug allopurinol (Zyloprim, Aloprim).
Allopurinol inhibits the synthesis of uric aciD. Concomitant administration of allopurinol prevents:
A. Myelosuppression.
B. Gout and hyperuricemia.
C. Pancytopenia.
D. Cancer cell growth and replication
22. Superficial bladder cancer can be treated by direct instillation of the antineoplastic antibiotic agent
mitomycin (Mutamycin). This process is termed:

A. Intraventricular administration.
B. Intravesical administration.
C. Intravascular administration.
D. Intrathecal administration.
23. The most common dose-limiting toxicity of chemotherapy is:
A. Nausea and vomiting.
B. Bloody stools.
C. Myelosuppression.
D. Inability to ingest food orally due to stomatitis and mucositis.
24. Chemotherapy induces vomiting by:
A. Stimulating neuroreceptors in the medulla.
B. Inhibiting the release of catecholamines.
C. Autonomic instability.
D. Irritating the gastric mucosa.
25. Myeloablation using chemotherapeutic agents is useful in cancer treatment because:
A. It destroys the myelocytes (muscle cells).
B. It reduces the size of the cancer tumor.
C. After surgery, it reduces the amount of chemotherapy needed.
D. It destroys the bone marrow prior to transplant.
26. Anticipatory nausea and vomiting associated with chemotherapy occurs:
A. Within the first 24 hours after chemotherapy.
B. 1-5 days after chemotherapy.
C. Before chemotherapy administration.
D. While chemotherapy is being administered.
27. Medications bound to protein have the following effect:
A. Enhancement of drug availability.
B. Rapid distribution of the drug to receptor sites.
C. The more drug bound to protein, the less available for desired effect.
D. Increased metabolism of the drug by the liver.
28. Some drugs are excreted into bile and delivered to the intestines. Prior to elimination from the body, the
drug may be absorbed. This process is known as:

A. Hepatic clearance.
B. Total clearance.
C. Enterohepatic cycling.
D. First-pass effect.
29. An adult patient has been taking a drug (Drug A) that is highly metabolized by the cytochrome p-450
system. He has been on this medication for 6 months. At this time, he is started on a second medication
(Drug B) that is an inducer of the cytochrome p-450 system. You should monitor this patient for:
A. Increased therapeutic effects of Drug A.
B. Increased adverse effects of Drug B.
C. Decreased therapeutic effects of Drug A.
D. Decreased therapeutic effects of Drug B.
30. Epinephrine is administered to a female patient. The nurse should expect this agent to rapidly affect:
A. Adrenergic receptors.
B. Muscarinic receptors.
C. Cholinergic receptors.
D. Nicotinic receptors.

Answers & Rationale


Here are the answers and rationale for this exam. Counter check your answers to those below and tell us your
scores. If you have any disputes or need more clarification to a certain question, please direct them to the comments
section.
1. Answer C. It competes with the antibiotic for binding sites on the microbe.
Glycocalyx is a viscous polysaccharide or polypeptide slime that covers microbes. It enhances adherence to
surfaces, resists phagocytic engulfment by the white blood cells, and prevents antibiotics from contacting the
microbe. Glycocalyx does not have the effects in options B-D.
2. Answer B. Use of the arms is not restricted.
The child can move his extremities and function in a normal fashion. This lessens stress associated with position
restriction and promotes normal activity. Fear may not be eliminated. All lines can be dislodged. Even small
catheters can be readily seen.
3. Answer C. Concentrated hyperalimentation fluid can be administered through the CVAD.

In patients unable to take oral nutrition, parenteral hyperalimentation is an option for providing nutritional support.
High concentrations of dextrose, protein, minerals, vitamins, and trace elements can be provided. Dosing is not
affected with options a and d. Crystalloid can provide free water but has very little nutritional benefits.
Hyperalimentation can provide free water and considerable nutritional benefits.
4. Answer D. Allow different medications or solutions to be administered simultaneously.
A multilumen catheter contains separate ports and means to administer agents. An agent infusing in one port cannot
mix with an agent infusing into another port. Thus, agents that would be incompatible if given together can be given
in separate ports simultaneously.
5. Answer A. Lipid residue may accumulate in the CVAD and occlude the catheter.
Occlusion occurs with slow infusion rates and concurrent administration of some medications. Lipid occlusions may
be treated with 70 percent ethanol or with 0.1 mmol/mL NaOH. Lipids provide essential fatty acids. It is
recommended that approximately 4 percent of daily calories be EFAs. A deficiency can quickly develop. Daily
essential fatty acids are necessary for constant prostaglandin production. Lipids are almost isotonic with blood.
6. Answer B. To decrease the risk of infection, the doctor inserting the PICC will wear a cap, mask, and
sterile gown and gloves.
Strict aseptic technique including the use of cap, mask, and sterile gown and gloves is require when placing a
central venous line including a PICC. Options A, C, and D are incorrect statements. They increase the risk of
infection.
7. Answer C. The insertion site will be anesthetized. Threading the catheter through the vein is not painful.
Pain related to PICC insertion occurs with puncture of the skin. When inserting PICC lines, the insertion site is
anesthetized so no pain is felt. The patient will not receive general anesthesia or sedation. Statement 2 is false.
Unnecessary pain should be prevented.
8. Answer D. None.
Any air entering the right heart can lead to a pulmonary embolus. All air should be purged from central venous lines;
none should enter the patient.
9. Answer A. The sample will be withdrawn into a syringe attached to the portacath needle and then placed
into a vacutainer.
A special port-a-cath needle is used to access the port-a-cath device. A syringe is attached and the sample is
obtained. One of the primary reasons for insertion of a portacath device is the need for frequent or long-term blood

sampling. A vacutainer will exert too much suction on the central line resulting in collapse of the line. Only special
portacath needles should be used to access the portacath device.
10. Answer B. Decreases the risk of infection.
The actual access to the subclavian vein is still just under the clavicle, but by tunneling the distal portion of the
catheter several inches under the skin the risk of migratory infection is reduces compared to a catheter that enters
the subclavian vein directly and is not tunneled. The catheter is tunneled to prevent infection.
11. Answer C. Infection.
A foreign body in a blood vessel increases the risk of infection. Catheters that come outside the body have an even
higher risk of infection. Most infections are caused by skin bacteria. Other infective organisms include yeasts and
fungi. Options 1 and 4 are complications of a CVAD but are not the primary problem. Once placed, these lines do
not cause pain and discomfort.
12. Answer D. Heparin in instilled into the lumen of the catheter to decrease the risk of clotting.
A solution containing heparin is used to reduce catheter clotting and maintain patency. The concentration of heparin
used depends on the patients age, comorbidities, and the frequency of catheter access/flushing. Although patients
have few complications, the device is not risk free. Patients may develop infection, catheter clots, vascular
obstruction, pneumothorax, hemothorax, or mechanical problems (catheter breakage). Strict adherence to protocol
enhances the longevity of central access devices. They routinely last weeks to months and sometimes years. The
patient will be taught how to perform dressing changes at home.
13. Answer A. Cross-link DNA strands with covalent bonds between alkyl groups on the drug and guanine
bases on DNA.
Alkylating agents are highly reactive chemicals that introduce alkyl radicals into biologically active molecules and
thereby prevent their proper functioning, replication, and transcription. Alkylating agents have numerous side effects
including alopecia, nausea, vomiting, and myelosuppression. Nitrogen mustards have a broad spectrum of activity
against chronic lymphocytic leukemia, non-Hodgkins lymphoma, and breast and ovarian cancer, but they are
effective chemotherapeutic agents because of DNA cross-linkage. Alkylating agents are noncell cycle-specific
agents.
14. Answer C. Estrogen antagonists to treat breast cancer.
Estrogen antagonists are used to treat estrogen hormone-dependent cancer, such as breast carcinoma. A wellknown estrogen antagonist used in breast cancer therapy is tamoxifen (Nolvadex). This drug, in combination with
surgery and other chemotherapeutic drugs reduces breast cancer recurrence by 30 percent. Estrogen antagonists
can also be administered to prevent breast cancer in women who have a strong family history of the disease.
Thyroxine is a natural thyroid hormone. It does not treat thyroid cancer. ACTH is an anterior pituitary hormone, which

stimulates the adrenal glands to release glucocorticoids. It does not treat adrenal cancer. Glucagon is a pancreatic
alpha cell hormone, which stimulates glycogenolysis and gluconeogenesis. It does not treat pancreatic cancer.
15. Answer D. The current white cell count is not affected by chemotherapy.
The time required to clear circulating cells before the effect that chemotherapeutic drugs have on precursor cell
maturation in the bone marrow becomes evident. Leukopenia is an abnormally low white blood cell count. Answers
A-C pertain to red blood cells.
16. Answer A. Epoetin alfa (Epogen, Procrit).
Epoetin alfa (Epogen, Procrit) is a recombinant form of endogenous erythropoietin, a hematopoietic growth factor
normally produced by the kidney that is used to induce red blood cell production in the bone marrow and reduce the
need for blood transfusion. Glucagon is a pancreatic alpha cell hormone, which cause glycogenolysis and
gluconeogenesis. Fenofibrate (Tricor) is an antihyperlipidemic agent that lowers plasma triglycerides. Lamotrigine
(Lamictal) is an anticonvulsant.
17. Answer A. Prostate cancer.
Prostate tissue is stimulated by androgens and suppressed by estrogens. Androgen antagonists will block
testosterone stimulation of prostate carcinoma cells. The types of cancer in options 2-4 are not androgen dependent.
18. Answer A. ondansetron (Zofran).
Chemotherapy often induces vomiting centrally by stimulating the chemoreceptor trigger zone (CTZ) and
peripherally by stimulating visceral afferent nerves in the GI tract. Ondansetron (Zofran) is a serotonin antagonist
that blocks the effects of serotonin and prevents and treats nausea and vomiting. It is especially useful in single-day
highly emetogenic cancer chemotherapy (for example, cisplatin). The agents in options 2-4 are selective serotonin
reuptake inhibitors. They increase the available levels of serotonin.
19. Answer C. Intrathecally.
With intrathecal administration chemotherapy is injected through the theca of the spinal cord and into the
subarachnoid space entering into the cerebrospinal fluid surrounding the brain and spinal cord. The methods in
options A, B, and D are ineffective because the medication cannot enter the CNS.
20. Answer B. Consolidation therapy.
Leucovorin is used to save or rescue normal cells from the damaging effects of chemotherapy allowing them to
survive while the cancer cells die. Therapy to rapidly reduce the number of cancerous cells is the induction phase.
Consolidation therapy seeks to complete or extend the initial remission and often uses a different combination of

drugs than that used for induction. Chemotherapy is often administered in intermittent courses called pulse therapy.
Pulse therapy allows the bone marrow to recover function before another course of chemotherapy is given.
21. Answer B. Gout and hyperuricemia.
Prevent uric acid nephropathy, uric acid lithiasis, and gout during cancer therapy since chemotherapy causes the
rapid destruction of cancer cells leading to excessive purine catabolism and uric acid formation. Allopurinol can
induce myelosuppression and pancytopenia. Allopurinol does not have this function.
22. Answer B. Intravesical administration.
Medications administered intravesically are instilled into the bladder. Intraventricular administration involves the
ventricles of the brain. Intravascular administration involves blood vessels. Intrathecal administration involves the
fluid surrounding the brain and spinal cord.
23. Answer C. Myelosuppression.
The overall goal of cancer chemotherapy is to give a dose large enough to be lethal to the cancer cells, but small
enough to be tolerable for normal cells. Unfortunately, some normal cells are affected including the bone marrow.
Myelosuppression limits the bodys ability to prevent and fight infection, produce platelets for clotting, and
manufacture red blood cells for oxygen portage. Even though the effects in options a, b, and d are uncomfortable
and distressing to the patient, they do not have the potential for lethal outcomes that myelosuppression has.
24. Answer A. Stimulating neuroreceptors in the medulla.
Vomiting (emesis) is initiated by a nucleus of cells located in the medulla called the vomiting center. This center
coordinates a complex series of events involving pharyngeal, gastrointestinal, and abdominal wall contractions that
lead to expulsion of gastric contents. Catecholamine inhibition does not induce vomiting. Chemotherapy does not
induce vomiting from autonomic instability. Chemotherapy, especially oral agents, may have an irritating effect on the
gastric mucosa, which could result in afferent messages to the solitary tract nucleus, but these pathways do not
project to the vomiting center.
25. Answer A. It destroys the myelocytes (muscle cells).
Myelo comes from the Greek word myelos, which means marrow. Ablation comes from the Latin word ablatio, which
means removal. Thus, myeloablative chemotherapeutic agents destroy the bone marrow. This procedure destroys
normal bone marrow as well as the cancerous marrow. The patients bone marrow will be replaced with a bone
marrow transplant. Myelocytes are not muscle cells Tumors are solid masses typically located in organs. Surgery
may be performed to reduce tumor burden and require less chemotherapy afterward.
26. Answer C. Before chemotherapy administration.

Nausea and vomiting (N&V) are common side effects of chemotherapy. Some patients are able to trigger these
events prior to actually receiving chemotherapy by anticipating, or expecting, to have these effects. N&V occurring
post-chemotherapeutic administration is not an anticipatory event but rather an effect of the drug. N&V occurring
during the administration of chemotherapy is an effect of the drug.
27. Answer C. The more drug bound to protein, the less available for desired effect.
Only an unbound drug can be distributed to active receptor sites. Therefore, the more of a drug that is bound to
protein, the less it is available for the desired drug effect. Less drug is available if bound to protein. Distribution to
receptor sites is irrelevant since the drug bound to protein cannot bind with a receptor site. Metabolism would not be
increased. The liver will first have to remove the drug from the protein molecule before metabolism can occur. The
protein is then free to return to circulation and be used again.
28. Answer C. Enterohepatic cycling.
Drugs and drug metabolites with molecular weights higher than 300 may be excreted via the bile, stored in the
gallbladder, delivered to the intestines by the bile duct, and then reabsorbed into the circulation. This process
reduces the elimination of drugs and prolongs their half-life and duration of action in the body. Hepatic clearance is
the amount of drug eliminated by the liver. Total clearance is the sum of all types of clearance including renal,
hepatic, and respiratory. First-pass effect is the amount of drug absorbed from the GI tract and then metabolized by
the liver; thus, reducing the amount of drug making it into circulation.
29. Answer C. Decreased therapeutic effects of Drug A.
Drug B will induce the cytochrome p-450 enzyme system of the liver; thus, increasing the metabolism of Drug A.
Therefore, Drug A will be broken down faster and exert decreased therapeutic effects. Drug A will be metabolized
faster, thus reducing, not increasing its therapeutic effect. Inducing the cytochrome p-450 system will not increase
the adverse effects of Drug B. Drug B induces the cytochrome p-450 system but is not metabolized faster. Thus, the
therapeutic effects of Drug B will not be decreased.
30. Answer A. Adrenergic receptors.
Epinephrine (adrenaline) rapidly affects both alpha and beta adrenergic receptors eliciting a sympathetic (fight or
flight) response. Muscarinic receptors are cholinergic receptors and are primarily located at parasympathetic
junctions. Cholinergic receptors respond to acetylcholine stimulation. Cholinergic receptors include muscarinic and
nicotinic receptors. Nicotinic receptors are cholinergic receptors activated by nicotine and found in autonomic ganglia
and somatic neuromuscular junctions.
1. Walter, a teenage patient is admitted to the hospital because of acetaminophen (Tylenol) overdose.
Overdoses of acetaminophen can precipitate life-threatening abnormalities in which of the following
organs?

A. Lungs
B. Liver
C. Kidney
D. Adrenal Glands
2. A contraindication for topical corticosteroid usage in a male patient with atopic dermatitis (eczema) is:
A. Parasite infection.
B. Viral infection.
C. Bacterial infection.
D. Spirochete infection.
3. In infants and children, the side effects of first generation over-the-counter (OTC) antihistamines, such as
diphenhydramine (Benadryl) and hydroxyzine (Atarax) include:
A. Reyes syndrome.
B. Cholinergic effects.
C. Paradoxical CNS stimulation.
D. Nausea and diarrhea.
4. Reyes syndrome, a potentially fatal illness associated with liver failure and encephalopathy is associated
with the administration of which over-the-counter (OTC) medication?
A. acetaminophen (Tylenol)
B. ibuprofen (Motrin)
C. aspirin
D. brompheniramine/pseudoephedrine (Dimetapp)
5. The nurse is aware that the patients who are allergic to intravenous contrast media are usually also
allergic to which of the following products?
A. Eggs
B. Shellfish
C. Soy
D. acidic fruits
6. A 13-month-old child recently arrived in the United States from a foreign country with his parents and
needs childhood immunizations. His mother reports that he is allergic to eggs. Upon further questioning,
you determine that the allergy to eggs is anaphylaxis. Which of the following vaccines should he not
receive?

A. Hepatitis B
B. inactivated polio
C. diphtheria, acellular pertussis, tetanus (DTaP)
D. mumps, measles, rubella (MMR)
7. The cell and Coombs classification system categorizes allergic reactions and is useful in describing and
classifying patient reactions to drugs. Type I reactions are immediate hypersensitivity reactions and are
mediated by:
A. immunoglobulin E (IgE).
B. immunoglobulin G (IgG).
C. immunoglobulin A (IgA).
D. immunoglobulin M (IgM).
8. Drugs can cause adverse events in a patient. Bone marrow toxicity is one of the most frequent types of
drug-induced toxicity. The most serious form of bone marrow toxicity is:
A. aplastic anemia.
B. thrombocytosis.
C. leukocytosis.
D. granulocytosis.
9. Serious adverse effects of oral contraceptives include:
A. Increase in skin oil followed by acne.
B. Headache and dizziness.
C. Early or mid-cycle bleeding.
D. Thromboembolic complications.
10. The most serious adverse effect of Alprostadil (Prostin VR pediatric injection) administration in
neonates is:
A. Apnea.
B. Bleeding tendencies.
C. Hypotension.
D. Pyrexia.
11. Mandy, a patient calls the clinic today because he is taking atorvastatin (Lipitor) to treat his high
cholesterol and is having pain in both of his legs. You instruct him to:
A. Stop taking the drug and make an appointment to be seen next week.
B. Continue taking the drug and make an appointment to be seen next week.

C. Stop taking the drug and come to the clinic to be seen today.
D. Walk for at least 30 minutes and call if symptoms continue.
12. Which of the following adverse effects is associated with levothyroxine (Synthroid) therapy?
A. Tachycardia
B. Bradycardia
C. Hypotension
D. Constipation
13. Which of the following adverse effects is specific to the biguanide diabetic drug metformin (Glucophage)
therapy?
A. Hypoglycemia
B. GI distress
C. Lactic acidosis
D. Somnolence
14. The most serious adverse effect of tricyclic antidepressant (TCA) overdose is:
A. Seizures.
B. Hyperpyrexia.
C. Metabolic acidosis.
D. Cardiac arrhythmias.
15. The nurse is aware that the following solutions is routinely used to flush an IV device before and after
the administration of blood to a patient is:
A. 0.9 percent sodium chloride
B. 5 percent dextrose in water solution
C. Sterile water
D. Heparin sodium
16. Chris asks the nurse whether all donor blood products are cross-matched with the recipient to prevent a
transfusion reaction. Which of the following always require cross-matching?
A. packed red blood cells
B. platelets
C. plasma
D. granulocytes

17. A month after receiving a blood transfusion an immunocompromised male patient develops fever, liver
abnormalities, a rash, and diarrhea. The nurse would suspect this patient has:
A. Nothing related to the blood transfusion.
B. Graft-versus-host disease (GVHD).
C. Myelosuppression.
D. An allergic response to a recent medication.
18. Jonas comes into the local blood donation center. He says he is here to donate platelets only today. The
nurse knows this process is called:
A. Directed donation.
B. Autologous donation.
C. Allogeneic donation.
D. Apheresis.
19. Nurse Bryan knows that the age group that uses the most units of blood and blood products is:
A. Premature infants.
B. Children ages 1-20 years.
C. Adults ages 21-64 years.
D. The elderly above age 65 years.
20. A child is admitted with a serious infection. After two days of antibiotics, he is severely neutropeniC.
The physician orders granulocyte transfusions for the next four days. The mother asks the nurse why? The
nurse responds:
A. This is the only treatment left to offer the child.
B. This therapy is fast and reliable in treating infections in children.
C. The physician will have to explain his rationale to you.
D. Granulocyte transfusions replenish the low white blood cells until the body can produce its own.
21. A neighbor tells nurse Maureen he has to have surgery and is reluctant to have any blood product
transfusions because of a fear of contracting an infection. He asks the nurse what are his options. The
nurse teaches the person that the safest blood product is:
A. An allogeneic product.
B. A directed donation product.
C. An autologous product.
D. A cross-matched product.

22. A severely immunocompromised female patient requires a blood transfusion. To prevent GVHD, the
physician will order:
A. Diphenhydramine hydrochloride (Benadryl).
B. The transfusion to be administered slowly over several hours.
C. Irradiation of the donor blood.
D. Acetaminophen (Tylenol).
23. Louie who is to receive a blood transfusion asks the nurse what is the most common type of infection
he could receive from the transfusion. The nurse teaches him that approximately 1 in 250,000 patients
contract:
A. Human immunodeficiency disease (HIV).
B. Hepatitis C infection.
C. Hepatitis B infection.
D. West Nile viral disease.
24. A male patient with blood type AB, Rh factor positive needs a blood transfusion. The Transfusion
Service (blood bank) sends type O, Rh factor negative blood to the unit for the nurse to infuse into this
patient. The nurse knows that:
A. This donor blood is incompatible with the patients blood.
B. Premedicating the patient with diphenhydramine hydrochloride (Benadryl) and acetaminophen (Tylenol) will
prevent any transfusion reactions or side effects.
C. This is a compatible match.
D. The patient is at minimal risk receiving this product since it is the first time he has been transfused with type O,
Rh negative blooD.
25. Dr. Rodriguez orders 250 milliliters of packed red blood cells (RBC) for a patient. This therapy is
administered for treatment of:
A. Thrombocytopenia.
B. Anemia.
C. Leukopenia.
D. Hypoalbuminemia.
26. A female patient needs a whole blood transfusion. In order for transfusion services (the blood bank) to
prepare the correct product a sample of the patients blood must be obtained for:
A. A complete blood count and differential.
B. A blood type and crossmatch.

C. A blood culture and sensitivity.


D. A blood type and antibody screen.
27. A male patient needs to receive a unit of whole blooD. What type of intravenous (IV) device should the
nurse consider starting?
A. A small catheter to decrease patient discomfort
B. The type of IV device the patient has had in the past, which worked well
C. A large bore catheter
D. The type of device the physician prefers
28. Dr. Smith orders a gram of human salt poor albumin product for a patient. The product is available in a
50 milliliter vial with a concentration of 25 percent. What dosage will the nurse administer?
A. The nurse should use the entire 50 milliliter vial.
B. The nurse should determine the volume to administer from the physician.
C. This concentration of product should not be used.
D. The nurse will administer 4 milliliters.
29. Central venous access devices (CVADs) are frequently utilized to administer chemotherapy. What is a
distinct advantage of using the CVAD for chemotherapeutic agent administration?
A. CVADs are less expensive than a peripheral IV.
B. Once a week administration is possible.
C. Caustic agents in small veins can be avoided.
D. The patient or his family can administer the drug at home.
30. A female patients central venous access device (CVAD) becomes infecteD. Why would the physician
order antibiotics to be given through the line rather than through a peripheral IV line?
A. To prevent infiltration of the peripheral line
B. To reduce the pain and discomfort associated with antibiotic administration in a small vein
C. To lessen the chance of an allergic reaction to the antibiotic
D. To attempt to sterilize the catheter and prevent having to remove it

Answers & Rationale


Here are the answers and rationale for this exam. Counter check your answers to those below and tell us your
scores. If you have any disputes or need more clarification to a certain question, please direct them to the comments
section.
1. Answer B. Liver

Acetaminophen is extensively metabolized by pathways in the liver. Toxic doses of acetaminophen deplete hepatic
glutathione, resulting in accumulation of the intermediate agent, quinine, which leads to hepatic necrosis. Prolonged
use of acetaminophen may result in an increased risk of renal dysfunction, but a single overdose does not
precipitate life-threatening problems in the respiratory system, renal system, or adrenal glands.
2. Answer B. Viral infection.
Topical agents produce a localized, rather than systemic effect. When treating atopic dermatitis with a steroidal
preparation, the site is vulnerable to invasion by organisms. Viruses, such as herpes simplex or varicella-zoster,
present a risk of disseminated infection. Educate the patient using topical corticosteroids to avoid crowds or people
known to have infections and to report even minor signs of an infection. Topical corticosteroid usage results in little
danger of concurrent infection with these agents.
3. Answer C. Paradoxical CNS stimulation.
Typically, first generation OTC antihistamines have a sedating effect because of passage into the CNS. However, in
some individuals, especially infants and children, paradoxical CNS stimulation occurs and is manifested by
excitement, euphoria, restlessness, and confusion. For this reason, use of first generation OTC antihistamines has
declined, and second generation product usage has increased. Reyes syndrome is a systemic response to a virus.
First generation OTC antihistamines do not exhibit a cholinergic effect. Nausea and diarrhea are uncommon when
first generation OTC antihistamines are taken.
4. Answer C. aspirin
Virus-infected children who are given aspirin to manage pain, fever, and inflammation are at an increased risk of
developing Reyes syndrome. Use of acetaminophen has not been associated with Reyes syndrome and can be
safely given to patients with fever due to viral illnesses. Ibuprofen adverse effects include GI irritation and bleeding,
and in toxic doses, both renal and hepatic failure are reported. However, ibuprofen has not been associated with the
onset of Reyes disease. Brompheniramine/pseudoephedrine contains a first generation OTC antihistamine and a
decongestant. Neither agent has been associated with the development of Reyes syndrome.
5. Answer B. Shellfish
Some types of contrast media contain iodine as an ingredient. Shellfish also contain significant amounts of iodine.
Therefore, a patient who is allergic to iodine will exhibit an allergic response to both iodine containing contrast media
and shellfish. These products do not contain iodine.
6. Answer D. mumps, measles, rubella (MMR)
The measles portion of the MMR vaccine is grown in chick embryo cells. The current MMR vaccine does not contain
a significant amount of egg proteins, and even children with dramatic egg allergies are extremely unlikely to have an
anaphylactic reaction. However, patients that do respond to egg contact with anaphylaxis should be in a medically

controlled setting where full resuscitation efforts can be administered if anaphylaxis results. The vaccines in options
a,b and c do not contain egg protein.
7. Answer A. immunoglobulin E (IgE).
IgE, the least common serum immunoglobulin (Ig) binds very tightly to receptors on basophils and mast cells and is
involved in allergic reactions. Binding of the allergen to the IgE on the cells results in the release of various
pharmacological mediators that result in allergic symptoms. IgG is the major Ig (75 percent of serum Ig is IgG). Most
versatile Ig because it is capable of carrying out all of the functions of Ig molecules. IgG is the only class of Ig that
crosses the placenta. It is an opsonin, a substance that enhances phagocytosis. IgA, the second most common
serum Ig is found in secretions (tears, saliva, colostrum, and mucus). It is important in local (mucosal) immunity.
IgM, the third most common serum Ig, is the first Ig to be made by the fetus and the first Ig to be made by a virgin B
cell when it is stimulated by antigen. IgM antibodies are very efficient in leading to the lysis of microorganisms.
8. Answer A. aplastic anemia.
Aplastic anemia is the result of a hypersensitivity reaction and is often irreversible. It leads to pancytopenia, a severe
decrease in all cell types: red blood cells, white blood cells, and platelets. A reduced number of red blood cells
causes hemoglobin to drop. A reduced number of white blood cells make the patient susceptible to infection. And, a
reduced number of platelets cause the blood not to clot as easily. Treatment for mild cases is supportive.
Transfusions may be necessary. Severe cases require a bone marrow transplant. Option 2 is an elevated platelet
count. Option 3 is an elevated white count. Option 4 is an elevated granulocyte count. A granulocyte is a type of
white blood cell.
9. Answer D. Thromboembolic complications.
Oral contraceptives have been associated with an increased risk of stroke, myocardial infarction, and deep vein
thrombosis. These risks are increased in women who smoke. Increased skin oil and acne are effects of progestin
excess. Headache and dizziness are effects of estrogen excess. Early or mid-cycle bleeding are effects of estrogen
deficiency.
10. Answer A. Apnea.
All items are adverse reactions of the drug. However, apnea appearing during the first hour of drug infusion occurs in
10-12 percent of neonates with congenital heart defects. Clinicians deciding to utilize alprostadil must be prepared to
intubate and mechanically ventilate the infant. Careful monitoring for apnea or respiratory depression is mandatory.
In some institutions, elective intubation occurs prior to initiation of the medication.
11. Answer C. Stop taking the drug and come to the clinic to be seen today.

Muscle aches, soreness, and weakness may be early signs of myopathy such as rhabdomyolysis associated with
the HMG-CoA reductase class of antilipemic agents. This patient will need an immediate evaluation to rule out
myopathy. Additional doses may exacerbate the problem. Exercise will not reverse myopathy and delays diagnosis.
12. Answer A. Tachycardia
Levothyroxine, especially in higher doses, can induce hyperthyroid-like symptoms including tachycardia. An agent
that increases the basal metabolic rate would not be expected to induce a slow heart rate. Hypotension would be a
side effect of bradycardia. Constipation is a symptom of hypothyroid disease.
13. Answer C. Lactic acidosis
Lactic acidosis is the most dangerous adverse effect of metformin administration with death resulting in
approximately 50 percent of individuals who develop lactic acidosis while on this drug. Metformin does not induce
insulin production; thus, administration does not result in hypoglycemic events. Some nausea, vomiting, and
diarrhea may develop but is usually not severe. NVD is not specific for metformin. Metformin does not induce
sleepiness.
14. Answer D. Cardiac arrhythmias
Excessive ingestion of TCAs result in life-threatening wide QRS complex tachycardia. TCA overdose can induce
seizures, but they are typically not life-threatening. TCAs do not cause an elevation in body temperature. TCAs do
not cause metabolic acidosis.
15. Answer A. 0.9 percent sodium chloride
0.9 percent sodium chloride is normal saline. This solution has the same osmolarity as blood. Its use prevents red
cell lysis. The solutions given in options 2 and 3 are hypotonic solutions and can cause red cell lysis. The solution in
option 4 may anticoagulate the patient and result in bleeding.
16. Answer A. packed red blood cells
Red blood cells contain antigens and antibodies that must be matched between donor and recipient. The blood
products in options 2-4 do not contain red cells. Thus, they require no cross-match.
17.Answer B. Graft-versus-host disease (GVHD)
GVHD occurs when white blood cells in donor blood attack the tissues of an immunocompromised recipient. This
process can occur within a month of the transfusion. Options 1 and 4 may be a thought, but the nurse must
remember that immunocompromised transfusion recipients are at risk for GVHD.
18. Answer D. Apheresis

The process of apheresis involves removal of whole blood from a donor. Within an instrument that is essentially
designed as a centrifuge, the components of whole blood are separated. One of the separated portions is then
withdrawn, and the remaining components are retransfused into the donor. Directed donation is collected from a
blood donor other than the recipient, but the donor is known to the recipient and is usually a family member or friend.
Autologous donation is the collection and reinfusion of the patients own blood. Allogeneic donation is collected from
a blood donor other than the recipient.
19. Answer D. The elderly above age 65 years.
People older than 65 years use 43 percent of donated blood. This number is expected to increase as the population
ages.
20. Answer D. Granulocyte transfusions replenish the low white blood cells until the body can produce its own.
Granulocyte (neutrophil) replacement therapy is given until the patients blood values are normal and he is able to
fight the infection himself. Options 1 and 3 are not therapeutic responses. The treatment in option 2 takes days and
is not always able to prevent morbidity and mortality.
21. Answer C. An autologous product.
This process is the collection and reinfusion of the patients own blood. It is recommended by the American Medical
Associations Council on Scientific Affairs as the safest product since it eliminates recipient incompatibility and
infection. The product in option 1 is collected from a blood donor other than the recipient. The process in option 2 is
also collected from a blood donor other than the recipient, but the donor is known to the recipient and is usually a
family member or friend. Cross-matching significantly enhances compatibility. It does not detect infection.
22. Answer C. Irradiation of the donor blood.
This process eliminates white blood cell functioning, thus, preventing GVHD. Diphenhydramine HCl is an
antihistamine. Its use prior to a blood transfusion decreases the likelihood of a transfusion reaction. Option 2 will not
prevent GVHD. Use of acetaminophen prevents and treats the common side effects of blood administration caused
by the presence of white blood cells in the transfusion product: fever, headache, and chills.
23. Answer C. Hepatitis B infection.
Hepatitis B is the most common infection spread via blood transfusion. Donors are screened by a questionnaire that
includes symptoms. The donated blood is also tested for infection. The risk of infection with the agents in options 2
and 3 has decreased to approximately 1 in 2 million secondary to donor questioning and donor blood testing. The
incidence of West Nile viral transmission is unknown, but donor infection is still relatively rare.
24. Answer C. This is a compatible match.

Type O, Rh negative blood has none of the major antigens and is safely administered to patients of all blood types. It
is also known as the universal donor. Premedicating with these agents will not prevent a major transfusion reaction if
the blood type and Rh factors of the donor blood are incompatible with the recipients blood.
25. Answer B. Anemia.
A red blood cell transfusion is used to correct anemia in patients in which the low red blood cell count must be
rapidly corrected. RBC transfusion will not correct a low platelet count. RBC transfusion will not correct a low white
blood cell count. Packed RBCs contain very little plasma and, thus, only a small amount of albumin. This amount will
not correct low albumin levels.
26. Answer B. A blood type and crossmatch.
This is needed to utilize the correct type of donor blood and to match the donor product with the patient.
Incompatible matches would result in severe adverse events and possible death. The tests in options 1 and 3 are
unnecessary. The test in option 4 is utilized to determine the patients blood type and presence of antibodies to
blood antigens. It does not determine donor blood compatibility with the patient.
27. Answer C. A large bore catheter
Large bore catheters prevent damage to blood components and are less likely to develop clotting problems than a
small bore catheter. The nurse should determine the correct device without asking the patient what type has been
used before or asking the physician which type he prefers and start the IV.
28. Answer D. The nurse will administer 4 milliliters.
A 25 percent solution contains one quarter of a gram per milliliter. Thus, the nurse will administer 4 milliliters to
provide a complete gram of albumin. The volume in option 1 would provide 12.5 grams of albumin. The nurse should
determine the volume. It is unnecessary to seek the answer from the physician. A 25 percent solution is an
acceptable product and can safely be used.
29. Answer C. Caustic agents in small veins can be avoided.
Many chemotherapeutic drugs are vesicants (highly active corrosive materials that can produce tissue damage even
in low concentrations). Extravasations of a vesicant can result in significant tissue necrosis. Administration into a
large vein is optimal. CVADs are more expensive than a peripheral IV. Dosing depends on the drug. IV
chemotherapeutic agents are not administered at home. They are given in an outpatient or clinic setting if not given
during hospitalization.
30. Answer D. To attempt to sterilize the catheter and prevent having to remove it

Microorganisms that infect CVADs are often coagulase-negative staphylococci, which can be eliminated by antibiotic
administration through the catheter. If unsuccessful in eliminating the microorganism, the CVAD must be removed.
CVAD use lessens the need for peripheral IV lines and, thus, the risk of infiltration. In this case however, the
antibiotics are given to eradicate microorganisms from the CVAD. CVAD use has this effect, but in this case, the
antibiotics are given through the CVAD to eliminate the infective agent. The third option would not occur.
Situation 1 Mr. Ibarra is assigned to the triage area and while on duty, he assesses the condition of Mrs. Simon
who came in with asthma. She has difficulty breathing and her respiratory rate is 40 per minute. Mr. Ibarra is asked
to inject the client epinephrine 0.3mg subcutaneously
1. The indication for epinephrine injection for Mrs Simon is to:
a. Reduce anaphylaxis
b. Relieve hypersensitivity to allergen
c. Relieve respiratory distress due to bronchospasm
d. Restore clients cardiac rhythm
2. When preparing the epinephrine injection from an ampule, the nurse initially:
a. Taps the ampule at the top to allow fluid to flow to the base of the ampule
b. Checks expiration date of the medication ampule
c. Removes needle cap of syringe and pulls plunger to expel air
d. Breaks the neck of the ampule with a gauze wrapped around it
3. Mrs. Simon is obese. When administering a subcutaneous injection to an obese patient, it is best for the
nurse to:
a Inject needle at a 15 degree angle over the stretched skin of the client
b. Pinch skin at the Injection site and use airlock technique
c. Pull skin of patient down to administer the drug in a Z track
d. Spread skin or pinch at the injection site and inject needle at a 45-90 degree angle
4. When preparing for a subcutaneous injection, the proper size of syringe and needle would be:
a. Syringe 3-5ml and needle gauge 21 to 23
b. Tuberculin syringe 1 ml with needle gauge 26 or 27
c. Syringe 2ml and needle gauge 22
d. Syringe 1-3 ml and needle gauge 25 to 27
5. The rationale for giving medications through the subcutaneous route is:

a. There are many alternative sites for subcutaneous injection


b. Absorption time of the medicine is slower
c. There are less pain receptors in this area
d. The medication can be injected while the client is in any position
Situation 2 The use of massage and meditation to help decrease stress and pain have been strongly
recommended based on documented testimonials.
6. Martha wants to do a study on, this topic. Effects of massage and meditation on stress and pain. The
type of research that best suits this topic is:
a. applied research
b. qualitative research
c. basic research
d. quantitative research
7. The type of research design that does not manipulate independent variable is:
a. experimental design
b. quasi-experimental design
c. non-experimental design
d. quantitative design
8. This research topic has the potential to contribute to nursing because it seeks to:
a. include new modalities of care
b. resolve a clinical problem
c. clarify an ambiguous modality of care
d. enhance client care
9. Martha does review of related literature for the purpose of:
a. determine statistical treatment of data research
b. gathering data about what is already known or unknown
c. to identify if problem can be replicated
d. answering the research question
10. Clients rights should be protected when doing research using human subjects. Martha identifies these
rights as follows EXCEPT:
a. right of self-determination
b. right to compensation

c. right of privacy
d. right not to be harmed
Situation 3 Richard has a nursing diagnosis of ineffective airway clearance related to excessive secretions and is
at risk for infection because of retained secretions. Part of Nurse Marios nursing care plan is to loosen and remove
excessive secretions in the airway,
11. Mario listens to Richards bilateral sounds and finds that congestion is in the upper lobes of the lungs.
The appropriate position to drain the anterior and posterior apical segments of the lungs when Mario does
percussion would be:
a. Client lying on his back then flat on his abdomen on Trendelenburg position
b. Client seated upright in bed or on a chair then leaning forward in sitting position then flat on his back and on his
abdomen
c. Client lying flat on his back and then flat on his abdomen
d. Client lying on his right then left side on Trendelenburg position
12. When documenting outcome of Richards treatment Mario should include the following in his recording
EXCEPT:
a. Color, amount and consistent of sputum
b. Character of breath sounds and respiratory/rate before and after procedure
c. Amount of fluid intake of client before and after the procedure
d. Significant changes in vital signs
13. When assessing Richard for chest percussion or chest vibration and postural drainage Mario would
focus on the following EXCEPT:
a. Amount of food and fluid taken during the last meal before treatment
b. Respiratory rate, breath sounds and location of congestion
c. Teaching the clients relatives to perform the procedure
d. Doctors order regarding position restriction and clients tolerance for lying flat
14. Mario prepares Richard for postural drainage and percussion. Which of the following is a special
consideration when doing the procedure?
a. Respiratory rate of 16 to 20 per minute
b. Client can tolerate sitting and lying position
c. Client has no signs of infection
d. Time of fast food and fluid intake of the client

15. The purpose of chest percussion and vibration is to loosen secretions in the lungs. The difference
between the procedure is:
a. Percussion uses only one hand white vibration uses both hands
b. Percussion delivers cushioned blows to the chest with cupped palms while gently shakes secretion loose on the
exhalation cycle
c. In both percussion and vibration the hands are on top of each other and hand action is in tune with clients breath
rhythm
d. Percussion slaps the chest to loosen secretions while vibration shakes the secretions along with the inhalation of
air
Situation 4 A 61 year old man, Mr. Regalado, is admitted to the private ward for observation after complaints of
severe chest pain. You are assigned to take care of the client.
16. When doing an initial assessment, the best way for you to identify the clients priority problem is to:
a. Interview the client for chief complaints and other symptoms
b. Talk to the relatives to gather data about history of illness
c. Do auscultation to check for chest congestion
d. Do a physical examination white asking the client relevant questions
17. Upon establishing Mr. Regalados nursing needs, the next nursing approach would be to:
a. introduce the client to the ward staff to put the client and family at ease
b. Give client and relatives a brief tour of the physical set up the unit
c. Take his vital signs for a baseline assessment
d. Establish priority needs and implement appropriate interventions
18. Mr. Regalado says he has trouble going to sleep. In order to plan your nursing intervention you will.
a. Observe his sleeping patterns in the next few days
b. Ask him what he means by this statement
c. Check his physical environment to decrease noise level
d. Take his blood pressure before sleeping and upon waking up
19. Mr. Regalados lower extremities are swollen and shiny. He has pitting pedal edema. When taking care of
Mr. Regalado, which of the following intervention would be the most appropriate immediate nursing
approach.
a. Moisturize lower extremities to prevent skin irritation
b. Measure fluid intake and output to decrease edema

c. Elevate lower extremities for postural drainage


d. Provide the client a list of food low in sodium
20. Mr. Regalado will be discharged from your unit within the hour. Nursing actions when preparing a client
for discharge include all EXCEPT:
a. Making a final physical assessment before client leaves the hospital
b. Giving instructions about his medication regimen
c. Walking the client to the hospital exit to ensure his safety
d. Proper recording of pertinent data
Situation 5 Nancy, mother of 2 young kids. 36 years old, had a mammogram and was told that she has breast
cysts and that she may need surgery. This causes her anxiety as shown by increase in her pulse and respiratory
rate, sweating and feelings of tension.
21. Considering her level of anxiety, the nurse can best assist Nancy by:
a. Giving her activities to divert her attention
b. Giving detailed explanations about the treatments she will undergo
c. Preparing her and her family in case surgery is not successful
d. Giving her clear but brief information at the level of her understanding
22. Nancy blames God for her situation. She is easily provoked to tears and wants to be left alone, refusing
to eat or talk to her family. A religious person before, she now refuses to pray or go to church stating that
God has abandoned her. The nurse understands that Nancy is grieving for her self and is in the stage of:
a. bargaining
b. denial
c. anger
d. acceptance
23. The nurse visits Nancy and prods her to eat her food. Nancy replies whats the use? My time is running
out. The nurses best response would be:
a. The doctor ordered full diet for you so that you will be strong for surgery.
b. I understand how you fee! but you have 1o try for your childrens sake.
c. Have you told your, doctor how you feel? Are you changing your mind) about surgery?
d. You sound like you are giving up.
24. The nurse feels sad about Nancys illness and tells her head nurse during the end of shift endorsement
that its unfair for Nancy to have cancer when she is still so young and with two kinds. The best response
of the head nurse would be:

a. Advise the nurse to be strong and learn to control her feelings


b. Assign the nurse to another client to avoid sympathy for the client
c. Reassure the nurse that the client has hope if she goes through all statements prescribed for her
c. Ask the other nurses what they feel about the patient to find out if they share the same feelings
25. Realizing that she feels angry about Nancys condition, the nurse sees that being self-aware is a
conscious process that she should do in any situation like this because:
a. This is a necessary part of the nurse -client relationship process
b. The nurse is a role model for the client and should be strong
C. How the nurse thinks and feels affect her actions towards her client and her work
d. The nurse has to be therapeutic at all times and should not be affected
Situation 6 Mrs. Seva, 32 years old, asks you about possible problems regarding her elimination now that she is in
the menopausal stage.
26. Instruction on health promotion regarding urinary elimination is important. Which would you include?
a. Hold urine, as long as she can before emptying the bladder to strengthen her sphincters muscles
b. If burning sensation is experienced while voiding, drink pineapple-juice
c. After urination, wipe from anal area up towards the pubis
d. Jell client to empty the bladder at each voiding
27. Mrs. Seva also tells the nurse that she is often constipated. Because she is aging, what physical
changes predispose her to constipation?
a. inhibition of the parasympathetic reflex
b. weakness of sphincter muscles of the anus
c. loss of tone of the smooth muscles of the color
d. decreased ability to absorb fluids in the lower intestines
28. The nurse understands that one of these factors contributes to constipation:
a. excessive exercise
b. high fiber diet
c. no regular time for defecation daily
d. prolonged use of laxatives
29. Mrs. Seva talks about rear of being incontinent due to a prior experience of dribbling urine when
laughing or sneezing and when she has a full bladder. Your most appropriate .instruction would be to:

a. tell client to drink less fluids to avoid accidents


b. instruct client to start wearing thin adult diapers
c. ask the client to bring change of underwear just in case
d. teach client pelvic exercise to strengthen perineal muscles
30. Mrs. Seva asked for instructions for skin care for her mother who has urinary incontinence and is almost
always in bed. Your instruction would focus on prevention of skin irritation and breakdown by
a. Using thick diapers to absorb urine well
b. Drying the skin with baby powder to prevent or mask the smell of ammonia
c. Thorough washing, rising and during of skin area that get wet with urine
d. Making sure that linen are smooth and dry at all times
Situation 7 Using Maslows need theory, Airway, Breathing and Circulation are the physiological needs vital to life.
The nurses knowledge and ability to identify and immediately intervene to meet these needs is important to save
lives.
31. Which of these clients has a problem with the transport of oxygen from the lungs to the tissues:
a. Carol with a tumor in the brain
b. Theresa with anemia
c. Sonny Boy with a fracture in the femur
d. Brigette with diarrhea
32. You noted from the lab exams in the chart of Mr. Santos that he has reduced oxygen in the blood.
This condition is called:
a. Cyanosis
b. Hypoxia
c. Hypoxemia
d. Anemia
33. You will nasopharyngeal suctioning Mr. Abad. Your guide for the length of insertion of the tubing for an
adult would be:
a. tip of the nose to the base of the .neck
b. the distance from the tip of the nose to the middle of the cheek
c. the distance from the tip of the nose to the tip of the ear lobe
d. eight to ten inches
34. While doing nasopharyngeal suctioning on .Mr. Abad, the nurse can avoid trauma to the area by:

a. Apply suction for at least 20-30 seconds each time to ensure that all secretions are removed
b. Using gloves to prevent introduction of pathogens to the respiratory system
c. Applying no suction while inserting the catheter
d. Rotating catheter as it is inserted with gentle suction
35. Myrna has difficulty breathing when on her back and must sit upright in bed to breath, effectively and
comfortably. The nurse documents this condition as:
a. Apnea
b. Orthopnea
c. Dyspnea
d. Tachypnea
Situation 8 You are assigned to screen for hypertension: Your task is to take blood pressure readings and you are
informed about avoiding the common mistakes in BP taking that lead to false or inaccurate blood pressure readings.
36. When taking blood pressure reading the cuff should be:
a. deflated fully then immediately start second reading for same client
b deflated quickly after inflating up to 180 mmHg
c. large enough to wrap around upper arm of the adult client 1 cm above brachial artery
d. inflated to 30 mmHg above the estimated systolic BP based on palpation of radial or bronchial artery
37. Chronic Obstructive Pulmonary Disease (COPD) in one of the leading causes of death worldwide and is
a preventable disease. The primary cause of COPD is:
a. tobacco hack
b. bronchitis
c. asthma
d. cigarette smoking
38. In your health education class for clients with diabetes you teach, them the areas, for control . Diabetes
which include all EXCEPT:
a. regular physical activity
b. thorough knowledge of foot care
c. prevention nutrition
d. proper nutrition
39. You teach your clients the difference between, Type I (IDDM) and Type II (NDDM) Diabetes. Which of the
following is true?

a. both types diabetes mellitus clients are all prone to developing ketosis
b. Type II (NIDDM) is more common and is also preventable compared to Type I (IDDM) diabetes which is genetic in
etiology
c. Type I (IDDM) is characterized by fasting hyperglycemia
d. Type II (NIDDM) is characterized by abnormal immune response
40. Lifestyle-related diseases in general share areas common risk factors. These are the following except
a. physical activity
b. smoking
c. genetics
d. nutrition
Situation 9 Nurse Rivera witnesses a vehicular accident near the hospital where she works. She decides to get
involved and help the victims of the accident.
41. Her priority nursing action would be to:
a. Assess damage to property
b. Assist in the police investigation since she is a witness
c. Report the incident immediately to the local police authorities
d. Assess the extent of injuries incurred by the victims, of the accident
42. Priority attention should be given to which of these clients?
a. Linda who shows severe anxiety due to trauma of the accident
b. Ryan who has chest injury, is pate and with difficulty of breathing
c. Noel who has lacerations on the arms with mild-bleeding
c. Andy whose left ankle swelled and has some abrasions
43. In the emergency room, Nurse Rivera is assigned to attend to the client with .lacerations on the arms,
while assessing the extent of the wound the nurse observes that the wound is now starting to bleed
profusely. The most immediate nursing action would be to:
a. Apply antiseptic to prevent infection
b. Clean the wound vigorously of contaminants
c. Control and. reduce bleeding of the wound
d. Bandage the wound and elevate the arm
44. The nurse applies pressure dressing on the bleeding site. This intervention is done to:
a. Reduce the need to change dressing frequently
b. Allow the pus to surface faster

c. Protect the wound from micro organisms in the air


d. Promote hemostasis
45. After the treatment, the client is sent home and asked to come back for follow-up care. Your
responsibilities when the client is to be discharged include the following EXCEPT:
a. Encouraging the client to go to the, outpatient clinic for follow up care
b. Accurate recording, of treatment done and instructions given to client
c. Instructing the client to see you after discharge for further assistance
d. Providing instructions regarding wound care
Situation 10 While working in the clinic, a new client, Geline, 35 years old, arrives for her doctors appointment. As
the clinic nurse, you are to assist the client fill up forms, gather data and make an assessment.
46. The nurse purpose of your initial nursing interview is to:
a. Record pertinent information in the client chart for health team to read
b Assist the client find solutions to her health concerns
c. Understand her lifestyle, health needs and possible problems to develop a plan of care
d. Make nursing diagnoses for identified health problems
47. While interviewing Geline, she starts to moan and doubles up in pain, She tells you that this pain occurs
about an hour after taking black coffee without breakfast for a few weeks now. You will record this as
follows:
a. Claims to have abdominal pains after intake of coffee unrelieved by analgesics
b. After drinking coffee, the client experienced severe abdominal pain
c. Client complained of intermittent abdominal pain an hour after drinking coffee
d. Client reported abdominal pain an hour after drinking black coffee for three weeks now
48. Geline tells you that she drinks black coffee frequently within the day to have energy and be wide
awake and she eats nothing for breakfast and eats strictly vegetable salads for lunch and dinner to lose
weight. She has lost weight during the past two weeks, in planning a healthy balanced diet with Geline, you
will:
a. Start her off with a cleansing diet to free her body of toxins then change to a vegetarian, diet and drink plenty of
fluids
b. Plan a high protein, diet, low carbohydrate diet for her considering her favorite food
c. Instruct her to attend classes in nutrition to find food rich in complex carbohydrates to maintain daily high energy
level
d. Discuss with her the importance of eating a variety of food from the major food groups with plenty of fluids

49. Geline tells you that she drinks 4-5 cups of black coffee and diet cola drinks. She also smokes up to a
pack of cigarettes daily. She confesses that she is in her 2nd month of pregnancy but she does not want to
become fat that is why she limits her food intake. You warn or caution her about which of the following?
a. Caffeine products affect the central nervous system and may cause the mother to have a nervous breakdown
b. Malnutrition and its possible effects on growth and development problems in the unborn fetus
c. Caffeine causes a stimulant effect on both the mother and the baby
d. Studies show conclusively that caffeine causes mental retardation
50. Your health education plan for Geline stresses proper diet for a pregnant woman and the prevention
of noncommunicable diseases that are influenced by her lifestyle these include of the following EXCEPT:
a. Cardiovascular diseases
b. Cancer
c. Diabetes Mellitus
d. Osteoporosis
Situation 11 Management of nurse practitioners is done by qualified nursing leaders who have had clinical
experience and management experience.
51. An example of a management function of a nurse is:
a. Teaching patient do breathing and coughing exercises
b. Preparing for a surprise party for a client
c. Performing nursing procedures for clients
d. Directing and evaluating the staff nurses
52. Your head nurse in the unit believes that the staff nurses are not capable of decision making so she
makes the decisions for everyone without consulting anybody. This type of leadership is:
a. Laissez faire leadership
b. Democratic leadership
c. Autocratic leadership
d. Managerial leadership
53. When the head nurse in your ward plots and approves your work schedules and directs your work, she
is demonstrating:
a. Responsibility
b. Delegation
c. Accountability
d. Authority

54. The following tasks can be safely delegated by a nurse to a non-nurse health worker EXCEPT:
a. Transfer a client from bed to chair
b. Change IV infusions
c. Irrigation of a nasogastric tube
d. Take vital signs
55. You made a mistake in giving the medicine to the wrong client You notify the clients doctor and write an
incident report. You are demonstrating:
a. Responsibility
b. Accountability
c. Authority
d. Autocracy
Situation 12 Mr. Dizon, 84 years old, is brought to the Emergency Room for complaint of hypertension flushed
face, severe headache, and nausea. You are doing the initial assessment of vital signs.
56. You are to measure the clients initial blood pressure reading by doing all of the following EXCEPT:
a. Take the blood pressure reading on both arms for comparison
b. Listen to and identify the phases of Korotkoffs sounds
c. Pump the cuff up to around 50 mmHg above the point where the pulse is obliterated
d. Observe procedures for infection control
57. A pulse oximeter is attached to Mr. Dizons finger to:
a. Determine if the clients hemoglobin level is low and if he needs blood transfusion
b. Check level of clients tissue perfusion
c. Measure the efficacy of the clients antihypertensive medications
d. Detect oxygen saturation of arterial blood before symptoms of hypoxemia develops
58. After a few hours in the Emergency Room, Mr. Dizon is admitted to the ward with an order of hourly
monitoring of blood pressure. The nurse finds that the cuff is too narrow and this will cause the blood
pressure reading to be:
a. Inconsistent
b. low systolic and high diastolic pressure
c. higher than what the reading should be
d. lower than what the reading should be

59. Through the clients health history, you gather that Mr. Dizon smokes and drinks coffee. When taking the
blood pressure of a client who recently smoked or drank coffee, how long should be the nurse wait before
taking the clients blood pressure for accurate reading?
a. 15 minutes
b. 30 minutes
c. 1 hour
d. 5 minutes
60. While the client has the pulse oximeter on his fingertip, you notice that the sunlight is shining on .the
area where the oximeter is. Your action will be to:
a. Set and turn on the alarm of the oximeter
b. Do nothing since there is no identified problem
c. Cover the fingertip sensor with a towel or bedsheet
d. Change the location of the sensor every four hours
Situation 13 The nurses understanding of ethico-legal responsibilities will guide his/her nursing practice.
61. The principles that .govern right and proper conducts of a person regarding life, biology and the health
professions is referred to as:
a. Morality
b. Religion
c. Values
d. Bioethics
62. The purpose of having nurses code of ethics is:
a. Delineate the scope and areas of nursing practice
b. Identify nursing action recommended for specific health care situations
c. To help the public understand professional conduct, expected of nurses
d. To define the roles and functions of the health caregiver, nurses, clients
63. The most important nursing responsibility where ethical situations emerge in patient care is to:
a. Act only when advised that the action is ethically sound
b. Not take sides remain neutral and fair
c. Assume that ethical questions are the responsibility: of the health team
d. Be accountable for his or her own actions
64. You inform the patient about his rights which include the following EXCEPT:

a. Right to expect reasonable continuity of care


b. Right to consent to or decline to participate in research studies or experiments
c. Right to obtain information about another patient
d. Right to expect that the records about his care will be treated as confidential
65. The principle states that a person has unconditional worth and has the capacity to determine his own
destiny.
a. Bioethics
b. Justice
c. Fidelity
d. Autonomy
Situation 14 Your director of nursing wants to improve the quality of health care offered in the hospital. As a staff
nurse in that hospital you know that this entails quality assurance programs.
66. The following mechanisms can be utilized as part of the quality assessment program of your hospital
EXCEPT:
a. Patient satisfaction surveys provided
b. Peer review clinical records of care of client
c. Review of the Nursing Intervention Classification
d. Use of Nursing Interventions Classification
67. The nurse of the Standards of Nursing Practice is important in the hospital. Which of the following
statements best describes what it is?
a. These are statements that describe the maximum or highest level of acceptable performance in nursing practice.
b. It refers to the scope of nursing as defined in Republic Act 9173
c. It is a license issued by the Professional Regulation Commission to protect the public from substandard nursing
practice.
d. The Standards of care includes the various steps of the nursing process and the standards of professional
performance.
68. You are taking care of critically ill client and the doctor in charge calls to order a DNR (do not
resuscitate) for the client. Which of the following is the appropriate action when getting DNR order over the
phone?
a. Have the registered nurse, family spokesperson, nurse supervisor and doctor sign
b. Have two nurses validate the phone order, both nurses sign the order and the doctor should sign his order within
24 hours.

c. Have the registered nurse, family and doctor sign the order
d. Have 1 nurse take the order and sign it and have the doctor sign it within 24 hours
69. To ensure the client safety before starting blood transfusion the following are needed before the
procedure can be done EXCEPT:
a. take baseline vital signs
b. blood should be warmed to room temperature for 30 minutes before blood transfusion is administered
c. have two nurses verify client identification, blood type, unit number and expiration date of blood
d. get a consent signed for blood transfusion
70. Part of standards of care has to do with the use of restraints. Which of the following statements is NOT
true?
a. Doctors order for restraints should be signed within 24 hours
b. Remove and reapply restraints every two hours
c. Check clients pulse, blood pressure and circulation every four hours
d. Offer food and toileting every two hours
Situation 15 During the NUTRITION EDUCATION class discussion a 58 year old man, Mr. Bruno shows increased
interest.
71. Mr. Bruno asks what the normal allowable salt intake is. Your best response to Mr. Bruno is:
a. 1 tsp of salt/day with iodine and sprinkle of MSG
b. 5 gms per day or 1 tsp of table salt/day
c. 1 tbsp of salt/day with some patis and toyo
d. 1 tsp of salt/day but not patis or toyo
72. Your instructions to reduce or limit salt intake include all the following EXCEPT:
a. eat natural food with little or no salt added
b. limit use of table salt and use condiments instead
c. use herbs and spices
d. limit intake of preserved or processed food
73. Teaching strategies and approaches when giving nutrition education is influenced by age, sex and
immediate concerns of the group. Your presentation for a group of young mothers would be best if you
focus on:
a. diets limited in salt and fat
b. harmful effect on drugs and alcohol intake

c. commercial preparation of dishes


d. cooking demonstration and meal planning
74. Cancer cure is dependent on
a. use of alternative methods of healing
b. watching out for warning signs of cancer
c. proficiency in doing breast self-examination
d. early detection and prompt treatment
75. The role of the health worker in health education is to:
a. report incidence of non-communicable disease to community health center
b. educate as many people about warning signs of non-communicable diseases
c. focus on smoking cessation projects
d. monitor clients with hypertension
Situation 16 You are assigned to take care of 10 patients during the morning shift. The endorsement includes the
IV infusion and medications for these clients.
76. Mr. Felipe, 36 years old is to be given 2700 ml of D5RL to infuse for 18 hours starting at 8am. At what
rate should the IV fluid be flowing hourly?
a. 100 ml/hour
b. 210 ml/hour
c. 150 ml/hour
d. 90 ml/hour
77. Mr. Atienza is to receive 150 mg/hour of D5W IV infusion for 12 hours for a total of 1800 ml. He is
also losing gastric fluid which must be replaced every two hours. Between 8am to 10am. Mr. Atienza has
lost 250 ml of gastric fluid. How much fluid should he receive at 11am?
a. 350 ml/hour
b. 275 ml/hour
c. 400 ml/hour
d. 200 ml/hour
78. You are to apply a transdermal patch of nitroglycerin to your client. The following important guidelines
to observe EXCEPT:
a. Apply to hairlines clean are of the skin not subject to much wrinkling
b. Patches may be applied to distal part of the extremities like forearm

c. Change application and site regularly to prevent irritation of the skin


d. Wear gloves to avoid any medication of your hand
79. You will be applying eye drops to Miss Romualdez. After checking all the necessary information and
cleaning the affected eyelid and eyelashes you administer the ophthalmic drops by instilling the eye drops.
a. directly onto the cornea
b. pressing on the lacrimal duct
c. into the outer third of the lower conjunctival sac
d. from the inner canthus going towards the side of the eye
80. When applying eye ointment, the following guidelines apply EXCEPT:
a. squeeze about 2 cm of ointment and gently close but not squeeze eye
b. apply ointment from the inner canthus going outward of the affected eye
c. discard the first bead of the eye ointment before application because the tube likely to expel more than desired
amount of ointment
d. hold the tube above the conjunctival sac do not let tip touch the conjunctiva
Situation 17 The staff nurse supervisor request all the staff nurses to brainstorm and learn ways to instruct
diabetic clients on self-administration of insulin. She wants to ensure that there are nurses available daily to do
health education classes.
81. The plan of the nurse supervisor is an example of
a. in service education process
b. efficient management of human resources
c. increasing human resources
d. primary prevention
82. When Mrs. Guevarra, a nurse, delegates aspects of the clients care to the nurse-aide who is an
unlicensed staff, Mrs. Guevarra.
a. makes the assignment to teach the staff member
b. is assigning the responsibility to the aide but not the accountability for those tasks
c. does not have to supervise or evaluate the aide
d. most know how to perform task delegated
83. Connie, the-new nurse, appears tired and sluggish and lacks the enthusiasms she give six weeks ago
when she started the job. The nurse supervisor should:

a. empathize with the nurse and listen to her


b. tell her to take the day off
c. discuss how she is adjusting to her new job
d. ask about her family life
84. Process of formal negotiations of working conditions between a group of registered nurses and
employer is:
a. grievance
b. arbitration
c. collective bargaining
d. strike
85. You are attending a certification program on cardiopulmonary resuscitation (CPR) offered and required
by the hospital employing you. This is:
a. professional course towards credits
b. in-service education
c. advance training
d. continuing education
Situation 18 There are various developments in health education that the nurse should know about.
86. The provision of health information in the rural areas nationwide through television and radio programs
and video conferencing is referred to as:
a. Community health program
b. Telehealth program
c. Wellness program
d. Red cross program
87. A nearby community provides blood pressure screening, height and weight measurement smoking
cessation classes and aerobics class services. This type of program is referred to as:
a. outreach program
b. hospital extension program
c. barangay health center
d. wellness center
88. Part of teaching client in health promotion is responsibility for ones health. When Danica states she
need to improve her nutritional status this means:

a. Goals and interventions to be followed by client are based on nurses priorities


b. Goals and intervention developed by nurse and client should be approved by the doctor
c. Nurse will decide goals and, interventions needed to meet client goals
d. Client will decide the goals and interventions required to meet her goals
89. Nurse Beatrice is providing tertiary prevention to Mrs. De Villa. An example of tertiary prevention is:
a. Marriage counseling
b. Self-examination for breast cancer
c. Identifying complication of diabetes
d. Poison, control
90. Mrs. Ostrea has a schedule for Pap Smear. She has a strong family history of cervical cancer. This is an
example of:
a. tertiary prevention
b. secondary prevention
c. health screening
d. primary prevention
Situation: 19 Ronnie has a vehicular accident where he sustained injury to his left ankle. In the Emergency Room,
you notice how anxious he looks.
91. You establish rapport with him and to reduce his anxiety you initially
a. Take him to the radiology, section for X-ray of affected extremity
b. Identify yourself and state your purpose in being with the client
c. Talk to the physician for an order of Valium
d. Do inspection and palpation to check extent of his injuries
92. While doing your assessment, Ronnie asks you Do I have a fracture? I dont want to have a cast. The
most appropriate nursing response would be:
a. You have to have an X-ray first to know if you have a fracture.
b. Why do you sound so scared? It is just a cast and its not painful
c. You seem to be concerned about being in a cast.
d. Based on my assessment, there doesnt seem to be a fracture.
93. A technique in physical examination that is use to assess the movement of air through the
tracheobronchial tree:

A. Palpation
B. Auscultation
C. Inspection
D. Percussion
94. An instrument used for auscultation is:
A. Percussion-hammer
B. Audiometer
C. Stethoscope
D. Sphygmomanometer
95. Resonance is best describe as:
A. Sounds created by air filled lungs
B. Short, high pitch and thudding
C. Moderately loud with musical quality
D. Drum-like
96. The best position for examining the rectum is:
A. Prone
B. Sims
C. Knee-chest
D. Lithotomy
97. It refers to the manner of walking
A. Gait
B. Range of motion
C. Flexion and extension
D. Hopping
98. The nurse asked the client to read the Snellen chart. Which of the following is tested:
A. Optic
B. Olfactory
C. Oculomotor
D. Trochlear
99. Another name for knee-chest position is:

A. Genu-dorsal
B. Genu-pectoral
C. Lithotomy
D. Sims
100. The nurse prepares an IM injection that is irritating to the subcutaneous tissue. Which of the following
is the best action in order to prevent tracking of the medication
A. Use a small gauge needle
B. Apply ice on the injection site
C. Administer at a 45 angle
D. Use the Z-track technique

Answers
Here are the answers for the exam. Unfortunately, rationales are not given. If you need clarifications or disputes,
please direct them to the comments section and well be glad to give you an explanation.
1. C
2. B
3. D
4. D
5. B
6. B
7. C
8. D
9. B
10. B
11. B
12. C
13. C
14. D
15. A
16. A
17. C
18. B
19. A
20. C
21. D
22. C

23. D
24. D
25. C
26. D
27. C
28. D
29. D
30. C
31. B
32. C
33. C
34. C
35. B
36. D
37. D
38. B
39. B
40. C
41. D
42. B
43. D
44. D
45. C
46. C
47. D
48. D
49. B
50. D
51. D
52. C
53. D
54. B
55. B
56. C
57. D
58. C
59. B
60. C
61. D
62. C

63. D
64. C
65. D
66. D
67. A
68. D
69. D
70. C
71. B
72. B
73. D
74. D
75. B
76. C
77. B
78. B
79. B
80. C
81. C
82. B
83. C
84. C
85. B
86. B
87. A
88. D
89. C
90. B
91. B
92. C
93. B
94. C
95. A
96. C
97. A
98. A
99. B
100. D

1. The nurse-in-charge in labor and delivery unit administered a dose of terbutaline to a client without
checking the clients pulse. The standard that would be used to determine if the nurse was negligent is:
a. The physicians orders.
b. The action of a clinical nurse specialist who is recognized expert in the field.
c. The statement in the drug literature about administration of terbutaline.
d. The actions of a reasonably prudent nurse with similar education and experience.
2. Nurse Trish is caring for a female client with a history of GI bleeding, sickle cell disease, and a platelet
count of 22,000/l. The female client is dehydrated and receiving dextrose 5% in half-normal saline solution
at 150 ml/hr. The client complains of severe bone pain and is scheduled to receive a dose of morphine
sulfate. In administering the medication, Nurse Trish should avoid which route?
a. I.V
b. I.M
c. Oral
d. S.C
3. Dr. Garcia writes the following order for the client who has been recently admitted Digoxin .125 mg P.O.
once daily. To prevent a dosage error, how should the nurse document this order onto the medication
administration record?
a. Digoxin .1250 mg P.O. once daily
b. Digoxin 0.1250 mg P.O. once daily
c. Digoxin 0.125 mg P.O. once daily
d. Digoxin .125 mg P.O. once daily
4. A newly admitted female client was diagnosed with deep vein thrombosis. Which nursing diagnosis
should receive the highest priority?
a. Ineffective peripheral tissue perfusion related to venous congestion.
b. Risk for injury related to edema.
c. Excess fluid volume related to peripheral vascular disease.
d. Impaired gas exchange related to increased blood flow.
5. Nurse Betty is assigned to the following clients. The client that the nurse would see first after
endorsement?
a. A 34 year-old postoperative appendectomy client of five hours who is complaining of pain.
b. A 44 year-old myocardial infarction (MI) client who is complaining of nausea.
c. A 26 year-old client admitted for dehydration whose intravenous (IV) has infiltrated.

d. A 63 year-old postoperative abdominal hysterectomy client of three days whose incisional dressing is saturated
with serosanguinous fluid.
6. Nurse Gail places a client in a four-point restraint following orders from the physician. The client care
plan should include:
a. Assess temperature frequently.
b. Provide diversional activities.
c. Check circulation every 15-30 minutes.
d. Socialize with other patients once a shift.
7. A male client who has severe burns is receiving H2 receptor antagonist therapy. The nurse In-charge
knows the purpose of this therapy is to:
a. Prevent stress ulcer
b. Block prostaglandin synthesis
c. Facilitate protein synthesis.
d. Enhance gas exchange
8. The doctor orders hourly urine output measurement for a postoperative male client. The nurse Trish
records the following amounts of output for 2 consecutive hours: 8 a.m.: 50 ml; 9 a.m.: 60 ml. Based on
these amounts, which action should the nurse take?
a. Increase the I.V. fluid infusion rate
b. Irrigate the indwelling urinary catheter
c. Notify the physician
d. Continue to monitor and record hourly urine output
9. Tony, a basketball player twist his right ankle while playing on the court and seeks care for ankle pain and
swelling. After the nurse applies ice to the ankle for 30 minutes, which statement by Tony suggests that ice
application has been effective?
a. My ankle looks less swollen now.
b. My ankle feels warm.
c. My ankle appears redder now.
d. I need something stronger for pain relief
10. The physician prescribes a loop diuretic for a client. When administering this drug, the nurse anticipates
that the client may develop which electrolyte imbalance?
a. Hypernatremia
b. Hyperkalemia

c. Hypokalemia
d. Hypervolemia
11. She finds out that some managers have benevolent-authoritative style of management. Which of the
following behaviors will she exhibit most likely?
a. Have condescending trust and confidence in their subordinates.
b. Gives economic and ego awards.
c. Communicates downward to staffs.
d. Allows decision making among subordinates.
12. Nurse Amy is aware that the following is true about functional nursing
a. Provides continuous, coordinated and comprehensive nursing services.
b. One-to-one nurse patient ratio.
c. Emphasize the use of group collaboration.
d. Concentrates on tasks and activities.
13. Which type of medication order might read Vitamin K 10 mg I.M. daily 3 days?
a. Single order
b. Standard written order
c. Standing order
d. Stat order
14. A female client with a fecal impaction frequently exhibits which clinical manifestation?
a. Increased appetite
b. Loss of urge to defecate
c. Hard, brown, formed stools
d. Liquid or semi-liquid stools
15.Nurse Linda prepares to perform an otoscopic examination on a female client. For proper visualization,
the nurse should position the clients ear by:
a. Pulling the lobule down and back
b. Pulling the helix up and forward
c. Pulling the helix up and back
d. Pulling the lobule down and forward
16. Which instruction should Nurse Tommy would give to a male client who is having external radiation
therapy:

a. Protect the irritated skin from sunlight.


b. Eat 3 to 4 hours before treatment.
c. Wash the skin over regularly.
d. Apply lotion or oil to the irradiated area when it is red or sore.
17. In assisting a female client for immediate surgery, the nurse In-charge is aware that she should:
a. Encourage the client to void following preoperative medication.
b. Explore the clients fears and anxieties about the surgery.
c. Assist the client in removing dentures and nail polish.
d. Encourage the client to drink water prior to surgery.
18. A male client is admitted and diagnosed with acute pancreatitis after a holiday celebration of excessive
food and alcohol. Which assessment finding reflects this diagnosis?
a. Blood pressure above normal range.
b. Presence of crackles in both lung fields.
c. Hyperactive bowel sounds
d. Sudden onset of continuous epigastric and back pain.
19. Which dietary guidelines are important for nurse Oliver to implement in caring for the client with burns?
a. Provide high-fiber, high-fat diet
b. Provide high-protein, high-carbohydrate diet.
c. Monitor intake to prevent weight gain.
d. Provide ice chips or water intake.
20. Nurse Hazel will administer a unit of whole blood, which priority information should the nurse have
about the client?
a. Blood pressure and pulse rate.
b. Height and weight.
c. Calcium and potassium levels
d. Hgb and Hct levels.
21. Nurse Michelle witnesses a female client sustain a fall and suspects that the leg may be broken. The
nurse takes which priority action?
a. Takes a set of vital signs.
b. Call the radiology department for X-ray.
c. Reassure the client that everything will be alright.
d. Immobilize the leg before moving the client.

22. A male client is being transferred to the nursing unit for admission after receiving a radium implant for
bladder cancer. The nurse in-charge would take which priority action in the care of this client?
a. Place client on reverse isolation.
b. Admit the client into a private room.
c. Encourage the client to take frequent rest periods.
d. Encourage family and friends to visit.
23. A newly admitted female client was diagnosed with agranulocytosis. The nurse formulates which
priority nursing diagnosis?
a. Constipation
b. Diarrhea
c. Risk for infection
d. Deficient knowledge
24. A male client is receiving total parenteral nutrition suddenly demonstrates signs and symptoms of an air
embolism. What is the priority action by the nurse?
a. Notify the physician.
b. Place the client on the left side in the Trendelenburg position.
c. Place the client in high-Fowlers position.
d. Stop the total parenteral nutrition.
25. Nurse May attends an educational conference on leadership styles. The nurse is sitting with a nurse
employed at a large trauma center who states that the leadership style at the trauma center is task-oriented
and directive. The nurse determines that the leadership style used at the trauma center is:
a. Autocratic.
b. Laissez-faire.
c. Democratic.
d. Situational
26. The physician orders DS 500 cc with KCl 10 mEq/liter at 30 cc/hr. The nurse in-charge is going to hang a
500 cc bag. KCl is supplied 20 mEq/10 cc. How many ccs of KCl will be added to the IV solution?
a. .5 cc
b. 5 cc
c. 1.5 cc
d. 2.5 cc

27. A child of 10 years old is to receive 400 cc of IV fluid in an 8 hour shift. The IV drip factor is 60. The IV
rate that will deliver this amount is:
a. 50 cc/ hour
b. 55 cc/ hour
c. 24 cc/ hour
d. 66 cc/ hour
28. The nurse is aware that the most important nursing action when a client returns from surgery is:
a. Assess the IV for type of fluid and rate of flow.
b. Assess the client for presence of pain.
c. Assess the Foley catheter for patency and urine output
d. Assess the dressing for drainage.
29. Which of the following vital sign assessments that may indicate cardiogenic shock after myocardial
infarction?
a. BP 80/60, Pulse 110 irregular
b. BP 90/50, Pulse 50 regular
c. BP 130/80, Pulse 100 regular
d. BP 180/100, Pulse 90 irregular
30.Which is the most appropriate nursing action in obtaining a blood pressure measurement?
a. Take the proper equipment, place the client in a comfortable position, and record the appropriate information in
the clients chart.
b. Measure the clients arm, if you are not sure of the size of cuff to use.
c. Have the client recline or sit comfortably in a chair with the forearm at the level of the heart.
d. Document the measurement, which extremity was used, and the position that the client was in during the
measurement.
31. Asking the questions to determine if the person understands the health teaching provided by the nurse
would be included during which step of the nursing process?
a. Assessment
b. Evaluation
c. Implementation
d. Planning and goals
32. Which of the following item is considered the single most important factor in assisting the health
professional in arriving at a diagnosis or determining the persons needs?

a. Diagnostic test results


b. Biographical date
c. History of present illness
d. Physical examination
33. In preventing the development of an external rotation deformity of the hip in a client who must remain in
bed for any period of time, the most appropriate nursing action would be to use:
a. Trochanter roll extending from the crest of the ileum to the midthigh.
b. Pillows under the lower legs.
c. Footboard
d. Hip-abductor pillow
34. Which stage of pressure ulcer development does the ulcer extend into the subcutaneous tissue?
a. Stage I
b. Stage II
c. Stage III
d. Stage IV
35. When the method of wound healing is one in which wound edges are not surgically approximated and
integumentary continuity is restored by granulations, the wound healing is termed
a. Second intention healing
b. Primary intention healing
c. Third intention healing
d. First intention healing
36. An 80-year-old male client is admitted to the hospital with a diagnosis of pneumonia. Nurse Oliver learns
that the client lives alone and hasnt been eating or drinking. When assessing him for dehydration, nurse
Oliver would expect to find:
a. Hypothermia
b. Hypertension
c. Distended neck veins
d. Tachycardia
37. The physician prescribes meperidine (Demerol), 75 mg I.M. every 4 hours as needed, to control a clients
postoperative pain. The package insert is Meperidine, 100 mg/ml. How many milliliters of meperidine
should the client receive?

a. 0.75
b. 0.6
c. 0.5
d. 0.25
38. A male client with diabetes mellitus is receiving insulin. Which statement correctly describes an insulin
unit?
a. Its a common measurement in the metric system.
b. Its the basis for solids in the avoirdupois system.
c. Its the smallest measurement in the apothecary system.
d. Its a measure of effect, not a standard measure of weight or quantity.
39. Nurse Oliver measures a clients temperature at 102 F. What is the equivalent Centigrade temperature?
a. 40.1 C
b. 38.9 C
c. 48 C
d. 38 C
40. The nurse is assessing a 48-year-old client who has come to the physicians office for his annual
physical exam. One of the first physical signs of aging is:
a. Accepting limitations while developing assets.
b. Increasing loss of muscle tone.
c. Failing eyesight, especially close vision.
d. Having more frequent aches and pains.
41. The physician inserts a chest tube into a female client to treat a pneumothorax. The tube is connected to
water-seal drainage. The nurse in-charge can prevent chest tube air leaks by:
a. Checking and taping all connections.
b. Checking patency of the chest tube.
c. Keeping the head of the bed slightly elevated.
d. Keeping the chest drainage system below the level of the chest.
42. Nurse Trish must verify the clients identity before administering medication. She is aware that the
safest way to verify identity is to:
a. Check the clients identification band.
b. Ask the client to state his name.

c. State the clients name out loud and wait a client to repeat it.
d. Check the room number and the clients name on the bed.
43. The physician orders dextrose 5 % in water, 1,000 ml to be infused over 8 hours. The I.V. tubing delivers
15 drops/ml. Nurse John should run the I.V. infusion at a rate of:
a. 30 drops/minute
b. 32 drops/minute
c. 20 drops/minute
d. 18 drops/minute
44. If a central venous catheter becomes disconnected accidentally, what should the nurse in-charge do
immediately?
a. Clamp the catheter
b. Call another nurse
c. Call the physician
d. Apply a dry sterile dressing to the site.
45. A female client was recently admitted. She has fever, weight loss, and watery diarrhea is being admitted
to the facility. While assessing the client, Nurse Hazel inspects the clients abdomen and notice that it is
slightly concave. Additional assessment should proceed in which order:
a. Palpation, auscultation, and percussion.
b. Percussion, palpation, and auscultation.
c. Palpation, percussion, and auscultation.
d. Auscultation, percussion, and palpation.
46. Nurse Betty is assessing tactile fremitus in a client with pneumonia. For this examination, nurse Betty
should use the:
a. Fingertips
b. Finger pads
c. Dorsal surface of the hand
d. Ulnar surface of the hand
47. Which type of evaluation occurs continuously throughout the teaching and learning process?
a. Summative
b. Informative
c. Formative
d. Retrospective

48. A 45-year-old client, has no family history of breast cancer or other risk factors for this disease. Nurse
John should instruct her to have mammogram how often?
a. Twice per year
b. Once per year
c. Every 2 years
d. Once, to establish baseline
49. A male client has the following arterial blood gas values: pH 7.30; Pao2 89 mmHg; Paco2 50 mmHg; and
HCO3 26 mEq/L. Based on these values, Nurse Patricia should expect which condition?
a. Respiratory acidosis
b. Respiratory alkalosis
c. Metabolic acidosis
d. Metabolic alkalosis
50. Nurse Len refers a female client with terminal cancer to a local hospice. What is the goal of this referral?
a. To help the client find appropriate treatment options.
b. To provide support for the client and family in coping with terminal illness.
c. To ensure that the client gets counseling regarding health care costs.
d. To teach the client and family about cancer and its treatment.
51. When caring for a male client with a 3-cm stage I pressure ulcer on the coccyx, which of the following
actions can the nurse institute independently?
a. Massaging the area with an astringent every 2 hours.
b. Applying an antibiotic cream to the area three times per day.
c. Using normal saline solution to clean the ulcer and applying a protective dressing as necessary.
d. Using a povidone-iodine wash on the ulceration three times per day.
52. Nurse Oliver must apply an elastic bandage to a clients ankle and calf. He should apply the bandage
beginning at the clients:
a. Knee
b. Ankle
c. Lower thigh
d. Foot
53. A 10 year old child with type 1 diabetes develops diabetic ketoacidosis and receives a continuous
insulin infusion. Which condition represents the greatest risk to this child?

a. Hypernatremia
b. Hypokalemia
c. Hyperphosphatemia
d. Hypercalcemia
54. Nurse Len is administering sublingual nitroglycerin (Nitrostat) to the newly admitted client. Immediately
afterward, the client may experience:
a. Throbbing headache or dizziness
b. Nervousness or paresthesia.
c. Drowsiness or blurred vision.
d. Tinnitus or diplopia.
55. Nurse Michelle hears the alarm sound on the telemetry monitor. The nurse quickly looks at the monitor
and notes that a client is in a ventricular tachycardia. The nurse rushes to the clients room. Upon reaching
the clients bedside, the nurse would take which action first?
a. Prepare for cardioversion
b. Prepare to defibrillate the client
c. Call a code
d. Check the clients level of consciousness
56. Nurse Hazel is preparing to ambulate a female client. The best and the safest position for the nurse in
assisting the client is to stand:
a. On the unaffected side of the client.
b. On the affected side of the client.
c. In front of the client.
d. Behind the client.
57. Nurse Janah is monitoring the ongoing care given to the potential organ donor who has been diagnosed
with brain death. The nurse determines that the standard of care had been maintained if which of the
following data is observed?
a. Urine output: 45 ml/hr
b. Capillary refill: 5 seconds
c. Serum pH: 7.32
d. Blood pressure: 90/48 mmHg
58. Nurse Amy has an order to obtain a urinalysis from a male client with an indwelling urinary catheter. The
nurse avoids which of the following, which contaminate the specimen?

a. Wiping the port with an alcohol swab before inserting the syringe.
b. Aspirating a sample from the port on the drainage bag.
c. Clamping the tubing of the drainage bag.
d. Obtaining the specimen from the urinary drainage bag.
59. Nurse Meredith is in the process of giving a client a bed bath. In the middle of the procedure, the unit
secretary calls the nurse on the intercom to tell the nurse that there is an emergency phone call. The
appropriate nursing action is to:
a. Immediately walk out of the clients room and answer the phone call.
b. Cover the client, place the call light within reach, and answer the phone call.
c. Finish the bed bath before answering the phone call.
d. Leave the clients door open so the client can be monitored and the nurse can answer the phone call.
60. Nurse Janah is collecting a sputum specimen for culture and sensitivity testing from a client who has a
productive cough. Nurse Janah plans to implement which intervention to obtain the specimen?
a. Ask the client to expectorate a small amount of sputum into the emesis basin.
b. Ask the client to obtain the specimen after breakfast.
c. Use a sterile plastic container for obtaining the specimen.
d. Provide tissues for expectoration and obtaining the specimen.
61. Nurse Ron is observing a male client using a walker. The nurse determines that the client is using the
walker correctly if the client:
a. Puts all the four points of the walker flat on the floor, puts weight on the hand pieces, and then walks into it.
b. Puts weight on the hand pieces, moves the walker forward, and then walks into it.
c. Puts weight on the hand pieces, slides the walker forward, and then walks into it.
d. Walks into the walker, puts weight on the hand pieces, and then puts all four points of the walker flat on the floor.
62. Nurse Amy has documented an entry regarding client care in the clients medical record. When
checking the entry, the nurse realizes that incorrect information was documented. How does the nurse
correct this error?
a. Erases the error and writes in the correct information.
b. Uses correction fluid to cover up the incorrect information and writes in the correct information.
c. Draws one line to cross out the incorrect information and then initials the change.
d. Covers up the incorrect information completely using a black pen and writes in the correct information
63. Nurse Ron is assisting with transferring a client from the operating room table to a stretcher. To provide
safety to the client, the nurse should:

a. Moves the client rapidly from the table to the stretcher.


b. Uncovers the client completely before transferring to the stretcher.
c. Secures the client safety belts after transferring to the stretcher.
d. Instructs the client to move self from the table to the stretcher.
64. Nurse Myrna is providing instructions to a nursing assistant assigned to give a bed bath to a client who
is on contact precautions. Nurse Myrna instructs the nursing assistant to use which of the following
protective items when giving bed bath?
a. Gown and goggles
b. Gown and gloves
c. Gloves and shoe protectors
d. Gloves and goggles
65. Nurse Oliver is caring for a client with impaired mobility that occurred as a result of a stroke. The client
has right sided arm and leg weakness. The nurse would suggest that the client use which of the following
assistive devices that would provide the best stability for ambulating?
a. Crutches
b. Single straight-legged cane
c. Quad cane
d. Walker
66. A male client with a right pleural effusion noted on a chest X-ray is being prepared for thoracentesis.
The client experiences severe dizziness when sitting upright. To provide a safe environment, the nurse
assists the client to which position for the procedure?
a. Prone with head turned toward the side supported by a pillow.
b. Sims position with the head of the bed flat.
c. Right side-lying with the head of the bed elevated 45 degrees.
d. Left side-lying with the head of the bed elevated 45 degrees.
67. Nurse John develops methods for data gathering. Which of the following criteria of a good instrument
refers to the ability of the instrument to yield the same results upon its repeated administration?
a. Validity
b. Specificity
c. Sensitivity
d. Reliability
68. Harry knows that he has to protect the rights of human research subjects. Which of the following
actions of Harry ensures anonymity?

a. Keep the identities of the subject secret


b. Obtain informed consent
c. Provide equal treatment to all the subjects of the study.
d. Release findings only to the participants of the study
69. Patients refusal to divulge information is a limitation because it is beyond the control of the researcher.
What type of research is appropriate for this study?
a. Descriptive- correlational
b. Experiment
c. Quasi-experiment
d. Historical
70. Nurse Ronald is aware that the best tool for data gathering is?
a. Interview schedule
b. Questionnaire
c. Use of laboratory data
d. Observation
71. Monica is aware that there are times when only manipulation of study variables is possible and the
elements of control or randomization are not attendant. Which type of research is referred to this?
a. Field study
b. Quasi-experiment
c. Solomon-Four group design
d. Posttest only design
72. Cherry notes down ideas that were derived from the description of an investigation written by the
person who conducted it. Which type of reference source refers to this?
a. Footnote
b. Bibliography
c. Primary source
d. Endnotes
73. When Nurse Trish is providing care to his patient, she must remember that her duty is bound not to do
doing any action that will cause the patient harm. This is the meaning of the bioethical principle:
a. Non-maleficence
b. Beneficence

c. Justice
d. Solidarity
74. When a nurse in-charge causes an injury to a female patient and the injury caused becomes the proof of
the negligent act, the presence of the injury is said to exemplify the principle of:
a. Force majeure
b. Respondeat superior
c. Res ipsa loquitur
d. Holdover doctrine
75. Nurse Myrna is aware that the Board of Nursing has quasi-judicial power. An example of this power is:
a. The Board can issue rules and regulations that will govern the practice of nursing
b. The Board can investigate violations of the nursing law and code of ethics
c. The Board can visit a school applying for a permit in collaboration with CHED
d. The Board prepares the board examinations
76. When the license of nurse Krina is revoked, it means that she:
a. Is no longer allowed to practice the profession for the rest of her life
b. Will never have her/his license re-issued since it has been revoked
c. May apply for re-issuance of his/her license based on certain conditions stipulated in RA 9173
d. Will remain unable to practice professional nursing
77. Ronald plans to conduct a research on the use of a new method of pain assessment scale. Which of the
following is the second step in the conceptualizing phase of the research process?
a. Formulating the research hypothesis
b. Review related literature
c. Formulating and delimiting the research problem
d. Design the theoretical and conceptual framework
78. The leader of the study knows that certain patients who are in a specialized research setting tend to
respond psychologically to the conditions of the study. This referred to as:
a. Cause and effect
b. Hawthorne effect
c. Halo effect
d. Horns effect

79. Mary finally decides to use judgment sampling on her research. Which of the following actions of is
correct?
a. Plans to include whoever is there during his study.
b. Determines the different nationality of patients frequently admitted and decides to get representations samples
from each.
c. Assigns numbers for each of the patients, place these in a fishbowl and draw 10 from it.
d. Decides to get 20 samples from the admitted patients
80. The nursing theorist who developed transcultural nursing theory is:
a. Florence Nightingale
b. Madeleine Leininger
c. Albert Moore
d. Sr. Callista Roy
81. Marion is aware that the sampling method that gives equal chance to all units in the population to get
picked is:
a. Random
b. Accidental
c. Quota
d. Judgment
82. John plans to use a Likert Scale to his study to determine the:
a. Degree of agreement and disagreement
b. Compliance to expected standards
c. Level of satisfaction
d. Degree of acceptance
83. Which of the following theorem addresses the four modes of adaptation?
a. Madeleine Leininger
b. Sr. Callista Roy
c. Florence Nightingale
d. Jean Watson
84. Ms. Garcia is responsible to the number of personnel reporting to her. This principle refers to:
a. Span of control
b. Unity of command

c. Downward communication
d. Leader
85. Ensuring that there is an informed consent on the part of the patient before a surgery is done, illustrates
the bioethical principle of:
a. Beneficence
b. Autonomy
c. Veracity
d. Non-maleficence
86. Nurse Reese is teaching a female client with peripheral vascular disease about foot care; Nurse Reese
should include which instruction?
a. Avoid wearing cotton socks.
b. Avoid using a nail clipper to cut toenails.
c. Avoid wearing canvas shoes.
d. Avoid using cornstarch on feet.
87. A client is admitted with multiple pressure ulcers. When developing the clients diet plan, the nurse
should include:
a. Fresh orange slices
b. Steamed broccoli
c. Ice cream
d. Ground beef patties
88. The nurse prepares to administer a cleansing enema. What is the most common client position used for
this procedure?
a. Lithotomy
b. Supine
c. Prone
d. Sims left lateral
89. Nurse Marian is preparing to administer a blood transfusion. Which action should the nurse take first?
a. Arrange for typing and cross matching of the clients blood.
b. Compare the clients identification wristband with the tag on the unit of blood.
c. Start an I.V. infusion of normal saline solution.
d. Measure the clients vital signs.

90. A 65 years old male client requests his medication at 9 p.m. instead of 10 p.m. so that he can go to sleep
earlier. Which type of nursing intervention is required?
a. Independent
b. Dependent
c. Interdependent
d. Intradependent
91. A female client is to be discharged from an acute care facility after treatment for right leg
thrombophlebitis. The Nurse Betty notes that the clients leg is pain-free, without redness or edema. The
nurses actions reflect which step of the nursing process?
a. Assessment
b. Diagnosis
c. Implementation
d. Evaluation
92. Nursing care for a female client includes removing elastic stockings once per day. The Nurse Betty is
aware that the rationale for this intervention?
a. To increase blood flow to the heart
b. To observe the lower extremities
c. To allow the leg muscles to stretch and relax
d. To permit veins in the legs to fill with blood.
93. Which nursing intervention takes highest priority when caring for a newly admitted client whos
receiving a blood transfusion?
a. Instructing the client to report any itching, swelling, or dyspnea.
b. Informing the client that the transfusion usually take 1 to 2 hours.
c. Documenting blood administration in the client care record.
d. Assessing the clients vital signs when the transfusion ends.
94. A male client complains of abdominal discomfort and nausea while receiving tube feedings. Which
intervention is most appropriate for this problem?
a. Give the feedings at room temperature.
b. Decrease the rate of feedings and the concentration of the formula.
c. Place the client in semi-Fowlers position while feeding.
d. Change the feeding container every 12 hours.

95. Nurse Patricia is reconstituting a powdered medication in a vial. After adding the solution to the powder,
she nurse should:
a. Do nothing.
b. Invert the vial and let it stand for 3 to 5 minutes.
c. Shake the vial vigorously.
d. Roll the vial gently between the palms.
96. Which intervention should the nurse Trish use when administering oxygen by face mask to a female
client?
a. Secure the elastic band tightly around the clients head.
b. Assist the client to the semi-Fowler position if possible.
c. Apply the face mask from the clients chin up over the nose.
d. Loosen the connectors between the oxygen equipment and humidifier.
97. The maximum transfusion time for a unit of packed red blood cells (RBCs) is:
a. 6 hours
b. 4 hours
c. 3 hours
d. 2 hours
98. Nurse Monique is monitoring the effectiveness of a clients drug therapy. When should the nurse
Monique obtain a blood sample to measure the trough drug level?
a. 1 hour before administering the next dose.
b. Immediately before administering the next dose.
c. Immediately after administering the next dose.
d. 30 minutes after administering the next dose.
99. Nurse May is aware that the main advantage of using a floor stock system is:
a. The nurse can implement medication orders quickly.
b. The nurse receives input from the pharmacist.
c. The system minimizes transcription errors.
d. The system reinforces accurate calculations.
100. Nurse Oliver is assessing a clients abdomen. Which finding should the nurse report as abnormal?
a. Dullness over the liver.
b. Bowel sounds occurring every 10 seconds.

c. Shifting dullness over the abdomen.


d. Vascular sounds heard over the renal arteries

Answers & Rationale


Gauge your performance by counter checking your answers to the answers below. Learn more about the question
by reading the rationale. If you have any disputes or questions, please direct them to the comments section.
1. Answer: (D) The actions of a reasonably prudent nurse with similar education and experience.
Rationale: The standard of care is determined by the average degree of skill, care, and diligence by nurses in similar
circumstances.
2. Answer: (B) I.M
Rationale: With a platelet count of 22,000/l, the clients tends to bleed easily. Therefore, the nurse should avoid
using the I.M. route because the area is a highly vascular and can bleed readily when penetrated by a needle. The
bleeding can be difficult to stop.
3. Answer: (C) Digoxin 0.125 mg P.O. once daily
Rationale: The nurse should always place a zero before a decimal point so that no one misreads the figure, which
could result in a dosage error. The nurse should never insert a zero at the end of a dosage that includes a decimal
point because this could be misread, possibly leading to a tenfold increase in the dosage.
4. Answer: (A) Ineffective peripheral tissue perfusion related to venous congestion.
Rationale: Ineffective peripheral tissue perfusion related to venous congestion takes the highest priority because
venous inflammation and clot formation impede blood flow in a client with deep vein thrombosis.
5. Answer: (B) A 44 year-old myocardial infarction (MI) client who is complaining of nausea.
Rationale: Nausea is a symptom of impending myocardial infarction (MI) and should be assessed immediately so
that treatment can be instituted and further damage to the heart is avoided.
6. Answer: (C) Check circulation every 15-30 minutes.
Rationale: Restraints encircle the limbs, which place the client at risk for circulation being restricted to the distal
areas of the extremities. Checking the clients circulation every 15-30 minutes will allow the nurse to adjust the
restraints before injury from decreased blood flow occurs.
7. Answer: (A) Prevent stress ulcer
Rationale: Curlings ulcer occurs as a generalized stress response in burn patients. This results in a decreased
production of mucus and increased secretion of gastric acid. The best treatment for this prophylactic use of antacids
and H2 receptor blockers.

8. Answer: (D) Continue to monitor and record hourly urine output


Rationale: Normal urine output for an adult is approximately 1 ml/minute (60 ml/hour). Therefore, this clients output
is normal. Beyond continued evaluation, no nursing action is warranted.
9. Answer: (A) My ankle looks less swollen now.
Rationale: Ice application decreases pain and swelling. Continued or increased pain, redness, and increased
warmth are signs of inflammation that shouldnt occur after ice application
10. Answer: (C) Hypokalemia
Rationale: A loop diuretic removes water and, along with it, sodium and potassium. This may result in hypokalemia,
hypovolemia, and hyponatremia.
11. Answer:(A) Have condescending trust and confidence in their subordinates
Rationale: Benevolent-authoritative managers pretentiously show their trust and confidence to their followers.
12. Answer: (A) Provides continuous, coordinated and comprehensive nursing services.
Rationale: Functional nursing is focused on tasks and activities and not on the care of the patients.
13. Answer: (B) Standard written order
Rationale: This is a standard written order. Prescribers write a single order for medications given only once. A stat
order is written for medications given immediately for an urgent client problem. A standing order, also known as a
protocol, establishes guidelines for treating a particular disease or set of symptoms in special care areas such as
the coronary care unit. Facilities also may institute medication protocols that specifically designate drugs that a
nurse may not give.
14. Answer: (D) Liquid or semi-liquid stools
Rationale: Passage of liquid or semi-liquid stools results from seepage of unformed bowel contents around the
impacted stool in the rectum. Clients with fecal impaction dont pass hard, brown, formed stools because the feces
cant move past the impaction. These clients typically report the urge to defecate (although they cant pass stool)
and a decreased appetite.
15. Answer: (C) Pulling the helix up and back
Rationale: To perform an otoscopic examination on an adult, the nurse grasps the helix of the ear and pulls it up and
back to straighten the ear canal. For a child, the nurse grasps the helix and pulls it down to straighten the ear canal.
Pulling the lobule in any direction wouldnt straighten the ear canal for visualization.
16. Answer: (A) Protect the irritated skin from sunlight.
Rationale: Irradiated skin is very sensitive and must be protected with clothing or sunblock. The priority approach is
the avoidance of strong sunlight.

17. Answer: (C) Assist the client in removing dentures and nail polish.
Rationale: Dentures, hairpins, and combs must be removed. Nail polish must be removed so that cyanosis can be
easily monitored by observing the nail beds.
18. Answer: (D) Sudden onset of continuous epigastric and back pain.
Rationale: The autodigestion of tissue by the pancreatic enzymes results in pain from inflammation, edema, and
possible hemorrhage. Continuous, unrelieved epigastric or back pain reflects the inflammatory process in the
pancreas.
19. Answer: (B) Provide high-protein, high-carbohydrate diet.
Rationale: A positive nitrogen balance is important for meeting metabolic needs, tissue repair, and resistance to
infection. Caloric goals may be as high as 5000 calories per day.
20. Answer: (A) Blood pressure and pulse rate.
Rationale: The baseline must be established to recognize the signs of an anaphylactic or hemolytic reaction to the
transfusion.
21. Answer: (D) Immobilize the leg before moving the client.
Rationale: If the nurse suspects a fracture, splinting the area before moving the client is imperative. The nurse
should call for emergency help if the client is not hospitalized and call for a physician for the hospitalized client.
22. Answer: (B) Admit the client into a private room.
Rationale: The client who has a radiation implant is placed in a private room and has a limited number of visitors.
This reduces the exposure of others to the radiation.
23. Answer: (C) Risk for infection
Rationale: Agranulocytosis is characterized by a reduced number of leukocytes (leucopenia) and neutrophils
(neutropenia) in the blood. The client is at high risk for infection because of the decreased body defenses against
microorganisms. Deficient knowledge related to the nature of the disorder may be appropriate diagnosis but is not
the priority.
24. Answer: (B) Place the client on the left side in the Trendelenburg position.
Rationale: Lying on the left side may prevent air from flowing into the pulmonary veins. The Trendelenburg position
increases intrathoracic pressure, which decreases the amount of blood pulled into the vena cava during aspiration.
25. Answer: (A) Autocratic.
Rationale: The autocratic style of leadership is a task-oriented and directive.
26. Answer: (D) 2.5 cc
Rationale: 2.5 cc is to be added, because only a 500 cc bag of solution is being medicated instead of a 1 liter.

27. Answer: (A) 50 cc/ hour


Rationale: A rate of 50 cc/hr. The child is to receive 400 cc over a period of 8 hours = 50 cc/hr.
28. Answer: (B) Assess the client for presence of pain.
Rationale: Assessing the client for pain is a very important measure. Postoperative pain is an indication of
complication. The nurse should also assess the client for pain to provide for the clients comfort.
29. Answer: (A) BP 80/60, Pulse 110 irregular
Rationale: The classic signs of cardiogenic shock are low blood pressure, rapid and weak irregular pulse, cold,
clammy skin, decreased urinary output, and cerebral hypoxia.
30. Answer: (A) Take the proper equipment, place the client in a comfortable position, and record the
appropriate information in the clients chart.
Rationale: It is a general or comprehensive statement about the correct procedure, and it includes the basic ideas
which are found in the other options
31. Answer: (B) Evaluation
Rationale: Evaluation includes observing the person, asking questions, and comparing the patients behavioral
responses with the expected outcomes.
32. Answer: (C) History of present illness
Rationale: The history of present illness is the single most important factor in assisting the health professional in
arriving at a diagnosis or determining the persons needs.
33. Answer: (A) Trochanter roll extending from the crest of the ileum to the mid-thigh.
Rationale: A trochanter roll, properly placed, provides resistance to the external rotation of the hip.
34. Answer: (C) Stage III
Rationale: Clinically, a deep crater or without undermining of adjacent tissue is noted.
35. Answer: (A) Second intention healing
Rationale: When wounds dehisce, they will allowed to heal by secondary intention
36. Answer: (D) Tachycardia
Rationale: With an extracellular fluid or plasma volume deficit, compensatory mechanisms stimulate the heart,
causing an increase in heart rate.
37. Answer: (A) 0.75
Rationale: To determine the number of milliliters the client should receive, the nurse uses the fraction method in the
following equation.
75 mg/X ml = 100 mg/1 ml
To solve for X, cross-multiply:

75 mg x 1 ml = X ml x 100 mg
75 = 100X
75/100 = X
0.75 ml (or ml) = X
38. Answer: (D) Its a measure of effect, not a standard measure of weight or quantity.
Rationale: An insulin unit is a measure of effect, not a standard measure of weight or quantity. Different drugs
measured in units may have no relationship to one another in quality or quantity.
39. Answer: (B) 38.9 C
Rationale: To convert Fahrenheit degrees to Centigrade, use this formula
C = (F 32) 1.8
C = (102 32) 1.8
C = 70 1.8
C = 38.9
40. Answer: (C) Failing eyesight, especially close vision.
Rationale: Failing eyesight, especially close vision, is one of the first signs of aging in middle life (ages 46 to 64).
More frequent aches and pains begin in the early late years (ages 65 to 79). Increase in loss of muscle tone occurs
in later years (age 80 and older).
41. Answer: (A) Checking and taping all connections
Rationale: Air leaks commonly occur if the system isnt secure. Checking all connections and taping them will
prevent air leaks. The chest drainage system is kept lower to promote drainage not to prevent leaks.
42. Answer: (A) Check the clients identification band.
Rationale: Checking the clients identification band is the safest way to verify a clients identity because the band is
assigned on admission and isnt be removed at any time. (If it is removed, it must be replaced). Asking the clients
name or having the client repeated his name would be appropriate only for a client whos alert, oriented, and able to
understand what is being said, but isnt the safe standard of practice. Names on bed arent always reliable
43. Answer: (B) 32 drops/minute
Rationale: Giving 1,000 ml over 8 hours is the same as giving 125 ml over 1 hour (60 minutes). Find the number of
milliliters per minute as follows:
125/60 minutes = X/1 minute
60X = 125 = 2.1 ml/minute
To find the number of drops per minute:
2.1 ml/X gtt = 1 ml/ 15 gtt
X = 32 gtt/minute, or 32 drops/minute

44. Answer: (A) Clamp the catheter


Rationale: If a central venous catheter becomes disconnected, the nurse should immediately apply a catheter clamp,
if available. If a clamp isnt available, the nurse can place a sterile syringe or catheter plug in the catheter hub. After
cleaning the hub with alcohol or povidone-iodine solution, the nurse must replace the I.V. extension and restart the
infusion
45. Answer: (D) Auscultation, percussion, and palpation.
Rationale: The correct order of assessment for examining the abdomen is inspection, auscultation, percussion, and
palpation. The reason for this approach is that the less intrusive techniques should be performed before the more
intrusive techniques. Percussion and palpation can alter natural findings during auscultation.
46. Answer: (D) Ulnar surface of the hand
Rationale: The nurse uses the ulnar surface, or ball, of the hand to asses tactile fremitus, thrills, and vocal vibrations
through the chest wall. The fingertips and finger pads best distinguish texture and shape. The dorsal surface best
feels warmth.
47. Answer: (C) Formative
Rationale: Formative (or concurrent) evaluation occurs continuously throughout the teaching and learning process.
One benefit is that the nurse can adjust teaching strategies as necessary to enhance learning. Summative, or
retrospective, evaluation occurs at the conclusion of the teaching and learning session. Informative is not a type of
evaluation.
48. Answer: (B) Once per year
Rationale: Yearly mammograms should begin at age 40 and continue for as long as the woman is in good health. If
health risks, such as family history, genetic tendency, or past breast cancer, exist, more frequent examinations may
be necessary.
49. Answer: (A) Respiratory acidosis
Rationale: The client has a below-normal (acidic) blood pH value and an above-normal partial pressure of arterial
carbon dioxide (Paco2) value, indicating respiratory acidosis. In respiratory alkalosis, the pH value is above normal
and in the Paco2 value is below normal. In metabolic acidosis, the pH and bicarbonate (Hco3) values are below
normal. In metabolic alkalosis, the pH and Hco3 values are above normal.
50. Answer: (B) To provide support for the client and family in coping with terminal illness.
Rationale: Hospices provide supportive care for terminally ill clients and their families. Hospice care doesnt focus on
counseling regarding health care costs. Most client referred to hospices have been treated for their disease without
success and will receive only palliative care in the hospice.
51. Answer: (C) Using normal saline solution to clean the ulcer and applying a protective dressing as
necessary.
Rationale: Washing the area with normal saline solution and applying a protective dressing are within the nurses

realm of interventions and will protect the area. Using a povidone-iodine wash and an antibiotic cream require a
physicians order. Massaging with an astringent can further damage the skin.
52. Answer: (D) Foot
Rationale: An elastic bandage should be applied form the distal area to the proximal area. This method promotes
venous return. In this case, the nurse should begin applying the bandage at the clients foot. Beginning at the ankle,
lower thigh, or knee does not promote venous return.
53. Answer: (B) Hypokalemia
Rationale: Insulin administration causes glucose and potassium to move into the cells, causing hypokalemia.
54. Answer: (A) Throbbing headache or dizziness
Rationale: Headache and dizziness often occur when nitroglycerin is taken at the beginning of therapy. However, the
client usually develops tolerance
55. Answer: (D) Check the clients level of consciousness
Rationale: Determining unresponsiveness is the first step assessment action to take. When a client is in ventricular
tachycardia, there is a significant decrease in cardiac output. However, checking the unresponsiveness ensures
whether the client is affected by the decreased cardiac output.
56. Answer: (B) On the affected side of the client.
Rationale: When walking with clients, the nurse should stand on the affected side and grasp the security belt in the
mid spine area of the small of the back. The nurse should position the free hand at the shoulder area so that the
client can be pulled toward the nurse in the event that there is a forward fall. The client is instructed to look up and
outward rather than at his or her feet.
57. Answer: (A) Urine output: 45 ml/hr
Rationale: Adequate perfusion must be maintained to all vital organs in order for the client to remain visible as an
organ donor. A urine output of 45 ml per hour indicates adequate renal perfusion. Low blood pressure and delayed
capillary refill time are circulatory system indicators of inadequate perfusion. A serum pH of 7.32 is acidotic, which
adversely affects all body tissues.
58. Answer: (D ) Obtaining the specimen from the urinary drainage bag.
Rationale: A urine specimen is not taken from the urinary drainage bag. Urine undergoes chemical changes while
sitting in the bag and does not necessarily reflect the current client status. In addition, it may become contaminated
with bacteria from opening the system.
59. Answer: (B) Cover the client, place the call light within reach, and answer the phone call.
Rationale: Because telephone call is an emergency, the nurse may need to answer it. The other appropriate action
is to ask another nurse to accept the call. However, is not one of the options. To maintain privacy and safety, the

nurse covers the client and places the call light within the clients reach. Additionally, the clients door should be
closed or the room curtains pulled around the bathing area.
60. Answer: (C) Use a sterile plastic container for obtaining the specimen.
Rationale: Sputum specimens for culture and sensitivity testing need to be obtained using sterile techniques
because the test is done to determine the presence of organisms. If the procedure for obtaining the specimen is not
sterile, then the specimen is not sterile, then the specimen would be contaminated and the results of the test would
be invalid.
61. Answer: (A) Puts all the four points of the walker flat on the floor, puts weight on the hand pieces, and
then walks into it.
Rationale: When the client uses a walker, the nurse stands adjacent to the affected side. The client is instructed to
put all four points of the walker 2 feet forward flat on the floor before putting weight on hand pieces. This will ensure
client safety and prevent stress cracks in the walker. The client is then instructed to move the walker forward and
walk into it.
62. Answer: (C) Draws one line to cross out the incorrect information and then initials the change.
Rationale: To correct an error documented in a medical record, the nurse draws one line through the incorrect
information and then initials the error. An error is never erased and correction fluid is never used in the medical
record.
63. Answer: (C) Secures the client safety belts after transferring to the stretcher.
Rationale: During the transfer of the client after the surgical procedure is complete, the nurse should avoid exposure
of the client because of the risk for potential heat loss. Hurried movements and rapid changes in the position should
be avoided because these predispose the client to hypotension. At the time of the transfer from the surgery table to
the stretcher, the client is still affected by the effects of the anesthesia; therefore, the client should not move self.
Safety belts can prevent the client from falling off the stretcher.
64. Answer: (B) Gown and gloves
Rationale: Contact precautions require the use of gloves and a gown if direct client contact is anticipated. Goggles
are not necessary unless the nurse anticipates the splashes of blood, body fluids, secretions, or excretions may
occur. Shoe protectors are not necessary.
65. Answer: (C) Quad cane
Rationale: Crutches and a walker can be difficult to maneuver for a client with weakness on one side. A cane is
better suited for client with weakness of the arm and leg on one side. However, the quad cane would provide the
most stability because of the structure of the cane and because a quad cane has four legs.
66. Answer: (D) Left side-lying with the head of the bed elevated 45 degrees.
Rationale: To facilitate removal of fluid from the chest wall, the client is positioned sitting at the edge of the bed

leaning over the bedside table with the feet supported on a stool. If the client is unable to sit up, the client is
positioned lying in bed on the unaffected side with the head of the bed elevated 30 to 45 degrees.
67. Answer: (D) Reliability
Rationale: Reliability is consistency of the research instrument. It refers to the repeatability of the instrument in
extracting the same responses upon its repeated administration.
68. Answer: (A) Keep the identities of the subject secret
Rationale: Keeping the identities of the research subject secret will ensure anonymity because this will hinder
providing link between the information given to whoever is its source.
69. Answer: (A) Descriptive- correlational
Rationale: Descriptive- correlational study is the most appropriate for this study because it studies the variables that
could be the antecedents of the increased incidence of nosocomial infection.
70. Answer: (C) Use of laboratory data
Rationale: Incidence of nosocomial infection is best collected through the use of biophysiologic measures,
particularly in vitro measurements, hence laboratory data is essential.
71. Answer: (B) Quasi-experiment
Rationale: Quasi-experiment is done when randomization and control of the variables are not possible.
72. Answer: (C) Primary source
Rationale: This refers to a primary source which is a direct account of the investigation done by the investigator. In
contrast to this is a secondary source, which is written by someone other than the original researcher.
73. Answer: (A) Non-maleficence
Rationale: Non-maleficence means do not cause harm or do any action that will cause any harm to the patient/client.
To do good is referred as beneficence.
74. Answer: (C) Res ipsa loquitur
Rationale: Res ipsa loquitur literally means the thing speaks for itself. This means in operational terms that the injury
caused is the proof that there was a negligent act.
75. Answer: (B) The Board can investigate violations of the nursing law and code of ethics
Rationale: Quasi-judicial power means that the Board of Nursing has the authority to investigate violations of the
nursing law and can issue summons, subpoena or subpoena duces tecum as needed.
76. Answer: (C) May apply for re-issuance of his/her license based on certain conditions stipulated in RA
9173
Rationale: RA 9173 sec. 24 states that for equity and justice, a revoked license maybe re-issued provided that the

following conditions are met: a) the cause for revocation of license has already been corrected or removed; and, b)
at least four years has elapsed since the license has been revoked.
77. Answer: (B) Review related literature
Rationale: After formulating and delimiting the research problem, the researcher conducts a review of related
literature to determine the extent of what has been done on the study by previous researchers.
78. Answer: (B) Hawthorne effect
Rationale: Hawthorne effect is based on the study of Elton Mayo and company about the effect of an intervention
done to improve the working conditions of the workers on their productivity. It resulted to an increased productivity
but not due to the intervention but due to the psychological effects of being observed. They performed differently
because they were under observation.
79. Answer: (B) Determines the different nationality of patients frequently admitted and decides to get
representations samples from each.
Rationale: Judgment sampling involves including samples according to the knowledge of the investigator about the
participants in the study.
80. Answer: (B) Madeleine Leininger
Rationale: Madeleine Leininger developed the theory on transcultural theory based on her observations on the
behavior of selected people within a culture.
81. Answer: (A) Random
Rationale: Random sampling gives equal chance for all the elements in the population to be picked as part of the
sample.
82. Answer: (A) Degree of agreement and disagreement
Rationale: Likert scale is a 5-point summated scale used to determine the degree of agreement or disagreement of
the respondents to a statement in a study
83. Answer: (B) Sr. Callista Roy
Rationale: Sr. Callista Roy developed the Adaptation Model which involves the physiologic mode, self-concept
mode, role function mode and dependence mode.
84. Answer: (A) Span of control
Rationale: Span of control refers to the number of workers who report directly to a manager.
85. Answer: (B) Autonomy
Rationale: Informed consent means that the patient fully understands about the surgery, including the risks involved
and the alternative solutions. In giving consent it is done with full knowledge and is given freely. The action of
allowing the patient to decide whether a surgery is to be done or not exemplifies the bioethical principle of autonomy.

86. Answer: (C) Avoid wearing canvas shoes.


Rationale: The client should be instructed to avoid wearing canvas shoes. Canvas shoes cause the feet to perspire,
which may, in turn, cause skin irritation and breakdown. Both cotton and cornstarch absorb perspiration. The client
should be instructed to cut toenails straight across with nail clippers.
87. Answer: (D) Ground beef patties
Rationale: Meat is an excellent source of complete protein, which this client needs to repair the tissue breakdown
caused by pressure ulcers. Oranges and broccoli supply vitamin C but not protein. Ice cream supplies only some
incomplete protein, making it less helpful in tissue repair.
88. Answer: (D) Sims left lateral
Rationale: The Sims left lateral position is the most common position used to administer a cleansing enema
because it allows gravity to aid the flow of fluid along the curve of the sigmoid colon. If the client cant assume this
position nor has poor sphincter control, the dorsal recumbent or right lateral position may be used. The supine and
prone positions are inappropriate and uncomfortable for the client.
89. Answer: (A) Arrange for typing and cross matching of the clients blood.
Rationale: The nurse first arranges for typing and cross matching of the clients blood to ensure compatibility with
donor blood. The other options, although appropriate when preparing to administer a blood transfusion, come later.
90. Answer: (A) Independent
Rationale: Nursing interventions are classified as independent, interdependent, or dependent. Altering the drug
schedule to coincide with the clients daily routine represents an independent intervention, whereas consulting with
the physician and pharmacist to change a clients medication because of adverse reactions represents an
interdependent intervention. Administering an already-prescribed drug on time is a dependent intervention. An
intradependent nursing intervention doesnt exist.
91. Answer: (D) Evaluation
Rationale: The nursing actions described constitute evaluation of the expected outcomes. The findings show that the
expected outcomes have been achieved. Assessment consists of the clients history, physical examination, and
laboratory studies. Analysis consists of considering assessment information to derive the appropriate nursing
diagnosis. Implementation is the phase of the nursing process where the nurse puts the plan of care into action.
92. Answer: (B) To observe the lower extremities
Rationale: Elastic stockings are used to promote venous return. The nurse needs to remove them once per day to
observe the condition of the skin underneath the stockings. Applying the stockings increases blood flow to the heart.
When the stockings are in place, the leg muscles can still stretch and relax, and the veins can fill with blood.
93. Answer:(A) Instructing the client to report any itching, swelling, or dyspnea.
Rationale: Because administration of blood or blood products may cause serious adverse effects such as allergic
reactions, the nurse must monitor the client for these effects. Signs and symptoms of life-threatening allergic

reactions include itching, swelling, and dyspnea. Although the nurse should inform the client of the duration of the
transfusion and should document its administration, these actions are less critical to the clients immediate health.
The nurse should assess vital signs at least hourly during the transfusion.
94. Answer: (B) Decrease the rate of feedings and the concentration of the formula.
Rationale: Complaints of abdominal discomfort and nausea are common in clients receiving tube feedings.
Decreasing the rate of the feeding and the concentration of the formula should decrease the clients discomfort.
Feedings are normally given at room temperature to minimize abdominal cramping. To prevent aspiration during
feeding, the head of the clients bed should be elevated at least 30 degrees. Also, to prevent bacterial growth,
feeding containers should be routinely changed every 8 to 12 hours.
95. Answer: (D) Roll the vial gently between the palms.
Rationale: Rolling the vial gently between the palms produces heat, which helps dissolve the medication. Doing
nothing or inverting the vial wouldnt help dissolve the medication. Shaking the vial vigorously could cause the
medication to break down, altering its action.
96. Answer: (B) Assist the client to the semi-Fowler position if possible.
Rationale: By assisting the client to the semi-Fowler position, the nurse promotes easier chest expansion, breathing,
and oxygen intake. The nurse should secure the elastic band so that the face mask fits comfortably and snugly
rather than tightly, which could lead to irritation. The nurse should apply the face mask from the clients nose down to
the chin not vice versa. The nurse should check the connectors between the oxygen equipment and humidifier to
ensure that theyre airtight; loosened connectors can cause loss of oxygen.
97. Answer: (B) 4 hours
Rationale: A unit of packed RBCs may be given over a period of between 1 and 4 hours. It shouldnt infuse for
longer than 4 hours because the risk of contamination and sepsis increases after that time. Discard or return to the
blood bank any blood not given within this time, according to facility policy.
98. Answer: (B) Immediately before administering the next dose.
Rationale: Measuring the blood drug concentration helps determine whether the dosing has achieved the
therapeutic goal. For measurement of the trough, or lowest, blood level of a drug, the nurse draws a blood sample
immediately before administering the next dose. Depending on the drugs duration of action and half-life, peak blood
drug levels typically are drawn after administering the next dose.
99. Answer: (A) The nurse can implement medication orders quickly.
Rationale: A floor stock system enables the nurse to implement medication orders quickly. It doesnt allow for
pharmacist input, nor does it minimize transcription errors or reinforce accurate calculations.
100. Answer: (C) Shifting dullness over the abdomen.
Rationale: Shifting dullness over the abdomen indicates ascites, an abnormal finding. The other options are normal
abdominal findings.

SITUATION: A Nurse utilizes the nursing process in managing patient care. Knowledge of this process is essential
to deliver high quality care and to focus on the clients response to their illness.
1. During the planning phase of the nursing process, which of the following is the product developed?
A. Nursing care plan
B. Nursing diagnosis
C. Nursing history
D. Nursing notes
2. Objective data are also known as?
A. Covert data
B. Inferences
C. Overt data
D. Symptoms
3. Data or information obtained from the assessment of a patient is primarily used by nurse to:
A. Ascertain the patients response to health problems
B. Assist in constructing the taxonomy of nursing intervention
C. Determine the effectiveness of the doctors order
D. Identify the patients disease process
4. What is an example of a subjective data?
A. Color of wound drainage
B. Odor of breath
C. Respiration of 14 breaths/minute
D. The patients statement of I feel sick to my stomach
SITUATION : Correct application of the Nursing Process is vital in providing quality care. The nurse must use her
skills and knowledge in proper assessment, planning and evaluating to meet the patients need and address the
priority response of the client to his or her illness.
5. Which statement is a difference between comprehensive and focused assessment?
A. Comprehensive assessments cant include any focus assessments
B. Focused assessments are more important than comprehensive assessments

C. Focused assessments are usually ongoing and concerning specific problems


D. Objective data are included only in comprehensive assessments
6. Two-year-old Bens mother states Ben vomited 8 ounces of his formula this morning. This statement is
an example of:
A. Objective data from a primary source
B. Objective data from a secondary source
C. Subjective data from a primary source
D. Subjective data from a secondary source
7. Which expected outcome is correctly written?
A. The patient will be less edematous in 24 hours
B. The patient will drink an adequate amount of fluid daily
C. The patient will identify 5 high-salt foods from prepared list by discharge
D. The patient will soon sleep well through the night
8. An expected outcome on a patients care plan reads: Patient will state seven warning signs of cancer by
discharge. When the nurse evaluates the patient progress, the patient is able to state that a change in wart
or mole, a sore that doesnt heal and a change in bowel or bladder habits are warning signals of cancer.
Which of the following would be an appropriate evaluative statement for the nurse to place on the patients
nursing care plan?
A. Patient understands the warning signals of cancer
B. Goal met; Patient cited a change in wart of mole, sore that doesnt heal and a change in bowel or bladder habits
as warning signals of cancer.
C. Goal not met
D. Goal partially met
9. A quality assurance nurse sends questionnaire to patients after discharge to determine their level of
satisfaction with the nurse care they received in the facility. What type of nursing audit is this?
A. Concurrent
B. Outcome
C. Terminal
D. Retrospective
10. The nurse makes the following entry in the patients record: Goal not met; patient refuses to attend
smoking cessation classes. Because this goal hasnt been met, the nurse should:

A. Develop a completely new nursing care plan


B. Assign the patient to a more experienced nurse
C. Critique the steps involved in the development of the goal
D. Transfer the patient to another facility
SITUATION : Health Care Delivery System affects the health status of every Filipino. As a nurse knowledgeable of
this system, she is expected to ensure quality of life.
11. When should rehabilitation commence?
A. The day before discharge
B. When the patient desires
C. Upon admission
D. 24 hours after discharge
12. What exemplified the preventive and promotive programs in the hospital?
A. Hospital as a center to prevent and control infection
B. Program for smokers
C. Program for alcoholics and drug addicts
D. Hospital Wellness Center
13. Which makes nursing dynamic?
A. Every patient is a unique physical, emotional, social and spiritual being
B. The patient participate in the overall nursing care plan
C. Nursing practice is expanding in the light of modern developments that takes place
D. The health status of the patient is constantly changing and the nurse must be cognizant and responsive to these
changes
14. Prevention is an important responsibility of the nurse in:
A. Hospitals
B. Community
C. Workplace
D. All of the above
15. This form of Health Insurance provides comprehensive prepaid health services to enrollees for a fixed
periodic payment.

A. Health Maintenance Organization


B. Medicare
C. Philippine Health Insurance Act
D. Hospital Maintenance Organization
SITUATION : Maslows hierarchy of needs is an integral component of prioritization. As a nurse, you must know the
clients needs that request for an immediate action.
16. The client with history of pulmonary emboli is scheduled for insertion of an inferior vena cava filter. The
nurse checks on the client 1 hour after the physician has explained the procedure and obtained consent
from the client. The client is lying in bed, wringing the hands, and says to the nurse, Im not sure about
this. What if it doesnt work, and Im just as bad off as before? The nurse addresses which of the following
primary concerns of the client?
A. Fear related to the potential risk and outcome of surgery
B. Anxiety related to the fear of death
C. Ineffective individual coping related to the therapeutic regimen
D. Knowledge deficit related to the surgical procedure
17. A nurse is formulating a plan of care for a client receiving enteral feedings. The nurse identifies which
nursing diagnosis as the highest priority for this client?
A. Altered nutrition, less than body requirements
B. High risk for aspiration
C. High risk for fluid volume deficit
D. Diarrhea
18. A home care nurse finds a client in the bedroom, unconscious, with pill bottle in hand. The pill bottle
contained the SSRI Sertraline (Zoloft). The nurse immediately assesses the clients:
A. Blood pressure
B. Respirations
C. Pulse
D. Urine Output
19. A nurse is caring for a client admitted to the hospital for subclavian line placement. Which psychosocial
area of assessment should the nurse address with the client?
A. Strict restrictions of neck mobility
B. Loss of ability to ambulate as tolerated

C. Possible body image disturbance


D. Continuous pain related to ongoing placement of the subclavian line
20. A hospitalized client who has a living will is being fed through a nasogastric tube. During a bolus
feeding, the client vomits and begins choking. Which of the following actions is most appropriate for the
nurse to take?
A. Clear the clients airway
B. Make the client comfortable
C. Start CPR
D. Stop feeding and remove the NG tube
SITUATION: Health Promotion is the priority nursing responsibility. The nurse should be able to promote the
clients well being and identify ways on improving the clients quality of life.
21. The nurse is caring for a 16 year old female client who isnt sexually active. The client asks if she needs
a Papanicolaou test. The nurse should reply:
A. Yes, she should have a Pap test after the onset of menstruation
B. No, because she isnt sexually active
C. Yes, because shes 16 years old
D. No, because she is not yet 21 years old
22. The nurse is assessing a client who complains of abdominal pain, nausea and diarrhea. When
examining the clients abdomen, which sequence should the nurse use?
A. Inspection, palpation, percussion, auscultation
B. Inspection, auscultation, percussion, palpation
C. Auscultation, inspection, percussion, palpation
D. Palpation, auscultation, percussion, inspection
23. The nurse is speaking to a group of women about early detection of breast cancer. The average age of
the women in the group is 45. Following the Philippine Cancer Society guidelines, the nurse should
recommend that the women:
A. Perform breast self examination annually
B. Have a mammogram annually
C. Have a hormonal receptor assay annually
D. Have a physician conduct a clinical examination every 2 years

24. The school nurse is planning a program for a group of teenagers on skin cancer prevention. Which of
the following instruction should the nurse emphasize in her talk?
A. Stay out of the sun between 1 p.m and 3 p.m
B. Tanning booth are a safe alternative for those who wish to tan
C. Sun exposure is safe, provided the client wears protective clothing
D. Examine skin once per month, looking for suspicious lesions or changes in moles
25. An employer establishes a physical exercise area in the workplace and encourages all employees to use
it. This is an example of which level of prevention?
A. Primary prevention
B. Secondary prevention
C. Tertiary prevention
D. Passive prevention
SITUATION : Basic nursing skills are essential for they are vital in many nursing procedures. Such skills are needed
in order to promote health, prevent illness, cure a disease and rehabilitate infirmities.
26. A nurse has just been told by the physician that an order has been written to administer an iron injection
to a client. The nurse plans to give the medication in the:
A. Gluteal muscle using Z-Track technique
B. Deltoid muscle using an air lock
C. Subcutaneous tissue of the abdomen
D. Anterolateral thigh using 5/8-inch needle
27. A client has just been told by the physician that a cerebral angiogram will be obtained. The nurse then
collects data from the client about which of the following pieces of information?
A. Claustrophobia
B. Excessive weight
C. Allergy to eggs
D. Allergy to iodine or shellfish
28. A client has an order for a wound culture to be performed with the next wound irrigation and dressing
change. The nurse would plan to use which of the following solutions for irrigation before this particular
procedure?

A. Povidone-iodine (Betadine)
B. One-half-strength hydrogen peroxide
C. Normal saline
D. Acetic acid
29. Which of the following is the best indicator in determining that the clients airway needs suctioning?
A. Oxygen saturation measurement
B. Respiratory rate
C. Breath sounds
D. Arterial blood gas results
30. A nurse is planning to assist a diabetic client to perform self-monitoring of blood glucose levels. The
nurse should incorporate which of the following strategies to best help the client obtain an adequate
capillary sample?
A. Cleanse the hands beforehand using cool water
B. Let the arm hang dependently and milk the digit
C. Puncture the center of the finger pad
D. Puncture the finger as deeply as possible
SITUATION : Mastery of Intravenous therapy and all aspects that address the response of the client to complication
related to it will help the new nurse in providing quality care.
31. One hour after the IV was inserted, Nurse Net found out that the 1 liter of D5NSS was empty. Patient was
in severe respiratory distress with pinkish frothy sputum. The most probable complication is:
A. Speed shock
B. Renal failure
C. Congestive heart failure
D. Pulmonary edema
32. When an IV of D5W is not infusing well on patient, it is best to first:
A. Pinch the rubber part to flush out clogged blood
B. Coil tube and squeeze hard
C. Lower IV to check for return flow
D. Restart the IV

33. The doctor ordered venoclysis of dextrose 5% in water one liter KVO for 24 hours as a vehicle for IV
medications. How many drops per minute should the fluid be regulated?
A. 20 drops per minute
B. 5 drops per minute
C. 10 drops per minute
D. 15 drops per minute
34. The nurse is administering I.V fluids to an infant. Infants receiving I.V Therapy are particularly vulnerable
to:
A. Hypotension
B. Fluid overload
C. Cardiac arrhythmias
D. Pulmonary emboli
35. A client with severe inflammatory bowel disease is receiving TPN. When administering TPN, the nurse
must take care to maintain the prescribed flow rate because giving TPN too rapidly may cause:
A. Hyperglycemia
B. Air embolism
C. Constipation
D. Dumping syndrome
SITUATION : Teaching and learning is a nursing responsibility. The ability of a nurse to educate and change the
clients behavior through health teaching is one important goal of nursing.
36. A clients hypertension has been recently diagnosed. The nurse would plan to do which of the following
as the first step in teaching the client about the disorder?
A. Gather all available resource material
B. Plan for the evaluation of the session
C. Assess the clients knowledge and needs
D. Decide on the teaching approach
37. A diabetic client who is performing self-monitoring of blood glucose at home asks a clinic nurse why a
glycosylated hemoglobin level needs to be measured. The nurse should plan to incorporate which of the
following into a response?

A. This laboratory test is done yearly to predict likelihood of long term complication
B. This laboratory test gives an indication of glycemic control over the last 3 months
C. It is done as a method of verifying the accuracy of the meter used at Home
D. It is done to predict risk of hypoglycemia with the current diet and medication regimen
38. In teaching the sister of a diabetic client about the proper use of a glucometer in determining the blood
sugar level of the client, The nurse is focusing in which domain of learning according to bloom?
A. Cognitive
B. Affective
C. Psychomotor
D. Affiliative
39. Which of the following is the most important condition for diabetic client to learn how to control their
diet?
A. Use of pamphlets and effective teaching devices during health instruction
B. Motivation to be symptom free
C. Ability of the client to understand teaching instruction
D. Language and appropriateness of the instruction
40. When you prepare your teaching plan for a group of hypertensive clients, you first formulate your
learning objectives. Which of the following steps in the nursing process corresponds to the writing of
learning objectives?
A. Planning
B. Implementing
C. Evaluating
D. Assessing
SITUATION : Nursing is a science. It involves a wide spectrum of theoretical foundation applied in different health
care situation. The nurse must use these theories in order to deliver the most needed quality care.
41. The theorist who believes that adaptation and manipulation of stressors are needed to foster change is:
A. Betty Neuman
B. Dorothea Orem
C. Martha Rogers
D. Sister Callista Roy

42. The theorist whose theory can be defined as the development of a science of humankind, incorporating
the concepts of energy fields, openness, pattern and organization is:
A. Dorothy Johnson
B. Hildegard Peplau
C. Martha Rogers
D. Myra Levine
43. A theorist whose major theme is the idea of transcultural nursing and caring nursing is:
A. Dorothea Orem
B. Madeleine Leininger
C. Sister Callista Roy
D. Virginia Henderson
44. Florence Nightingale was born in
A. Italy, May 12, 1840
B. Italy, May 12, 1820
C. England, May 12, 1840
D. England, May 12, 1820
45. Smith conceptualizes this health model as a condition of actualization or realization of persons
potential. Avers that the highest aspiration of people is fulfillment and complete developmental
actualization.
A. Clinical Model
B. Role performance Model
C. Adaptive Model
D. Eudaemonistic Model
SITUATION : Oxygen is the most important physiologic requirement of the body. Absence of this vital element for
over 6 minutes leads to irreversible brain damage. Measures that promotes oxygenation is integral in successfully
managing clients response to illnesses.
46. The primary effect of oxygen therapy is to:
A. Increase oxygen in the tissues and cells
B. Increase oxygen carrying capacity of the blood

C. Increase respiratory rate


D. Increase oxygen pressure in the alveolar sac
47. A nurse suctions a client who has an endotracheal tube in place. Following the procedure, which of the
following would indicate to the nurse that the client is experiencing an adverse effect of this procedure?
A. Hypertension
B. Cardiac Irregularities
C. A reddish coloration in the clients face
D. Oxygen saturation level of 95%
48. The GAUGE size in ET tubes determines:
A. The external circumference of the tube
B. The internal diameter of the tube
C. The length of the tube
D. The tubes volumetric capacity
49. The nurse is correct in performing suctioning when she applies the suction intermittently during:
A. Insertion of the suction catheter
B. Withdrawing of the suction catheter
C. both insertion and withdrawing of the suction catheter
D. When the suction catheter tip reaches the bifurcation of the trachea
50. The purpose of the cuff in Tracheostomy tube is to:
A. Separate the upper and lower airway
B. Separate trachea from the esophagus
C. Separate the larynx from the nasopharynx
D. Secure the placement of the tube
SITUATION : To deliver a safe and quality care, Knowledge about wound care is necessary. The nurse will have to
deal with different types of wound during practice. It is of outmost important to apply this knowledge to ensure
optimum wound healing.
51. Based on the nurses knowledge of surgical wounds, simple surgical incisions heal by:
A. Primary intention
B. Secondary intention

C. Tertiary intention
D. Quarternary intention
52. The nurse documents that the wound edges are approximated. When the edges of an incision are said
to be approximated, this means edges are:
A. Brought together by sutures, tapes or staples
B. Eythematous and swollen
C. Gaping and draining
D. Necrotic and draining
53. Which vitamin is most essential for collagen synthesis?
A. Vitamin A
B. Vitamin B
C. Vitamin C
D. Vitamin D
54. When assessing the clients wound for sign of infection, the nurse should look for the presence of which
of the following?
A. Granulation tissue
B. Pink tissue
C. Purulent drainage
D. Well approximated edges
55. The nurse is changing dressing and providing wound care. Which activity should she perform first?
A. Assess the drainage in the dressing
B. Slowly remove the soiled dressing
C. Wash hands thoroughly
D. Put on latex gloves
SITUATION : Physical examination and health assessment are important in rendering care. The nurse must use
assessment knowledge in order to determine and prioritize clients response to his or her illness.
56. The component that should receive the highest priority before physical examination is the:
A. Psychological preparation of the client
B. Physical Preparation of the client

C. Preparation of the Environment


D. Preparation of the Equipments
57. When inspecting a clients skin, the nurse finds a vesicle on the clients arm. Which description applies
to a vesicle?
A. A flat, nonpalpable, and colored
B. Solid, elevated, and circumscribed
C. Circumscribed, elevated, and filled with serous fluid
D. Elevated, pus-filled, and circumscribed
58. When assessing a client with abdominal pain, the nurse should assess:
A. Any quadrant first
B. The symptomatic quadrant first
C. The symptomatic quadrant last
D. The sympomatic quadrant either second or third
59. To assess the clients dorsalis pedis pulse, the nurse should palpate the:
A. Medial surface of the ankle
B. Lateral surface of the ankle
C. Ventral aspect of the top of the foot
D. Medial aspect of the dorsum of the foot
60. Which of the following assessment would be a priority for a 2 year old after bronchoscopy?
A. Cardiac rate
B. Respiratory quality
C. Sputum color
D. Pulse pressure changes
61. The nurse checks the clients gag reflex. The recommended technique for testing the gag reflex is to:
A. touch the back of the clients throat with a tongue depressor
B. observe the client for evidence of spontaneous swallowing when the neck is stroked
C. place a few milliliters of water on the clients tongue and note whether or not he swallows
D. observe the clients response to the introduction of a catheter for endotracheal suctioning

62. The nurse is evaluating a clients lung sounds. Which of the following breath sounds indicate adequate
ventilation when auscultated over the lung fields?
A. Vesicular
B. Bronchial
C. Bronchovesicular
D. Adventitious
63. The night nurse informs the primary nurse that a client receiving intermittent gavage feedings is not
tolerating them. The primary nurse should first:
A. Change the feeding schedule to omit nights
B. Request that the type of solution be changed
C. Observe the night nurse administering a feeding
D. Suggest that the prescribed antiemetic be given first
64. A client has a chest tubes attached to a pleural drainage system. When caring for this client, the nurse
should:
A. Palpate the surrounding are for crepitus
B. Clamp the chest tubes when suctioning the client
C. Change the dressing daily using aseptic technique
D. Empty drainage chamber at the end of the shift
65. The nurse, aware of a clients 25 year history of excessive alcohol use, would expect the physical
assessment to reveal a:
A. Liver infection
B. Low blood ammonia
C. Small liver with a rough surface
D. High fever with a generalizedrash
SITUATION : Nursing is a profession. Miss Linda is a newly appointed nurse in a hospital in Manila. Born May 1985,
Her knowledge of nursings professional development is required in order to project the profession in a way that it
lifts the standards of nursing.
66. Mrs. Linda defined nursing as one of the following except:
A. Assisting individual, family and community in attaining health
B. Assisting basic health needs

C. Establishing nursing diagnosis and implementing nursing care


D. Diagnosing, treating, prescribing medication and doing minor surgery
67. Philippine Nurses Association (PNA) was established in:
1. 1922
2. 1926
3. With Mrs. Rosario Delgado as first president
4. With Mrs. Anastacia Tupas as first president
A. 1, 3
B. 1, 4
C. 2, 3
D. 2, 4
68. As a national nurses association, it is characterized as follows except:
A. Both a professional body and a labor union
B. Affiliated with the International Council Of Nurses
C. Advocating for improved work and life condition of Nurses
D. Accrediting body for continuing education program
69. CPE Units per year is needed for license renewal. The number of CPE units per year should be:
A. 20 units
B. 30 units
C. 60 units
D. 100 units
70. As a nurse, R.A 1080 exempts her from:
A. Paying her professional tax
B. Examination for civil service eligibility
C. Qualifying for the CGFNS
D. Paying business permit
71. In resigning for her job as a staff nurse, she must give advance notice of:
A. 15 days
B. 30 days

C. 45 days
D. 60 days
72. Why is there an ethical dilemma?
A. Because the law do not clearly state what is right from what is wrong
B. Because morality is subjective and it differs from each individual
C. Because the patients right coincide with the nurses responsibility
D. Because the nurse lacks ethical knowledge to determine what action is correct and what action is unethical
73. Who among the following can work as a practicing nurse in the Philippines without taking the Licensure
examination?
A. Internationally well known experts which services are for a fee
B. Those that are hired by local hospitals in the country
C. Expert nurse clinicians hired by prestigious hospitals
D. Those involved in medical mission whos services are for free
74. The nurse is correct in determining that a License is:
A. A personal property
B. Can be revoked by the Board of Nursing
C. A Right
D. Can be revoked by the PNA
75. A License is renewed every:
A. 1 year
B. 2 years
C. 3 years
D. 4 years
76. Which of the following persons cannot have free access to a patients record?
A. The patient
B. The physical therapist
C. The pharmacist
D. The lawyer
77. Ms. Lindas license will expire in 2007, She must renew her license when?

A. January 2007
B. December 2007
C. May 2007
D. May 2008
78. The practice of Nursing in the Philippines is regulated by:
A. RA 9163
B. RA 9173
C. RA 7164
D. RA 7146
79. This quality is being demonstrated by a Nurse who raise the side rails of a confuse and disoriented
patient?
A. Autonomy
B. Responsibility
C. Prudence
D. Resourcefulness
80. Nurse Joel and Ana is helping a 16 year old Nursing Student in a case filed against the student. The
case was frustrated homicide. Nurse Joel and Ana are aware of the different circumstances of crimes. They
are correct in identifying which of the following Circumstances that will be best applied in this case?
A. Justifying
B. Aggravating
C. Mitigating
D. Exempting
SITUATION : This is the first day of Mark, R.N. to report as a staff nurse in a tertiary hospital. As a morning duty
nurse, she is about to chart her nursing care.
81. Which of the following is not an accepted medical abbreviation?
A. NPO
B. PRN
C. OD
D. NON

82. Communication is best undertaken if barriers are first removed. Considering this statement, which of
the following is considered as deterrent factor in communication?
A. Not universally accepted abbreviation
B. Wrong Grammar
C. Poor Penmanship
D. Old age of the client
83. Which of the following chart entries are not acceptable?
A. Patient complained of chest pain
B. Patient ambulated to B/R
C. Vital signs 130/70; 84; 20
D. Pain relieved by Nitrogylcerine gr 1/150 sublingually
84. Which of the following indicates narrative charting?
A. Written descriptive nurses notes
B. Date recorded on nurse activity sheet
C. Use of checklist
D. Use of flowsheet
85. Being a new staff nurse, Mark remembers that information needed for daily nursing care of clients are
readily available in which of the following?
A. Kardex
B. Order sheet
C. Admission notes
D. Nurses notes
SITUATION : Mr. Orlando is assigned to Mang Carlos, A 60 years old, newly diagnosed diabetes patient. He is
beginning to write objectives for his teaching plan.
86. Which of the following is written in behavioral term?
A. Mang Carlos will know about diabetes related foot care and the techniques and equipment necessary to carry it
out
B. Mang Carlos sister will be able to determine his insulin requirement based on blood glucose levels obtained from
glucometer in two days

C. Mang Carlos daugter should learn about diabetes milletus within the week
D. Mang Carlos wife needs to understand the side effects of insulin
87. Which of the following is the best rationale for written objectives?
A. Document the quality of care
B. Facilitate evaluation of the performance of the nurse
C. Ensure learning on the part of the nurse
D. Ensure communication among staff members
88. Mang Carlos has been terminally ill for 5 years. He asked his wife to decide for him when he is no longer
capable to do so. As a Nurse, You know that this is called:
A. Last will and testament
B. DNR
C. Living will
D. Durable Power of Attorney
89. Which of the following behavior of Mang Carlos least indicates readiness to learn?
A. Talking with the nurse in charge and doctor
B. Reading brochures and pamphlets about diabetes
C. Inquiring about date of discharge
D. Asking question about diabetes milletus
90. Which of the following behaviors best contribute to the learning of Mang Carlos regarding his disease
condition?
A. Drawing him into discussion about diabetes
B. Frequent use of technical terms
C. Loosely structured teaching session
D. Detailed lengthy explanation
SITUATION : Mrs. Purificacion is now the Chief nurse of a hospital in Manila. She is carefully reviewing different
management styles and theories that will best help her in running the nursing services in the hospital.
91. Which leadership style best empower the staff towards excellence?
A. Autocratic
B. Situational

C. Democratic
D. Laissez Faire
92. As a Nurse Manager, DSJ enjoys his staff of talented and self motivated individuals. He knew that the
leadership style to suit the needs of this kind of people is called:
A. Autocratic
B. Participative
C. Democratic
D. Laissez Faire
93. A fire has broken in the unit of DSJ R.N. The best leadership style suited in cases of emergencies like
this is:
A. Autocratic
B. Participative
C. Democratic
D. Laissez Faire
94. Mrs. Purificacion is thinking of introducing the Primary Nursing Model Approach. You understand that
this nursing model is:
A. The nurse manager assigns tasks to the staff members
B. Critical paths are used in providng nursing care
C. A single registered nurse is responsible for planning and providing individualized nursing care
D. Nursing staff are led by an RN leader in providing care to a group of clients
95. Structure, Process and Outcome are components of which step of the management process?
A. Planning
B. Organizing
C. Directing
D. Controlling
SITUATION : Nursing research is the term used to describe the evidence used to support nursing practice. Nursing,
as an evidence based area of practice, has been developing since the time of Florence Nightingale to the present
day, where many nurses now work as researchers based in universities as well as in the health care setting.
96. Mr. DSJ Plans to undertake a research of Community 1 and 2 on how they manage their health using
Primary health care after an organization and training seminars. This type of research is:

A. Experimental
B. Historical
C. Descriptive
D. Basic
97. The independent variable is:
A. Primary Health Care
B. Community 1 and 2
C. Organization and training seminars
D. Management of their health
98. In this design, the variable that is being manipulated is
1. Independent
2. Organization and training seminars
3. Dependent
4. Management of Primary Health Care
A. 1,2
B. 1,4
C. 2,3
D. 3,4
99. In general, the research process follows the ff. ordered sequence:
1. Determination of design
2. Statement of the problem
3. Definition of variables
4. Collection and analysis of data
5. Review of related literature
A. 2,5,3,1,4
B. 3,5,4,1,2
C. 2,5,3,4,1
D. 2,5,1,3,4
100. Studies done in natural setting such as this one, posses difficulty of controlling which variable?

A. Independent
B. Dependent
C. Extraneous
D. Organismic

Answers
Here are the answers for the exam. Unfortunately, rationales are not given. If you need clarifications or disputes,
please direct them to the comments section and well be glad to give you an explanation.
1. A. Nursing care plan
2. C. Overt data
3. A. Ascertain the patients response to health problems
4. D. The patients statement of I feel sick to my stomach
5. C. Focused assessments are usually ongoing and concerning specific problems
6. B. Objective data from a secondary source
7. C. The patient will identify 5 high-salt foods from prepared list by discharge
8. D. Goal partially met
9. C. Terminal
10. C. Critique the steps involved in the development of the goal
11. C. Upon admission
12. D. Hospital Wellness Center
13. D. The health status of the patient is constantly changing and the nurse must be cognizant and
responsive to these changes
14. D. All of the above
15. A. Health Maintenance Organization
16. A. Fear related to the potential risk and outcome of surgery
17. B. High risk for aspiration
18. B. Respirations
19. C. Possible body image disturbance
20. A. Clear the clients airway
21. B. No, because she isnt sexually active
22. B. Inspection, auscultation, percussion, palpation
23. B. Have a mammogram annually
24. D. Examine skin once per month, looking for suspicious lesions or changes in moles
25. A. Primary prevention
26. A. Gluteal muscle using Z-Track technique
27. D. Allergy to iodine or shellfish
28. C. Normal saline

29. C. Breath sounds


30. D. Puncture the finger as deeply as possible
31. D. Pulmonary edema
32. C. Lower IV to check for return flow
33. C. 10 drops per minute
34. B. Fluid overload
35. A. Hyperglycemia
36. C. Assess the clients knowledge and needs
37. B. This laboratory test gives an indication of glycemic control over the last 3 months
38. C. Psychomotor
39. B. Motivation to be symptom free
40. A. Planning
41. D. Sister Callista Roy
42. C. Martha Rogers
43. B. Madeleine Leininger
44. B. Italy, May 12, 1820
45. D. Eudaemonistic Model
46. A. Increase oxygen in the tissues and cells
47. B. Cardiac Irregularities
48. B. The internal diameter of the tube
49. B. Withdrawing of the suction catheter
50. A. Separate the upper and lower airway
51. A. Primary intention
52. A. Brought together by sutures, tapes or staples
53. C. Vitamin C
54. C. Purulent drainage
55. C. Wash hands thoroughly
56. A. Psychological preparation of the client
57. C. Circumscribed, elevated, and filled with serous fluid
58. C. The symptomatic quadrant last
59. D. Medial aspect of the dorsum of the foot
60. B. Respiratory quality
61. A. touch the back of the clients throat with a tongue depressor
62. A. Vesicular
63. C. Observe the night nurse administering a feeding
64. A. Palpate the surrounding are for crepitus
65. C. Small liver with a rough surface
66. D. Diagnosing, treating, prescribing medication and doing minor surgery
67. A. 1,3
68. A. Both a professional body and a labor union

69. A. 20 units
70. B. Examination for civil service eligibility
71. B. 30 days
72. C. Because the patients right coincide with the nurses responsibility
73. D. Those involved in medical mission whos services are for free
74. B. Can be revoked by the Board of Nursing
75. C. 3 years
76. D. The lawyer
77. C. May 2007
78. B. RA 9173
79. C. Prudence
80. C. Mitigating
81. D. NON
82. A. Not universally accepted abbreviation
83. C. Vital signs 130/70; 84; 20
84. A. Written descriptive nurses notes
85. A. Kardex
86. B. Mang Carlos sister will be able to determine his insulin requirement based on blood glucose levels
obtained from glucometer in two days
87. A. Document the quality of care
88. D. Durable Power of Attorney
89. C. Inquiring about date of discharge
90. C. Loosely structured teaching session
91. C. Democratic
92. D. Laissez Faire
93. A. Autocratic
94. C. A single registered nurse is responsible for planning and providing individualized nursing care
95. D. Controlling
96. A. Experimental
97. C. Organization and training seminars
98. A. 1,2
99. A. 2,5,3,1,4
100.

C. Extraneous

Situation 1: Nursing is a profession. The nurse should have a background on the theories and foundation of nursing
as it influenced what is nursing today.
1. Nursing is the protection, promotion and optimization of health and abilities, prevention of illness and
injury, alleviation of suffering through the diagnosis and treatment of human response and advocacy in the
care of the individuals, families, communities and population. This is the most accepted definition of
nursing as defined by the:

A. PNA
B. ANA
C. Nightingale
D. Henderson
2. Advancement in Nursing leads to the development of the Expanded Career Roles. Which of the following
is NOT expanded career role for nurse?
A. Nurse practitioner
B. Clinical Nurse Specialist
C. Nurse Researcher
D. Nurse anaesthesiologist
3. The Board of Nursing regulated the Nursing profession in the Philippines and is responsible for the
maintenance of the quality of nursing in the country. Powers and duties of the board of nursing are the
following EXCEPT:
A. Issue, suspend revoke certificates of registration
B. Issue subpoena duces tecum, ad testificandum
C. Open and close colleges of nursing
D. Supervise and regulate the practice
4. A nursing student or a beginning staff nurse who has not yet experienced enough in a situation to make
judgments about them is in what stage of Nursing Expertise?
A. Novice
B. Newbie
C. Advanced Beginner
D. Competent
5. Benners Proficient nurse level is different from the other levels in nursing expertise in the context of
having:
A. The ability to organize and plan activities
B. Having attained an advanced level of education
C. A holistic understanding and perception of the client
D. Intuitive and analytic ability in new situations
Situation 2: The nurse has been asked to administer an injection via Z TRACK technique. Questions 6 to 10 refer
this.

6. The nurse prepares an IM injection for an adult client using the Z track techniques, 4 ml of medication is
to be administered to the client. Which of the following site will you choose?
A. Deltoid
B. Rectus Femoris
C. Ventrogluteal
D. Vastus lateralis
7. In infants 1 year old and below, which of the following is the site of choice in intramuscular injection?
A. Deltoid
B. Rectus Femoris
C. Ventrogluteal
D. Vastus lateralis
8. In order to decrease discomfort in Z track administration, which of the following should be done?
A. Pierce the skin quickly and smoothly at 90 degree angle
B. Inject the medication at around 10 minutes per millilitre
C. Pull back the plunger and aspirate for 1 minute t make sure that the needle did not hit a blood vessel
D. Pierce the skin slowly and carefully at a 90 degree angle
9. After injection using the Z track technique, the nurse should know that she needs to wait for few second
before withdrawing the needle and this is to allow the medication to disperse into the muscle tissue thus
decreasing the clients discomfort. How many seconds should the nurse wait before withdrawing the
needle?
A. 2 second
B. 5 seconds
C. 10 seconds
D. 15 seconds
10. The rationale in using the Z track technique in an intramuscular injection is:
A. It decreases the leakage of discolouring and irritating medication into the subcutaneous tissue.
B. It will allow a faster absorption of the medication
C. The Z track technique prevent irritation of the muscle
D. It is much more convenient for the nurse

Situation 3: A client was rushed to the emergency room and you are his attending nurse. You are
performing a vital sign assessment:
11. All of the following are correct methods in assessment of the blood pressure EXCEPT:
A. Take the blood pressure reading on both arms for comparison
B. Listen to and identify the phases of Korotkoff sound
C. Pump the cuff to around 50mmHg above the point where the pulse is obliterated
D. Observe procedures for infection control
12. You attached a pulse oximeter to the client. You know that the purpose id to:
A. Determine if the clients hemoglobin level is low and if he needs blood transfusion
B. Check level of clients tissue perfusion
C. Measure the efficacy of the clients anti-hypertension medications
D. Detect oxygen saturation of arterial blood before symptoms of hypoxemia develops
13. After a few hours in the Emergency Room, the client is admitted to the ward with an order of hourly
monitoring of blood pressure. The nurse finds that the cuff is too narrow and this will cause the blood
pressure reading to be:
A. Inconsistent
B. Low systolic and high diastolic
C. Higher than what the reading should be
D. Lower than what the reading should be
14. Through the clients health history, you gather that the patient smokes and drinks coffee. When taking
the blood pressure of a client who recently smoked or drank coffee, how long should the nurse wait before
taking the clients blood pressure for accurate reading?
A. 15 minutes
B. 30 minutes
C. 1 hour
D. 5 minutes
15. While the client has pulse oximeter on his fingertip, you notice that the sunlight is shining on the area
where the oximeter is. Your action will be to:
A. Set and turn on the alarm of the oximeter
B. Do nothing since there is no identified problem

C. Cover the fingertip sensor with a towel or bedsheet


D. Change the location of the sensor every four hours
16. The nurse finds it necessary to recheck the blood pressure reading. In case of such reassessment, the
nurse should wait for a period of:
A. 15 seconds
B. 1 to 2 minutes
C. 30 minutes
D. 15 minutes
17. If the arm is said to be elevated when taking the blood pressure. It will create a:
A. False high reading
B. False low reading
C. True False reading
D. Indeterminate
18. You are to assessed the temperature of the client the next morning and found out that he ate ice cream.
How many minutes should you wait before assessing the clients oral temperature?
A. 10 minutes
B. 20 minutes
C. 30 minutes
D. 15 minutes
19. When auscultating the clients blood pressure the nurse hears the following: From 150 mmHg to 130
mmHg: Silence, Then: a thumping sound continuing down to 100 mmHg: muffled sound continuing down to
80 mmHg and then silence. What is the clients pressure?
A. 130/80
B. 150/100
C. 100/80
D. 150/100
20. In a client with a previous blood pressure of 130/80 4 hours ago, how long will it take to release the
blood pressure cuff to obtain an accurate reading?
A. 10 20 seconds
B. 30 45 seconds

C. 1 1.5 minutes
D. 3 3.5 minutes
Situation 4 Oral care is an important part of hygienic practices and promoting client comfort.
21. An elderly client, 84 years old, is unconscious. Assessment of the mouth reveals excessive dryness and
presence of sores. Which of the following is BEST to use for oral care?
A. lemon glycerine
B. hydrogen peroxide
C. Mineral oil
D. Normal saline solution
22. When performing oral care to an unconscious client, which of the following is a special consideration to
prevent aspiration of fluids into the lungs?
A. Put the client on a side lying position with head of bed lowered
B. Keep the client dry by placing towel under the chin
C. Wash hands and observe appropriate infection control
D. Clean mouth with oral swabs in a careful and an orderly progression
23. The advantages of oral care for a client include all of the following, EXCEPT:
A. decreases bacteria in the mouth and teeth
B. reduces need to use commercial mouthwash which irritate the buccal mucosa
C. improves clients appearance and self-confidence
D. improves appetite and taste of food
24. A possible problem while providing oral care to unconscious clients is the risk of fluid aspiration to
lungs. This can be avoided by:
A. Cleaning teeth and mouth with cotton swabs soaked with mouthwash to avoid rinsing the buccal cavity
B. swabbing the inside of the cheeks and lips, tongue and gums with dry cotton swabs
C. use fingers wrapped with wet cotton washcloth to rub inside the cheeks, tongue, lips and ums
D. suctioning as needed while cleaning the buccal cavity
25. Your client has difficulty of breathing and is mouth breathing most of the time. This causes dryness of
the mouth with unpleasant odor. Oral hygiene is recommended for the client and in addition, you will keep
the mouth moistened by using:

A. salt solution
B. water
C. petroleum jelly
D. mentholated ointment
Situation 5: Ensuring safety before, during and after a diagnostic procedure is an important responsibility of the
nurse.
26. To help Fernan better tolerate the bronchoscopy, you should instruct him to practice which of the
following prior to the procedure:
A. Clenching his fist every 2 minutes
B. Breathing in and out through the nose with his mouth open
C. Tensing the shoulder muscles while lying on his back
D. Holding his breath periodically for 30 seconds
27. Following a bronchoscopy, which of the following complains to Fernan should be noted as a possible
complication:
A. Nausea and vomiting
B. Shortness of breath and laryngeal stridor
C. Blood tinged sputum and coughing
D. Sore throat and hoarseness
28. Immediately after bronchoscopy, you instructed Fernan to:
A. Exercise the neck muscles
B. Breathe deeply
C. Refrain from coughing and talking
D. Clear his throat
29. Thoracentesis may be performed for cytologic study of pleural fluid. As a nurse your most important
function during the procedure is to:
A. Keep the sterile equipment from contamination
B. Assist the physician
C. Open and close the three-way stopcock
D. Observe the patients vital signs
30. Right after thoracentesis, which of the following is most appropriate intervention?

A. Instruct the patient not to cough or deep breathe for two hours
B. Observe for symptoms of tightness of chest or bleeding
C. Place an ice pack to the puncture site
D. Remove the dressing to check for bleeding
Situation 6: Knowledge of the acid base disturbance and the functions of the electrolytes is necessary to determine
appropriate intervention and nursing actions.
31. A client with diabetes mellitus has glucose level of 644 mg/dL. The nurse interprets that this client is at
most risk for the involvement at which type of acid base imbalance?
A. Respiratory acidosis
B. Respiratory alkalosis
C. Metabolic acidosis
D. Metabolic alkalosis
32. In a client in the health care clinic, arterial blood gas analysis gives the following results: pH 7.48, PCO2
32mmHg, PO2 94 mmHg, HCO3 24 mEq/L. The nurse interprets that the client has which acid base
disturbance?
A. Respiratory acidosis
B. Respiratory alkalosis
C. Metabolic acidosis
D. Metabolic alkalosis
33. A client has an order for ABG analysis on radial artery specimens. The nurse ensures that which of the
following has been performed or tested before the ABG specimen are drawn?
A. Guthing test
B. Allens test
C. Rombergs test
D. Webers test
34. A nurse is reviewing the arterial blood gas values of a client and notes that the pH is 7.31, Pco2 is 500
mmHg, and the bicarbonate is 27 mEq/L. The nurse concludes that which acid base disturbance is present
in this client?
A. Respiratory acidosis
B. Respiratory alkalosis

C. Metabolic acidosis
D. Metabolic alkalosis
35. Allens test checks the patency of the:
A. Ulnar artery
B. Radial artery
C. Carotid artery
D. Brachial artery
36. (skip) (Pretend Im not here.)
37. After IVP a renal stone was confirmed, a left nephrectomy was done. Her post operative order includes
daily urine specimen to be sent to the laboratory. Eileen has a foley catheter attached to a urinary
drainage system. How will you collect the urine specimen?
A. remove urine from drainage tube with sterile needle and syringe and empty urine from the syringe into the
specimen container
B. empty a sample urine from the collecting bag into the specimen container
C. disconnect the drainage tube from the indwelling catheter and allow urine to flow from catheter into the specimen
container.
D. disconnect the drainage the from the collecting bag and allow the urine to flow from the catheter into the
specimen container.
38. Where would the nurse tape Eileens indwelling catheter in order to reduce urethral irritation?
A. to the patients inner thigh
B. to the patients lower thigh
C. to the patients buttocks
D. to the patient lower abdomen
39. Which of the following menu is appropriate for one with low sodium diet?
A. instant noodles, fresh fruits and ice tea
B. ham and cheese sandwich, fresh fruits and vegetables
C. white chicken sandwich, vegetable salad and tea
D. canned soup, potato salad, and diet soda
40. How will you prevent ascending infection to Eileen who has an indwelling catheter?

A. see to it that the drainage tubing touches the level of the urine
B. change he catheter every eight hours
C. see to it that the drainage tubing does not touch the level of the urine
D. clean catheter may be used since urethral meatus is not a sterile area
Situation 7: Hormones are secreted by the various glands in the body. Basic knowledge of the endocrine system is
necessary.
41. Somatotropin or the Growth Hormone releasing hormone is secreted by the anterior pituitary gland:
A. Hypothalamus
B. Anterior pituitary gland
C. Posterior pituitary gland
D. Thyroid gland
42. All of the following are secreted by the anterior pituitary gland except:
A. Somatotropin/Growth hormone
B. Follicle stimulating hormone
C. Thyroid stimulating hormone
D. Gonadotropin hormone releasing hormone
43. All of the following hormones are hormones secreted by the Posterior pituitary gland except:
A. Vasopressin
B. Oxytocin
C. Anti-diuretic hormone
D. Growth hormone
44. Calcitonin, a hormone necessary for calcium regulation is secreted in the:
A. Thyroid gland
B. Hypothalamus
C. Parathyroid gland
D. Anterior pituitary gland
45. While Parathormone, a hormone that regulates the effect of calcitonin is secreted by the:
A. Thyroid gland
B. Hypothalamus

C. Parathyroid gland
D. Anterior pituitary gland
Situation 8 The staff nurse supervisor requests all the staff nurses to brainstorm and learn ways to instruct
diabetic clients on self-administration of insulin. She wants to ensure that there are nurses available daily to do
health education classes.
46. The plan of the nurse supervisor is an example of
A. in service education process
B. efficient management of human resources
C. increasing human resources
D. primary prevention
47. When Mrs. Guevarra, a nurse, delegates aspects of the clients care to the nurse-aide who is an
unlicensed staff, Mrs. Guevarra
A. makes the assignment to teach the staff member
B. is assigning the responsibility to the aide but not the accountability for those tasks
C. does not have to supervise or evaluate the aide
D. most know how to perform task delegated
48. Connie, the new nurse, appears tired and sluggish and lacks the enthusiasm she had six weeks ago
when she started the job. The nurse supervisor should
A. empathize with the nurse and listen to her
B. tell her to take the day off
C. discuss how she is adjusting to her new job
D. ask about her family life
49. Process of formal negotiations of working conditions between a group of registered nurses and
employer is
A. grievance
B. arbitration
C. collective bargaining
D. strike
50. You are attending a certification on cardiopulmonary resuscitation (CPR) offered and required by the
hospital employing you. This is

A. professional course towards credits


B. inservice education
C. advance training
D. continuing education
Situation 9: As a nurse, you are aware that proper documentation in the patient chart is your responsibility.
51. Which of the following is NOT a legally binding document but nonetheless very important in the care of
all patients in any setting?
A. Bill of rights as provided in the Philippine Constitution
B. Scope of nursing practice as defined in R.A. 9173
C. Board of Nursing resolution adopting the Code of Ethics
D. Patients Bill of Rights
52. A nurse gives a wrong medication to the client. Another nurse employed by the same hospital as a risk
manager will expect to receive which of the following communication?
A. Incident Report
B. Oral report
C. Nursing kardex
D. Complain report
53. Performing a procedure on a client in the absence of an informed consent can lead to which of the
following charges?
A. Fraud
B. Assault and Battery
C. Harassment
D. Breach of confidentiality
54. Which of the following is the essence of informed consent?
A. It should have a durable power of attorney
B. It should have coverage from an insurance company
C. It should respect the clients freedom from coercion
D. It should discloses previous diagnosis, prognosis and alternative treatments available for the client.

55. Delegation is the process of assigning tasks that can be performed by a subordinate. The RN should
always be accountable and should not lose his accountability. Which of the following is a role included in
delegation?
A. The RN must supervise all delegated tasks
B. After a task has been delegated. It is no longer a responsibility of the RN.
C. The RN is responsible and accountable for the delegated task in a adjunct with the delegate.
D. Follow up with a delegated task necessary only if the assistive personnel is not trustworthy.
Situation 10 When creating your lesson plan for cerebrovascular disease or STROKE. It is important to include the
risk factors of stroke.
56. The most important risk factor is:
A. Cigarette smoking
B. Hypertension
C. binge drinking
D. heredity
57. Part of your lesson plan is to talk about etiology or cause of stroke. The types of stroke based on cause
are the following EXCEPT:
A. Embolic stroke
B. Hemorrhagic stroke
C. diabetic stroke
D. thrombotic stroke
58. Hemmorhagic stroke occurs suddenly usually when the person is active. All are causes of hemorrhage,
EXCEPT:
A. phlebitis
B. trauma
C. damage to blood vessel
D. aneurysm
59. The nurse emphasizes that intravenous drug abuse carries a high risk of stroke. Which drug is closely
linked to this?
A. Amphetamines
B. Cocaine

C. shabu
D. Demerol
60. A participant in the STROKE class asks what is a risk factor of stroke. Your best response is:
A. More red blood cells thicken blood and make clots more possible.
B. Increased RBC count is linked to high cholesterol.
C. More red blood cell increases hemoglobin content.
D. High RBC count increases blood pressure.
Situation 11: Recognition of normal values is vital in assessment of clients with various disorders.
61. A nurse is reviewing the laboratory test results for a client with a diagnosis of severe dehydration. The
nurse would expect the hematocrit level for this client to be which of the following?
A. 60%
B. 47%
C. 45%
D. 32%
62. A nurse is reviewing the electrolyte results of an assigned client and notes that the potassium level is
5.6 mEq/L. Which of the following would the nurse expect to note on the ECG as a result of this laboratory
value?
A. ST depression
B. Inverted t wave
C. Prominent U wave
D. Tall peaked T waves
63. A nurse is reviewing the electrolyte results of an assigned client and notes that the potassium level is
3.2 mEq/L. Which of the following would the nurse expect to note on the ECG as a result of this laboratory
value?
A. U waves
B. Absent P waves
C. Elevated T waves
D. Elevated ST segment
64. Dorothy underwent diagnostic test and the result of the blood examination are back. On reviewing the
result the nurse notices which of the following as abnormal finding?

A. Neutrophils 60%
B. White blood cells (WBC) 9000/mm
C. Erythrocyte sedimentation rate (ESR) is 39 mm/hr
D. Iron 75 mg/100 ml
65. Which of the following laboratory test result indicate presence of an infectious process?
A. Erythrocyte sedimentation rate (ESR) 12 mm/hr
B. White blood cells (WBC) 18,000/mm3
C. Iron 90 g/100ml
D. Neutrophils 67%
Situation 12: Pleural effusion is the accumulation of fluid in the pleural space.
66. Which of the following is a finding that the nurse will be able to assess in a client with pleural effusion?
A. Reduced or absent breath sound at the base of the lungs, dyspnea, tachypnea and shortness of breath.
B. Hypoxemia
C. Noisy respiration, crackles, stridor and wheezing
D. Tracheal deviation towards the affected side, increased fremitus and loud breath sounds
67. Thoracentesis is performed to the client with effusion. The nurse knows that he removal of fluid should
be slow. Rapid removal of fluid in thoracentesis might cause:
A. Pneumothorax
B. Pleurisy or Pleuritis
C. Cardiovascular collapse
D. Hypertension
68. Three days after thoracentesis, the client again exhibited respiratory distress. The nurse will know that
the pleural effusion has reoccurred when she noticed a sharp stabbing pain during inspiration. The
physician ordered a closed tube thoracotomy for the client. The nurse knows that the primary function of
the chest tube is to:
A. Restore positive intrathoracic pressure
B. Restore negative intrathoracic pressure
C. To visualize the intrathoracic content
D. As a method of air administration
69. The chest tube is functioning properly if:

A. There is an oscillation
B. There is no bubbling in the drainage bottle
C. There is a continuous bubbling in the water seal.
D. The suction control bottle has a continuous bubbling
70. In a client with pleural effusion, the nurse is instructing a appropriate breathing technique. Which of the
following is included in the teaching?
A. Breath normally
B. Hold the breath after each inspiration for 1 full minute
C. Practice abdominal breathing
D. Inhale slowly and hold the breath for 3-5 seconds after each inhalation.
Situation 13: Health care delivery system affects the health status of every Filipino. As a Nurse, Knowledge of this
system is expected to ensure quality of life.
71. When should rehabilitation commence?
A. The day before discharge
B. When the patient desires
C. Upon admission
D. 24hours after discharge
72. What exemplified the preventive and promotive programs in the hospital?
A. Hospitals as a center to prevent and control infection
B. Program for smokers
C. Program for alcoholics and drug addicts
D. Wellness Center
73. Which makes nursing dynamic?
A. Every patient is a unique physical, emotional, social and spiritual being
B. The patient participate in the over all nursing care plan
C. Nursing practice is expanding in the light of modern development that takes place
D. The health status of the patient is constantly changing and the nurse must be cognizant and responsive to these
changes.
74. Prevention is an important responsibility of the nurse in:

A. Hospitals
B. Community
C. Workplace
D. All of the above
75. This form of Health Insurance provides comprehensive prepaid health services to enrollees for a
periodic payment.
A. Health Maintenance Organization
B. Medicare
C. Philippine Health Insurance Act
D. Hospital Maintenance Organization
76. Health care reports have different purposes. The availability of patients record to all health ream
members demonstrates which of the following purposes:
A. Legal documentation
B. Education
C. Research
D. Vehicle for communication
77. When a nurse commits medication error she should accurately document clients response and her
corresponding action. This is very important for which of the following purposes:
A. Research
B. Nursing Audit
C. Legal documentation
D. Vehicle for communication
78. POMR has been widely used in many teaching hospitals. One of its unique features is SOAPIE charting.
The P in SOAPIE charting should include:
A. Prescription of the doctor to the patients illness
B. Plan of care for patient
C. Patients perception of ones illness
D. Nursing Problem and Nursing Diagnosis
79. The medical records that are organized into separate section from doctors or nurses has more
disadvantages than advantages. This is classified as what type of recording?

A. POMR
B. SOAPIE
C. Modified POMR
D. SOMR
80. Which of the following is the advantage of SOMR or Traditional Recording?
A. Increase efficiency of Data gathering
B. Reinforces the use of the nursing process
C. The caregiver can easily locate proper section for making charting entries
D. Enhances effective communication among health care team members
Situation 17: June is 24 year old client with symptoms of dyspnea, absent breath sounds on the right lung and chest
X-ray revealed pleural effusion. The physician will perform thoracentesis
81. Thoracentesis is useful in treating which of the following pulmonary disorders except:
A. Hemothorax
B. Tuberculosis
C. Hydrothorax
D. Empyema
82. Which of the following psychological preparation is not relevant for him?
A. Telling him that the gauge of the needle and anesthesia to be used
B. Telling him to keep still during the procedure to facilitate the insertion of the needle in the correct place.
C. Allow June to express his feeling and concerns
D. Physicians explanation on the purpose of the procedure and how it will be done.
83. Before thoracentesis, the legal consideration you must check is:
A. Consent is signed by the client
B. Medicine preparation is correct
C. Position of the client is correct
D. Consent is signed by relative and physician
84. As a nurse, you know that the position for June before thoracentesis is:
A. Orthopneic
B. Knee-chest

C. Low fowlers
D. Sidelying position on the affected side
85. Which of the following anesthetic drug is used for thoracentesis?
A. Procaine 2 %
B. Valium 250 mg
C. Demerol 75 mg
D. Phenobarbital

Answers
Here are the answers for the exam. Unfortunately, rationales are not given. If you need clarifications or disputes,
please direct them to the comments section and well be glad to give you an explanation.
1. B. ANA
2. D. Nurse anaesthesiologist
3. C. Open and close colleges of nursing
4. A. Novice
5. C. A holistic understanding and perception of the client
6. C. Ventrogluteal
7. D. Vastus lateralis
8. A. Pierce the skin quickly and smoothly at 90 degree angle
9. C. 10 seconds
10. A. It decreases the leakage of discolouring and irritating medication into the subcutaneous tissue.
11. C. Pump the cuff to around 50 mmHg above the point where the pulse is obliterated
12. D. Detect oxygen saturation of arterial blood before symptoms of hypoxemia develops
13. C. Higher than what the reading should be
14. B. 30 minutes
15. C. Cover the fingertip sensor with a towel or bed sheet
16. B. 1 to 2 minutes
17. B. False low reading
18. C. 30 minutes
19. A. 130/80
20. C. 1 1.5 minutes
21. D. Normal saline solution
22. A. Put the client on a side lying position with head of bed lowered
23. B. reduces need to use commercial mouthwash which irritate the buccal mucosa
24. D. suctioning as needed while cleaning the buccal cavity
25. C. petroleum jelly

26. B. Breathing in and out through the nose with his mouth open
27. B. Shortness of breath and laryngeal stridor
28. B. Breathe deeply
29. D. Observe the patients vital signs
30. B. Observe for symptoms of tightness of chest or bleeding
31. A. Respiratory acidosis
32. B. Respiratory alkalosis
33. B. Allens test
34. A. Respiratory acidosis
35. B. Radial artery
36. (skip)
37. A. remove urine from drainage tube with sterile needle and syringe and empty urine from the syringe into
the specimen container
38. A. to the patients inner thigh
39. C. white chicken sandwich, vegetable salad and tea
40. C. see to it that the drainage tubing does not touch the level of the urine
41. A. Hypothalamus
42. D. Gonadotropin hormone releasing hormone
43. D. Growth hormone
44. A. Thyroid gland
45. C. Parathyroid gland
46. B. efficient management of human resources
47. B. is assigning the responsibility to the aide but not the accountability for those tasks
48. C. discuss how she is adjusting to her new job
49. C. collective bargaining
50. B. in-service education
51. D. Patients Bill of Rights
52. A. Incident Report
53. B. Assault and Battery
54. D. It should discloses previous diagnosis, prognosis and alternative treatments available for the client.
55. C. The RN is responsible and accountable for the delegated task in a adjunct with the delegate.
56. B. Hypertension
57. C. diabetic stroke
58. A. phlebitis
59. B. Cocaine
60. A. More red blood cells thicken blood and make clots more possible.
61. A. 60%
62. D. Tall peaked T waves
63. A. U waves
64. C. Erythrocyte sedimentation rate (ESR) is 39 mm/hr

65. B. White blood cells (WBC) 18,000/mm3


66. A. Reduced or absent breath sound at the base of the lungs, dyspnea, tachypnea and shortness of
breath.
67. C. Cardiovascular collapse
68. B. Restore negative intrathoracic pressure
69. A. There is an oscillation
70. D. Inhale slowly and hold the breath for 3-5 seconds after each inhalation.
71. C. Upon admission
72. D. Wellness Center
73. D. The health status of the patient is constantly changing and the nurse must be cognizant and
responsive to these changes.
74. D. All of the above
75. A. Health Maintenance Organization
76. D. Vehicle for communication
77. C. Legal documentation
78. B. Plan of care for patient
79. D. SOMR
80. C. The caregiver can easily locate proper section for making charting entries
81. B. Tuberculosis
82. A. Telling him that the gauge of the needle and anesthesia to be used
83. A. Consent is signed by the client
84. A. Orthopneic
85. A. Procaine 2 %

CHN
1. Which is the primary goal of community health nursing?
A. To support and supplement the efforts of the medical profession in the promotion of health and prevention of
B. To enhance the capacity of individuals, families and communities to cope with their health needs
C. To increase the productivity of the people by providing them with services that will increase their level of health
D. To contribute to national development through promotion of family welfare, focusing particularly on mothers and
children
2. CHN is a community-based practice. Which best explains this statement?
A. The service is provided in the natural environment of people
B. The nurse has to conduct community diagnosis to determine nursing needs and problems
C. The service are based on the available resources within the community
D. Priority setting is based on the magnitude of the health problems identified

3. Population- focused nursing practice requires which of the following processes?


A. Community organizing .
B. Nursing, process
C. Community diagnosis
D. Epidemiologic process
4. RA 1054 is also known as the Occupational Health Act. Aside from the number of employees, what other
factor must be considered in determining the occupational health privileges to which the workers will be
entitled?
A. Type of occupation,: agriculture, commercial, industrial
B. Location of the workplace in relation to health facilities
C. Classification of the business enterprise based on net profit
D. Sex and age composition of employees
5. A business firm must employ an occupational health nurse when it has at least how many employees.
A. 21
B. 101
C. 201
D. 301
6. When the occupational health nurse employs ergonomic principles, she is performing which of her
roles?
A. Health care provider
B. Health educator
C. Health care coordinator
D. Environment manager
7. A garment factory does not have an occupational nurse. Who shall provide the occupational health needs
of the factory workers?
A. Occupational health nurse at the Provincial Health Office
B. Physician employed by the factory
C. Public Health nurse of the RHU of their municipality
D. Rural Sanitary inspector of the RHU in their municipality
8. Public health services are given free of charge. Is this statement true or false?

A. The statement is true; it is the responsibility of government to provide haste services


B. The statement is false; people pay indirectly for public health services
C. The statement may be true or false; depending on the Specific service required
D. The statement may be true or false; depending on policies of the government concerned.
9. According to C.E. Winslow, which of the following is the goal of Public Health?
A. For people to attain their birthrights and longevity
B. For promotion of health and prevention and diseases
C. For people to have access to basic health services
D. For people to be organized in their health efforts
10. We say that a Filipino has attained longevity when he is able to reach the average life span of Filipinos.
What other statistic may be used to determine attainment of longevity?
A. Age-specific mortality rate
B. Proportionate mortality rate
C. Swaroops index
D. Case fatality rate
11. Which of the following is the most prominent feature of public health nursing?
A. It involves providing home care to sick people who are not confined in the hospital
B. Services are provided free of charge to people within the catchment area
C. The public health nurse functions as part of a team providing a public health nursing service
D. Public health nursing focuses on preventive, not curative services
12. According to Margaret Shetland, the philosophy of public health nursing is based on which of the
following?
A. Health and longevity as birthrights
B. The mandate of the state to protect the birthrights of its citizens
C. Public health nursing as a specialized field of nursing
D. The worth and dignity of man
13. Which of the following is the mission of the Department of Health?
A. Health for all Filipinos
B. Ensure the accessibility and quality of health

C. Improve the general health status of the population


D. Health in the hands of the Filipino people by the year 2020
14. Region IV Hospital is classified as what level of facility?
A. Primary
B. Secondary
C. Intermediate
D. Tertiary
15. What is true of primary facilities?
A. They are usually government-run
B. Their services are provided on an out-patient basis
C. They are training facilities for health professionals
D. A community hospital is an example of this level of health facilities
16. Which is an example of the school nurses health care provider function?
A. Requesting for BCG from the RHU for school entrance immunization
B. Conducting random classroom inspection during measles epidemic
C. Taking remedial action on an accident hazard in the school playground
D. Observing places in the school where pupils spend their free times
17. When the nurse determines whether resources were maximized in implementing Ligtas Tigdas, she is
evaluating:
A. Effectiveness
B. Efficiency
C. Adequacy
D. Appropriateness
18. You are a new B.S.N. graduate. You want to become a Public Health Nurse. Where will you apply?
A. Department of Health
B. Provincial Health Office
C. Regional Health Office
D. Rural Health Unit

19. RA 7160 mandates devolution of basic services from the national government to local government units.
Which of the following is the major goal of devolution?
A. To strengthen local government units
B. To allow greater autonomy to local government units.
C. To empower the people and promote their self-reliance
D. To make basic services more accessible to the people
20. Who is the Chairman of the Municipal Health Board?
A. Mayor
B. Municipal Health Officer
C. Public Health Nurse
D. Any qualified physician
21. Which level of health facility is the usual point of entry of a client into the health care delivery system?
A. Primary
B. Secondary
C. Intermediate
D. Tertiary
22. The public health nurse is the supervisor of rural health midwives. Which of the following is a
supervisory function of the pubic health nurse?
A. Referring cases or patients to the midwife
B. Providing technical guidance to the midwife
C. Proving nursing care to cases referred by the midwife
D. Formulating and implementing training programs for midwives
23. One of the participants in a hilot training class asked you to whom she should refer a patient in labor
who develops a complication. You will answer, to the;
A. Public health nurse
B. Rural health midwife
C. Municipal health officer
D. Any of these health professionals
24. You are the public health nurse in a municipality with a total population of about 20,000. There are3
health midwives among the RHU personnel. How many more midwife items will the RHU need?

A. 1
B. 2
C. 3
D. 4
25. If the RHU needs additional midwife items, you will submit the request for additional midwife items for
approval to the:
A. Rural Health Unit
B. District Health Office
C. Provincial Health Office
D. Municipal Health Board
26. As an epidemiologist, the nurse is responsible for reporting cases or notifiable diseases. What law
mandates reporting cases of notifiable diseases?
A. Act 3573
B. RA.3753
C. RA 1054
D. RA 1082
27. According to Freeman and Heinrich, community health nursing is a developmental service. Which of the
following best illustrates this statement?
A. The community health nurse continuously develops himself personally and professionally
B. Health education and community organizing are necessary in providing community health services
C. Community health nursing in intended primarily for health promotion and prevention and treatment of disease.
D. The goal of community health nursing is to provide nursing services to people in their own place of .residence
28. Which disease was declared through Presidential Proclamation No. 4 as a target for, eradication in the
Philippines?
A. Pioliomyelitis
B. Measles
C. Rabies
D. Neonatal Tetanus
29. The public health nurse is responsible for presenting the municipal health statistics using graphs and
tables. To compare the frequency of the leading causes of mortality in the municipality, which graph will
you prepare?

A. Line
B. Bar
C. Pie
D. Scatter diagram
30. Which step in community organizing involves training of potential leaders in the community?
A. Integration
B. Community organization
C. Community study
D. Core group formation
31. In which step are plans formulated for solving community problems?
A. Mobilization
B. Community organization
C. Follow-up/extension
D. Core group formation
32. The public health nurse takes an active role in community participation. What is the primary goal of
community organizing?
A. To educate the people regarding community health problems
B. To mobilize the people to resolve community health problems
C. To maximize the communitys resources in dealing with health problems
33. An indicator of success in community organizing is when people are able to:
A. Participate in community activities for the solution of a community problem
B. Implement activities for the solution of the community problem
C. Plan activities for the solution of the community problem
D. Identify the health problem as a common concern
34. Tertiary prevention is needed in which stage of the natural history of disease?
A. Pre-pathogenesis
B. Pathogenesis
C. Predromal
D. Terminal

35. Isolation of a child with measles belongs to what level of prevention?


A. Primary
B. Secondary
C. Intermediate
D. Tertiary
36. On the other hand, Operation Timbang is_____ prevention?
A. Primary
B. Secondary
C. Intermediate
D. Tertiary
37. Which type of family-nurse contact will provide you with the best opportunity to observe family
dynamics?
A. Clinic consultation
B. Group conferences
C. Home visit
D. Written communication
38. The typology of family nursing problems is used in the statement of nursing diagnosis in the care of
families. The youngest child of the delos Reyes family has been diagnosed as mentally retarded. This is
classified as:
A. Health threat
B. Health deficit
C. Foreseeable crisis
D. Stress point
39. The delos Reyes couple have 6-year old child entering school for the first time. The delos Reyes family
has a:
A. Health threat
B. Health deficit
C. Foreseeable crisis
D. Stress point
40. Which of the following is an advantage of a home visit?

A. It allows the nurse to provide nursing care to a greater number of people


B. It provides an opportunity to do first hand appraisal of the home situation
C. It allows sharing of experience among people with similar health problems
D. It develops the familys initiative in providing for health needs of its members
41. Which is CONTRARY to the principles in planning a home visit?
A. A home visit should have a purpose of objective
B. The plan should revolve around the family health .needs
C. A home visit should be conducted in the manner prescribed by RHU
D. Planning of continuing care should involve a responsible-family member
42. The PHN bag is an important tool in providing nursing care during a home visit. The most important
principle in bag technique states that it;
A. Should save time and effort
B. Should minimize if not totally prevent the spread of infection
C. Should not overshadow concern for the patient and his family
D. May be done in variety of ways depending on the home situation, etc.
43. To maintain the cleanliness of the bag and its contents, which of the following must the nurse do?
A. Wash his/her hands before and after providing nursing care to the family members
B. In the care of family members, as much as possible, use only articles taken from the bag
C. Put on an apron to protect her uniform and fold it with the right side out before putting it back into the bag.
D. At the end of the visit, fold the lining on which the bag was placed, ensuring that the contaminated side is on the
outside.
44. The public health conducts a study on the factors contributing to the high morality rate due to heart
diseases in the municipality where she works. Which branch of epidemiology does the nurse practice in
this situation?
A. Descriptive
B. Analytical
C. Therapeutic
D. Evaluation
45. Which of the following is a function of epidemiology?

A. Identifying the disease condition based on manifestations presented by a client


B. Determining factors that contributed to the occurrence of pneumonia in a 3 year old
C. Determining the efficacy of the antibiotic used in the treatment of the 3 year old client with pneumonia
D. Evaluating the effectiveness of the implementation of the Integrated Management of Childhood Illness
46. Which of the following is an epidemiologic function of the nurse during an epidemic?
A Conducting assessment of suspected cases to detect the communicable diseases
B. Monitoring the condition of the cases affected by the communicable disease
C. Participating in the investigation to determine the source of epidemic
D. Teaching the community on preventive measures against the disease
47. The primary purpose of conducting an epidemiologic investigation is to;
A. Delineate the etiology of the epidemic
B. Encourage cooperation and support of the community
C. Identify groups who are at risk of contracting the disease
D. Identify geographical location of cases of the disease in the community
48. Which is a characteristic of person-to-person propagated epidemic?
A. There are more cases of the disease than expected
B. The disease must necessarily be transmitted through a vector
C. The spread of the disease can be attributed to a common vehicle
D. There is gradual build up of cases before we epidemic becomes easily noticeable
49. In the investigation of an epidemic, you compare the present frequency of the disease with the usual
frequency at this time of the year in this community. This is done during which stage of the investigation?
A. Establishing the epidemic
B. Testing the hypothesis
C. Formulation of the hypothesis
D. Appraisal of facts
50. The number of cases of Dengue fever usually increases towards the end of the rainy season. This
pattern of occurrence of Dengue fever is best described as;
A. Epidemic occurrence
B. Cyclical variation

C. Sporadic occurrence
D. Secular occurrence
51. In the year 1980, the World Health Organization declared the Philippines, together with some other
countries in the Western Pacific Region, free of which disease?
A. Pneumonic plaque
B. Poliomyelitis
C. Small pox
D. Anthrax
52. In the census of the Philippines in 1995, there were about 35,299,000 males and about 34,968,000
females. What is the sex ratio?
A. 99.06:100
B. 100.94:100
C. 50.23%
D. 49.76%
53. Primary health care is a total approach to community development. Which of the following is an
indicator of success in the use of the primary health care approach?
A. Health services are provided free of charge to individuals and families
B. Local officials are empowered as the major decision makers in matters of health
C. Health workers are able too provide care based on identified health needs of the people
D. Health programs are sustained according to the level of development of the community
54. Sputum examination is the major screening tool for pulmonary tuberculosis. Clients would sometimes
get false negative results in this exam. This means that the test is not perfect in terms of which
characteristic of a diagnostic examination?
A. Effectiveness
B. Efficacy
C. Specificity
D. Sensitivity
55. Use of appropriate technology requires knowledge of indigenous technology. Which medical herb is
given for fever, headache and cough?

A. Sambong
B. Tsaang gubat
C. Akapulko
D. Lagundi
56. What law created the Philippine institute of Traditional and Alternative Health Care?
A. RA 8483
B. RA4823
C. RA 2483
D. RA 3482
57. In traditional Chinese medicine, the yielding, negative and feminine force is termed:
A. Yin
B. Yang
C. Qi
D. Chai
58. What is the legal basis of Primary Health Care approach in the Philippines?
A. Alma Ata Declaration of PHC
B. Letter of Instruction No 949
C. Presidential Decree No. 147
D. Presidential Decree 996
59. Which of the following demonstrates inter-sectoral linkages?
A. Two-way referral system
B. Team approach
C. Endorsement done by a midwife to another midwife
D. Cooperation between PHN and public school teacher
60. The municipality assigned to you has a population of about 20/000. Estimate the number of 1-4 year old
children who be given Retinol capsule 200.000 every 6 months.
A. 1,500
B. 1,800
C. 2,000
D. 2,300

61. Estimate the number of pregnant women who will be given tetanus toxoid during an immunization
outreach activity in a barangay with a population of about 1,500.
A. 265
B. 300
C. 375
D. 400
62. To describe the sex composition of the population, which demographic tool may be used?
A. Sex ratio
B. Sex proportion
C. Population pyramid
D. Any of these maybe used
63. Which of the following is a natality rate?
A. Crude birth rate
B. Neonatal mortality rate
C. Infant mortality rate
D. General fertility rate
64. You are computing the crude rate of your municipality, with a total population o about 18,000 for last
year. There were 94 deaths. Among those who died, 20 died because of diseases of the heart and 32 were
aged 50 years or older. What is the crude death rate?
A. 4.1/1000
B. 5.2/1000
C. 6.3/1000
D. 7.3/1000
65. Knowing that malnutrition is a frequent community health problem, you decided to conduct nutritional
assessment. What population is particularly susceptible to protein energy malnutrition (PEM)?
A. Pregnant women and the elderly
B. Under 5 year old children
C. 1-4 year old children
D. School age children
66. Which statistic can give the most accurate reflection of the health status of a community?

A. 1-4 year old age-specific mortality rate


B. Infant mortality rate
C. Swaroops index
D. Crude death rate
67. In the past year, Barangay A had an average population of 1655. 46 babies were born in that year, 2 of
whom died less than 4 weeks after they were born. They were 4 recorded stillbirths. What is the neonatal
mortality rate?
A. 27.8/1000
B. 43.5/1000
C. 86.9/1000
D. 130.4/1000
68. Which statistic best reflects the nutritional status of a population?
A. 1-4 year old age-specific mortality rate
B. Proportionate mortality rate
C. Infant mortality rate
D. Swaroops index
69. What numerator is used in computing general fertility rate?
A. Estimated midyear population
B. Number of registered live births
C. Number of pregnancies in the year
D. Number of females of reproductive age
70. You will gather data for nutritional assessment of a purok. You will gather information only from families
with members who belong to the target population for PEM. What method of delta gathering is best for this
purpose?
A. Census
B. Survey
C. Record Review
D. Review of civil registry
71. In the conduct of a census, the method of population assignment based on the actual physical location
of the people is termed;

A. De jure
B. De locus
C. De facto
D. De novo
72. The Field Health Services and information System (FHSIS) is the recording and reporting system in
public health) care in the Philippines. The monthly field health service activity report is a form used in
which of the components of the FHSIS?
A. Tally report
B. Output report
C. Target/client list
D. Individual health record
73. To monitor clients registered in long-term regimens, such as the Multi-Drug Therapy, which component
will be most useful?
A. Tally report
B. Output report
C. Target/client list
D. Individual health record
74. Civil registries are important sources of data. Which law requires registration of births within 30 days
from the occurrence of the birth?
A. PD 651
B. Act 3573
C. RA 3753
D. RA 3375
75. Which of the following professionals can sign the birth certificate?
A. Public health nurse
B. Rural health midwife
C. Municipal health officer
D. Any of these health professionals
76. Which criterion in priority setting of health problems is used only in community health care?

A. Modifiability of the problem


B. Nature of the problem presented
C. Magnitude of the health problem
D. Preventive potential of the health problem
77. The Sentrong Sigla Movement has been launched to improve health service delivery. Which of the
following is/are true of this movement?
A. This is a project spearheaded by local government units
B. It is a basis for increasing funding from local government units
C. It encourages health centers to focus on disease prevention and control
D. Its main strategy is certification of health centers able to comply with standards
78. Which of the following women should be considered as special targets for family planning?
A. Those who have two children or more
B. Those with medical conditions such as anemia
C. Those younger than 20 years and older than 35 years
D. Those who just had a delivery within the past 15 months
79. Freedom of choice in one of the policies of the Family Planning Program of the Philippines. Which of the
following illustrates this principle?
A. Information dissemination about the need for family planning
B. Support of research and development in family planning methods
C. Adequate information for couples regarding the different methods
D. Encouragement of couples to take family planning as a joint responsibility
80. A woman, 6 months pregnant, came to the center for consultation. Which of the following substances is
contraindicated?
A. Tetanus toxoid
B. Retinol 200,000 IU
C. Ferrous sulfate 200mg
D. Potassium iodate 200 mg, capsule
81. During prenatal consultation, a client asked you if she can have her delivery at home. After history
taking and physical examination, you advised her against a home delivery. Which of the following findings
disqualifies her for a home delivery?

A. Her OB score is G5P3


B. She has some palmar pallor
C. Her blood pressure is 130/80
D. Her baby is in cephalic presentation
82. Inadequate intake by the pregnant woman of which vitamin may cause neural tube defects?
A. Niacin
B. Riboflavin
C. Folic Acid
D. Thiamine
83. You are in a clients home to attend to a delivery. Which of the following will you do first?
A. Set up a sterile area
B. Put on a clean gown and apron
C. Cleanse the clients vulva with soap and water
D. Note the interval, duration and intensity of labor and contractions
84. In preparing a primigravida for breastfeeding, which of the following will you do?
A. Tell her that lactation begins within a day after delivery
B. Teach her nipple stretching exercises if her nipples are everted
C. Instruct her to wash her nipples before and after each breastfeeding
D. Explain to her that putting the baby to breast will lessen blood loss after delivery
85. A primigravida is instructed to offer her breast to the baby for the first time within 30 minutes after
delivery. What is the purpose of offering the breast this early?
A. To initiate the occurrence of milk letdown
B. To stimulate milk production by the mammary acini
C. To make sure that the baby is able to get the colustrum
D. To allow the woman to practice breastfeeding in the presence of the health worker
86. In a mothers class, you discuss proper breastfeeding technique. Which of these is a sign that the baby
has lactated on the breast property?
A. The baby takes shallow, rapid sucks
B. The mother does not feel nipple pain

C. The babys mouth is only partly open


D. Only the mothers nipple is inside the babys mouth
87. You explain to a breastfeeding mother that breastmilk is sufficient for all of the babys nutrient needs
only up to:
A. 3 months
B. 6 months
C. 1 year
D. 2 years
88. What is given to a woman within a month after the delivery of a baby?
A. Malunggay capsule
B. Ferrous sutfate l00mg O.D.
C. Retinol 200.000 IU 1 capsule
D. Potassium Iodate 200 mg, 1 capsule
89. Which biological used in EPI is stored in the freezer?
A. DPT
B. Tetanus toxoid
C. Measles vaccine
D. Hepatitis B vaccine
90. Unused BCG should be discarded how many hours after reconstitution?
A. 2
B. 4
C. 6
D. At the end of the day
91. In immunity school entrants with BCG, you not obliged to secure parental consent. This is because of
which legal document?
A. PD 996
B. RA 7864
C. Presidential Proclamation No. 6
D. Presidential Proclamation No. 46

92. Which immunization produces a permanent scar?


A. DPT
B. BCG
C. Measles vaccination
D. Hepatitis B vaccination
93. A 4 week old baby was brought to the health center for his first immunization. Which can be given to
him?
A. DPT1
B. OPV1
C. Infant BCG
D. Hepatitis B Vaccin
94. You will not give DPT 2 if the mother says that the infant had?
A. Seizures a day after DPT1
B. Fever for 3 days after DPT1
C. Abscess formation after DPT1
D. Local tenderness for 3 days after DPT1
95. A 2-month old infant was brought to the health center for immunization. During assessment, the infants
temperature registered at 38.1 C. Which is the best course of action that you will take?
A. Go on with the infants immunization
B. Give paracetamol and wait for his fever to subside
C. Refer the infant to the physician for further assessment
D. Advise the infants mother to bring him back for immunization when he is well
96. A pregnant woman had just received her 4th dose of tetanus toxoid. Subsequently, her baby will have
protection against tetanus for how long?
A. 1 year
B. 3 years
C. 10 years
D. Lifetime
97. A 4-month old infant was brought to the health center of cough. Her respiratory rate is 42/minute. Using
the IMCI guidelines of assessment, her breathing is considered;

A. Fast
B. Slow
C. Normal
D. Insignificant
98. Which of the following signs will indicate that a young child is suffering from severe pneumonia?
A. Dyspnea
B. Wheezing
C. Fast breathing
D. Chest indrawing
99. Using IMCI guidelines, you classify a child as having severe pneumonia. What is the best management
for the child?
A. Prescribe antibiotic
B. Refer him urgently to the hospital
C. Instruct the mother to increase fluid intake
D. Instruct the mother to continue breastfeeding
100. A 5-month old infant was brought by his mother to the health center because of diarrhea occurring 4 to
5 times a day. His skin goes back slowly after a skin pinch and his eyes are sunken. Using the IMCI
guidelines, you will classify this infant in which category?
A. No signs of dehydration
B. Some dehydration
C. Severe dehydration
D. The data is insufficient
101. Based on the assessment, you classified a 3-month old infant with the chief complaint of diarrhea in
the category of SOME DEHYDRATION. Based on the IMCI management guidelines, which of the following
will you do?
A. Bring the infant to the nearest facility where IV fluids can be given
B. Supervise the mother in giving 200 to 400 ml of Oresol in 4 hours
C. Give the infants mother instructions on home management
D. Keep the infant in your health center for close observation
102. A mother is using Oresol in the management of diarrhea of her 3-year old child. She asked you what to
do if her child vomits. You will tell her to:

A. Bring the child to the nearest hospital for further assessment


B. Bring the child to the health center for IV therapy
C. Bring the child to the health center for assessment by the physician
D. Let the child rest for 10 minutes then continue giving Oresol more slowly
103. A 1 1/2 year old child was classified as having 3rd degree of protein energy malnutrition, kwashjorkor.
Which of the following signs will be most apparent in this child?
A. Voracious appetite
B. Wasting
C. Apathy
D. Edema
104. Assessment of a 2-year old child revealed baggy pants. Using the IMCI guidelines, how will you
manage this child?
A. Refer the child urgently to a hospital for confinement
B. Coordinate with the social worker to enroll the child in a feeding program
C. Make a teaching plan for the mother, focusing on the menu planning for her child
D. Assess and treat the child for health problems like infections and intestinal parasitism
105. During the physical examination of a young child, what is the earliest sign of xerophthalmia that may
observe?
A. Keratomalacia
B. Corneal opacity
C. Night blindness
D. Conjunctival xerosis
106. To prevent xerophthalmia, young children are given Retinol capsule every 6 months. What is the dose
given to preschoolers?
A. 10, 000 IU
B. 20, 000 IU
C. 100, 000 IU
D. 200, 000 IU
107. The major sign of iron deficiency anemia is pallor. What part is best examined for pallor?
A. Palms
B. Nailbeds

C. Around the lips


D. Lower conjunctival sac
108. Food fortification is one of the strategies to prevent micronutrient deficiency conditions. RA 8976
mandates fortification of certain food items. Which of the following is among these food items?
A. Sugar
B. Bread
C. Margarine
D. Filled milk
109. What is the best course of action when there is a measles epidemic in a nearby municipality?
A. Give measles vaccine to babies aged 6 to 3 months
B. Give babies aged 6 to 11 months one dose of 100,000 IU of Retinol
C. Instruct mother to keep their babies at home to prevent disease transmission
D. Instruct mothers to feed their babies adequately to enhance their babies resistance
110. A mother brought her daughter, 4 years old, to the RHU because of cough and colds. Following the
IMCI assessment guide, which of the following is a danger sign that indicates the need for urgent referral to
a hospital?
A. Inability to drink
B. High grade fever
C. Signs of severe dehydration
D. Cough for more than 30 days
111. Management of a child with measles includes the administration of which of the following?
A. Gentian violet on mouth lesions
B. Antibiotic to prevent pneumonia
C. Tetracycline eye ointment for corneal opacity
D. Retinol capsule regardless of when the last dose was given
112. A mother brought her 10 month old infant for consultation because of fever which started 4 days prior
to consultation. To determine malaria risk, what will you do?
A. Do a tourniquet test
B. Ask where the family resides

C. Get a specimen for blood smear


D. Ask if the fever is present everyday
113. The following are strategies implemented by the DOH to prevent mosquito-borne diseases. Which of
these is most effective in the control of Dengue fever?
A. Stream seeding with larva-eating fish
B. Destroying breeding places of mosquitoes
C. Chemoprophylaxis of non-immune persons going to endemic areas
D. Teaching people in endemic areas to use chemically treated mosquito nets
114. Secondary prevention for malaria includes?
A. Planting of neem or eucalyptus trees
B. Residual spraying of insecticides at night
C. Determining whether a place is endemic or not
D. Growing larva-eating fish in mosquito breeding places
115. Scotch tape swab is done to check for which intestinal parasite?
A. Ascaris
B. Pinworm
C. Hookworm
D. Schistosoma
116. Which of the following signs indicates the need for sputum examination for AFB?
A. Hematemesis
B. Fever for 1 week
C. Cough for 3 weeks
D. Chest pain for 1 week
117. Which clients are considered targets for DOTS category?
A. Sputum negative cavitary cases
B. Clients returning after default
C. Relapses and failures of previous PTB treatment regimens
D. Clients diagnosed for the first time through a positive sputum exam
118. To improve compliance to treatment, what innovation is being implemented in DOTS?

A. Having the health worker follow up the client at home


B. Having the health worker or a responsible family member monitor drug intake
C. Having the patient come to the health center every month to get his medications
D. Having a target list to check on whether the patient has collected his monthly supply of drugs
119. Diagnosis of leprosy is highly dependent on recognition of symptoms. Which of the following is an
early sign of leprosy?
A. Macular lesions
B. Inability to close eyelids
C. Thickened painful nerves
D. Sinking of the nose bridge
120. Which of the following clients should be classified as a case of mutibacillary leprosy?
A. 3 skin lesions, negative slit skin smear
B. 3 skin lesions, positive slit skin smear
C. 5 skin lesions, negative slit skin smear
D. 5 skin lesions, positive slit skin smear
121. In the Philippines, which condition is the most frequent cause of death associated by schistosomiasis?
A. Liver cancer
B. Liver cirrhosis
C. Bladder cancer
D. Intestinal perforation
122. What is the most effective way of controlling schistosomiasis in an endemic area?
A. Use of molluscicides
B. Building of foot bridges
C. Proper use of sanitary toilets
D. Use of protective footwear, such as rubber boots
123. When residents obtain water from an artesian well in the neighborhood, the level of this approved type
of water facility is:
A. I
B. II

C. III
D. IV
124. For prevention of Hepatitis A, you decided to conduct health education activities. Which of the
following is Irrelevant?
A. Use of sterile syringes and needles
B. Safe food preparation and food handling by vendors
C. Proper disposal of human excreta and personal hygiene
D. Immediate reporting of water pipe leaks and illegal water connections
125. Which biological used in EPI should not be stored in the freezer?
A. DPT
B. OPV
C. Measles vaccine
D. MMR
126. You will conduct outreach immunization in a barangay with a population of about 1500. Estimate the
number of infants in the barangay.
A. 45
B. 50
C. 55
D. 60
127. In IMCI, severe conditions generally require urgent referral to a hospital. Which of the following severe
conditions Does not always require urgent referral to hospital?
A. Mastoiditis
B. Severe dehydration
C. Severe pneumonia
D. Severe febrile disease.
128. A client was diagnosed as having Dengue Fever. You will say that there is slow capillary refill when the
color of the nailbed that you pressed does not return within how many seconds?
A. 3
B. 5

C. 8
D. 10
129. A 3-year old child was brought by his mother to the health center because of fever of 4-day duration.
The child had a positive tourniquet test result. In the absence of other signs, which of the most appropriate
measure that the PHN may carry out to prevent Dengue shock syndrome?
A. Insert an NGT and give fluids per NGT
B. Instruct the mother to give the child Oresol
C. Start the patient on IV Stat
D. Refer the client to the physician for appropriate management
130. The pathognomonic sign of measles is Kopliks spot. You may see Kopliks spot by inspecting the:
A. Nasal Mucosa
B. Buccal mucosa
C. Skin on the abdomen
D. Skin on the antecubital surface
131. Among the following diseases, which is airborne?
A. Viral conjunctivitis
B. Acute poliomyelitis
C. Diphtheria
D. Measles
132. Among children aged 2 months to 3 years, the most prevalent form of meningitis is caused by which
microorganism?
A. Hemophilus Influenzae
B. Morbillivirus
C. Streptococcus Pneumoniae
D. Neisseria meningitides
133. Human beings are the major reservoir of malaria. Which of the following strategies in malaria control is
based on this fact?
A. Stream seeding
B. Stream clearing

C. Destruction of breeding places


D. Zooprophylaxis
134. The use of larvivorous fish in malaria control is the basis for which strategy of malaria control?
A. Stream seeding
B. Stream clearing
C. Destruction of breeding places
D. Zooprophylaxis .
135. Mosquito-borne diseases are prevented mostly with the use of mosquito control measures. Which of
the following is NOT appropriate for malaria control?
A. Use of chemically treated mosquito nets
B. Seeding of breeding places with larva-eating fish
C. Destruction of breeding places of the mosquito vector
D. Use of mosquito-repelling soaps, such as those with basil or citronella
136. A 4-year old client was brought to the health center with chief complaint of severe diarrhea and the
passage of rice water. The client is most probably suffering from which condition?
A. Giardiasis
B. Cholera
C. Amebiasis
D. Dysentery
137. In the Philippines, which specie of schistosoma is endemic in certain regions?
A. S. mansoni
B. S. japonicum
C. S. malayensis
D. S. haematobium
138. A 32 year old client came for consultation at the health center with the chief complaint of fever for a
week. Accompanying symptoms were muscle pains and body malaise. A week after the start of fever, the
client noted yellowish discoloration of his sclera. History showed that he waded in flood waters about 2
weeks before the onset of symptoms. Based on this history/ which disease condition will you suspect?
A. Hepatitis A
B. Hepatitis B

C. Tetanus
D. Leptospirosis
139. MWSS provides water to Manila and other cities in Metro Manila. This is an example of which level of
water facility?
A. I
B. II
C. III
D. IV
140. You are the PHN in the city health center. A client underwent screening for AIDS using ELISA. His
result was positive. What is the best course of action that you may take?
A. Get a thorough history of the client, focusing on the practice of high risk behavior
B. Ask the client to be accompanied by a significant person before revealing the result.
C. Refer the client to the physician since he is the best person to reveal the result to the client
D. Refer the client for a supplementary test, such as Western blot, since the ELISA result maybe false
141. Which is the BEST control measure for AIDS?
A. Being faithful to a single sexual partner
B. Using a condom during each sexual contact
C. Avoiding sexual contact with commercial sex workers
D. Making sure that ones sexual partner does not have signs of AIDS
142. The most frequent causes of death among clients with AIDS are opportunistic diseases. Which of the
following opportunistic infections is characterized by tonsilllopharyngitis?
A. Respiratory candidiasis
B. Infectious mononucleosis
C. Cytomegalovirus disease
D. Pneumocystis carinii pneumonia
143. To determine the possible sources of sexually transmitted infections, which is the BEST method that
may be undertaken by the public health nurse?
A. Contact tracing
B. Community survey

C. Mass screening tests


D. Interview suspects
144. Antiretroviral agents, such as AZT are used in the management of AIDS. Which of the following is not
an action expected of these drugs?
A. They prolong the life of the client with AIDS
B. They reduce the risk of opportunistic infections
C. They shorten the period of communicability of the disease
D. They are able to bring about a cure of the disease condition
145. A barangay had an outbreak of German measles. To prevent congenital rubella, what is the BEST
advice that you can give to women in the first trimester of pregnancy in the barangay?
a. Advice them on the sign of German Measles
b. Avoid crowded places, such as markets and moviehouses
c. Consult at the health center where rubella vaccine may be given
d. Consult a physician who may give them rubella immunoglobulin

Answers and Rationale


Gauge your performance by counter-checking your answers to those below. If you have disputes or further
questions, please direct them to the comments section.
1. Answer: (B) To enhance the capacity of individuals, families and communities to cope with their health needs.
To contribute to national development through promotion of family welfare, focusing particularly on mothers and
children.
2. Answer: (B) The nurse has to conduct community diagnosis to determine nursing needs and problems.
Community-based practice means providing care to people in their own natural environments: the home, school and
workplace, for example.
3. Answer: (C) Community diagnosis
Population-focused nursing care means providing care based on the greater need of the majority of the population.
The greater need is identified through community diagnosis.
4. Answer: (B) Location of the workplace in relation to health facilities
Based on R.A. 1054, an occupational nurse must be employed when there are 30 to 100 employees and the

workplace is more than 1 km. away from the nearest health center.
5. Answer: (B) 101
6. Answer: (D) Environmental manager
Ergonomics is improving efficiency of workers by improving the workers environment through appropriately
designed furniture, for example.
7. Answer: (C) Public health nurse of the RHU of their municipality
8. Answer: (B) The statement is false; people pay indirectly for public health services.
Community health services, including public health services, are prepaid paid services, through taxation, for
example.
9. Answer: (A) For people to attain their birthrights of health and longevity
According to Winslow, all public health efforts are for people to realize their birthrights of health and longevity.
10. Answer: (C) Swaroops index
Swaroops index is the percentage of the deaths aged 50 years or older. Its inverse represents the percentage of
untimely deaths (those who died younger than 50 years).
11. Answer: (D) Public health nursing focuses on preventive, not curative, services.
The catchment area in PHN consists of a residential community, many of whom are well individuals who have
greater need for preventive rather than curative services.
12. Answer: (D) The worth and dignity of man
This is a direct quote from Dr. Margaret Shetlands statements on Public Health Nursing.
13. Answer: (B) Ensure the accessibility and quality of health care
14. Answer: (D) Tertiary
Regional hospitals are tertiary facilities because they serve as training hospitals for the region.
15. Answer: (B) Their services are provided on an out-patient basis.
Primary facilities government and non-government facilities that provide basic out-patient services.
16. Answer: (B) Conducting random classroom inspection during a measles epidemic
Random classroom inspection is assessment of pupils/students and teachers for signs of a health problem prevalent
in the community.

17. Answer: (B) Efficiency


Efficiency is determining whether the goals were attained at the least possible cost.
18. Answer: (D) Rural Health Unit
R.A. 7160 devolved basic health services to local government units (LGUs). The public health nurse is an employee
of the LGU.
19. Answer: (C) To empower the people and promote their self-reliance
People empowerment is the basic motivation behind devolution of basic services to LGUs.
20. Answer: (A) Mayor
The local executive serves as the chairman of the Municipal Health Board.
21. Answer: (A) Primary
The entry of a person into the health care delivery system is usually through a consultation in out-patient services.
22. Answer: (B) Providing technical guidance to the midwife
The nurse provides technical guidance to the midwife in the care of clients, particularly in the implementation of
management guidelines, as in Integrated Management of Childhood Illness.
23. Answer: (C) Municipal Health Officer
A public health nurse and rural health midwife can provide care during normal childbirth. A physician should attend
to a woman with a complication during labor.
24. Answer: (A) 1
Each rural health midwife is given a population assignment of about 5,000.
25. Answer: (D) Municipal Health Board
As mandated by R.A. 7160, basic health services have been devolved from the national government to local
government units.
26. Answer: (A) Act 3573
Act 3573, the Law on Reporting of Communicable Diseases, enacted in 1929, mandated the reporting of diseases
listed in the law to the nearest health station.
27. Answer: (B) Health education and community organizing are necessary in providing community health services.
The community health nurse develops the health capability of people through health education and community
organizing activities.

28. Answer: (B) Measles


Presidential Proclamation No. 4 is on the Ligtas Tigdas Program.
29. Answer: (B) Bar
A bar graph is used to present comparison of values, a line graph for trends over time or age, a pie graph for
population composition or distribution, and a scatter diagram for correlation of two variables.
30. Answer: (D) Core group formation
In core group formation, the nurse is able to transfer the technology of community organizing to the potential or
informal community leaders through a training program.
31. Answer: (B) Community organization
Community organization is the step when community assemblies take place. During the community assembly, the
people may opt to formalize the community organization and make plans for community action to resolve a
community health problem.
32. Answer: (D) To maximize the communitys resources in dealing with health problems
Community organizing is a developmental service, with the goal of developing the peoples self-reliance in dealing
with community health problems. A, B and C are objectives of contributory objectives to this goal.
33. Answer: (A) Participate in community activities for the solution of a community problem
Participation in community activities in resolving a community problem may be in any of the processes mentioned in
the other choices.
34. Answer: (D) Terminal
Tertiary prevention involves rehabilitation, prevention of permanent disability and disability limitation appropriate for
convalescents, the disabled, complicated cases and the terminally ill (those in the terminal stage of a disease)
35. Answer: (A) Primary
The purpose of isolating a client with a communicable disease is to protect those who are not sick (specific disease
prevention).
36. Answer: (B) Secondary
Operation Timbang is done to identify members of the susceptible population who are malnourished. Its purpose is
early diagnosis and, subsequently, prompt treatment.
37. Answer: (C) Home visit
Dynamics of family relationships can best be observed in the familys natural environment, which is the home.

38. Answer: (B) Health deficit


Failure of a family member to develop according to what is expected, as in mental retardation, is a health deficit.
39. Answer: (C) Foreseeable crisis
Entry of the 6-year old into school is an anticipated period of unusual demand on the family.
40. Answer: (B) It provides an opportunity to do first hand appraisal of the home situation.
Choice A is not correct since a home visit requires that the nurse spend so much time with the family. Choice C is an
advantage of a group conference, while choice D is true of a clinic consultation.
41. Answer: (C) A home visit should be conducted in the manner prescribed by the RHU.
The home visit plan should be flexible and practical, depending on factors, such as the familys needs and the
resources available to the nurse and the family.
42. Answer: (B) Should minimize if not totally prevent the spread of infection.
Bag technique is performed before and after handling a client in the home to prevent transmission of infection to and
from the client.
43. Answer: (A) Wash his/her hands before and after providing nursing care to the family members.
Choice B goes against the idea of utilizing the familys resources, which is encouraged in CHN. Choices C and D
goes against the principle of asepsis of confining the contaminated surface of objects.
44. Answer: (B) Analytical
Analytical epidemiology is the study of factors or determinants affecting the patterns of occurrence and distribution of
disease in a community.
45. Answer: (D) Evaluating the effectiveness of the implementation of the Integrated Management of Childhood
Illness
Epidemiology is used in the assessment of a community or evaluation of interventions in community health practice.
46. Answer: (C) Participating in the investigation to determine the source of the epidemic
Epidemiology is the study of patterns of occurrence and distribution of disease in the community, as well as the
factors that affect disease patterns. The purpose of an epidemiologic investigation is to identify the source of an
epidemic, i.e., what brought about the epidemic.
47. Answer: (A) Delineate the etiology of the epidemic
Delineating the etiology of an epidemic is identifying its source.
48. Answer: (D) There is a gradual build up of cases before the epidemic becomes easily noticeable.
A gradual or insidious onset of the epidemic is usually observable in person-to-person propagated epidemics.

49. Answer: (A) Establishing the epidemic


Establishing the epidemic is determining whether there is an epidemic or not. This is done by comparing the present
number of cases with the usual number of cases of the disease at the same time of the year, as well as establishing
the relatedness of the cases of the disease.
50. Answer: (B) Cyclical variation
A cyclical variation is a periodic fluctuation in the number of cases of a disease in the community.
51. Answer: (C) Smallpox
The last documented case of Smallpox was in 1977 at Somalia.
52. Answer: (B) 100.94:100
Sex ratio is the number of males for every 100 females in the population.
53. Answer: (D) Health programs are sustained according to the level of development of the community.
Primary health care is essential health care that can be sustained in all stages of development of the community.
54. Answer: (D) Sensitivity
Sensitivity is the capacity of a diagnostic examination to detect cases of the disease. If a test is 100% sensitive, all
the cases tested will have a positive result, i.e., there will be no false negative results.
55. Answer: (D) Lagundi
Sambong is used as a diuretic. Tsaang gubat is used to relieve diarrhea. Akapulko is used for its antifungal property.
56. Answer: (A) R.A. 8423 or AN ACT CREATING THE PHILIPPINE INSTITUTE OF TRADITIONAL AND
ALTERNATIVE HEALTH CARE (PITAHC) TO ACCELERATE THE DEVELOPMENT OF TRADITIONAL AND
ALTERNATIVE HEALTH CARE IN THE PHILIPPINES, PROVIDING FOR A TRADITIONAL AND ALTERNATIVE
HEALTH CARE DEVELOPMENT FUND AND FOR OTHER PURPOSES signed to a law on December 9, 1997.
57. Answer: (A) Yin
Yang is the male dominating, positive and masculine force.
58. Answer: (B) Letter of Instruction No. 949
Letter of Instruction 949 was issued by then President Ferdinand Marcos, directing the formerly called Ministry of
Health, now the Department of Health, to utilize Primary Health Care approach in planning and implementing health
programs.
59. Answer: (D) Cooperation between the PHN and public school teacher
Intersectoral linkages refer to working relationships between the health sector and other sectors involved in
community development.

60. Answer: (D) 2,300


Based on the Philippine population composition, to estimate the number of 1-4 year old children, multiply total
population by 11.5%.
61. Answer: (A) 265
To estimate the number of pregnant women, multiply the total population by 3.5%.
62. Answer: (D) Any of these may be used.
Sex ratio and sex proportion are used to determine the sex composition of a population. A population pyramid is
used to present the composition of a population by age and sex.
63. Answer: (A) Crude birth rate
Natality means birth. A natality rate is a birth rate.
64. Answer: (B) 5.2/1,000
To compute crude death rate divide total number of deaths (94) by total population (18,000) and multiply by 1,000.
65. Answer: (C) 1-4 year old children
Preschoolers are the most susceptible to PEM because they have generally been weaned. Also, this is the
population who, unable to feed themselves, are often the victims of poor intrafamilial food distribution.
66. Answer: (C) Swaroops index
Swaroops index is the proportion of deaths aged 50 years and above. The higher the Swaroops index of a
population, the greater the proportion of the deaths who were able to reach the age of at least 50 years, i.e., more
people grew old before they died.
67. Answer: (B) 43.5/1,000
To compute for neonatal mortality rate, divide the number of babies who died before reaching the age of 28 days by
the total number of live births, then multiply by 1,000.
68. Answer: (A) 1-4 year old age-specific mortality rate
Since preschoolers are the most susceptible to the effects of malnutrition, a population with poor nutritional status
will most likely have a high 1-4 year old age-specific mortality rate, also known as child mortality rate.
69. Answer: (B) Number of registered live births
To compute for general or total fertility rate, divide the number of registered live births by the number of females of
reproductive age (15-45 years), then multiply by 1,000.
70. Answer: (B) Survey
A survey, also called sample survey, is data gathering about a sample of the population.

71. Answer: (C) De facto


The other method of population assignment, de jure, is based on the usual place of residence of the people.
72. Answer: (A) Tally report
A tally report is prepared monthly or quarterly by the RHU personnel and transmitted to the Provincial Health Office.
73. Answer: (C) Target/client list
The MDT Client List is a record of clients enrolled in MDT and other relevant data, such as dates when clients
collected their monthly supply of drugs.
74. Answer: (A) P.D. 651
P.D. 651 amended R.A. 3753, requiring the registry of births within 30 days from their occurrence.
75. Answer: (D) Any of these health professionals
R.A. 3753 states that any birth attendant may sign the certificate of live birth.
76. Answer: (C) Magnitude of the health problem
Magnitude of the problem refers to the percentage of the population affected by a health problem. The other choices
are criteria considered in both family and community health care.
77. Answer: (D) Its main strategy is certification of health centers able to comply with standards.
Sentrong Sigla Movement is a joint project of the DOH and local government units. Its main strategy is certification
of health centers that are able to comply with standards set by the DOH.
78. Answer: (D) Those who just had a delivery within the past 15 months
The ideal birth spacing is at least two years. 15 months plus 9 months of pregnancy = 2 years.
79. Answer: (C) Adequate information for couples regarding the different methods
To enable the couple to choose freely among different methods of family planning, they must be given full
information regarding the different methods that are available to them, considering the availability of quality services
that can support their choice.
80. Answer: (B) Retinol 200,000 IU
Retinol 200,000 IU is a form of megadose Vitamin A. This may have a teratogenic effect.
81. Answer: (A) Her OB score is G5P3.
Only women with less than 5 pregnancies are qualified for a home delivery. It is also advisable for a primigravida to
have delivery at a childbirth facility.

82. Answer: (C) Folic acid


It is estimated that the incidence of neural tube defects can be reduced drastically if pregnant women have an
adequate intake of folic acid.
83. Answer: (D) Note the interval, duration and intensity of labor contractions.
Assessment of the woman should be done first to determine whether she is having true labor and, if so, what stage
of labor she is in.
84. Answer: (D) Explain to her that putting the baby to breast will lessen blood loss after delivery.
Suckling of the nipple stimulates the release of oxytocin by the posterior pituitary gland, which causes uterine
contraction. Lactation begins 1 to 3 days after delivery. Nipple stretching exercises are done when the nipples are
flat or inverted. Frequent washing dries up the nipples, making them prone to the formation of fissures.
85. Answer: (B) To stimulate milk production by the mammary acini
Suckling of the nipple stimulates prolactin reflex (the release of prolactin by the anterior pituitary gland), which
initiates lactation.
86. Answer: (B) The mother does not feel nipple pain.
When the baby has properly latched on to the breast, he takes deep, slow sucks; his mouth is wide open; and much
of the areola is inside his mouth. And, youre right! The mother does not feel nipple pain.
87. Answer: (B) 6 months
After 6 months, the babys nutrient needs, especially the babys iron requirement, can no longer be provided by
mothers milk alone.
88. Answer: (C) Retinol 200,000 I.U., 1 capsule
A capsule of Retinol 200,000 IU is given within 1 month after delivery. Potassium iodate is given during pregnancy;
malunggay capsule is not routinely administered after delivery; and ferrous sulfate is taken for two months after
delivery.
89. Answer: (C) Measles vaccine
Among the biologicals used in the Expanded Program on Immunization, measles vaccine and OPV are highly
sensitive to heat, requiring storage in the freezer.
90. Answer: (B) 4
While the unused portion of other biologicals in EPI may be given until the end of the day, only BCG is discarded 4
hours after reconstitution. This is why BCG immunization is scheduled only in the morning.

91. Answer: (A) P.D. 996


Presidential Decree 996, enacted in 1976, made immunization in the EPI compulsory for children under 8 years of
age. Hepatitis B vaccination was made compulsory for the same age group by R.A. 7846.
92. Answer: (B) BCG
BCG causes the formation of a superficial abscess, which begins 2 weeks after immunization. The abscess heals
without treatment, with the formation of a permanent scar.
93. Answer: (C) Infant BCG
Infant BCG may be given at birth. All the other immunizations mentioned can be given at 6 weeks of age.
94. Answer: (A) Seizures a day after DPT 1.
Seizures within 3 days after administration of DPT is an indication of hypersensitivity to pertussis vaccine, a
component of DPT. This is considered a specific contraindication to subsequent doses of DPT.
95. Answer: (A) Go on with the infants immunizations.
In the EPI, fever up to 38.5C is not a contraindication to immunization. Mild acute respiratory tract infection, simple
diarrhea and malnutrition are not contraindications either.
96. Answer: (A) 1 year
The baby will have passive natural immunity by placental transfer of antibodies. The mother will have active artificial
immunity lasting for about 10 years. 5 doses will give the mother lifetime protection.
97. Answer: (C) Normal
In IMCI, a respiratory rate of 50/minute or more is fast breathing for an infant aged 2 to 12 months.
98. Answer: (D) Chest indrawing
In IMCI, chest indrawing is used as the positive sign of dyspnea, indicating severe pneumonia.
99. Answer: (B) Refer him urgently to the hospital.
Severe pneumonia requires urgent referral to a hospital. Answers A, C and D are done for a client classified as
having pneumonia.
100. Answer: (B) Some dehydration
Using the assessment guidelines of IMCI, a child (2 months to 5 years old) with diarrhea is classified as having
SOME DEHYDRATION if he shows 2 or more of the following signs: restless or irritable, sunken eyes, the skin goes
back slow after a skin pinch.
101. Answer: (B) Supervise the mother in giving 200 to 400 ml. of Oresol in 4 hours.
In the IMCI management guidelines, SOME DEHYDRATION is treated with the administration of Oresol within a

period of 4 hours. The amount of Oresol is best computed on the basis of the childs weight (75 ml/kg body weight).
If the weight is unknown, the amount of Oresol is based on the childs age.
102. Answer: (D) Let the child rest for 10 minutes then continue giving Oresol more slowly.
If the child vomits persistently, that is, he vomits everything that he takes in, he has to be referred urgently to a
hospital. Otherwise, vomiting is managed by letting the child rest for 10 minutes and then continuing with Oresol
administration. Teach the mother to give Oresol more slowly.
103. Answer: (D) Edema
Edema, a major sign of kwashiorkor, is caused by decreased colloidal osmotic pressure of the blood brought about
by hypoalbuminemia. Decreased blood albumin level is due a protein-deficient diet.
104. Answer: (A) Refer the child urgently to a hospital for confinement.
Baggy pants is a sign of severe marasmus. The best management is urgent referral to a hospital.
105. Answer: (D) Conjunctival xerosis
The earliest sign of Vitamin A deficiency (xerophthalmia) is night blindness. However, this is a functional change,
which is not observable during physical examination.The earliest visible lesion is conjunctival xerosis or dullness of
the conjunctiva due to inadequate tear production.
106. Answer: (D) 200,000 IU
Preschoolers are given Retinol 200,000 IU every 6 months. 100,000 IU is given once to infants aged 6 to 12 months.
The dose for pregnant women is 10,000 IU.
107. Answer: (A) Palms
The anatomic characteristics of the palms allow a reliable and convenient basis for examination for pallor.
108. Answer: (A) Sugar
R.A. 8976 mandates fortification of rice, wheat flour, sugar and cooking oil with Vitamin A, iron and/or iodine.
109. Answer: (A) Give measles vaccine to babies aged 6 to 8 months.
Ordinarily, measles vaccine is given at 9 months of age. During an impending epidemic, however, one dose may be
given to babies aged 6 to 8 months. The mother is instructed that the baby needs another dose when the baby is 9
months old.
110. Answer: (A) Inability to drink
A sick child aged 2 months to 5 years must be referred urgently to a hospital if he/she has one or more of the
following signs: not able to feed or drink, vomits everything, convulsions, abnormally sleepy or difficult to awaken.

111. Answer: (D) Retinol capsule regardless of when the last dose was given
An infant 6 to 12 months classified as a case of measles is given Retinol 100,000 IU; a child is given 200,000 IU
regardless of when the last dose was given.
112. Answer: (B) Ask where the family resides.
Because malaria is endemic, the first question to determine malaria risk is where the clients family resides. If the
area of residence is not a known endemic area, ask if the child had traveled within the past 6 months, where he/she
was brought and whether he/she stayed overnight in that area.
113. Answer: (B) Destroying breeding places of mosquitoes
Aedes aegypti, the vector of Dengue fever, breeds in stagnant, clear water. Its feeding time is usually during the
daytime. It has a cyclical pattern of occurrence, unlike malaria which is endemic in certain parts of the country.
114. Answer: (C) Determining whether a place is endemic or not
This is diagnostic and therefore secondary level prevention. The other choices are for primary prevention.
115. Answer: (B) Pinworm
Pinworm ova are deposited around the anal orifice.
116. Answer: (C) Cough for 3 weeks
A client is considered a PTB suspect when he has cough for 2 weeks or more, plus one or more of the following
signs: fever for 1 month or more; chest pain lasting for 2 weeks or more not attributed to other conditions;
progressive, unexplained weight loss; night sweats; and hemoptysis.
117. Answer: (D) Clients diagnosed for the first time through a positive sputum exam
Category I is for new clients diagnosed by sputum examination and clients diagnosed to have a serious form of
extrapulmonary tuberculosis, such as TB osteomyelitis.
118. Answer: (B) Having the health worker or a responsible family member monitor drug intake
Directly Observed Treatment Short Course is so-called because a treatment partner, preferably a health worker
accessible to the client, monitors the clients compliance to the treatment.
119. Answer: (C) Thickened painful nerves
The lesion of leprosy is not macular. It is characterized by a change in skin color (either reddish or whitish) and loss
of sensation, sweating and hair growth over the lesion. Inability to close the eyelids (lagophthalmos) and sinking of
the nosebridge are late symptoms.
120. Answer: (D) 5 skin lesions, positive slit skin smear
A multibacillary leprosy case is one who has a positive slit skin smear and at least 5 skin lesions.

121. Answer: (B) Liver cirrhosis


The etiologic agent of schistosomiasis in the Philippines is Schistosoma japonicum, which affects the small intestine
and the liver. Liver damage is a consequence of fibrotic reactions to schistosoma eggs in the liver.
122. Answer: (C) Proper use of sanitary toilets
The ova of the parasite get out of the human body together with feces. Cutting the cycle at this stage is the most
effective way of preventing the spread of the disease to susceptible hosts.
123. Answer: (B) II
A communal faucet or water standpost is classified as Level II.
124. Answer: (A) Use of sterile syringes and needles
Hepatitis A is transmitted through the fecal oral route. Hepatitis B is transmitted through infected body secretions like
blood and semen.
125. Answer: (A) DPT
DPT is sensitive to freezing. The appropriate storage temperature of DPT is 2 to 8 C only. OPV and measles
vaccine are highly sensitive to heat and require freezing. MMR is not an immunization in the Expanded Program on
Immunization.
126. Answer: (A) 45
To estimate the number of infants, multiply total population by 3%.
127. Answer: (B) Severe dehydration
The order of priority in the management of severe dehydration is as follows: intravenous fluid therapy, referral to a
facility where IV fluids can be initiated within 30 minutes, Oresol/nasogastric tube, Oresol/orem. When the foregoing
measures are not possible or effective, tehn urgent referral to the hospital is done.
128. Answer: (A) 3
Adequate blood supply to the area allows the return of the color of the nailbed within 3 seconds.
129. Answer: (B) Instruct the mother to give the child Oresol.
Since the child does not manifest any other danger sign, maintenance of fluid balance and replacement of fluid loss
may be done by giving the client Oresol.
130. Answer: (B) Buccal mucosa
Kopliks spot may be seen on the mucosa of the mouth or the throat.
131. Answer: (D) Measles
Viral conjunctivitis is transmitted by direct or indirect contact with discharges from infected eyes. Acute poliomyelitis

is spread through the fecal-oral route and contact with throat secretions, whereas diphtheria is through direct and
indirect contact with respiratory secretions.
132. Answer: (A) Hemophilus influenzae
Hemophilus meningitis is unusual over the age of 5 years. In developing countries, the peak incidence is in children
less than 6 months of age. Morbillivirus is the etiology of measles. Streptococcus pneumoniae and Neisseria
meningitidis may cause meningitis, but age distribution is not specific in young children.
133. Answer: (D) Zooprophylaxis
Zooprophylaxis is done by putting animals like cattle or dogs close to windows or doorways just before nightfall. The
Anopheles mosquito takes his blood meal from the animal and goes back to its breeding place, thereby preventing
infection of humans.
134. Answer: (A) Stream seeding
Stream seeding is done by putting tilapia fry in streams or other bodies of water identified as breeding places of the
Anopheles mosquito.
135. Answer: (C) Destruction of breeding places of the mosquito vector
Anopheles mosquitoes breed in slow-moving, clear water, such as mountain streams.
136. Answer: (B) Cholera
Passage of profuse watery stools is the major symptom of cholera. Both amebic and bacillary dysentery are
characterized by the presence of blood and/or mucus in the stools. Giardiasis is characterized by fat malabsorption
and, therefore, steatorrhea.
137. Answer: (B) S. japonicum
S. mansoni is found mostly in Africa and South America; S. haematobium in Africa and the Middle East; and S.
malayensis only in peninsular Malaysia.
138. Answer: (D) Leptospirosis
Leptospirosis is transmitted through contact with the skin or mucous membrane with water or moist soil
contaminated with urine of infected animals, like rats.
139. Answer: (C) III
Waterworks systems, such as MWSS, are classified as level III.
140. Answer: (D) Refer the client for a supplementary test, such as Western blot, since the ELISA result may be
false.
A client having a reactive ELISA result must undergo a more specific test, such as Western blot. A negative
supplementary test result means that the ELISA result was false and that, most probably, the client is not infected.

141. Answer: (A) Being faithful to a single sexual partner


Sexual fidelity rules out the possibility of getting the disease by sexual contact with another infected person.
Transmission occurs mostly through sexual intercourse and exposure to blood or tissues.
142. Answer: (B) Infectious mononucleosis
Cytomegalovirus disease is an acute viral disease characterized by fever, sore throat and lymphadenopathy.
143. Answer: (A) Contact tracing
Contact tracing is the most practical and reliable method of finding possible sources of person-to-person transmitted
infections, such as sexually transmitted diseases.
144. Answer: (D) They are able to bring about a cure of the disease condition.
There is no known treatment for AIDS. Antiretroviral agents reduce the risk of opportunistic infections and prolong
life, but does not cure the underlying immunodeficiency.
145. Answer: (D) Consult a physician who may give them rubella immunoglobulin.
Rubella vaccine is made up of attenuated German measles viruses. This is contraindicated in pregnancy. Immune
globulin, a specific prophylactic against German measles, may be given to pregnant women.

LMR
1. Ms. Caputo is newly-promoted to a patient care manager position. She updates her knowledge on the
theories in management and leadership in order to become effective in her new role. She learns that some
managers have low concern for services and high concern for staff. Which style of management refers to
this?
A. Organization Man
B. Impoverished Management
C. Country Club Management
D. Team Management
2. Her former manager demonstrated passion for serving her staff rather than being served. She takes time
to listen, prefers to be a teacher first before being a leader, which is characteristic of
A. Transformational leader
B. Transactional leader
C. Servant leader
D. Charismatic leader
3. On the other hand, Ms. Caputo notices that the Chief Nurse Executive has charismatic leadership style.
Which of the following behaviors best describes this style?

A. Possesses inspirational quality that makes followers gets attracted of him and regards him with reverence
B. Acts as he does because he expects that his behavior will yield positive results
C. Uses visioning as the core of his leadership
D. Matches his leadership style to the situation at hand.
4. Which of the following conclusions of Ms. Caputo about leadership characteristics is TRUE?
A. There is a high correlation between the communication skills of a leader and the ability to get the job done.
B. A manager is effective when he has the ability to plan well.
C. Assessment of personal traits is a reliable tool for predicting a managers potential.
D. There is good evidence that certain personal qualities favor success in managerial role.
5. She reads about Path Goal theory. Which of the following behaviors is manifested by the leader who uses
this theory?
A. Recognizes staff for going beyond expectations by giving them citations
B. Challenges the staff to take individual accountability for their own practice
C. Admonishes staff for being laggards.
D. Reminds staff about the sanctions for non performance.
6. One leadership theory states that leaders are born and not made, which refers to which of the following
theories?
A. Trait
B. Charismatic
C. Great Man
D. Situational
7. She came across a theory which states that the leadership style is effective dependent on the situation.
Which of the following styles best fits a situation when the followers are self-directed, experts and are
matured individuals?
A. Democratic
B. Authoritarian
C. Laissez faire
D. Bureaucratic
8. She surfs the internet for more information about leadership styles. She reads about shared leadership
as a practice in some magnet hospitals. Which of the following describes this style of leadership?
A. Leadership behavior is generally determined by the relationship between the leaders personality and the specific
situation

B. Leaders believe that people are basically good and need not be closely controlled
C. Leaders rely heavily on visioning and inspire members to achieve results
D. Leadership is shared at the point of care.
9. Ms. Caputo learns that some leaders are transactional leaders. Which of the following does NOT
characterize a transactional leader?
A. Focuses on management tasks
B. Is a caretaker
C. Uses trade-offs to meet goals
D. Inspires others with vision
10. She finds out that some managers have benevolent-authoritative style of management. Which of the
following behaviors will she exhibit most likely?
A. Have condescending trust and confidence in their subordinates
B. Gives economic or ego awards
C. Communicates downward to the staff
D. Allows decision making among subordinates
11. Henry is a Unit Manager I the Medical Unit. He is not satisfied with the way things are going in his unit.
Patient satisfaction rate is 60% for two consecutive months and staff morale is at its lowest. He decides to
plan and initiate changes that will push for a turnaround in the condition of the unit. Which of the following
actions is a priority for Henry?
A. Call for a staff meeting and take this up in the agenda.
B. Seek help from her manager.
C. Develop a strategic action on how to deal with these concerns.
D. Ignore the issues since these will be resolved naturally.
12. He knows that there are external forces that influence changes in his unit. Which of the following is NOT
an external force?
A. Memo from the CEO to cut down on electrical consumption
B. Demands of the labor sector to increase wages
C. Low morale of staff in his unit
D. Exacting regulatory and accreditation standards
13. After discussing the possible effects of the low patient satisfaction rate, the staff started to list down
possible strategies to solve the problems head-on. Should they decide to vote on the best change strategy,
which of the following strategies is referred to this?

A. Collaboration
B. Majority rule
C. Dominance
D. Compromise
14. One staff suggests that they review the pattern of nursing care that they are using, which is described
as a:
A. job description
B. system used to deliver care
C. manual of procedure
D. rules to be followed
15. Which of the following is TRUE about functional nursing?
A. Concentrates on tasks and activities
B. Emphasizes use of group collaboration
C. One-to-one nurse-patient ratio
D. Provides continuous, coordinated and comprehensive nursing services
16. Functional nursing has some advantages, which one is an EXCEPTION?
A. Psychological and sociological needs are emphasized.
B. Great control of work activities.
C. Most economical way of delivering nursing services.
D. Workers feel secure in dependent role
17. He raised the issue on giving priority to patient needs. Which of the following offers the best way for
setting priority?
A. Assessing nursing needs and problems
B. Giving instructions on how nursing care needs are to be met
C. Controlling and evaluating the delivery of nursing care
D. Assigning safe nurse: patient ratio
18. Which of the following is the best guarantee that the patients priority needs are met?
A. Checking with the relative of the patient
B. Preparing a nursing care plan in collaboration with the patient
C. Consulting with the physician
D. Coordinating with other members of the team

19. When Henry uses team nursing as a care delivery system, he and his team need to assess the priority of
care for a group of patients, which of the following should be a priority?
A. Each patient as listed on the worksheet
B. Patients who needs least care
C. Medications and treatments required for all patients
D. Patients who need the most care
20. He is hopeful that his unit will make a big turnaround in the succeeding months. Which of the following
actions of Henry demonstrates that he has reached the third stage of change?
A. Wonders why things are not what it used to be
B. Finds solutions to the problems
C. Integrate the solutions to his day-to-day activities
D. Selects the best change strategy
21. Joey is a newly-appointed nurse manager of The Holy Spirit Medical Center, a tertiary hospital located
within the heart of the metropolis. He thinks of scheduling planning workshop with his staff in order to
ensure an effective and efficient management of the department. Should he decide to conduct a strategic
planning workshop, which of the following is NOT a characteristic of this activity?
A. Long-term goal-setting
B. Extends to 3-5 years in the future
C. Focuses on routine tasks
D. Determines directions of the organization
22. Which of the following statements refer to the vision of the hospital?
A. The Holy Spirit Medical Center is a trendsetter in tertiary health care in the next five year
B. The officers and staff of The Holy Spirit Medical Center believe in the unique nature of the human person
C. All the nurses shall undergo continuing competency training program.
D. The Holy Spirit Medical Center aims to provide a patient-centered care in a total healing environment.
23. The statement, The Holy Spirit Medical Center aims to provide patient-centered care in a total healing
environment refers to which of the following?
A. Vision
B. Goal
C. Philosophy
D. Mission

24. Joey plans to revisit the organizational chart of the department. He plans to create a new position of a
Patient Educator who has a coordinating relationship with the head nurse in the unit. Which of the following
will likely depict this organizational relationship?
A. Box
B. Solid line
C. Broken line
D. Dotted line
25. He likewise stresses the need for all the employees to follow orders and instructions from him and not
from anyone else. Which of the following principles does he refer to?
A. Scalar chain
B. Discipline
C. Unity of command
D. Order
26. Joey orients his staff on the patterns of reporting relationship throughout the organization. Which of the
following principles refer to this?
A. Span of control
B. Hierarchy
C. Esprit d corps
D. Unity of direction
27. He emphasizes to the team that they need to put their efforts together towards the attainment of the
goals of the program. Which of the following principles refers to this?
A. Span of control
B. Unity of direction
C. Unity of command
D. Command responsibility
28. Joey stresses the importance of promoting esprit d corps among the members of the unit. Which of the
following remarks of the staff indicates that they understand what he pointed out?
A. Lets work together in harmony; we need to be supportive of one another
B. In order that we achieve the same results; we must all follow the directives of Julius and not from other
managers.
C. We will ensure that all the resources we need are available when needed.
D. We need to put our efforts together in order to raise the bar of excellence in the care we provide to all our
patients.

29. He discusses the goal of the department. Which of the following statements is a goal?
A. Increase the patient satisfaction rate
B. Eliminate the incidence of delayed administration of medications
C. Establish rapport with patients.
D. Reduce response time to two minutes.
30. He wants to influence the customary way of thinking and behaving that is shared by the members of the
department. Which of the following terms refer to this?
A. Organizational chart
B. Cultural network
C. Organizational structure
D. Organizational culture
31. He asserts the importance of promoting a positive organizational culture in their unit. Which of the
following behaviors indicate that this is attained by the group?
A. Proactive and caring with one another
B. Competitive and perfectionist
C. Powerful and oppositional
D. Obedient and uncomplaining
32. Stephanie is a new Staff Educator of a private tertiary hospital. She conducts orientation among new
staff nurses in her department. Joseph, one of the new staff nurses, wants to understand the channel of
communication, span of control and lines of communication. Which of the following will provide this
information?
A. Organizational structure
B. Policy
C. Job description
D. Manual of procedures
33. Stephanie is often seen interacting with the medical intern during coffee breaks and after duty hours.
What type of organizational structure is this?
A. Formal
B. Informal
C. Staff
D. Line

34. She takes pride in saying that the hospital has a decentralized structure. Which of the following is NOT
compatible with this type of model?
A. Flat organization
B. Participatory approach
C. Shared governance
D. Tall organization
35. Centralized organizations have some advantages. Which of the following statements are TRUE?
1. Highly cost-effective
2. Makes management easier
3. Reflects the interest of the worker
4. Allows quick decisions or actions.
A. 1 & 2
B. 2 & 4
C. 2, 3& 4
D. 1, 2, & 4
36. Stephanie delegates effectively if she has authority to act, which is BEST defined as:
A. having responsibility to direct others
B. being accountable to the organization
C. having legitimate right to act
D. telling others what to do
37. Regardless of the size of a work group, enough staff must be available at all times to accomplish certain
purposes. Which of these purposes is NOT included?
A. Meet the needs of patients
B. Provide a pair of hands to other units as needed
C. Cover all time periods adequately.
D. Allow for growth and development of nursing staff.
38. Which of the following guidelines should be least considered in formulating objectives for nursing care?
A. Written nursing care plan
B. Holistic approach
C. Prescribed standards
D. Staff preferences

39. Stephanie considers shifting to transformational leadership. Which of the following statements best
describes this type of leadership?
A. Uses visioning as the essence of leadership.
B. Serves the followers rather than being served.
C. Maintains full trust and confidence in the subordinates
D. Possesses innate charisma that makes others feel good in his presence.
40. As a manager, she focuses her energy on both the quality of services rendered to the patients as well as
the welfare of the staff of her unit. Which of the following management styles does she adopt?
A. Country club management
B. Organization man management
C. Team management
D. Authority-obedience management

Answers and Rationale


Answers and rationale are given below for this exam. Check your correct answers and read the rationales for
additional learning. Tell us your scores on the comments section.
1. Answer: A. Country Club Management
Country club management style puts concern for the staff as number one priority at the expense of the delivery of
services. He/she runs the department just like a country club where everyone is happy including the manager.
2. Answer: C. Servant leader
Servant leaders are open-minded, listen deeply, try to fully understand others and not being judgmental
3. Answer: A. Possesses inspirational quality that makes followers gets attracted of him and regards him
with reverence
Charismatic leaders make the followers feel at ease in their presence. They feel that they are in good hands
whenever the leader is around.
4. Answer: C. Assessment of personal traits is a reliable tool for predicting a managers potential.
It is not conclusive that certain qualities of a person would make him become a good manager. It can only predict a
managers potential of becoming a good one.
5. Answer: A. Recognizes staff for going beyond expectations by giving them citations

Path Goal theory according to House and associates rewards good performance so that others would do the same
6. Answer: C. Great Man
Leaders become leaders because of their birth right. This is also called Genetic theory or the Aristotelian theory
7. Answer: C. Laissez faire
Laissez faire leadership is preferred when the followers know what to do and are experts in the field. This leadership
style is relationship-oriented rather than task-centered.
8. Answer: D. Leadership is shared at the point of care.
Shared governance allows the staff nurses to have the authority, responsibility and accountability for their own
practice.
9. Answer: D. Inspires others with vision
Inspires others with a vision is characteristic of a transformational leader. He is focused more on the day-to-day
operations of the department/unit.
10. Answer: A. Have condescending trust and confidence in their subordinates
Benevolent-authoritative managers pretentiously show their trust and confidence to their followers
11. Answer: A. Call for a staff meeting and take this up in the agenda.
This will allow for the participation of every staff in the unit. If they contribute to the solutions of the problem, they will
own the solutions; hence the chance for compliance would be greater.
12. Answer: C. Low morale of staff in his unit
Low morale of staff is an internal factor that affects only the unit. All the rest of the options emanate from the top
executive or from outside the institution.
13. Answer: B. Majority rule
Majority rule involves dividing the house and the highest vote wins. 1/2 + 1 is a majority.
14. Answer: B. system used to deliver care
A system used to deliver care. In the 70s it was termed as methods of patient assignment; in the early 80s it was
called modalities of patient care then patterns of nursing care in the 90s until recently authors called it nursing care
systems.

15. Answer: A. Concentrates on tasks and activities


Functional nursing is focused on tasks and activities and not on the holistic care of the patients
16. Answer: A. Psychological and sociological needs are emphasized.
When the functional method is used, the psychological and sociological needs of the patients are neglected; the
patients are regarded as tasks to be done
17. Answer: A. Assessing nursing needs and problems
This option follows the framework of the nursing process at the same time applies the management process of
planning, organizing, directing and controlling
18. Answer: B. Preparing a nursing care plan in collaboration with the patient
The best source of information about the priority needs of the patient is the patient himself. Hence using a nursing
care plan based on his expressed priority needs would ensure meeting his needs effectively.
19. Answer: D. Patients who need the most care
In setting priorities for a group of patients, those who need the most care should be number-one priority to ensure
that their critical needs are met adequately. The needs of other patients who need less care ca be attended to later
or even delegated to assistive personnel according to rules on delegation.
20. Answer: C. Integrate the solutions to his day-to-day activities
Integrate the solutions to his day-to-day activities is expected to happen during the third stage of change when the
change agent incorporate the selected solutions to his system and begins to create a change.
21. Answer: C. Focuses on routine tasks
Strategic planning involves options A, B and D except C which is attributed to operational planning
22. Answer: A. The Holy Spirit Medical Center is a trendsetter in tertiary health care in the next five years
A vision refers to what the institution wants to become within a particular period of time.
23. Answer: B. Goal
24. Answer: C. Broken line
This is a staff relationship hence it is depicted by a broken line in the organizational structure

25. Answer: C. Unity of command


The principle of unity of command means that employees should receive orders coming from only one manager and
not from two managers. This averts the possibility of sowing confusion among the members of the organization
26. Answer: B. Hierarchy
Hierarchy refers to the pattern of reporting or the formal line of authority in an organizational structure.
27. Answer: B. Unity of direction
Unity of direction means having one goal or one objective for the team to pursue; hence all members of the
organization should put their efforts together towards the attainment of their common goal or objective.
28. Answer: A. Lets work together in harmony; we need to be supportive of one another
The principle of esprit d corps refers to promoting harmony in the workplace, which is essential in maintaining a
climate conducive to work.
29. Answer: A. Increase the patient satisfaction rate
Goal is a desired result towards which efforts are directed.
30. Answer: D. Organizational culture
An organizational culture refers to the way the members of the organization think together and do things around
them together. Its their way of life in that organization
31. Answer: A. Proactive and caring with one another
32. Answer: A. Organizational structure
Organizational structure provides information on the channel of authority, (i.e., who reports to whom and with what
authority) the number of people who directly reports to the various levels of hierarchy and the lines of communication
whether line or staff.
33. Answer: B. Informal
This is usually not published and oftentimes concealed.
34. Answer: D. Tall organization
Tall organizations are highly centralized organizations where decision making is centered on one authority level.

35. Answer: A. 1 & 2


Centralized organizations are needs only a few managers hence they are less expensive and easier to manage
36. Answer: C. having legitimate right to act
Authority is a legitimate or official right to give command. This is an officially sanctioned responsibility
37. Answer: B. Provide a pair of hands to other units as needed
Providing a pair of hands for other units is not a purpose in doing an effective staffing process. This is a function of a
staffing coordinator at a centralized model.
38. Answer: D. Staff preferences
Staff preferences should be the least priority in formulating objectives of nursing care. Individual preferences should
be subordinate to the interest of the patients.
39. Answer: A. Uses visioning as the essence of leadership.
Transformational leadership relies heavily on visioning as the core of leadership.
40. Answer: C. Team management
Team management has a high concern for services and high concern for staff.
1. Katherine is a young Unit Manager of the Pediatric Ward. Most of her staff nurses are senior to her, very
articulate, confident and sometimes aggressive. Katherine feels uncomfortable believing that she is the
scapegoat of everything that goes wrong in her department. Which of the following is the best action that
she must take?
A. Identify the source of the conflict and understand the points of friction
B. Disregard what she feels and continue to work independently
C. Seek help from the Director of Nursing
D. Quit her job and look for another employment.
2. As a young manager, she knows that conflict occurs in any organization. Which of the following
statements regarding conflict is NOT true?
A. Can be destructive if the level is too high
B. Is not beneficial; hence it should be prevented at all times
C. May result in poor performance
D. May create leaders

3. Katherine tells one of the staff, I dont have time to discuss the matter with you now. See me in my office
later when the latter asks if they can talk about an issue. Which of the following conflict resolution
strategies did she use?
A. Smoothing
B. Compromise
C. Avoidance
D. Restriction
4. Kathleen knows that one of her staff is experiencing burnout. Which of the following is the best thing for
her to do?
A. Advise her staff to go on vacation.
B. Ignore her observations; it will be resolved even without intervention
C. Remind her to show loyalty to the institution.
D. Let the staff ventilate her feelings and ask how she can be of help.
5. She knows that performance appraisal consists of all the following activities EXCEPT:
A. Setting specific standards and activities for individual performance.
B. Using agency standards as a guide.
C. Determine areas of strength and weaknesses
D. Focusing activity on the correction of identified behavior.
6. Which of the following statements is NOT true about performance appraisal?
A. Informing the staff about the specific impressions of their work help improve their performance.
B. A verbal appraisal is an acceptable substitute for a written report
C. Patients are the best source of information regarding personnel appraisal.
D. The outcome of performance appraisal rests primarily with the staff.
7. There are times when Katherine evaluates her staff as she makes her daily rounds. Which of the following
is NOT a benefit of conducting an informal appraisal?
A. The staff member is observed in natural setting.
B. Incidental confrontation and collaboration is allowed.
C. The evaluation is focused on objective data systematically.
D. The evaluation may provide valid information for compilation of a formal report.
8. She conducts a 6-month performance review session with a staff member. Which of the following actions
is appropriate?

A. She asks another nurse to attest the session as a witness.


B. She informs the staff that she may ask another nurse to read the appraisal before the session is over.
C. She tells the staff that the session is manager-centered.
D. The session is private between the two members.
9. Alexandra is tasked to organize the new wing of the hospital. She was given the authority to do as she
deems fit. She is aware that the director of nursing has substantial trust and confidence in her capabilities,
communicates through downward and upward channels and usually uses the ideas and opinions of her
staff. Which of the following is her style of management?
A. Benevolent authoritative
B. Consultative
C. Exploitive-authoritative
D. Participative
10. She decides to illustrate the organizational structure. Which of the following elements is NOT included?
A. Level of authority
B. Lines of communication
C. Span of control
D. Unity of direction
11. She plans of assigning competent people to fill the roles designed in the hierarchy. Which process
refers to this?
A. Staffing
B. Scheduling
C. Recruitment
D. Induction
12. She checks the documentary requirements for the applicants for staff nurse position. Which one is NOT
necessary?
A. Certificate of previous employment
B. Record of related learning experience (RLE)
C. Membership to accredited professional organization
D. Professional identification card
13. Which phase of the employment process includes getting on the payroll and completing documentary
requirements?

A. Orientation
B. Induction
C. Selection
D. Recruitment
14. She tries to design an organizational structure that allows communication to flow in all directions and
involve workers in decision making. Which form of organizational structure is this?
A. Centralized
B. Decentralized
C. Matrix
D. Informal
15. In a horizontal chart, the lowest level worker is located at the
A. Left most box
B. Middle
C. Right most box
D. Bottom
16. She decides to have a decentralized staffing system. Which of the following is an advantage of this
system of staffing?
A. greater control of activities
B. Conserves time
C. Compatible with computerization
D. Promotes better interpersonal relationship
17. Aubrey thinks about primary nursing as a system to deliver care. Which of the following activities is
NOT done by a primary nurse?
A. Collaborates with the physician
B. Provides care to a group of patients together with a group of nurses
C. Provides care for 5-6 patients during their hospital stay.
D. Performs comprehensive initial assessment
18. Which pattern of nursing care involves the care given by a group of paraprofessional workers led by a
professional nurse who take care of patients with the same disease conditions and are located
geographically near each other?
A. Case method
B. Modular nursing

C. Nursing case management


D. Team nursing
19. St. Raphael Medical Center just opened its new Performance Improvement Department. Ms. Valencia is
appointed as the Quality Control Officer. She commits herself to her new role and plans her strategies to
realize the goals and objectives of the department. Which of the following is a primary task that they should
perform to have an effective control system?
A. Make an interpretation about strengths and weaknesses
B. Identify the values of the department
C. Identify structure, process, outcome standards & criteria
D. Measure actual performances
20. Ms. Valencia develops the standards to be followed. Among the following standards, which is
considered as a structure standard?
A. The patients verbalized satisfaction of the nursing care received
B. Rotation of duty will be done every four weeks for all patient care personnel.
C. All patients shall have their weights taken recorded
D. Patients shall answer the evaluation form before discharge
21. When she presents the nursing procedures to be followed, she refers to what type of standards?
A. Process
B. Outcome
C. Structure
D. Criteria
22. The following are basic steps in the controlling process of the department. Which of the following is
NOT included?
A. Measure actual performance
B. Set nursing standards and criteria
C. Compare results of performance to standards and objectives
D. Identify possible courses of action
23. Which of the following statements refers to criteria?
A. Agreed on level of nursing care
B. Characteristics used to measure the level of nursing care
C. Step-by-step guidelines
D. Statement which guide the group in decision making and problem solving

24. She wants to ensure that every task is carried out as planned. Which of the following tasks is NOT
included in the controlling process?
A. Instructing the members of the standards committee to prepare policies
B. Reviewing the existing policies of the hospital
C. Evaluating the credentials of all nursing staff
D. Checking if activities conform to schedule
25. Ms. Valencia prepares the process standards. Which of the following is NOT a process standard?
A. Initial assessment shall be done to all patients within twenty four hours upon admission.
B. Informed consent shall be secured prior to any invasive procedure
C. Patients reports 95% satisfaction rate prior to discharge from the hospital.
D. Patient education about their illness and treatment shall be provided for all patients and their families.
26. Which of the following is evidence that the controlling process is effective?
A. The things that were planned are done
B. Physicians do not complain.
C. Employees are contended
D. There is an increase in customer satisfaction rate.
27. Ms. Valencia is responsible to the number of personnel reporting to her. This principle refers to:
A. Span of control
B. Unity of command
C. Carrot and stick principle
D. Esprit d corps
28. She notes that there is an increasing unrest of the staff due to fatigue brought about by shortage of
staff. Which action is a priority?
A. Evaluate the overall result of the unrest
B. Initiate a group interaction
C. Develop a plan and implement it
D. Identify external and internal forces.
29. Kevin is a member of the Nursing Research Council of the hospital. His first assignment is to determine
the level of patient satisfaction on the care they received from the hospital. He plans to include all adult
patients admitted from April to May, with average length of stay of 3-4 days, first admission, and with no
complications. Which of the following is an extraneous variable of the study?

A. Date of admission
B. Length of stay
C. Age of patients
D. Absence of complications
30. He thinks of an appropriate theoretical framework. Whose theory addresses the four modes of
adaptation?
A. Martha Rogers
B. Sr. Callista Roy
C. Florence Nightingale
D. Jean Watson
31. He opts to use a self-report method. Which of the following is NOT TRUE about this method?
A. Most direct means of gathering information
B. Versatile in terms of content coverage
C. Most accurate and valid method of data gathering
D. Yields information that would be difficult to gather by another method
32. Which of the following articles would Kevin least consider for his review of literature?
A. Story-Telling and Anxiety Reduction Among Pediatric Patients
B. Turnaround Time in Emergency Rooms
C. Outcome Standards in Tertiary Health Care Institutions
D. Environmental Manipulation and Client Outcomes
33. Which of the following variables will he likely EXCLUDE in his study?
A. Competence of nurses
B. Caring attitude of nurses
C. Salary of nurses
D. Responsiveness of staff
34. He plans to use a Likert Scale to determine
A. degree of agreement and disagreement
B. compliance to expected standards
C. level of satisfaction
D. degree of acceptance

35. He checks if his instruments meet the criteria for evaluation. Which of the following criteria refers to the
consistency or the ability to yield the same response upon its repeated administration?
A. Validity
B. Reliability
C. Sensitivity
D. Objectivity
36. Which criteria refer to the ability of the instrument to detect fine differences among the subjects being
studied?
A. Sensitivity
B. Reliability
C. Validity
D. Objectivity
37. Which of the following terms refer to the degree to which an instrument measures what it is supposed to
be measure?
A. Validity
B. Reliability
C. Meaningfulness
D. Sensitivity
38. He plans for his sampling method. Which sampling method gives equal chance to all units in the
population to get picked?
A. Random
B. Accidental
C. Quota
D. Judgment
39. Raphael is interested to learn more about transcultural nursing because he is assigned at the family
suites where most patients come from different cultures and countries. Which of the following designs is
appropriate for this study?
A. Grounded theory
B. Ethnography
C. Case study
D. Phenomenology
40. The nursing theorist who developed transcultural nursing theory is

A. Dorothea Orem
B. Madeleine Leininger
C. Betty Newman
D. Sr. Callista Roy

Answers and Rationale


Here are the answers and rationales for the exam. This exam is out of this world difficult so dont feel bad if you got a
low score but be sure to read the rationales below. If you need clarifications, direct them to the comments below.
1. Answer: A. Identify the source of the conflict and understand the points of friction
This involves a problem solving approach, which addresses the root cause of the problem.
2. Answer: B. Is not beneficial; hence it should be prevented at all times
Conflicts are beneficial because it surfaces out issues in the open and can be solved right away. Likewise, members
of the team become more conscientious with their work when they are aware that other members of the team are
watching them.
3. Answer: C. Avoidance
This strategy shuns discussing the issue head-on and prefers to postpone it to a later time. In effect the problem
remains unsolved and both parties are in a lose-lose situation.
4. Answer: D. Let the staff ventilate her feelings and ask how she can be of help.
Reaching out and helping the staff is the most effective strategy in dealing with burn out. Knowing that someone is
ready to help makes the staff feel important; hence her self-worth is enhanced.
5. Answer: D. Focusing activity on the correction of identified behavior.
Performance appraisal deal with both positive and negative performance; is not meant to be a fault-finding activity
6. Answer: C. Patients are the best source of information regarding personnel appraisal.
The patient can be a source of information about the performance of the staff but it is never the best source. Directly
observing the staff is the best source of information for personnel appraisal.
7. Answer: C. The evaluation is focused on objective data systematically.
Collecting objective data systematically can not be achieved in an informal appraisal. It is focused on what actually
happens in the natural work setting.
8. Answer: D. The session is private between the two members.
The session is private between the manager and the staff and remains to be so when the two parties do not divulge
the information to others.

9. Answer: B. Consultative
A consultative manager is almost like a participative manager. The participative manager has complete trust and
confidence in the subordinate, always uses the opinions and ideas of subordinates and communicates in all
directions.
10. Answer: D. Unity of direction
Unity of direction is a management principle, not an element of an organizational structure.
11. Answer: A. Staffing
Staffing is a management function involving putting the best people to accomplish tasks and activities to attain the
goals of the organization.
12. Answer: B. Record of related learning experience (RLE)
Record of RLE is not required for employment purposes but it is required for the nurses licensure examination.
13. Answer: B. Induction
This step in the recruitment process gives time for the staff to submit all the documentary requirements for
employment.
14. Answer: B. Decentralized
Decentralized structures allow the staff to make decisions on matters pertaining to their practice and communicate in
downward, upward, lateral and diagonal flow.
15. Answer: C. Rightmost box
The leftmost box is occupied by the highest authority while the lowest level worker occupies the rightmost box.
16. Answer: D. Promotes better interpersonal relationship
Decentralized structures allow the staff to solve decisions by themselves, involve them in decision making; hence
they are always given opportunities to interact with one another.
17. Answer: B. Provides care to a group of patients together with a group of nurses
This function is done in team nursing where the nurse is a member of a team that provides care for a group of
patients.
18. Answer: B. Modular nursing
Modular nursing is a variant of team nursing. The difference lies in the fact that the members in modular nursing are
paraprofessional workers.
19. Answer: B. Identify the values of the department
Identify the values of the department will set the guiding principles within which the department will operate its
activities

20. Answer: B. Rotation of duty will be done every four weeks for all patient care personnel.
Structure standards include management system, facilities, equipment, materials needed to deliver care to patients.
Rotation of duty is a management system.
21. Answer: A. Process
Process standards include care plans, nursing procedure to be done to address the needs of the patients.
22. Answer: D. Identify possible courses of action
This is a step in a quality control process and not a basic step in the control process.
23. Answer: B. Characteristics used to measure the level of nursing care
Criteria are specific characteristics used to measure the standard of care.
24. Answer: A. Instructing the members of the standards committee to prepare policies
Instructing the members involves a directing function.
25. Answer: C. Patients reports 95% satisfaction rate prior to discharge from the hospital.
This refers to an outcome standard, which is a result of the care that is rendered to the patient.
26. Answer: A. The things that were planned are done
Controlling is defined as seeing to it that what is planned is done.
27. Answer: A. Span of control
Span of control refers to the number of workers who report directly to a manager.
28. Answer: B. Initiate a group interaction
Initiate a group interaction will be an opportunity to discuss the problem in the open.
29. Answer: C. Age of patients
An extraneous variable is not the primary concern of the researcher but has an effect on the results of the study.
Adult patients may be young, middle or late adult.
30. Answer: B. Sr. Callista Roy
Sr. Callista Roy developed the Adaptation Model which involves the physiologic mode, self-concept mode, role
function mode and dependence mode
31. Answer: C. Most accurate and valid method of data gathering
The most serious disadvantage of this method is accuracy and validity of information gathered
32. Answer: B. Turnaround Time in Emergency Rooms
The article is for pediatric patients and may not be relevant for adult patients.

33. Answer: C. Salary of nurses


Salary of staff nurses is not an indicator of patient satisfaction, hence need not be included as a variable in the
study.
34. Answer: A. degree of agreement and disagreement
Likert scale is a 5-point summated scale used to determine the degree of agreement or disagreement of the
respondents to a statement in a study.
35. Answer: B. Reliability
Reliability is repeatability of the instrument; it can elicit the same responses even with varied administration of the
instrument
36. Answer: A. Sensitivity
Sensitivity is an attribute of the instrument that allow the respondents to distinguish differences of the options where
to choose from
37. Answer: A. Validity
Validity is ensuring that the instrument contains appropriate questions about the research topic
38. Answer: A. Random
Random sampling gives equal chance for all the elements in the population to be picked as part of the sample.
39. Answer: B. Ethnography
Ethnography is focused on patterns of behavior of selected people within a culture
40. Answer: B. Madeleine Leininger
Madeleine Leininger developed the theory on transcultural theory based on her observations on the behavior of
selected people within a culture.
1. Which one of the following statements about hypotheses is most accurate?
A. Hypotheses represent the main idea to be studied and are the foundations of research studies.
B. Hypotheses help frame a test of the validity of a theory.
C. Hypotheses provide the means to test nursing theory.
D. A hypothesis can also be called a problem statement.
2. A nurse wants to study the effectiveness of meditation on people with anxiety disorder. Which variable
would be most relevant to explore in the literature on this topic?
A. Use of meditation during childbirth
B. Meditation techniques found to be effective

C. Pain management for people with anxiety disorders


D. Outcomes of meditation when used by elderly populations
3. What is a characteristic of an independent variable?
A. It is the variable that is predicted to change.
B. It varies with a change in the dependent variable.
C. It is manipulated by the researcher.
D. It can be identified only by changes in the dependent variable.
4. Which statement is most accurate regarding hypotheses?
A. Hypotheses operationally define the dependent variables.
B. Hypotheses are statements about the relationships among variables.
C. Hypotheses describe the effect of the dependent variable on the independent variable.
D. Hypotheses must include a definition of the treatment or intervention used.
5. What is a characteristic of a hypothesis?
A. It flows from interpretation of the data collected.
B. It operationally defines the variable to be studied.
C. It eliminates the need to designate a dependent variable.
D. It implies a causative or associative relationship.
6. When should a hypothesis be developed by the researcher during the research process?
A. Before development of the research question
B. After development of the research question
C. After a research design is determined
D. Before any statistical analysis
7. Which research hypothesis is most testable?
A. There is a relationship between meditation and anxiety disorders.
B. Patients with anxiety disorders who learn meditation techniques have less anxiety than those who do not.
C. Teaching one meditation technique to patients with anxiety disorders will be better than teaching multiple
techniques.
D. The ability to meditate causes lower anxiety in patients with anxiety disorder than those who do not meditate.
8. What is a characteristic of a statistical hypothesis?

A. It is a null hypothesis.
B. It predicts a positive relationship among variables.
C. It is a complex hypothesis.
D. It describes data-analysis methods.
9. When will a null hypothesis be rejected?
A. There is no association among variables.
B. There is evidence of significance.
C. The independent and dependent variables are related.
D. The research hypothesis is rejected.
10. Which level is characteristic of the strength of the evidence provided by the results of a quasiexperimental study?
A. Level I
B. Level II
C. Level III
D. Level IV
11. A researcher wants to discover why patients of certain ethnic backgrounds are reluctant to ask for pain
medication. Because there are little data in the literature on this topic, the researcher designs a study to
explore the relationships between cultural belief systems, the experience of pain, and the effective use of
medication to relieve pain. The researcher plans to use the findings of this study to formulate hypotheses
for a future study. What is a characteristic of this study?
A. It is a quasi-experimental study.
B. It will lead to level II data.
C. It has a directional hypothesis.
D. It is a hypothesis-generating study.
12. The nurse develops the following hypothesis: Elderly women receive less aggressive treatment for
breast cancer than do younger women. Which variable would be considered to be the dependent variable?
A. Degree of treatment received
B. Age of the patient
C. Type of cancer being treated
D. Use of inpatient treatment
13. The nurse develops the following hypothesis: Elderly women receive less aggressive treatment for
breast cancer than do younger women. Which variable would be considered to be the independent
variable?

A. Degree of treatment received


B. Age of the patient
C. Type of cancer being treated
D. Use of inpatient treatment
14. The following are considered steps in the qualitative research process, except?
A. Literature review
B. Hypothesis
C. Sample
D. Data collection
15. Which of the following could be considered the context of a study? Select all that apply.
A. Cultural understandings and beliefs of study participants
B. The physical setting of the study
C. The sample selected for the study
D. The number of subjects in the study
16. Which beliefs guide the constructivist paradigm? Select all that apply.
A. There are multiple realities.
B. The truth is objective.
C. Context does not matter as much as truth.
D. The participant (subject) is an active part of the study
17. Which of the following are consistent with the constructivist paradigm? Select all that apply.
A. Subjectivism is valued.
B. Natural laws exist.
C. Time and place are important.
D. Generalizability is valued.
18. Which paradigm provides the basis for qualitative research?
A. Empirical analytical research
B. Constructivism
C. Postpositivism
D. Naturalistic research
19. Which type of research allows researchers to be neutral observers?

A. Qualitative research
B. Ethnographic research
C. Quantitative research
D. Case studies
20. Which type of research study can be affected by detracting values of the researcher?
A. Qualitative
B. Naturalistic
C. Ethnographic
D. Quantitative

Answers and Rationale


1. Answer: B. Hypotheses help frame a test of the validity of a theory.
Although theories cannot be tested directly, hypotheses provide a bridge between theory and the real world. It is the
research question that represents the main idea to be studied (A). Theories cannot be tested directly (C). The
research question is also called the problem statement (D).
2. Answer: B. Meditation techniques found to be effective
3. Answer: C. It is manipulated by the researcher.
The independent variable is manipulated by the researcher and has a presumed effect on the dependent variable. It
is the dependent variable that is predicted to change (A). The independent variable is presumed to change the
dependent variable (B). The independent variable is manipulated by the researcher and is identified at the beginning
of the study (D).
4. Answer: B. Hypotheses are statements about the relationships among variables.
Hypotheses are statements about the relationships between two or more variables that suggest an answer to the
research question. Hypotheses are not concerned with operationally defining the variables involved in the study (A).
The independent variable is not affected or changed by the dependent variable (C). Hypotheses are not concerned
with operationally defining the variables involved in the study, including treatments or interventions (D).
5. Answer: D. It implies a causative or associative relationship.
A hypothesis implies a causative or associative relationship. A hypothesis guides the research design and collection
of data (A). Operational definitions are not included in the hypothesis (B). The hypothesis indicates the dependent
variable (C).

6. Answer: B. After development of the research question


The hypothesis is developed after development of the research question.
7. Answer: D. The ability to meditate causes lower anxiety in patients with anxiety disorder than those who
do not meditate.
This hypothesis meets the criteria of testability.
8. Answer: A. It is a null hypothesis.
Statistical hypotheses, called null hypotheses, state that there is no relationship between the independent and
dependent variables.
9. Answer: C. The independent and dependent variables are related.
Because the null hypothesis states that there is no relationship between the independent and dependent variables, it
is rejected if they are related.
10. Answer: C. Level III
Evidence provided by quasi-experimental studies is level III. Level I evidence is obtained from a systematic review of
all randomized, controlled trials. Level II evidence is obtained from at least one well-designed randomized, controlled
trial. Level IV evidence is obtained from nonexperimental studies.
11. Answer: D. It is a hypothesis-generating study.
Not enough is known in this area at this time to formulate hypotheses, so the researcher will conduct this qualitative
study and use the findings to generate hypotheses for future studies. This is a qualitative study, not a quasiexperimental study. Level II evidence is obtained from at least one well-designed randomized, controlled trial. This
study has no hypothesis.
12. Answer: A. Degree of treatment received
The degree of treatment received is considered the dependent variable.
13. Answer: B. Age of the patient
The age of the patient would be the independent variable.
14. Answer: B. Hypothesis
A hypothesis is the tool of quantitative studies, and is only found in such studies.

15. Answer: A, B
16. Answer: A, D
17. Answer: A, C
18. Answer: B. Constructivism
The paradigm that provides the basis for qualitative research is constructivism.
19. Answer: C. Quantitative research
In qualitative research, researchers are never considered neutral (A). In ethnography, a type of qualitative research,
researchers are never considered neutral (B). In case studies, a type of qualitative research, researchers are never
considered neutral (D).
20. Answer: D. Quantitative
The values of the researcher must be acknowledged in qualitative research (A). The values of the researcher must
be acknowledged in naturalistic research (B). The values of the researcher must be acknowledged in qualitative
research (C).
1. What is the purpose of grounded theory?
A. To support theoretical frameworks
B. To generate theory from data
C. To develop explanatory models
D. To find significant differences among groups of people
2. Why is it important to understand the philosophy underlying each type of research?
A. Conclusions reached should be congruent with the research question.
B. The research method that best meets intended purpose of the study should be used.
C. The paradigm of the method should be the same as that of the researcher.
D. The reader should understand the level of abstraction of the study.
3. Which conceptual analysis point of the framework for rigor used for interpretive phenomenology refers to
how the study findings will continue to have meaning for the reader?
A. Resonance
B. Concreteness
C. Actualization
D. Openness

4. Which question will critique the sampling of a research project?


A. Is the strategy used for analysis compatible with the purpose of the study?
B. What is the projected significance of the work to nursing?
C. Are the informants who were chosen appropriate to inform the research?
D. What are the philosophic underpinnings of the research method?
5. Which question will critique the method of a research project?
A. Is the strategy used for analysis compatible with the purpose of the study?
B. What is the projected significance of the work to nursing?
C. Are the informants who were chosen appropriate to inform the research?
D. What are the philosophic underpinnings of the research method?
6. Which question will critique the purpose of a research project?
A. Is the strategy used for analysis compatible with the purpose of the study?
B. What is the projected significance of the work to nursing?
C. Are the informants who were chosen appropriate to inform the research?
D. What are the philosophic underpinnings of the research method?
7. Which question will critique the credibility of a research project?
A. Is the strategy used for analysis compatible with the purpose of the study?
B. Does the researcher document the research process?
C. Are the researchers conceptualizations true to the data?
D. Has adequate time been allowed to understand fully the phenomenon?
8. Which question will critique the auditability of a research project?
A. Is the strategy used for analysis compatible with the purpose of the study?
B. Does the researcher document the research process?
C. Are the researchers conceptualizations true to the data?
D. Has adequate time been allowed to fully understand the phenomenon?
9. Which question will critique the fittingness of a research project?
A. Is the strategy used for analysis compatible with the purpose of the study?
B. Does the researcher document the research process?
C. Are the researchers conceptualizations true to the data?
D. Has adequate time been allowed to fully understand the phenomenon?

10. What are the uses of qualitative research methods? Select all that apply.
A. Guiding nursing practice
B. Studying the effects of nursing care on an outcome variable
C. Developing survey instruments
D. Developing nursing theory
11. What are scientific criteria appropriate for qualitative research? Select all that apply.
A. Auditability
B. Credibility
C. Fittingness
D. Reliability
12. What are ethical concerns for qualitative researchers? Select all that apply.
A. Because the study emerges over time, the researcher may not anticipate and inform the participants of a potential
threat.
B. To maintain a naturalistic environment for interviews, formal documents such as consent forms are not used.
C. Because there are so few participants in a qualitative study, no participant can opt out of the study.
D. Because the researcher and participant interact over a period of time, relationships developed between them may
change the focus of the interaction
13. Which of the following is most accurate regarding the grounded-theory method?
A. Data are collected using an etic perspective.
B. It is a process of constructing human experience.
C. Secondary sources are sometimes used.
D. It is an inductive approach.
14. What is the term used for the coding and clustering of data to form categories in the grounded-theory
method?
A. Theoretical sampling
B. Constant-comparative method
C. Emic method
D. Metasynthesis
15. What is a characteristic of an intrinsic case study?
A. It yields a better understanding of each case.
B. It provides a foundation to challenge a generalization.

C. It does not include quantitative data.


D. It can scrutinize only uncomplicated phenomena.
16. What is a characteristic of metasynthesis?
A. It is useful for triangulating research.
B. It synthesizes critical masses of qualitative findings.
C. It leads to a higher reliability of research findings.
D. It cannot be conducted on historical or case study findings.
17. What is meant by the fittingness of a research study?
A. Truth of findings as judged by the participants
B. The appropriateness of the interview questions posed
C. Faithfulness to everyday reality of the participants
D. The adequacy of the coding system used
18. How can qualitative outcome analysis be used? Select all that apply.
A. To determine the reliability of intervention outcomes in a study
B. To confirm the applicability of clinical strategies
C. To develop interventions and then test those selected
D. To build theory
19. When critiquing a qualitative study, which of the following questions are helpful in determining the
studys auditability? Select all that apply.
A. Has adequate time been allowed to understand the phenomenon fully?
B. Can the reader follow the researchers thinking?
C. Are the results meaningful to individuals not involved in the research?
D. Does the researcher document the research process?
20. Which question is helpful in determining the studys credibility?
A. Do the participants recognize the experience as their own?
B. What strategies were used to analyze the data?
C. How were human subjects protected?
D. Are the findings applicable outside the study situation?

Answers and Rationale


1. Answer: B. To generate theory from data

The grounded theory method refers to a qualitative approach of building theory about a phenomenon about which
little is known.
2. Answer: B. The research method that best meets intended purpose of the study should be used.
Different research methods accomplish different goals and offer different types and levels of evidence that inform
practice.
3. Answer: C. Actualization
4. Answer: C. Are the informants who were chosen appropriate to inform the research?
5. Answer: A. Is the strategy used for analysis compatible with the purpose of the study?
6. Answer: B. What is the projected significance of the work to nursing?
7. Answer: D. Has adequate time been allowed to understand fully the phenomenon?
8. Answer: B. Does the researcher document the research process?
9. Answer: A. Is the strategy used for analysis compatible with the purpose of the study?
10. Answer: A, C, D
11. Answer: A, B, C
12. Answer: A, D
13. Answer: D. It is an inductive approach.
Data are collected using the emic perspective (A). The grounded-theory method is a process of constructing theory
from human experience (B). In grounded-theory only primary sources (the participants) are used (C).
14. Answer: B. Constant-comparative method
15. Answer: A. It yields a better understanding of each case.
An intrinsic case study is undertaken to have a better understanding of the case.
16. Answer: B. It synthesizes critical masses of qualitative findings.
17. Answer: C. Faithfulness to everyday reality of the participants

Credibility is the truth of findings as judged by the participants (A). Auditability assists the reader to judge the
appropriateness of the interview questions posed (B). Auditability assists the reader to judge the adequacy of the
coding system used (D).
18. Answer: B, C, D
19. Answer: B, C
20. Answer: A. Do the participants recognize the experience as their own?

ABG
1. George Kent is a 54 year old widower with a history of chronic obstructive pulmonary disease and was
rushed to the emergency department with increasing shortness of breath, pyrexia, and a productive cough
with yellow-green sputum. He has difficulty in communicating because of his inability to complete a
sentence. One of his sons, Jacob, says he has been unwell for three days. Upon examination, crackles and
wheezes can be heard in the lower lobes; he has a tachycardia and a bounding pulse. Measurement of
arterial blood gas shows pH 7.3, PaCO2 68 mm Hg, HCO3 28 mmol/L, and PaO2 60 mm Hg. How would you
interpret this?
A. Respiratory Acidosis, Uncompensated
B. Respiratory Acidosis, Partially Compensated
C. Metabolic Alkalosis, Uncompensated
D. Metabolic Acidosis, Partially Compensated
2. Carl, an elementary student, was rushed to the hospital due to vomiting and a decreased level of
consciousness. The patient displays slow and deep (Kussmaul breathing), and he is lethargic and irritable
in response to stimulation. He appears to be dehydratedhis eyes are sunken and mucous membranes are
dryand he has a two week history of polydipsia, polyuria, and weight loss. Measurement of arterial blood
gas shows pH 7.0, PaO2 90 mm Hg, PaCO2 23 mm Hg, and HCO3 12 mmol/L; other results are Na+ 126
mmol/L, K+ 5 mmol/L, and Cl- 95 mmol/L. What is your assessment?
A. Respiratory Acidosis, Uncompensated
B. Respiratory Acidosis, Partially Compensated
C. Metabolic Alkalosis, Uncompensated
D. Metabolic Acidosis, Partially, Compensated
3. A cigarette vendor was brought to the emergency department of a hospital after she fell into the ground
and hurt her left leg. She is noted to be tachycardic and tachypneic. Painkillers were carried out to lessen
her pain. Suddenly, she started complaining that she is still in pain and now experiencing muscle cramps,
tingling, and paraesthesia. Measurement of arterial blood gas reveals pH 7.6, PaO2 120 mm Hg, PaCO2 31
mm Hg, and HCO3 25 mmol/L. What does this mean?

A. Respiratory Alkalosis, Uncompensated


B. Respiratory Acidosis, Partially Compensated
C. Metabolic Alkalosis, Uncompensated
D. Metabolic Alkalosis, Partially Compensated
4. Rickys grandmother is suffering from persistent vomiting for two days now. She appears to be lethargic
and weak and has myalgia. She is noted to have dry mucus membranes and her capillary refill takes >4
seconds. She is diagnosed as having gastroenteritis and dehydration. Measurement of arterial blood gas
shows pH 7.5, PaO2 85 mm Hg, PaCO2 40 mm Hg, and HCO3 34 mmol/L. What acid-base disorder is shown?
A. Respiratory Alkalosis, Uncompensated
B. Respiratory Acidosis, Partially Compensated
C. Metabolic Alkalosis, Uncompensated
D. Metabolic Alkalosis, Partially Compensated
5. Mrs. Johansson, who had undergone surgery in the post-anesthesia care unit (PACU), is difficult to
arouse two hours following surgery. Nurse Florence in the PACU has been administering Morphine Sulfate
intravenously to the client for complaints of post-surgical pain. The clients respiratory rate is 7 per minute
and demonstrates shallow breathing. The patient does not respond to any stimuli! The nurse assesses the
ABCs (remember Airway, Breathing, Circulation!) and obtains ABGs STAT! Measurement of arterial blood
gas shows pH 7.10, PaCO2 70 mm Hg and HCO3 24 mEq/L. What does this mean?
A. Respiratory Alkalosis, Partially Compensated
B. Respiratory Acidosis, Uncompensated
C. Metabolic Alkalosis, Partially Compensated
D. Metabolic Acidosis, Uncompensated
6. Baby Angela was rushed to the Emergency Room following her mothers complaint that the infant has
been irritable, difficult to breastfeed and has had diarrhea for the past 3 days. The infants respiratory rate is
elevated and the fontanels are sunken. The Emergency Room physician orders ABGs after assessing the
ABCs. The results from the ABG results show pH 7.39, PaCO2 27 mmHg and HCO3 19 mEq/L. What does
this mean?
A. Respiratory Alkalosis, Fully Compensated
B. Metabolic Acidosis, Uncompensated
C. Metabolic Acidosis, Fully Compensated
D. Respiratory Acidosis, Uncompensated
7. Mr. Wales, who underwent post-abdominal surgery, has a nasogastric tube. The nurse on duty notes that
the nasogastric tube (NGT) is draining a large amount (900 cc in 2 hours) of coffee ground secretions. The
client is not oriented to person, place, or time. The nurse contacts the attending physician and STAT ABGs

are ordered. The results from the ABGs show pH 7.57, PaCO2 37 mmHg and HCO3 30 mEq/L. What is your
assessment?
A. Metabolic Acidosis, Uncompensated
B. Metabolic Alkalosis, Uncompensated
C. Respiratory Alkalosis, Uncompensated
D. Metabolic Alkalosis, Partially Compensated
8. Client Z is admitted to the hospital and is to undergo brain surgery. The client is very anxious and scared
of the upcoming surgery. He begins to hyperventilate and becomes very dizzy. The client loses
consciousness and the STAT ABGs reveal pH 7.61, PaCO2 22 mmHg and HCO3 25 mEq/L. What is the ABG
interpretation based on the findings?
A. Metabolic Acidosis, Uncompensated
B. Respiratory Alkalosis, Partially Compensated
C. Respiratory Alkalosis, Uncompensated
D. Metabolic Alkalosis, Partially Compensated
9. Three-year-old Adrian is admitted to the hospital with a diagnosis of asthma and respiratory distress
syndrome. The mother of the child reports to the nurse on duty that she has witnessed slight tremors and
behavioral changes in her child over the past four days. The attending physician orders routine ABGs
following an assessment of the ABCs. The ABG results are pH 7.35, PaCO2 72 mmHg and HCO3 38 mEq/L.
What acid-base disorder is shown?
A. Respiratory Acidosis, Uncompensated
B. Respiratory Acidosis, Fully Compensated
C. Respiratory Alkalosis, Fully Compensated
D. Metabolic Alkalosis, Partially Compensated
10. Anne, who is drinking beer at a party, falls and hits her head on the ground. Her friend Liza dials 911
because Anne is unconscious, depressed ventilation (shallow and slow respirations), rapid heart rate, and
is profusely bleeding from both ears. Which primary acid-base imbalance is Anne at risk for if medical
attention is not provided?
A. Metabolic Acidosis
B. Metabolic Alkalosis
C. Respiratory Acidosis
D. Respiratory Alkalosis

Answers and Rationale

1. Answer: B. Respiratory Acidosis, Partially Compensated


The patient has respiratory acidosis (raised carbon dioxide) resulting from an acute exacerbation of chronic
obstructive pulmonary disease, with partial compensation.
2. Answer: D. Metabolic Acidosis, Partially, Compensated
The student was diagnosed having diabetes mellitus. The results show that he has metabolic acidosis (low HCO3 -)
with respiratory compensation (low CO2).
3. Answer: A. Respiratory Alkalosis, Uncompensated
The primary disorder is acute respiratory alkalosis (low CO2) due to the pain and anxiety causing her to
hyperventilate. There has not been time for metabolic compensation.
4. Answer: C. Metabolic Alkalosis, Uncompensated
The primary disorder is uncompensated metabolic alkalosis (high HCO3 -). As CO2 is the strongest driver of
respiration, it generally will not allow hypoventilation as compensation for metabolic alkalosis.
5. Answer: B. Respiratory Acidosis, Uncompensated
The results show that Mrs. Johansson has respiratory acidosis because of decreased pH and increased PaCO2
which mean acidic in nature. Meanwhile, it is uncompensated because HCO3 is within the normal range.
6. Answer: C. Metabolic Acidosis, Fully Compensated
Baby Angela has metabolic acidosis due to decreased HCO3 and slightly acidic pH. Her pH value is within the
normal range which made the result fully compensated.
7. Answer: B. Metabolic Alkalosis, Uncompensated
The postoperative clients ABG results show that he has metabolic alkalosis because of an increased pH and HCO3.
It is uncompensated due to the normal PaCO2 which is within 35 to 45 mmHg.
8. Answer: C. Respiratory Alkalosis, Uncompensated
The results show that client Z has respiratory alkalosis since there is an increase in the pH value and a decrease in
PaCO2 which are both basic. It is uncompensated due to the normal HCO3 which is within 22-26 mEq/L.
9. Answer: B. Respiratory Acidosis, Fully Compensated

The patient has respiratory acidosis (raised carbon dioxide) resulting from asthma and respiratory distress
syndrome, with compensation having normal pH value within 7.35to 7.45, increased PaCO2 which is acidic and
increased HCO3 which is basic.
10. Answer: C. Respiratory Acidosis
One of the risk factors of having respiratory acidosis is hypoventilation which may be due to brain trauma, coma, and
hypothyroidism or myxedema. Other risk factors include COPD, Respiratory conditions such as pneumothorax,
pneumonia and status asthmaticus. Drugs such as Morphine and MgSO4 toxicity are also risk factors of respiratory
acidosis.
1. Dave, a 6-year-old boy, was rushed to the hospital following her mothers complaint that her son has been
vomiting, nauseated and has overall weakness. After series of tests, the nurse notes the laboratory results:
potassium: 2.9 mEq. Which primary acid-base imbalance is this boy at risk for if medical intervention is not
carried out?
A. Respiratory Acidosis
B. Respiratory Alkalosis
C. Metabolic Acidosis
D. Metabolic Alkalosis
2. An old beggar was admitted to the emergency department due to shortness of breath, fever, and a
productive cough. Upon examination, crackles and wheezes are noted in the lower lobes; he appears to be
tachycardic and has a bounding pulse. Measurement of arterial blood gas shows pH 7.2, PaCO2 66 mm Hg,
HCO3 27 mmol/L, and PaO2 65 mm Hg. As a knowledgeable nurse, you know that the normal value for pH
is:
A. 7.20
B. 7.30
C. 7.40
D. 7.50
3. Lizas mother is seen in the emergency department at a community hospital. She admits that her mother
is taking many tablets of aspirin (salicylates) over the last 24-hour period because of a severe headache.
Also, the mother complains of an inability to urinate. The nurse on duty took her vital signs and noted the
following: Temp = 97.8 F; apical pulse = 95; respiration = 32 and deep. Which primary acid-base imbalance
is the gentleman at risk for if medical attention is not provided?
A. Respiratory Acidosis
B. Respiratory Alkalosis

C. Metabolic Acidosis
D. Metabolic Alkalosis
4. A patient who is hospitalized due to vomiting and a decreased level of consciousness displays slow and
deep (Kussmaul breathing), and he is lethargic and irritable in response to stimulation. The doctor
diagnosed him of having dehydration. Measurement of arterial blood gas shows pH 7.0, PaO2 90 mm Hg,
PaCO2 22 mm Hg, and HCO3 14 mmol/L; other results are Na+ 120 mmol/L, K+ 2.5 mmol/L, and Cl- 95
mmol/L. As a knowledgeable nurse, you know that the normal value for PaCO2 is:
A. 22 mm Hg
B. 36 mm Hg
C. 48 mm Hg
D. 50 mm Hg
5. A company driver is found at the scene of an automobile accident in a state of emotional distress. He tells
the paramedics that he feels dizzy, tingling in his fingertips, and does not remember what happened to his
car. Respiratory rate is rapid at 34/minute. Which primary acid-base disturbance is the young man at risk for
if medical attention is not provided?
A. Respiratory Acidosis
B. Respiratory Alkalosis
C. Metabolic Acidosis
D. Metabolic Alkalosis
6. An elderly client was admitted to hospital in a coma. Analysis of the arterial blood gave the following
values: PCO2 16 mm Hg, HCO3- 5 mmol/L and pH 7.1. As a well-rounded nurse, you know that the normal
value for HCO3 is:
A. 20 mmol/L
B. 24 mmol/L
C. 29 mmol/L
D. 31 mmol/L
7. In a patient undergoing surgery, it was vital to aspirate the contents of the upper gastrointestinal tract.
After the operation, the following values were acquired from an arterial blood sample: pH 7.55, PCO2 52 mm
Hg and HCO3- 40 mmol/l. What is the underlying disorder?
A. Respiratory Acidosis
B. Respiratory Alkalosis
C. Metabolic Acidosis
D. Metabolic Alkalosis

8. A mountaineer attempts an assault on a high mountain in the Andes and reaches an altitude of 5000
meters (16,400 ft) above sea level. What will happen to his arterial PCO2 and pH?
A. Both will be lower than normal.
B. The pH will rise and PCO2 will fall.
C. Both will be higher than normal due to the physical exertion.
D. The pH will fall and PCO2 will rise
9. A young woman is found comatose, having taken an unknown number of sleeping pills an unknown time
before. An arterial blood sample yields the following values: pH 6.90, HCO3- 13 meq/liter and PaCO2 68
mmHg. This patients acid-base status is most accurately described as:
A. Metabolic Acidosis
B. Respiratory Acidosis
C. Simultaneous Respiratory and Metabolic Acidosis
D. Respiratory Acidosis with Complete Renal Compensation
10. A mother is admitted in the emergency department following complaints of fever and chills. The nurse
on duty took her vital signs and noted the following: Temp = 100 F; apical pulse = 95; respiration = 20 and
deep. Measurement of arterial blood gas shows pH 7.37, PaO2 90 mm Hg, PaCO2 40 mm Hg, and HCO3 24
mmol/L. What is your assessment?
A. Hyperthermia
B. Hyperthermia and Respiratory Alkalosis
C. Hypothermia
D. Hypothermia and Respiratory Alkalosis

Answers and Rationale


1. Answer: D. Metabolic Alkalosis
Vomiting, hypokalemia, overdosage of NaHCO3 and NGT suctioning are considered risk factors of metabolic
alkalosis.
2. Answer: C. 7.40
Normal blood pH must be maintained within a narrow range of 7.35-7.45 to ensure the proper functioning of
metabolic processes and the delivery of the right amount of oxygen to tissues. Acidosis refers to an excess of acid in
the blood that causes the pH to fall below 7.35, and alkalosis refers to an excess of base in the blood that causes
the pH to rise above 7.45.
3. Answer. C. Metabolic Acidosis

Salicylate overdose causes a high anion gap metabolic acidosis in both children and adults. Adults commonly
develop a mixed acid-base disorder as a respiratory alkalosis due to direct respiratory centre stimulation occurs as
well. This second disorder is uncommon in children.
4. Answer: B. 36 mm Hg
The normal range for PaCO2 is from 35 to 35 mm Hg.
5. Answer: B. Respiratory Alkalosis
Hyperventilation is typically the underlying cause of respiratory alkalosis. Hyperventilation is also known as
overbreathing. When someone is hyperventilating, they tend to breathe very deeply or very rapidly.
6. Answer: B. 24 mmol/L
The normal value for bicarbonate (HCO3) is 22-26 mmol/L or mEq/L. It may vary slightly among different
laboratories. The given values show the common measurement range of results for these tests. Some laboratories
use different measurements or may test different specimens.
7. Answer: D. Metabolic Alkalosis
NGT suctioning, vomiting, hypokalemia and overdosage of NaHCO3 are considered risk factors of metabolic
alkalosis.
8. Answer: B. The pH will rise and PCO2 will fall.
The mountaineer will suffer from a respiratory alkalosis. The decline in the PO2 with altitude will stimulate breathing
to offset the hypoxia. Carbon dioxide is driven from the blood faster than it is produced in the tissues so PCO2 falls
and pH rises.
9. Answer: C. Simultaneous Respiratory and Metabolic Acidosis
Whenever the PCO2 and HCO3 are abnormal in opposite directions, ie, one above normal while the other is
reduced, a mixed respiratory and metabolic acid-base disorder exists. When the PCO2 is elevated and the [HCO3-]
reduced, respiratory acidosis and metabolic acidosis coexist.
10. Answer: A. Hyperthermia
An individual is considered to have hyperthermia if he or she has a temperature of >37.5 or 38.3 C (99.5 or 100.9
F). Measurement of arterial blood gases are normal.
In acid-base balance the normal plasma PCO2 and bicarbonate levels are disturbed. Match the changes in
these parameters given below with the disorders in the given choices.

1. Low plasma PaCO2


A. Metabolic Acidosis
B. Respiratory Alkalosis
C. Metabolic Alkalosis
D. Respiratory Acidosis
2. High plasma PaCO2
A. Metabolic Acidosis
B. Respiratory Alkalosis
C. Metabolic Alkalosis
D. Respiratory Acidosis
3. Decreased plasma bicarbonate (HCO3-)
A. Metabolic Acidosis
B. Respiratory Alkalosis
C. Metabolic Alkalosis
D. Respiratory Acidosis
4. Increased plasma bicarbonate (HCO3-)
A. Metabolic Acidosis
B. Respiratory Alkalosis
C. Metabolic Alkalosis
D. Respiratory Acidosis
5. What two organs in the body serve as a compensatory function to maintain acid base balance?
A. Kidneys and Lungs
B. Lungs and Spleen
C. Heart and Liver
D. Gallbladder and Appendix
6. Arterial blood gas (ABG) measurement will give the information needed to determine if the primary
disturbance of acid-base balance is respiratory or metabolic in nature.
A. True
B. False
C. Both Carbonic Acid Excess and Deficit Only
D. Both Bicarbonate Excess and Deficit Only

7. The major effect of acidosis is overexcitement of the central nervous system.


A. True
B. False
C. Maybe
D. Both Acidosis and Alkalosis result in overexcitement of the central nervous system.
8. Alkalosis is characterized by overexcitement of the nervous system.
A. True
B. False
C. The major effect of Alkalosis is a depression of the central nervous system.
D. Both Acidosis and Alkalosis result in overexcitement of the central nervous system.
9. The human body functions optimally in a state of homeostasis.
A. True
B. False
C. Maybe
D. Homeostasis has nothing to do with metabolic balance.
10. Acids have no hydrogen ions and are able to bind in a solution.
A. True
B. False
C. Acid is a substance that is not capable of donating hydrogen ions.
D. Acids and bases have nothing to do with hydrogen ions.
Match the acid-base status of the following blood samples to the disorders in the given choices. (PaCO2
values are in mm Hg and bicarbonate values in mmol/l).
11. pH 7.57, PaCO2 22, HCO3- 17
A. Respiratory Acidosis, Partially Compensated
B. Respiratory Alkalosis, Uncompensated
C. Metabolic Acidosis, Partially Compensated
D. Respiratory Alkalosis, Partially Compensated
12. pH 7.39, PaCO2 44, HCO3- 26
A. Respiratory Acidosis
B. Metabolic Acidosis

C. Respiratory Alkalosis
D. Normal
13. pH 7.55, PaCO2 25, HCO3- 22
A. Respiratory Acidosis, Partially Compensated
B. Respiratory Alkalosis, Uncompensated
C. Metabolic Alkalosis, Partially Compensated
D. Metabolic Acidosis, Uncompensated
14. pH 7.17, PaCO2 48, HCO3- 36
A. Respiratory Acidosis, Uncompensated
B. Metabolic Acidosis, Partially Compensated
C. Respiratory Alkalosis, Partially Compensated
D. Respiratory Acidosis, Partially Compensated
15. pH 7.34, PaCO2 24, HCO3- 20
A. Respiratory Acidosis, Partially Compensated
B. Metabolic Acidosis, Partially Compensated
C. Metabolic Acidosis, Uncompensated
D. Metabolic Alkalosis, Partially Compensated
16. pH 7.64, PaCO2 25, HCO3- 19
A. Respiratory Acidosis, Uncompensated
B. Respiratory Alkalosis, Partially Compensated
C. Respiratory Alkalosis, Uncompensated
D. Metabolic Alkalosis, Partially Compensated
17. pH 7.45, PaCO2 50, HCO3- 30
A. Metabolic Alkalosis, Fully Compensated
B. Respiratory Alkalosis, Fully Compensated
C. Metabolic Alkalosis, Partially Compensated
D. Respiratory Acidosis, Partially Compensated
18. pH 7.6, PaCO2 53, HCO3- 38
A. Metabolic Alkalosis, Partially Compensated
B. Metabolic Alkalosis, Fully Compensated

C. Respiratory Acidosis, Partially Compensated


D. Respiratory Alkalosis, Fully Compensated
19. pH 7.5, PaCO2 19, HCO3- 22
A. Respiratory Alkalosis, Partially Compensated
B. Metabolic Alkalosis, Partially Compensated
C. Respiratory Acidosis, Uncompensated
D. Respiratory Alkalosis, Uncompensated
20. pH 7.4, PaCO2 59, HCO3- 35
A. Respiratory Acidosis, Uncompensated
B. Metabolic Alkalosis, Uncompensated
C. Respiratory Acidosis, Fully Compensated
D. Metabolic Alkalosis, Partially Compensated

Answers and Rationale


1. Answer: B. Respiratory Alkalosis
Excessive pulmonary ventilation decreases hydrogen ion concentration and thus causes respiratory alkalosis. It can
become dangerous when it leads to cardiac dysrhythmias caused partly by a decrease in serum potassium levels.
2. Answer: D. Respiratory Acidosis
An excess of carbon dioxide (hypercapnia) can cause carbon dioxide narcosis. In this condition, carbon dioxide
levels are so high that they no longer stimulate respirations but depress them.
3. Answer: A. Metabolic Acidosis
The body compensates by using body fat for energy, producing abnormal amounts of ketone bodies. In an effort to
neutralize the ketones and maintain the acid-base balance of the body, plasma bicarbonate is exhausted. This
condition can develop in anyone who does not eat an adequate diet and whose body fat must be burned for energy.
Symptoms include headache and mental dullness.
4. Answer: C. Metabolic Alkalosis
In metabolic alkalosis, breathing becomes depressed in an effort to conserve carbon dioxide for combination with
water in the blood to raise the blood level of carbonic acid. Symptoms include confusion, dizziness, numbness or
tingling of fingers or toes.

5. Answer: A. Kidneys and Lungs


The carbonic acid concentration is controlled by the amount of carbon dioxide excreted by the lungs. The
bicarbonate concentration is controlled by the kidneys, which selectively retain or excrete bicarbonate in response to
the bodys needs.
6. Answer: A. True
ABGs are blood tests that are useful in identifying the cause and extent of the acid-base disturbance and in guiding
and monitoring treatment.
7. Answer: B. False
The major effect is a depression of the central nervous system, as evidenced by disorientation followed by coma.
8. Answer: A. True
The muscles may go into a state of tetany and convulsions.
9. Answer: A. True
The maintenance of acid-base balance, which in one part of homeostasis, is evidenced by an arterial plasma pH
value of 7.35-7.45. Many mechanisms in the body work together to achieve and maintain this delicate narrow range
of pH that is essential for normal cell function.
10. Answer: B. False
Acids are substances having one or more hydrogen ions that can be liberated into a solution.
Bases are substances that can bind hydrogen ions in a solution.
11. Answer: D. Respiratory Alkalosis, Partially Compensated
12. Answer: D. Normal
13. Answer: B. Respiratory Alkalosis, Uncompensated
14. Answer: D. Respiratory Acidosis, Partially Compensated
15. Answer: B. Metabolic Acidosis, Partially Compensated
16. Answer: B. Respiratory Alkalosis, Partially Compensated
17. Answer: A. Metabolic Alkalosis, Fully Compensated

18. Answer: A. Metabolic Alkalosis, Partially Compensated


19. Answer: D. Respiratory Alkalosis, Uncompensated
20. Answer: C. Respiratory Acidosis, Fully Compensated

You might also like